Civil Law Compilation Bar Q&a 1990-2017.pdf

  • Uploaded by: Reynaldo Yu
  • 0
  • 0
  • January 2021
  • PDF

This document was uploaded by user and they confirmed that they have the permission to share it. If you are author or own the copyright of this book, please report to us by using this DMCA report form. Report DMCA


Overview

Download & View Civil Law Compilation Bar Q&a 1990-2017.pdf as PDF for free.

More details

  • Words: 206,462
  • Pages:
Loading documents preview...
COMPILATION OF THE CIVIL LAW BAR EXAMINATIONS QUESTIONS AND SUGGESTED ANSWERS (1990-2017)

*I do not own any of the materials I’ve compiled in this pdf file. It’s all found in the internet, just have the patience to look for it since it’s scattered. Giving credits to the authors of these materials. So sharing all of it for free to all my fellow law students. - Bek

MATERIALS COMPILED: 1) Answers to Bar Examinations Questions in Civil Law 1990-2006 -> https://www.vbook.pub.com/doc/ 36387239/209-Suggested-Answers-in-Civil-Law-BarExams-1990-2006 2) A Compilation of the Questions and Suggested Answers in the Philippine Bar Examinations 2007-2013 in Civil Law -> https://www.vbook.pub.com/ doc/262939078/2007-2013-Civil-Law-Philippine-BarExamination-Questions-and-Suggested-AnswersJayArhSals-Ladot 3) UST 2013 2015 CIVIL LAW BAR ESSAY Q & SUGGESTED ANSWERS -> https://www.academia.edu/ 36905601UST_2013_2015_CIVIL_LAW_BAR_ESSAY_ Q_and_SUGGESTED_ANSWERS 4) 2016 CIVIL LAW BAR QA -> https://edoc.site/2016civil-law-bar-qa-pdf-free.html 5) 2017 Bar Suggested Answers By Rabuya -> https:// edoc.site/civ-2017-bar-suggested-answers-by-rabuyapdf-free.html

CIVIL LAW Answers to the BAR as Arranged by Topics (Year 1990-2006)

ANSWERS TO BAR EXAMINATION QUESTIONS IN

CIVIL LAW ARRANGED BY TOPIC (1990

– 2006)

First Edition - Edited and Arranged by:

July 26, 2005 Atty. Janette Laggui-Icao and Atty. Alex Andrew P. Icao (Silliman University College of Law) Latest Edition – Edited and Arranged by: ROMUALDO L. SEÑERIS II Silliman University College of Law

From the ANSWERS TO BAR EXAMINATION QUESTIONS by the UP LAW COMPLEX & Philippine Association of Law Schools

CIVIL LAW Answers to the BAR as Arranged by Topics (Year 1990-2006)

FORWARD This work is not intended for sale or commerce. This work is freeware. It may be freely copied and distributed, nevertheless, PERMISSION TO COPY from the editors is ADVISABLE to protect the interest of the ORIGINAL SOURCES/REFERENCES of this material…. It is primarily intended for all those who desire to have a deeper understanding of the issues touched by the Philippine Bar Examinations and its trend. It is specially intended for law students from the provinces who, very often, are recipients of deliberately distorted notes from other unscrupulous law schools and students. Share to others this work and you will be richly rewarded by God in heaven. It is also very good karma.

We would like to seek the indulgence of the reader for some Bar Questions which are improperly classified under a topic and for some topics which are improperly or ignorantly phrased, for the authors are just Bar Reviewees who have prepared this work while reviewing for the Bar Exams under time constraints and within their limited knowledge of the law. We would like to seek the reader’s indulgence for a lot of typographical errors in this work.

The Authors

CIVIL LAW Answers to the BAR as Arranged by Topics (Year 1990-2006)

Table of Contents GENERAL PRINCIPLES ....................................................................................................................................... 10 Civil law vs. Common Law (1997) ................................................................................................................................ 10 Effect of Obiter & Dissenting Opinion; SC Decisions (1994)......................................................................................... 10 Effectivity of Laws (1990) ............................................................................................................................................ 10 Equity follows the Law (2003)...................................................................................................................................... 10 Ignorance of the Law vs. Mistake of Fact (1996)........................................................................................................... 11 Inferior Courts Decisions (1994) .................................................................................................................................. 11 Prejudicial Questions (1997)........................................................................................................................................ 11 PERSONS................................................................................................................................................................... 11 Change of Name; Under RA 9048 (2006) ...................................................................................................................... 11 Death; Effects; Simultaneous Death (1998).................................................................................................................. 12 Death; Effects; Simultaneous Death (1999).................................................................................................................. 12 Death; Effects; Simultaneous Death (2000).................................................................................................................. 12 Juridical Capacity vs. Capacity to Act (1996) ............................................................................................................... 12 Juridical Capacity; Natural Persons (1999) .................................................................................................................. 13 Waiver of Rights (2004) ............................................................................................................................................... 13 CONFLICT OF LAWS............................................................................................................................................. 13 Appilicable Laws; laws governing contracts (1992) ..................................................................................................... 13 Applicable Laws; Arts 15, 16 & 17 (1998) ..................................................................................................................... 13 Applicable Laws; Arts 15, 16, 17 (2002)........................................................................................................................ 14 Applicable Laws; Capacity to Act (1998)...................................................................................................................... 14 Applicable Laws; Capacity to Buy Land (1995) ............................................................................................................ 15 Applicable Laws; Capacity to Contract (1995).............................................................................................................. 15 Applicable Laws; capacity to succeed (1991)............................................................................................................... 15 Applicable Laws; contracts contrary to public policy (1996) ........................................................................................ 15 Applicable Laws; Contracts of Carriage (1995) ............................................................................................................ 16 Applicable Laws; Labor Contracts (1991) .................................................................................................................... 16 Applicable Laws; laws governing marriages (1992) ..................................................................................................... 17 Applicable Laws; laws governing marriages (2003) ..................................................................................................... 17 Applicable Laws; Sale of Real Property (1995)............................................................................................................. 17 Applicable Laws; Succession; Intestate & Testamentary (2001)................................................................................... 18 Applicable Laws; Sucession of Aliens (1995)............................................................................................................... 18 Applicable Laws; Wills executed abroad (1993) ........................................................................................................... 18 Definition; Cognovit; Borrowing Statute; Characterization (1994) ................................................................................ 18 Definition; forum non-conveniens; long-arm statute (1994) ......................................................................................... 19 Divorce; effect of divorce granted to former Filipinos; Renvoi Doctrine (1997)............................................................. 19 Domiciliary theory vs. Nationality Theory (2004).......................................................................................................... 19 Forum Non Conveniens & Lex Loci Contractus (2002)................................................................................................. 19 Nationality Theory (2004)............................................................................................................................................. 20 Naturalization (2003) ................................................................................................................................................... 20 Theory; significant relationships theory (1994)............................................................................................................ 20 Torts; Prescriptive Period (2004) ................................................................................................................................. 21 ADOPTION................................................................................................................................................................. 21 Adoption; Use of Surname of her Natural Mother (2006) .............................................................................................. 21 Inter-Country Adoption; Formalities (2005).................................................................................................................. 21 Parental Authority; Rescission of Adoption (1994) ...................................................................................................... 21 Qualification of Adopter (2005).................................................................................................................................... 22 Qualification of Adopter; Applicable Law (2001) .......................................................................................................... 22 Qualifications of Adopter (2000) .................................................................................................................................. 22 Qualifications of Adopter (2003) .................................................................................................................................. 23 Successional Rights of Adopted Child (2004) .............................................................................................................. FAMILY CODE.......................................................................................................................................................... 23 23 Emancipation (1993).................................................................................................................................................... 23 Family Code; Retroactive Application; Vested Rights (2000)........................................................................................ 24

CIVIL LAW Answers to the BAR as Arranged by Topics (Year 1990-2006) Family Home; Dwelling House (1994)

.......................................................................................................................... 24 Family; Constitutional Mandates; Divorce (1991) ......................................................................................................... 24 Marriage; Annulment; Effects; Requisites Before Remarriage (1990) ........................................................................... 24 Marriage; Annulment; Grounds (1991)......................................................................................................................... 25 Marriage; Annulment; Judicial Declaration (1993)........................................................................................................ 25 Marriage; Annulment; Legal Separation; Prescription of Actions (1996) ...................................................................... 25 Marriage; Annulment; Proper Party (1990)................................................................................................................... 26 Marriage; Annulment; Proper Party (1995)................................................................................................................... 26 Marriage; Divorce Decree; Void Marriages (1992) ........................................................................................................ 26 Marriage; Divorce Decrees; Filiation of Children (2005) ............................................................................................... 26 Marriage; Divorce Decrees; Filipino Spouses becoming Alien (1996)........................................................................... 27 Marriage; Divorce Decrees; Filipino Spouses becoming Alien (1999)........................................................................... 27 Marriage; Donations by Reason of Marriage; Effect of Declaration of Nullity (1996) ..................................................... 28 Marriage; Grounds; Declaration of Nullity: Annulment: Legal Separation: Separation of Property (2003) ..................... 28 Marriage; Grounds; Nullity; Annulment; Legal Separation (1997)................................................................................. 29 Marriage; Legal Separation; Declaration of Nullity (2002)............................................................................................. 29 Marriage; Legal Separation; Grounds; Prescriptive Period (1994) ................................................................................ 29 Marriage; Legal Separation; Mutual guilt (2006) ........................................................................................................... 29 Marriage; Non-Bigamous Marriages (2006) .................................................................................................................. 30 Marriage; Property Relations; Void Marriages (1991) ................................................................................................... 30 Marriage; Psychological Incapacity (1996)................................................................................................................... 30 Marriage; Psychological Incapacity (2006)................................................................................................................... 31 Marriage; Psychological Incapacity (2006)................................................................................................................... 31 Marriage; Requisites (1995) ......................................................................................................................................... 31 Marriage; Requisites (1999) ......................................................................................................................................... 32 Marriage; Requisites; Marriage License (1996) ............................................................................................................ 32 Marriage; Requisites; Marriage License (2002) ............................................................................................................ 33 Marriage; Requisites; Solemnizing Officers (1994)....................................................................................................... 33 Marriage; Requisites; Void Marriage (1993) ................................................................................................................. 33 Marriage; Void Marriages (2004) .................................................................................................................................. 34 Marriage; Void Marriages (2006) .................................................................................................................................. 34 Marriage; Void Marriages; Psychological Incapacity (2002) ......................................................................................... 35 Parental Authority; Child under 7 years of age (2006) .................................................................................................. 35 Parental Authority; Special Parental Authority; Liability of Teachers (2003)................................................................. 35 Parental Authority; Substitute vs. Special (2004) ......................................................................................................... 35 Paternity & Filiation (1999) .......................................................................................................................................... 36 Paternity & Filiation; Artificial Insemination; Formalities (2006) ................................................................................... 36 Paternity & Filiation; Common-Law Union (2004)......................................................................................................... 36 Paternity & Filiation; Proofs; Limitations; Adopted Child (1995)................................................................................... 36 Paternity & Filiation; Recognition of illegitimate Child (2005)....................................................................................... 37 Paternity & Filiation; Rights of Legitimate Children (1990) ........................................................................................... 37 Presumptive Legitime (1999) ....................................................................................................................................... 38 Property Relations; Absolute Community (1994) ......................................................................................................... 38 Property Relations; Ante Nuptial Agreement (1995) ..................................................................................................... 39 Property Relations; Conjugal Partnership of Gains (1998) ........................................................................................... 39 Property Relations; Marriage Settlement; Conjugal Partnership of Gains (2005) .......................................................... 39 Property Relations; Marriage Settlements (1991) ......................................................................................................... 40 Property Relations; Marriage Settlements (1995) ......................................................................................................... 40 Property Relations; Obligations; Benefit of the Family (2000) ...................................................................................... 41 Property Relations; Unions without Marriage (1992) .................................................................................................... 41 Property Relations; Unions without Marriage (1997) .................................................................................................... 41 Property Relations; Unions without Marriage (2000) .................................................................................................... 42 SUCCESSION........................................................................................................................................................... 42 Amount of Successional Rights (2004) ........................................................................................................................ 42 Barrier between illegitimate & legitimate relatives (1993) ............................................................................................. 42 Barrier between illegitimate & legitimate relatives (1996) ............................................................................................. 43 Collation (1993) ........................................................................................................................................................... 43 Disinheritance vs. Preterition (1993) ............................................................................................................................ 43 Disinheritance; Ineffective (1999) ................................................................................................................................ 43 Disinheritance; Ineffective; Preterition (2000) .............................................................................................................. 44

CIVIL LAW Answers to the BAR as Arranged by Topics (Year 1990-2006) Heirs; Intestate Heirs; Reserva Troncal (1995)

............................................................................................................. 44 Heirs; Intestate Heirs; Shares (2003)............................................................................................................................ 45 Intestate Succession (1992)......................................................................................................................................... 45 Intestate Succession (1997)......................................................................................................................................... 45 Intestate Succession (1998)......................................................................................................................................... 46 Intestate Succession (1998)......................................................................................................................................... 46 Intestate Succession (1999)......................................................................................................................................... 46 Intestate Succession (2000)......................................................................................................................................... 46 Intestate Succession; Reserva Troncal (1999) ............................................................................................................. 47 Legitime (1997)............................................................................................................................................................ 47 Legitime; Compulsory Heirs (2003).............................................................................................................................. 47 Legitime; Compulsory Heirs vs. Secondary Compulsory Heirs (2005).......................................................................... 48 Preterition (2001)......................................................................................................................................................... 48 Preterition; Compulsory Heir (1999) ............................................................................................................................ 48 Proceedings; Intestate Proceedings; Jurisdiction (2004) ............................................................................................. 48 Succession; Death; Presumptive Legitime (1991) ........................................................................................................ 49 Wills; Codicil; Institution of Heirs; Substitution of Heirs (2002).................................................................................... 49 Wills; Formalities (1990) .............................................................................................................................................. 50 Wills; Holographic Wills; Insertions & Cancellations (1996) ......................................................................................... 50 Wills; Holographic Wills; Witnesses (1994).................................................................................................................. 50 Wills; Joint Wills (2000) ............................................................................................................................................... 50 Wills; Probate; Intrinsic Validity (1990) ........................................................................................................................ 51 Wills; Probate; Notarial and Holographic Wills (1997) .................................................................................................. 51 Wills; Revocation of Wills; Dependent Relative Revocation (2003)............................................................................... 51 Wills; Testamentary Disposition (2006)........................................................................................................................ 52 Wills; Testamentary Intent (1996) ................................................................................................................................ 52 DONATION ................................................................................................................................................................ 52 Donation vs. Sale (2003).............................................................................................................................................. 52 Donations; Condition; Capacity to Sue (1996) ............................................................................................................. 52 Donations; Conditions; Revocation (1991)................................................................................................................... 53 Donations; Effect; illegal & immoral conditions (1997)................................................................................................. 53 Donations; Formalities; Mortis Causa (1990) ............................................................................................................... 54 Donations; Formalities; Mortis Causa (1998) ............................................................................................................... 54 Donations; Inter Vivos; Acceptance (1993) .................................................................................................................. 54 Donations; Perfection (1998) ....................................................................................................................................... 54 Donations; Requisites; Immovable Property................................................................................................................ 55 Donations; Unregistered; Effects; Non-Compliance; Resolutory Condition (2006) ....................................................... 55 Donations; Validity; Effectivity; for Unborn Child (1999) .............................................................................................. 55 Donations; with Resolutory Condition (2003)............................................................................................................... 56 PROPERTY................................................................................................................................................................ 56 Accretion; Alluvion (2001) ........................................................................................................................................... 56 Accretion; Avulsion (2003) .......................................................................................................................................... 56 Builder; Good Faith (1992)........................................................................................................................................... 57 Builder; Good Faith vs. Bad Faith (1999) ..................................................................................................................... 57 Builder; Good Faith vs. Bad Faith (2000) ..................................................................................................................... 57 Builder; Good Faith vs. Bad Faith; Accession (2000) ................................................................................................... 58 Builder; Good Faith vs. Bad Faith; Presumption (2001)................................................................................................ 58 Chattel Mortgage vs. Pledge (1999) ............................................................................................................................. 58 Chattel Mortgage; Immovables (1994).......................................................................................................................... 59 Chattel Mortgage; Immovables (2003).......................................................................................................................... 59 Chattel Mortgage; Possession (1993) .......................................................................................................................... 60 Chattel Mortgage; Preference of Creditors (1995) ........................................................................................................ 60 Easement vs. Usufruct (1995)...................................................................................................................................... 60 Easement; Effects; Discontinuous Easements; Permissive Use (2005) ........................................................................ 61 Easement; Nuisance; Abatement (2002) ...................................................................................................................... 61 Easements; Classification (1998)................................................................................................................................. 62 Easements; Right of Way Page 5Right of of 119 (1993).................................................................................................................................. 62 Easements; Way (2000).................................................................................................................................. 62 Easements; Right of Way; Inseparability (2001) ........................................................................................................... 62

CIVIL LAW Answers to the BAR as Arranged by Topics (Year 1990-2006) Easements; Right of Way; Requisites (1996)

............................................................................................................... 63 Ejectment Suit vs. Cancellation of Title (2005) ............................................................................................................. 63 Ejectment Suit; Commodatum (2006) .......................................................................................................................... 63 Extra-Judicial Partition; Fraud (1990)........................................................................................................................... 63 Hidden Treasure (1995) ............................................................................................................................................... 64 Hidden Treasures (1997) ............................................................................................................................................. 64 Mortgage; Pactum Commissorium (1999) .................................................................................................................... 64 Mortgage; Pactum Commissorium (2001) .................................................................................................................... 65 Mortgage; Right of Redemption vs. Equity of Redemption (1999) ................................................................................ 65 Nuisance; Family House; Not Nuisance per se (2006) .................................................................................................. 65 Nuisance; Public Nuisance vs. Private Nuisance (2005)............................................................................................... 65 Ownership; Co-Ownership (1992) ................................................................................................................................ 66 Ownership; Co-Ownership; Prescription (2000) ........................................................................................................... 66 Ownership; Co-Ownership; Prescription (2002) ........................................................................................................... 67 Ownership; Co-Ownership; Redemption (1993) ........................................................................................................... 67 Ownership; Co-Ownership; Redemption (2000) ........................................................................................................... 67 Ownership; Co-Ownership; Redemption (2002) ........................................................................................................... 67 Possession (1998)....................................................................................................................................................... 68 Property; Real vs. Personal Property (1995) ................................................................................................................ 68 Property; Real vs. Personal Property (1997) ................................................................................................................ 68 Sower; Good Faith/ Bad Faith (2000) ........................................................................................................................... 69 Usufruct (1997)............................................................................................................................................................ 69 LAND TRANSFER & DEEDS............................................................................................................................... 69 Acquisition of Lands; Citizenship Requirement (2003)................................................................................................. 69 Adverse Claims; Notice of Levy (1998) ........................................................................................................................ 69 Annotation of Lis Pendens; When Proper (2001).......................................................................................................... 70 Foreshore Lands (2000)............................................................................................................................................... 70 Forgery; Innocent Purchaser; Holder in Bad Faith (2005)............................................................................................. 70 Forgery; Innocent Purchaser; Mirror Principle (1991) .................................................................................................. 71 Fraud; Procurement of Patent; Effect (2000) ................................................................................................................ 71 Homestead Patents; Void Sale (1999) .......................................................................................................................... 71 Innocent Purchaser for Value (2001)............................................................................................................................ 72 Mirror Principle (1990) ................................................................................................................................................. 72 Mirror Principle; Forgery; Innocent Purchaser (1999) .................................................................................................. 73 Notice of Lis Pendens (1995) ....................................................................................................................................... 73 Notice of Lis Pendens; Transferee Pendente Lite (2002) .............................................................................................. 73 Prescription & Laches; Elements of Laches (2000) ...................................................................................................... 74 Prescription & Laches; Indefeasibility Rule of Torrens Title (2002) .............................................................................. 74 Prescription (1990) ...................................................................................................................................................... 75 Prescription; Real Rights (1992) .................................................................................................................................. 75 Primary Entry Book; Acquisitive Prescription; Laches (1998) ...................................................................................... 76 Reclamation of Foreshore Lands; Limitations (2000) ................................................................................................... 76 Registration; Deed of Mortgage (1994)......................................................................................................................... 77 Remedies; Judicial Confirmation; Imperfect Title (1993) .............................................................................................. 77 Remedies; Judicial Reconstitution of Title (1996) ........................................................................................................ 77 Remedies; Procedure; Consulta (1994)........................................................................................................................ 77 Remedies; Reconveyance vs. Reopening of a Decree; Prescriptive Period (2003)........................................................ 78 Remedies; Reconveyance; Elements (1995) ................................................................................................................ 78 Remedies; Reconveyance; Prescriptive Period (1997) ................................................................................................. 79 Remedies; Reopening of a Decree; Elements (1992).................................................................................................... 79 Torrens System vs. Recording of Evidence of Title (1994) ........................................................................................... 80 Unregistered Land (1991) ............................................................................................................................................ 80 CONTRACTS ............................................................................................................................................................ 80 Consensual vs. Real Contracts; Kinds of Real Contracts (1998) .................................................................................. 80 Consideration; Validity (2000)...................................................................................................................................... 80 Contract of Option; Elements (2005)............................................................................................................................ 81 Inexistent Contracts vs. Annullable Contracts (2004)................................................................................................... 81 Nature of Contracts; Obligatoriness (1991).................................................................................................................. 81 Nature of Contracts; Privity of Contract (1996) ............................................................................................................ 82

CIVIL LAW Answers to the BAR as Arranged by Topics (Year 1990-2006) Nature of Contracts; Relativity of Contracts

(2002) ...................................................................................................... 82 Rescission of Contracts; Proper Party (1996) .............................................................................................................. 82 OBLIGATIONS ......................................................................................................................................................... 83 Aleatory Contracts; Gambling (2004)........................................................................................................................... 83 Conditional Obligations (2000) .................................................................................................................................... 83 Conditional Obligations (2003) .................................................................................................................................... 83 Conditional Obligations; Promise (1997) ..................................................................................................................... 84 Conditional Obligations; Resolutory Condition (1999) ................................................................................................. 84 Extinguishment; Assignment of Rights (2001) ............................................................................................................. 84 Extinguishment; Cause of Action (2004)...................................................................................................................... 85 Extinguishment; Compensation (2002) ........................................................................................................................ 85 Extinguishment; Compensation vs. Payment (1998) .................................................................................................... 85 Extinguishment; Compensation/Set-Off; Banks (1998) ................................................................................................ 85 Extinguishment; Condonation (2000) .......................................................................................................................... 85 Extinguishment; Extraordinary Inflation or Deflation (2001)......................................................................................... 86 Extinguishment; Loss (1994) ....................................................................................................................................... 86 Extinguishment; Loss; Impossible Service (1993) ....................................................................................................... 86 Extinguishment; Novation (1994)................................................................................................................................. 87 Extinguishment; Payment (1995) ................................................................................................................................. 87 Liability; Lease; Joint Liability (2001) .......................................................................................................................... 87 Liability; Solidary Liability (1998)................................................................................................................................. 87 Liability; Solidary Obligation (1992)............................................................................................................................. 88 Liability; Solidary Obligation; Mutual Guaranty (2003) ................................................................................................. 88 Loss of the thing due; Force Majeure (2000) ................................................................................................................ 88 Non-Payment of Amortizations; Subdivision Buyer; When justified (2005)................................................................... 89 Period; Suspensive Period (1991)................................................................................................................................ 89 TRUST......................................................................................................................................................................... 89 Express Trust; Prescription (1997) .............................................................................................................................. 89 Implied Trust (1998)..................................................................................................................................................... 90 Trust; Implied Resulting Trust (1995)........................................................................................................................... 91 SALES.......................................................................................................................................................................... 91 Assignment of Credit vs. Subrogation (1993)............................................................................................................... 91 Conditional Sale vs. Absolute Sale (1997).................................................................................................................... 91 Contract of Sale vs. Agency to Sell (1999) ................................................................................................................... 91 Contract of Sale; Marital Community Property; Formalities (2006) ............................................................................... 91 Contract to Sell (2001)................................................................................................................................................. 92 Contract to Sell vs. Contract of Sale (1997).................................................................................................................. 92 Contract to Sell; Acceptance; Right of First Refusal (1991).......................................................................................... 92 Double Sales (2001)..................................................................................................................................................... 92 Double Sales (2004)..................................................................................................................................................... 93 Equitable Mortgage (1991).......................................................................................................................................... 93 Equitable Mortgage vs. Sale (2005).............................................................................................................................. 93 Immovable Property; Rescission of Contract (2003) .................................................................................................... 94 Maceda Law (2000)...................................................................................................................................................... 94 Maceda Law; Recto Law (1999).................................................................................................................................... 95 Option Contract (2002) ................................................................................................................................................ 95 Option Contract; Earnest Money (1993) ....................................................................................................................... 95 Perfected Sale; Acceptance of Earnest Money (2002) .................................................................................................. 95 Redemption; Legal; Formalities (2001) ........................................................................................................................ 96 Redemption; Legal; Formalities (2002) ........................................................................................................................ 96 Right of First Refusal; Lessee; Effect (1996) ................................................................................................................ 96 Right of First Refusal; Lessee; Effect (1998) ................................................................................................................ 97 Right of Repurchase (1993) ......................................................................................................................................... 97 Transfer of Ownership; Non-Payment of the Price (1991)............................................................................................. 97 Transfer of Ownership; Risk of Loss (1990) ................................................................................................................. 97 LEASE.......................................................................................................................................................................... 97 Extinguishment; Total Distruction; Leased Property (1993) ......................................................................................... 97 Implied New Lease (1999)............................................................................................................................................ 98 Lease of Rural Lands (2000) ........................................................................................................................................ 98 Page 7 of 119

CIVIL LAW Answers to the BAR as Arranged by Topics (Year 1990-2006) Leasee & Lessor; Rights and Obligations (1990)

......................................................................................................... 98 Leasee; Death Thereof; Effects (1997) ......................................................................................................................... 98 Option to Buy; Expired (2001)...................................................................................................................................... 98 Sublease vs. Assignment of Lease; Rescission of Contract (2005) .............................................................................. 99 Sublease; Delay in Payment of Rentals (1994) ............................................................................................................. 99 Sublease; Sublessee; Liability (1999) ........................................................................................................................ 100 Sublease; Sublessee; Liability (2000) ........................................................................................................................ 100 Sublease; Validity; Assignment of Sublease (1990) ................................................................................................... 100 COMMON CARRIERS.......................................................................................................................................... 100 Extraordinary Diligence (2000) .................................................................................................................................. 100 AGENCY ................................................................................................................................................................... 101

Agency (2003) ........................................................................................................................................................... 101 Agency vs. Sale (2000) .............................................................................................................................................. 101 Agency; coupled with an interest (2001) .................................................................................................................... 101 Agency; Guarantee Commission (2004)..................................................................................................................... 101 Agency; Real Estate Mortgage (2004) ........................................................................................................................ 101 Appointment of Sub-Agent (1999).............................................................................................................................. 102 General Agency vs. Special Agency (1992)................................................................................................................ 102 Powers of the Agent (1994)........................................................................................................................................ 102 Termination; Effect of Death of Agent...................................................................................................................................................... (1997) ............................................................................................................. 103 PARTNERSHIP 103

Composition of Partnerships; Spouses; Corporations (1994) .................................................................................... 103 Conveyance of a Partner’s Share Dissolution (1998).................................................................................................. 103 Dissolution of Partnership (1995) .............................................................................................................................. 103 Dissolution of Partnership; Termination (1993).......................................................................................................... 104 Effect of Death of Partner (1997)................................................................................................................................ 104 Obligations of a Partner (1992) .................................................................................................................................. 104 Obligations of a Partner; Industrial Partner (2001) .................................................................................................................................... ..................................................................................................... 104 COMMODATUM & MUTUUM 104 Commodatum (1993) ................................................................................................................................................. 104 Commodatum (2005) ................................................................................................................................................. 105 Commodatum vs. Usufruct (1998) ............................................................................................................................. 105 Mutuum vs. Commodatum (2004) .............................................................................................................................. 106 Mutuum; Interests (2001)........................................................................................................................................... 106 Mutuum; Interests (2002)........................................................................................................................................... 106 Mutuum; Interests (2004) ........................................................................................................................................... DEPOSIT................................................................................................................................................................... 107

106 Compensation; Bank Loan (1997).............................................................................................................................. 107 Deposit; Exchange (1992).......................................................................................................................................... 107 SURETY.................................................................................................................................................................... 107 Recovery of Deficiency (1997) ................................................................................................................................... 107

ANTICHRESIS........................................................................................................................................................ 107 Antichresis (1995) ..................................................................................................................................................... 107 PLEDGE .................................................................................................................................................................... 108 Pledge (1994) ............................................................................................................................................................ 108 Pledge (2004) ............................................................................................................................................................ 108 Pledge; Mortgage; Antichresis (1996) ........................................................................................................................ 108 QUASI-CONTRACT.............................................................................................................................................. 108 Quasi-Contracts; Negotiorium Gestio (1992) ............................................................................................................. 109 Quasi-Contracts; Negotiorium Gestio (1993) ............................................................................................................. 109 Quasi-Contracts; Negotiorium Gestio (1995) ............................................................................................................. 109 Quasi-Contracts; Solutio Indebiti (2004) .................................................................................................................... 110 TORTS & DAMAGES ........................................................................................................................................... 110 Collapse of Structures; Last Clear Chance (1990)...................................................................................................... 110 Damages (1994)......................................................................................................................................................... 111 Damages arising from Death of Unborn Child (1991) ................................................................................................. 111 Damages arising from Death of Unborn Child (2003) ................................................................................................. 111 Death Indemnity (1994)..............................................................................................................................................Page 111 8 of 119

CIVIL LAW Answers to the BAR as Arranged by Topics (Year 1990-2006) Defense; Due Diligence in Selection

(2003)................................................................................................................ 112 Filing of Separate Civil Action; Need for Reservation (2003) ...................................................................................... 112 Fortuitous Event; Mechanical Defects (2002) ............................................................................................................. 112 Liability; Airline Company; Non-Performance of an Obligation (2004)........................................................................ 112 Liability; Airline Company; Non-Performance of an Obligation (2005)........................................................................ 113 Liability; Employer; Damage caused by Employees (1997) ........................................................................................ 113 Liability; owner who was in the vehicle (1996) ........................................................................................................... 114 Liability; owner who was in the vehicle (1998) ........................................................................................................... 114 Liability; owner who was in the vehicle (2002) ........................................................................................................... 114 Moral Damages & Atty Fees (2002) ............................................................................................................................ 114 Moral Damages; Non-Recovery Thereof (2006) .......................................................................................................... 115 Quasi-Delict (1992) .................................................................................................................................................... 115 Quasi-Delict (2005) .................................................................................................................................................... 115 Quasi-Delict; Acts contrary to morals (1996) ............................................................................................................. 115 Quasi-Delict; Mismanagement of Depositor’s Account (2006).................................................................................... 116 Vicarious Liability (1991) ........................................................................................................................................... 116 Vicarious Liability (2001) ........................................................................................................................................... 117 Vicarious Liability (2002) ........................................................................................................................................... 117 Vicarious Liability (2004) ........................................................................................................................................... 117 Vicarious Liability (2006) ........................................................................................................................................... 117 Vicarious Liability; Public Utility (2000) ..................................................................................................................... 118

INTELLECTUAL PROPERTY ........................................................................................................................... 118 Intellectual Creation (2004)........................................................................................................................................ 118

CIVIL LAW Answers to the BAR as Arranged by Topics (Year 1990-2006)

GENERAL PRINCIPLES

Civil law vs. Common Law (1997) How would you compare the Civil Law system in its governance and trend with that of the Common Law system?

cannot have the force of official precedents. It is as if the Court were turning aside from the main topic of the case to collateral subjects: a dissenting opinion affirms or overrules a claim, right or obligation. It neither disposes nor awards anything it merely expresses the view of the dissenter. (Civil Code, Paras]

SUGGESTED ANSWER:

As regards "governance": Governance in Civil Law is codal, statutory and written law. It is additionally derived from case law. Common law is basically derived from case law. As regards "trend": Civil law is now tending to rely more and more on decisions of the courts explaining the laws. Common law is now codifying laws more and more. So they are now merging towards similar systems. Additional Answers:

1. COMMON LAW refers to the traditional part of the law as distinct from legislation; it refers to the universal part of law as distinct from particular local customs (Encyclopedia Americana, Vol. 7). On the other hand, CIVIL LAW is understood to be that branch of law governing the relationship of persons in respect of their personal and private interests as distinguished from both public and international laws. In common law countries, the traditional responsibility has for the most part been with the judges; in civil law countries, the task is primarily reposed on the lawmakers. Contemporary practices, however, so indicate a trend towards centralizing that function to professional groups that may indeed, see the gradual assimilation in time of both systems. [Vitug, Civil. Law and Jurisprudence, p. XX) 2. In Civil Law, the statutes theoretically take precedence over court decisions interpreting them; while in Common Law, the court decisions resolving specific cases are regarded as law rather than the statutes themselves which are, at the start, merely embodiments of case law. Civil Law is code law or written law, while Common Law is case law. Civil Law adopts the deductive method - from the general to the particular, while the Common Law uses the inductive approach from the particular to the general. Common Law relies on equity. Civil Law anchors itself on the letter of the law. The civilists are for the judge-proof law even as the Common Law Is judge-made law. Civil Law judges are merely supposed to apply laws and not interpret them.

Effect of Obiter & Dissenting Opinion; SC Decisions (1994) 2) What are the binding effects of an obiter dictum and a dissenting opinion? 3) How can a decision of the Supreme Court be set aside? ALTERNATIVE ANSWERS: 2) None. Obiter dictum and opinions are not necessary to the determination of a case. They are not binding and

3) A decision of a division of the Supreme Court maybe set aside by the Supreme Court sitting en banc, a Supreme Court decision may be set aside by a contrary ruling of the Supreme Court itself or by a corrective legislative act of Congress, although said laws cannot adversely affect those favored prior to the Supreme Court decision. [Civil Code, Paras). Effectivity of Laws (1990) After a devastating storm causing widespread destruction in four Central Luzon provinces, the executive and legislative branches of the government agreed to enact a special law appropriating P1 billion for purposes of relief and rehabilitation for the provinces. In view of the urgent nature of the legislative enactment, it is provided in its effectivity clause that it shall take effect upon approval and after completion of publication in the Official Gazette and a newspaper of general circulation in the Philippines. The law was passed by the Congress on July 1, 1990. signed into law by the President on July 3, 1990, and published in such newspaper of general circulation on July 7, 1990 and in the Official Gazette on July 10, 1990. (a) As to the publication of said legislative enactment, is there sufficient observance or compliance with the requirements for a valid publication? Explain your answer. (b) When did the law take effect? Explain your (c) answer. Can the executive branch start releasing and disbursing funds appropriated by the said law the day following its approval? Explain your answer. SUGGESTED ANSWER:

(a) Yes, there is sufficient compliance. The law itself prescribes the requisites of publication for its effectivity, and all requisites have been complied with. (Article 2, Civil Code) (b) The law takes effect upon compliance with all the conditions for effectivity, and the last condition was complied with on July 10, 1990. Hence, the" law became effective on that date. (c) No. It was not yet effective when it was approved by Congress on July 1, 1990 and approved by the President on July 3, 1990. The other requisites for its effectivity were not yet complete at the time.

Equity follows the Law (2003) It is said that ―equity follows the law‖ What do you understand by this phrase, and what are its basic implications? 5% SUGGESTED ANSWER:

―Equity Follows the law‖ means that courts exercising equity jurisdiction are bound by rules of law and have no arbitrary discretion to disregard them. (Arsenal v IAC, 143 SCRA 40 [1986]). Equity is applied only in the absence of

Page 10 of 119

CIVIL LAW Answers to the BAR as Arranged by Topics (Year 1990-2006) but never against statutory law. (Toyota Motor Phil. V CA 1. The civil action involves an issue similar or intimately related to the issue raised in the criminal action, and 216 SCRA 236 [1992]).

Ignorance of the Law vs. Mistake of Fact (1996) Is there any difference in their legal effect between ignorance of the law and ignorance or mistake of fact? SUGGESTED ANSWER:

Yes, there is a difference. While ignorance of the law is not an excuse for not complying with it, ignorance of fact eliminates criminal intent as long as there is no negligence (Art, NCC). In addition, mistake on a doubtful or difficult question of law may be the basis of good faith (Art. 526. NCC). Mistake of fact may, furthermore, vitiate consent in a contract and make it voidable (Art. 1390. NCC). ALTERNATIVE ANSWER:

Yes. ignorance of the law differs in legal effect from Ignorance or mistake of fact. The former does not excuse a party from the legal consequences of his conduct while the latter does constitute an excuse and is a legal defense. Inferior Courts Decisions (1994) Are decisions of the Court of Appeals considered laws? ALTERNATIVE ANSWERS:

1) a) No, but decisions of the Court of Appeals may serve as precedents for inferior courts on points of law not covered by any Supreme Court decision, and a ruling of the Court of Appeals may become a doctrine. (Miranda vs.. Imperial 77 Phil. 1066).

b) No. Decisions of the Court of Appeals merely have persuasive, and therefore no mandatory effect. However, a conclusion or pronouncement which covers a point of law still undecided may still serve as judicial guide and it is possible that the same maybe raised to the status of doctrine. If after it has been subjected to test in the crucible of analysis, the Supreme Court should find that it has merits and qualities sufficient for its consideration as a rule of jurisprudence (Civil Code, Paras). Prejudicial Questions (1997) In the context that the term is used in Civil Law, state the (a) concept, (b) requisites and (c) consequences of a prejudicial question. SUGGESTED ANSWER:

(a) Concept A prejudicial question is one which must be decided first before a criminal action may be instituted or may proceed because a decision therein is vital to the judgment in the criminal case. In the case of People vs. Adelo Aragon (L5930, Feb. 17, 1954), the Supreme Court defined it as one which arises in a case, the resolution of which question is a logical antecedent of the issues involved in said case and the cognizance of which pertains to another tribunal (Paras, Vol. 1, Civil. Code Annotation, 1989 ed. p, 194).

2. the resolution of such issue determines whether or not the criminal action may proceed. (c) Consequences The criminal case must be suspended. Thus, in a criminal case for damages to one's property, a civil action that involves the ownership of said property should first be resolved (De Leon vs. Mabanag. 38 Phil. 202)

PERSONS Change of Name; Under RA 9048 (2006) Zirxthoussous delos Santos filed a petition for change of name with the Office of the Civil Registrar of Mandaluyong City under the administrative proceeding provided in Republic Act No. 9048. He alleged that his first name sounds ridiculous and is extremely difficult to spell and pronounce. After complying with the requirements of the law, the Civil Registrar granted his petition and changed his first name Zirxthoussous to "Jesus." His full name now reads "Jesus delos Santos." Jesus delos Santos moved to General Santos City to work in a multi-national company. There, he fell in love and married Mary Grace delos Santos. She requested him to have his first name changed because his new name "Jesus delos Santos" is the same name as that of her father who abandoned her family and became a notorious drug lord. She wanted to forget him. Hence, Jesus filed another petition with the Office of the Local Civil Registrar to change his first name to "Roberto." He claimed that the change is warranted because it will eradicate all vestiges of the infamy of Mary Grace's father. Will the petition for change of name of Jesus delos Santos to Roberto delos Santos under Republic Act No. 9048 prosper? Explain. (10%)

No, under the law, Jesus may only change his name once. In addition, the petition for change of name may be denied on the following grounds: (1) Jesus is neither ridiculous, nor tainted with dishonor nor extremely difficult to write or pronounce. (2) There is no confusion to be avoided or created with the use of the registered first name or nickname of the petitioner. (3) The petition involves the same entry in the same document, which was previously corrected or changed under this Order [Rules and Regulations Implementing RA 9048]. SUGGESTED ANSWER:

What entries in the Civil Registry may be changed or corrected without a judicial order? (2.5%)

SUGGESTED ANSWER: Only clerical or typographical errors (b) and first or nick names may be changed or corrected without 1 Requisites The prejudicial question must be determinative of the case a judicial order under RA 9048. before the court. 2 Jurisdiction to try said question must be lodged in another Clerical or typographical errors refer to mistakes committed tribunal. in the performance of clerical work in writing, copying, ADDITIONAL ANSWER:

transcribing or typing an entry in the civil register. The mistake is harmless and innocuous, such as errors in

CIVIL LAW Answers to the BAR as Arranged by Topics (Year 1990-2006)

spelling, visible to the eyes or obvious to the understanding, and can be corrected or changed only by reference to other existing records. Provided, however, that no correction must involve the change of nationality, age, status or sex of the petitioner. Death; Effects; Simultaneous Death (1998) Jaime, who is 65, and his son, Willy, who is 25, died in a plane crash. There is no proof as to who died first. Jaime's only surviving heir is his wife, Julia, who is also Willy's mother. Willy's surviving heirs are his mother, Julia and his wife, Wilma. 1. In the settlement of Jaime's estate, can Wilma successfully claim that her late husband, Willy had a hereditary share since he was much younger than his father and, therefore, should be presumed to have survived longer? [3%] 2. Suppose Jaime had a life insurance policy with his wife, Julia, and his son, Willy, as the beneficiaries. Can Wilma successfully claim that one-half of the proceeds should belong to Willy's estate? |2%J SUGGESTED ANSWER:

1. No, Wilma cannot successfully claim that Willy had a hereditary share in his father's estate. Under Art. 43, Civil Code, two persons "who are called to succeed each other" are presumed to have died at the same time, in the absence of proof as to which of them died first. This presumption of simultaneous death applies in cases involving the question of succession as between the two who died, who in this case are mutual heirs, being father and son. SUGGESTED ANSWER:

2. Yet, Wilma can invoke the presumption of survivorship and claim that one-half of the proceeds should belong to Willy's estate, under Sec. 3 (jj) par. 5 Rule 131, Rules of Court, as the dispute does not involve succession. Under this presumption, the person between the ages of 15 and 60 years is deemed to have survived one whose age was over 60 at the time of their deaths. The estate of Willy endowed with juridical personality stands in place and stead of Willy, as beneficiary. Death; Effects; Simultaneous Death (1999) Mr. and Mrs. Cruz, who are childless, met with a serious motor vehicle accident with Mr. Cruz at the wheel and Mrs. Cruz seated beside him, resulting in the instant death of Mr. Cruz. Mrs. Cruz was still alive when help came but she also died on the way to the hospital. The couple acquired properties worth One Million (P1 ,000,000.00) Pesos during their marriage, which are being claimed by the parents of both spouses in equal shares. Is the claim of both sets of parents valid and why? (3%) (b) Suppose in the preceding question, both Mr. and Mrs. Cruz were already dead when help came, so that no-body could say who died ahead of the other, would your answer be the same to the question as to who are entitled to the properties of the deceased couple? (2%) SUGGESTED ANSWER: (a) No, the claim of both parents is not valid. When Mr. Cruz died, he was succeeded by his wife and his parents as his intestate heirs who will share his estate equally. His estate was 0.5 Million pesos which is his half share in the

absolute community amounting to 1 Million Pesos. His wife, will, therefore, inherit O.25 Million Pesos and his parents will inherit 0.25 Million Pesos. When Mrs. Cruz died, she was succeeded by her parents as her intestate heirs. They will inherit all of her estate consisting of her 0.5 Million half share in the absolute community and her 0.25 Million inheritance from her husband, or a total of 0.750 Million Pesos.

In sum, the parents of Mr. Cruz will inherit 250,000 Pesos while the parents of Mrs. Cruz will inherit 750,000 Pesos. (b) This being a case of succession, in the absence of proof as to the time of death of each of the spouses, it is presumed they died at the same time and no transmission of rights from one to the other is deemed to have taken place. Therefore, each of them is deemed to have an estate valued at P500,000,00, or one-half of their conjugal property of P1 million. Their respective parents will thus inherit the entire P1 Million in equal shares, of P500,000.00 per set of parents. Death; Effects; Simultaneous Death (2000) b) Cristy and her late husband Luis had two children, Rose and Patrick, One summer, her mother-in-law, aged 70, took the two children, then aged 10 and 12, with her on a boat trip to Cebu. Unfortunately, the vessel sank en route, and the bodies of the three were never found. None of the survivors ever saw them on the water. On the settlement of her mother-in-law's estate, Cristy files a claim for a share of her estate on the ground that the same was inherited by her children from their grandmother in representation of their father, and she inherited the same from them. Will her action prosper? (2%) SUGGESTED ANSWER:

No, her action will not prosper. Since there was no proof as to who died first, all the three are deemed to have died at the same time and there was no transmission of rights from one to another, applying Article 43 of the New Civil Code. ALTERNATIVE ANSWER:

No, her action will not prosper. Under Article 43 of the New Civil Code, inasmuch as there is no proof as to who died first, all the three are presumed to have died at the same time and there could be no transmission of rights among them. Her children not having inherited from their grandmother. Cristy has no right to share in her mother-inlaw's estate. She cannot share in her own right as she is not a legal heir of her mother-in-law. The survivorship provision of Rule 131 of the Rules of Court does not apply to the problem. It applies only to those cases where the issue involved is not succession. Juridical Capacity vs. Capacity to Act (1996) Distinguish juridical capacity from capacity to act, SUGGESTED ANSWER:

JURIDICAL CAPACITY is the fitness to be the subject of legal relations while CAPACITY TO ACT is the power or to do acts with legal effect. The former is inherent in every natural person and is lost only through death while the latter is merely acquired and may be lost even before death (Art. 37, NCC). ALTERNATIVE ANSWER;

CIVIL LAW Answers to the BAR as Arranged by Topics (Year 1990-2006)

Juridical capacity, as distinguished from capacity to act: (a) the former is passive while the latter is active, (b) the former is inherent in a person while the latter is merely acquired, (c) the former is lost only through death while the latter may be lost through death or restricted by causes other than death, and Id) the former can exist without capacity to act while the latter cannot exist without juridical capacity.

Juridical Capacity; Natural Persons (1999) Elated that her sister who had been married for five years was pregnant for the first time, Alma donated P100,000.00 to the unborn child. Unfortunately, the baby died one hour after delivery. May Alma recover the P100.000.00 that she had donated to said baby before it was born considering that the baby died? Stated otherwise, is the donation valid and binding? Explain. (5%) SUGGESTED ANSWER:

The donation is valid and binding, being an act favorable to the unborn child, but only if the baby had an intra-uterine life of not less than seven months and pro-vided there was due acceptance of the donation by the proper person representing said child. If the child had less than seven months of intra-uterine life, it is not deemed born since it died less than 24 hours following its delivery, in which ease the donation never became effective since the donee never became a person, birth being determinative of personality. ALTERNATIVE ANSWER:

Even if the baby had an intra-uterine life of more than seven months and the donation was properly accepted, it would be void for not having conformed with the proper form. In order to be valid, the donation and acceptance of personal property exceeding five thousand pesos should be in writing. (Article 748, par. 3) Waiver of Rights (2004) B. DON, an American businessman, secured parental consent for the employment of five minors to play certain roles in two movies he was producing at home in Makati. They worked at odd hours of the day and night, but always accompanied by parents or other adults. The producer paid the children talent fees at rates better than adult wages. But a social worker, DEB, reported to OSWD that these children often missed going to school. They sometimes drank wine, aside from being exposed to drugs. In some scenes, they were filmed naked or in revealing costumes. In his defense, DON contended all these were part of artistic freedom and cultural creativity. None of the parents complained, said DON. He also said they signed a contract containing a waiver of their right to file any complaint in any office or tribunal concerning the working conditions of their children acting in the movies. Is the waiver valid and binding? Explain. (5%)

Why or why not?

SUGGESTED ANSWER: The waiver is not valid. Although the contracting parties may establish such stipulations, clauses, terms and conditions as they may deem convenient, they may not do so if such are contrary to law, morals, good customs, public order, or public policy (Article 1306, Civil Code). The parents' waiver to file a complaint concerning the working

conditions detrimental to the moral well-being of their children acting in the movies is in violation of the Family Code and Labor laws. Thus, the waiver is invalid and not binding.

The Child Labor Law is a mandatory and prohibitory law and the rights of the child cannot be waived as it is contrary to law and public policy.

CONFLICT OF LAWS Appilicable Laws; laws governing contracts (1992) X and Y entered into a contract in Australia, whereby it was agreed that X would build a commercial building for Y in the Philippines, and in payment for the construction, Y will transfer and convey his cattle ranch located in the United States in favor of X. What law would govern: a) The validity of the contract? b) The performance of the contract? c) The consideration of the contract? SUGGESTED ANSWER:

(a) The validity of the contract will be governed by Australian law, because the validity refers to the element of the making of the contract in this case. (Optional Addendum:"... unless the parties agreed to be bound by another law".}

(b) The performance will be governed by the law of the Philippines where the contract is to be performed. (c) The consideration will be governed by the law of United States where the ranch is located. (Optional Addendum: the In the foregoing cases, when the foreign law would apply, the absence of proof of that foreign law would render Philippine law applicable under the "eclectic theory".)

Applicable Laws; Arts 15, 16 & 17 (1998) Juan is a Filipino citizen residing in Tokyo, Japan. State what laws govern: 1 His capacity to contract marriage in Japan, [ 1%] 2 His successional rights as regards his deceased Filipino father's property in Texas, U.S.A. [1%] 3 The extrinsic validity of the last will and testament which Juan executed while sojourning in Switzerland. [2%] 4 The intrinsic validity of said will. (1%) SUGGESTED ANSWER:

1. Juan's capacity to contract marriage is governed by Philippine law -i.e., the Family Code -pursuant to Art. 15, Civil Code, which provides that our laws relating to, among others, legal capacity of persons are binding upon citizens of the Philippines even though living abroad. SUGGESTED ANSWER:

2. By way of exception to the general rule of lex rei sitae prescribed by the first paragraph of Art. 16. Civil Code, a person's successional rights are governed by the national law of the decedent (2nd par.. Art. 16). Since Juan's deceased Page 13 of 119

CIVIL LAW Answers to the BAR as Arranged by Topics (Year 1990-2006)

father was a Filipino citizen, Philippine law governs Juan's successional rights. ANOTHER ANSWER:

2. Juan's successional rights are governed by Philippine law, pursuant to Article 1039 and the second paragraph of Article 16, both of the Civil Code. Article 1039, Civil Code, provides that capacity to succeed shall be governed by the "law of the nation" of the decedent, i.e.. his national law. Article 16 provides in paragraph two that the amount of successional rights, order of succession, and intrinsic validity of testamentary succession shall be governed by the "national law" of the decedent who is identified as a Filipino in the present problem. SUGGESTED ANSWER:

3. The extrinsic validity of Juan's will is governed by (a) Swiss law, it being the law where the will was made (Art. 17. 1st par. Civil Code), or (b) Philippine law, by implication from the provisions of Art. 816, Civil Code, which allows even an alien who is abroad to make a will in conformity with our Civil Code. SUGGESTED ANSWER:

4. The intrinsic validity of his will is governed by Philippine law, it being his national law. (Art. 16, Civil Code) Applicable Laws; Arts 15, 16, 17 (2002) Felipe and Felisa, both Filipino citizens, were married in Malolos, Bulacan on June 1, 1950. In 1960 Felipe went to the United States, becoming a U.S. citizen in 1975. In 1980 they obtained a divorce from Felisa, who was duly notified of the proceedings. The divorce decree became final under California Law. Coming back to the Philippines in 1982, Felipe married Sagundina, a Filipino Citizen. In 2001, Filipe, then domiciled in Los Angeles, California, died, leaving one child by Felisa, and another one by Sagundina. He left a will which he left his estate to Sagundina and his two children and nothing to Felisa. Sagundina files a petition for the probate of Felipe’s will. Felisa questions the intrinsic validity of the will, arguing that her marriage to Felipe subsisted despite the divorce obtained by Felipe because said divorce is not recognized in the Philippines. For this reason, she claims that the properties and that Sagundina has no successional rights. A. Is the divorce secured by Felipe in California recognizable and valid in the Philippines? How does it affect Felipe’s marriage to Felisa? Explain. (2%). B. What law governs the formalities of the will? Explain. (1%) C. Will Philippine law govern the intrinsic validity of the will? Explain. (2%) SUGGESTED ANSWER:

A. (1.) The divorce secured by Felipe in California is recognizable and valid in the Philippines because he was no longer a Filipino at that time he secured it, Aliens may obtain divorces abroad which may be recognized in the Philippines provided that they are valid according to their national law (Van Dorn V. Romillo, Jr., 139 SCRA 139 [1985]; Quita v. Court of Appeals, 300 SCRA 406 [1998]; Llorente v. Court of Appeals, 345 SCRA 595 [2000] ).

(2). With respect to Felipe the divorce is valid, but with respect to Felisa it is not. The divorce will not capacitate Felisa to remarry because she and Felipe were both Filipinos at the time of their marriage. However, in DOJ Opinion No. 134 series of 1993, Felisa is allowed to remarry because the injustice sought to be corrected by Article 26 also obtains in her case. SUGGESTED ANSWER:

B. The foreigner who executes his will in the Philippines may observed the formalities described in: 1. The Law of the country of which he is a citizen under Article 817 of the New Civil Code, or 2. the law of the Philippines being the law of the place of execution under Article 17 of the New Civil Code. SUGGESTED ANSWER:

C. Philippine law will not govern the intrinsic validity of the will. Article 16 of the New Civil Code provides that intrinsic validity of testamentary provisions shall be governed by the National Law of the person whose succession is under consideration. California law will govern the intrinsic validity of the will. Applicable Laws; Capacity to Act (1998) Francis Albert, a citizen and resident of New Jersey, U.S.A., under whose law he was still a minor, being only 20 years of age, was hired by ABC Corporation of Manila to serve for two years as its chief computer programmer. But after serving for only four months, he resigned to join XYZ Corporation, which enticed him by offering more advantageous terms. His first employer sues him in Manila for damages arising from the breach of his contract of employment. He sets up his minority as a defense and asks for annulment of the contract on that ground. The plaintiff disputes this by alleging that since the contract was executed in the Philippines under whose law the age of majority is 18 years, he was no longer a minor at the time of perfection of the contract. 1 Will the suit prosper? [3%] 2 Suppose XYZ Corporation is impleaded as a codefendant, what would be the basis of its liability, if any? [2%] SUGGESTED ANSWER:

1. The suit will not prosper under Article 15, Civil Code, New Jersey law governs Francis Albert's capacity to act, being his personal law from the standpoint of both his nationality and his domicile. He was, therefore, a minor at the time he entered into the contract. ALTERNATIVE ANSWER:

1. The suit will not prosper. Being a U.S. national, Albert's capacity to enter into a contract is determined by the law of the State of which he is a national, under which he to still a minor. This is in connection with Article 15 of the Civil Code which embodies the said nationality principle of lex patriae. While this principle intended to apply to Filipino citizens under that provision, the Supreme Court in Recto v. Harden is of the view that the status or capacity of foreigners is to be determined on the basis of the same provision or principle, i.e., by U.S. law in the present problem.

CIVIL LAW Answers to the BAR as Arranged by Topics (Year 1990-2006)

Plaintiffs argument does not hold true, because status or capacity is not determined by lex loci contractus but by lex patriae. ANOTHER ANSWER:

1. Article 17 of the Civil Code provides that the forms and solemnities of contracts, wills and other public instruments shall be governed by the laws of the country in which they are executed. Since the contract of employment was executed in Manila, Philippine law should govern. Being over 18 years old and no longer a minor according to Philippine Law, Francis Albert can be sued. Thus, the suit of ABC Corporation against him for damages will prosper. SUGGESTED ANSWER:

2. XYZ Corporation, having enticed Francis Albert to break his contract with the plaintiff, may be held liable for damages under Art. 1314, Civil Code. ALTERNATIVE ANSWER:

2. The basis of liability of XYZ Corporation would be Article 28 of the Civil Code which states that: "Unfair competition in agricultural, commercial, or industrial enterprises or in labor through the use of force, intimidation, deceit, machination or any other unjust, oppressive or highhanded method shall give rise to a right of action by the person who thereby suffers damage." ANOTHER ANSWER: 2. No liability arises. The statement of the problem does not in any way suggest intent, malice, or even knowledge, on the part of XYZ Corporation as to the contractual relations between Albert and ABC Corporation. Applicable Laws; Capacity to Buy Land (1995) 3. What law governs the capacity of the Filipino to buy the land? Explain your answer and give its legal basis. SUGGESTED ANSWER:

Philippine law governs the capacity of the Filipino to buy the land. In addition to the principle of lex rei sitae given above. Article 15 of the NCC specifically provides that Philippine laws relating to legal capacity of persons are binding upon citizens of the Philippines no matter where they are. Applicable Laws; Capacity to Contract (1995) 2. What law governs the capacity of the Japanese to sell the land? Explain your answer and give its legal basis. SUGGESTED ANSWER:

Japanese law governs the capacity of the Japanese to sell the land being his personal law on the basis of an interpretation of Art. 15, NCC. ALTERNATIVE ANSWERS;

a) Since capacity to contract is governed by the personal law of an individual, the Japanese seller's capacity should be governed either by his national law (Japanese law) or by the law of his domicile, depending upon whether Japan follows the nationality or domiciliary theory of personal law for its citizens. b) Philippine law governs the capacity of the Japanese owner in selling the land. While as a general rule capacity of

persons is governed by the law of his nationality, capacity concerning transactions involving property is an exception. Under Article 16 of the NCC the capacity of persons in transactions involving title to property is governed by the law of the country where the property is situated. Since the property is in the Philippines, Philippine law governs the capacity of the seller. Applicable Laws; capacity to succeed (1991) Jacob, a Swiss national, married Lourdes, a Filipina, in Berne, Switzerland. Three years later, the couple decided to reside in the Philippines. Jacob subsequently acquired several properties in the Philippines with the money he inherited from his parents. Forty years later. Jacob died intestate, and is survived by several legitimate children and duly recognized illegitimate daughter Jane, all residing in the Philippines. (a) Suppose that Swiss law does not allow illegitimate children to inherit, can Jane, who is a recognized illegitimate child, inherit part of the properties of Jacob under Philippine law? (b) Assuming that Jacob executed a will leaving certain properties to Jane as her legitime in accordance with the law of succession in the Philippines, will such testamentary disposition be valid?

SUGGESTED ANSWER:

A. Yes. As stated in the problem. Swiss law does not allow illegitimate children to inherit Hence, Jane cannot inherit the property of Jacob under Philippine law. SUGGESTED ANSWER:

B. The testamentary disposition will not be valid if it would contravene Swill law; otherwise, the disposition would be valid. Unless the Swiss law is proved, it would be presumed to be the same as that of Philippine law under the Doctrine of Processual Presumption. Applicable Laws; contracts contrary to public policy (1996) Alma was hired as a domestic helper in Hongkong by the Dragon Services, Ltd., through its local agent. She executed a standard employment contract designed by the Philippine Overseas Workers Administration (POEA) for overseas Filipino workers. It provided for her employment for one year at a salary of US$1,000.00 a month. It was submitted to and approved by the POEA. However, when she arrived in Hongkong, she was asked to sign another contract by Dragon Services, Ltd. which reduced her salary to only US$600.00 a month. Having no other choice, Alma signed the contract but when she returned to the Philippines, she demanded payment of the salary differential of US$400.00 a month. Both Dragon Services, Ltd. and its local agent claimed that the second contract is valid under the laws of Hongkong, and therefore binding on Alma. Is their claim correct? Explain. SUGGESTED ANSWER:

Their claim is not correct. A contract is the law between the parties but the law can disregard the contract if it is contrary to public policy. The provisions of the 1987 Constitution on the protection of labor and on social justice (Sec. 10. Art II) embody a public policy of the Philippines. Since the application of Hongkong law in this case is in violation of

CIVIL LAW Answers to the BAR as Arranged by Topics (Year 1990-2006) that public policy, the application shall be disregarded by Court of Appeals (G.R No. 104235, Nov. 10, 1993) the

our Courts. (Cadalin v. POEA. 238 SCRA 762) ALTERNATIVE ANSWERS;

a) Their claim is not correct. Assuming that the second contract is binding under Hongkong law, such second contract is invalid under Philippine law which recognizes as valid only the first contract. Since the case is being litigated in the Philippines, the Philippine Court as the forum will not enforce any foreign claim obnoxious to the forum's public policy. There is a strong public policy enshrined in our Constitution on the protection of labor. Therefore, the second contract shall be disregarded and the first contract will be enforced. (Cadalin v. POEA, 238 SCRA 762). b) No, their claim is not correct. The second contract executed in Hongkong, partakes of the nature of a waiver that is contrary to Philippine law and the public policy governing Filipino overseas workers. Art. 17, provides that our prohibitive laws concerning persons, their acts, or their property or which have for their object public order, public policy and good customs shall not be rendered ineffective by laws or conventions agreed upon in a foreign country. Besides, Alma's consent to the second contract was vitiated by undue influence, being virtually helpless and under financial distress in a foreign country, as indicated by the given fact that she signed because she had no choice. Therefore, the defendants claim that the contract is valid under Hongkong law should be rejected since under the DOCTRINE OF PROCESSUAL PRESUMPTION a foreign law is deemed similar or identical to Philippine law in the absence of proof to the contrary, and such is not mentioned in the problem as having been adduced. Applicable Laws; Contracts of Carriage (1995) On 8 December 1991 Vanessa purchased from the Manila office of Euro-Aire an airline ticket for its Flight No. 710 from Dallas to Chicago on 16 January 1992. Her flight reservation was confirmed. On her scheduled departure Vanessa checked in on time at the Dallas airport. However, at the check-in counter she discovered that she was waitlisted with some other passengers because of intentional overbooking, a Euro-Aire policy and practice. Euro-Alre admitted that Vanessa was not advised of such policy when she purchased her plane ticket. Vanessa was only able to fly two days later by taking another airline. Vanessa sued Euro-Aire in Manila for breach of contract and damages. Euro-Aire claimed that it cannot be held liable for damages because its practice of overbooking passengers was allowed by the U.S. Code of Federal Regulations. Vanessa on the other hand contended that assuming that the U.S. Code of Federal Regulations allowed Intentional overbooking, the airline company cannot invoke the U.S. Code on the ground that the ticket was purchased in Manila, hence, Philippine law should apply, under which Vanessa can recover damages for breach of contract of carriage. Decide. Discuss fully. SUGGESTED ANSWER:

Vanessa can recover damages under Philippine law for breach of contract of carriage, Philippine law should govern as the law of the place where the plane tickets were bought and the contract of carriage was executed. In Zalamea v.

Supreme Court applied Philippine law in recovery of damages for breach of contract of carriage for the reason that it is the law of the place where the contract was executed. ALTERNATIVE ANSWER:

If the violation of the contract was attended with bad faith, there is a ground to recover moral damages. But since there was a federal regulation which was the basis of the act complained of, the airline cannot be in bad faith. Hence, only actual damages can be recovered. The same is true with regards to exemplary damages. Applicable Laws; Labor Contracts (1991) A. The Japan Air Lines (JAL), a foreigner corporation licensed to do business in the Philippines, executed in Manila a contract of employment with Maritess Guapa under which the latter was hired as a stewardess on the aircraft flying the Manila-Japan-Manila route. The contrast specifically provides that (1) the duration of the contract shall be two (2) years, (2) notwithstanding the above duration, JAL may terminate the agreement at any time by giving her notice in writing ten (10) days in advance, and (3) the contract shall be construed as governed under and by the laws of Japan and only the court in Tokyo, Japan shall have the jurisdiction to consider any matter arising from or relating to the contract. JAL dismissed Maritess on the fourth month of her employment without giving her due notice. Maritess then filed a complaint with the Labor Arbiter for reinstatement, backwages and damages. The lawyer of JAL contends that neither the Labor Arbiter nor any other agency or court in the Philippines has jurisdiction over the case in view of the above provision (3) of the contract which Maritess voluntarily signed. The contract is the law between her and JAL. Decide the issue. B. Where under a State's own conflicts rule that domestic law of another State should apply, may the courts of the former nevertheless refuse to apply the latter? If so, under what circumstance? SUGGESTED ANSWER:

A, Labor Legislations are generally intended as expressions of public policy on employer-employee relations. The contract therefore, between Japan Air Lines (JAL) and Maritess may apply only to the extent that its provisions are not inconsistent with Philippine labor laws intended particularly to protect employees. Under the circumstances, the dismissal of Maritess without complying with Philippine Labor law would be invalid and any stipulation in the contract to the contrary is considered void. Since the law of the forum in this case is the Philippine law the issues should-be resolved in accordance with Philippine law. B. The third paragraph of Art. 17 of the Civil Code provides that:

CIVIL LAW Answers to the BAR as Arranged by Topics (Year 1990-2006)

"Prohibitive laws concerning persons, their acts or property, and those which have for their object public order, public policy and good customs shall not be rendered ineffective by laws or judgments promulgated, or by determinations or conventions agreed upon in a foreign country."

Maris then returned to the Philippines and in a civil ceremony celebrated in Cebu City according to the formalities of Philippine law, she married her former classmate Vincent likewise a Filipino citizen. a) Was the marriage of Maris and Johnson valid when celebrated? Is their marriage still validly existing now? Reasons.

Accordingly, a state's own conflict of laws rule may, SUGGESTED ANSWER: exceptionally be inapplicable, given public policy (a) The marriage of Mans and Johnson was valid when celebrated because all marriages solemnized outside the considerations by the law of the forum. Philippines (Tokyo) in accordance with the laws in force in the country where they are solemnized (Japan), and valid Going into the specific provisions of the contract in question, there as such, are also valid in the Philippines. I would rule as follows: 1 The duration of the contract is not opposed to Philippine Their marriage no longer validly subsists, because it has been law and it can therefore be valid as stipulated; 2 The second provision to the effect that notwithstanding dissolved by the absolute divorce validly obtained by Johnson duration, Japan Air Lines (JAL) may terminate her employment is which capacitated Maris to remarry (Art. 26. Family Code). invalid, being inconsistent with our Labor laws; 3 That the contract shall be construed as governed under and by the laws of Japan and only the courts of Tokyo, Japan shall haveApplicable Laws; laws governing marriages (2003) jurisdiction, is invalid as clearly opposed to the aforecited third Gene and Jane, Filipino, met and got married in England paragraph of Arts. 17 and 1700 of the Civil Code, which provides:while both were taking up post-graduate courses there. A few years after their graduation, they decided to annul their marriage. Jane filed an action to annul her marriage to Gene in England on the ground of latter’s sterility, a ground for annulment of marriage in England. The English court "Art. 1700. The relations between capital and labor decreed the marriage annulled. Returning to the Philippines, are not merely contractual. They are so impressed Gene asked you whether or not he would be free to marry his with public interest that labor contracts must yield former girlfriend. What would your legal advice be? 5% to the common good. Therefore, such contracts are subject to the special laws on labor unions, SUGGESTED ANSWER: collective bargaining, strikes and lockouts, closed No, Gene is not free to marry his former girlfriend. His shop, wages, working conditions, hours of labor marriage to Jane is valid according to the forms and and similar subjects." solemnities of British law, is valid here (Article 17, 1st par., NCC). However, since Gene and Jane are still Filipinos ALTERNATIVE ANSWER; A. When a contract has a foreign element such as in the although living in England, the dissolution of their marriage is factual setting stated in the problem where one of the parties still governed by Philippine law (Article 15, NCC). Since, is a foreign corporation, the contract can be sustained as valid sterility is not one of the grounds for the annulment of a particularly the stipulation expressing that the contract is marriage under Article 45 of the Family Code, the annulment governed by the laws of the foreign country. Given this of Gene’s marriage to Jane on that ground is not valid in the generally accepted principle of international law, the contract Philippines (Article 17, NCC) between Maritess and JAL is valid and it should therefore be ALTERNATIVE ANSWER: Yes, Gene is free to marry his girlfriend because his marriage enforced. was validly annulled in England. The issue of whether or not a marriage is voidable, including the grounds therefore, is Applicable Laws; laws governing marriages (1992) governed by the law of the place where the marriage was In 1989, Maris, a Filipino citizen, married her boss Johnson, solemnized (lex loci celebrationis). Hence, even if sterility is an American citizen, in Tokyo in a wedding ceremony not a ground to annul the marriage under the Philippine law, celebrated according to Japanese laws. One year later, the marriage is nevertheless voidable because sterility makes Johnson returned to his native Nevada, and he validly the marriage voidable under English law. Therefore, obtained in that state an absolute divorce from his wife Maris. annulment of the marriage in England is valid in the Philippines. After Maris received the final judgment of divorce, she Applicable Laws; Sale of Real Property (1995) married her childhood sweetheart Pedro, also a Filipino While in Afghanistan, a Japanese by the name of Sato sold to citizen, in a religious ceremony in Cebu City, celebrated Ramoncito, a Filipino, a parcel of land situated in the according to the formalities of Philippine law. Pedro later left Philippines which Sato inherited from his Filipino mother. for the United States and became naturalized as an American 1. What law governs the formality in the execution of the citizen. Maris followed Pedro to the United States, and after a contract of sale? Explain your answer and give its legal basis. serious quarrel, Maris filed a suit and obtained a divorce decree issued by the court in the state of Maryland. SUGGESTED ANSWER:

CIVIL LAW Answers to the BAR as Arranged by Topics (Year 1990-2006)

Under Art. 16 par. 1, NCC, real property is subject to the law of the country where it is situated. Since the property is situated in the Philippines, Philippine law applies. The rule of lex rei sitae in Article 16 prevails over lex loci contractu in Article 17 of the NCC. ALTERNATIVE ANSWER:

Afghanistan law governs the formal requirements of the contract since the execution is in Afghanistan. Art. 17 of the Civil Code provides that the forms and solemnities of contracts, wills, and other public instruments shall be governed by the laws of the country in which they are executed. However, if the contract was executed before the diplomatic or consular officials of the Republic of the Philippines in Afghanistan, Philippine law shall apply. Applicable Laws; Succession; Intestate & Testamentary (2001) Alex was born a Filipino but was a naturalized Canadian citizen at the time of his death on December 25, 1998. He left behind a last will and testament in which he bequeathed all his properties, real and personal, in the Philippines to his acknowledged illegitimate Fillpina daughter and nothing to his two legitimate Filipino sons. The sons sought the annulment of the last will and testament on the ground that it deprived them of their legitimes but the daughter was able to prove that there were no compulsory heirs or legitimes under Canadian law. Who should prevail? Why? (5%) SUGGESTED ANSWER:

The daughter should prevail because Article 16 of the New Civil Code provides that intestate and testamentary succession shall be governed by the national law of the person whose succession is under consideration. Applicable Laws; Sucession of Aliens (1995) Michelle, the French daughter of Penreich, a German national, died in Spain leaving real properties in the Philippines as well as valuable personal properties in Germany. 1. What law determines who shall succeed the deceased? Explain your answer and give its legal basis. 2. What law regulates the distribution of the real properties in the Philippines? Explain your answer and give its legal basis. 3. What law governs the distribution of the personal properties in Germany? Explain your answer and give its legal basis. SUGGESTED ANSWER:

Assuming that the estate of the decedent is being settled in the Philippines) 1. The national law of the decedent (French law) shall govern in determining who will succeed to his estate. The legal basis is Art. 16 par. 2, NCC. ALTERNATIVE ANSWER:

French law shall govern the distribution of his real properties in the Philippines except when the real property is land which may be transmitted to a foreigner only by hereditary succession. SUGGESTED ANSWER:

2. The distribution of the real properties in the Philippines shall be governed by French law. The legal basis is Art. 16, NCC). SUGGESTED ANSWER:

3. The distribution of the personal properties in Germany shall be governed by French law. The legal basis is Art. 16, NCC). Applicable Laws; Wills executed abroad (1993) A, a Filipino, executed a will in Kuwait while there as a contract worker. Assume that under the laws of Kuwait, it is enough that the testator affix his signature to the presence of two witnesses and that the will need not be acknowledged before a notary public. May the will be probated in the Philippines? SUGGESTED ANSWER:

Yes. Under Articles 815 and 17 of the Civil Code, the formality of the execution of a will is governed by the law of the place of execution. If the will was executed with the formalities prescribed by the laws of Kuwait and valid there as such, the will is valid and may be probated in the Philippines. Definition; Cognovit; Borrowing Statute; Characterization(1994) In Private International Law (Conflict of Laws) what is: 1} Cognovit? 2) A borrowing statute? 3) Characterization? SUGGESTED ANSWER:

1) a) COGNOVIT is a confession of judgment whereby a portion of the complaint is confessed by the defendant who denies the rest thereof (Philippine law Dictionary, 3rd Ed.) (Ocampo v. Florenciano, L-M 13553, 2/23/50). b) COGNOVIT is a "statement of confession" Oftentimes, it is referred to as a "power of attorney" or simply as a "power", it is the written authority of the debtor and his direction to the clerk of the district court, or justice of the peace to enter judgment against the debtor as stated therein. (Words and Phrases, vol. 7, pp. 115-166). c) COGNOVIT is a plea in an action which acknowledges that the defendant did undertake and promise as the plaintiff in its declaration has alleged, and that it cannot deny that it owes and unjustly detains from the plaintiff the sum claimed by him in his declaration, and consents that judgment be entered against the defendant for a certain sum. [Words and Phrases, vol. 7, pp. 115-166). d) COGNOVIT is a note authorizing a lawyer for confession of judgment by defendant. 2) "BORROWING STATUTE" -Laws of the state or jurisdiction used by another state in deciding conflicts questioned involved in the choice of law (Black's Law Dictionary, 5th ed. 1979). 3) a) "CHARACTERIZATION" is otherwise called "classification" or "qualification." It is the process of assigning a disputed question to its correct legal category (Private International Law, Salonga). b) "CHARACTERIZATION" is a process in determining under what category a certain set of facts or rules fall. (Paras, Conflict of Laws, p. 94. 1984 ed.)

CIVIL LAW Answers to the BAR as Arranged by Topics (Year 1990-2006)

Definition; forum non-conveniens; long-arm statute (1994) 1) What is the doctrine of Forum non conveniens? 2) What is a "long arm statute"? SUGGESTED ANSWER:

1) a) FORUM NON CONVENIENS is a principle in Private International Law that where the ends of justice strongly indicate that the controversy may be more suitably tried elsewhere, then jurisdiction should be declined and the parties relegated to relief to be sought in another forum. (Moreno. Philippine Law Dictionary, p. 254, 1982 ed.). b) Where in a broad sense the ends of justice strongly indicate that the controversy may be more suitably tried elsewhere, then jurisdiction should be declined and the parties relegated to relief to be sought in another forum. (Handbook on Private International Law, Aruego). c) FORUM NON CONVENIENS means simply that a court may resist imposition upon its jurisdiction even when jurisdiction is authorized by the letter of a general venue statute. (Salonga. Private International Law. p, 51. 1967 ed.) d) Forum non conveniens is a doctrine whereby a court of law having full Jurisdiction over a case brought in a proper venue or district declines to determine the case on its merits because Justice would be better served by the trial over the case in another jurisdiction. (Webster's Dictionary) SUGGESTED ANSWER:

(2} a) LONG ARM STATUTE is a legislative act which provides for personal jurisdiction, via substituted service or process, over persons or corporations which are nonresidents of the state and which voluntarily go into the state, directly or by agent or communicate with persons in the state for limited purposes, inactions which concern claims relating to performance or execution of those purposes (Black's Law Dictionary, 5th Ed. 1979). b) Long arm statute refers simply to authorized substituted service. Divorce; effect of divorce granted to former Filipinos; Renvoi Doctrine (1997) In 1977, Mario and Clara, both Filipino citizens, were married in the Philippines. Three years later, they went to the United States of America and established their residence in San Francisco, California. In 1987, the couple applied for, and were granted, U.S. citizenship. In 1989, Mario, claiming to have been abandoned by Clara, was able to secure a decree of divorce in Reno, Nevada, U.S.A. In 1990, Mario returned to the Philippines and married Juana who knew well Mario's past life. (a) Is the marriage between Mario and Juana (b) Would the renvoi doctrine have any relevance to the case? valid? SUGGESTED ANSWER:

(a) Yes, because Phil law recognizes the divorce between Mario and Clara as valid. SUGGESTED ANSWER: (b) No, The renvoi doctrine is relevant in cases where one country applies the domiciliary theory and the other the

nationality theory, and the issue involved is which of the laws of the two countries should apply to determine the order of succession, the amount of successional rights, or, the intrinsic validity of testamentary provisions. Such issue is not involved in this case.

ALTERNATIVE ANSWER:

Yes. "Renvoi" - which means "referring back" is relevant because here, we are applying U.S. law to Mario, being already its citizen, although the formalities of the second marriage will be governed by Philippine law under the principle of lex loci celebrationis. Domiciliary theory vs. Nationality Theory (2004) Distinguish briefly but clearly between: Domiciliary theory and nationality theory of personal law. (5%) SUGGESTED ANSWER:

DOMICILIARY THEORY posits that the personal status and rights of a person are governed by the law of his domicile or the place of his habitual residence. The NATIONALITY THEORY, on the other hand, postulates that it is the law of the person's nationality that governs such status and rights Forum Non Conveniens & Lex Loci Contractus (2002) Felipe is a Filipino citizen. When he went to Sydney for vacation, he met a former business associate, who proposed to him a transaction which took him to Moscow. Felipe brokered a contract between Sydney Coals Corp. (Coals), an Australian firm, and Moscow Energy Corp. (Energy), a Russian firm, for Coals to supply coal to Energy on a monthly basis for three years. Both these firms were not doing, and still do not do, business in the Philippines. Felipe shuttled between Sydney and Moscow to close the contract. He also executed in Sydney a commission contract with Coals and in Moscow with Energy, under which contracts he was guaranteed commissions by both firms based on a percentage of deliveries for the three-year period, payable in Sydney and in Moscow, respectively, through deposits in accounts that he opened in the two cities. Both firms paid Felipe his commission for four months, after which they stopped paying him. Felipe learned from his contacts, who are residents of Sydney and Moscow, that the two firms talked to each other and decided to cut him off. He now files suit in Manila against both Coals and Energy for specific performance. A. Define or explain the principle of ―lex loci contractus‖. (2%) B. Define or explain the rule of ―forum non conveniens‖ (3%) C. Should the Philippine court assume jurisdiction over the case? Explain. (5%) SUGGESTED ANSWER:

A. LEX LOCI CONTRACTUS may be understood in two senses, as follows: (1) It is the law of the place where contracts, wills, and other public instruments are executed and governs their ―forms and solemnities‖, pursuant to the first paragraph, Article (2) It is 17 theofproper the New lawCivil of theCode; contract; or e.i., the system of law intended to govern the entire contract, including its essential requisites, indicating the law of the place with which the contract has its closest connection or

CIVIL LAW Answers to the BAR as Arranged by Topics (Year 1990-2006)

where the main elements of the contract converge. As illustrated by Zalamea v. Court of Appeals (228 SCRA 23 [1993]), it is the law of the place where the airline ticket was issued, where the passengers are nationals and residents of, and where the defendant airline company maintained its office.

ALTERNATIVE ANSWER:

A. Under the doctrine of lex loci contractus, as a general rule, the law of the place where a contract is made or entered into governs with respect to its nature and validity, obligation and interpretation. This has been said to be the rule even though the place where the contract was made is different from the place where it is to be performed, and particularly so, if the place of the making and the place of performance are the same (United Airline v. CA, G.R. No. 124110, April 20, 2001). SUGGESTED ANSWER:

B. FORUM NON CONVENIENS means that a court has discretionary authority to decline jurisdiction over a cause of action when it is of the view that the action may be justly and effectively adjudicated elsewhere. SUGGESTED ANSWER:

C. No, the Philippine courts cannot acquire jurisdiction over the case of Felipe. Firstly, under the rule of forum non conveniens, the Philippine court is not a convenient forum as all the incidents of the case occurred outside the Philippines. Neither are both Coals and Energy doing business inside the Philippines. Secondly, the contracts were not perfected in the Philippines. Under the principle of lex loci contractus, the law of the place where the contract is made shall apply. Lastly, the Philippine court has no power to determine the facts surrounding the execution of said contracts. And even if a proper decision could be reached, such would have no biding effect on Coals and Energy as the court was not able to acquire jurisdiction over the said corporations. (Manila Hotel Corp. v. NLRC. 343 SCRA 1, 1314[2000])

Nationality Theory (2004) PH and LV are HK Chinese. Their parents are now Filipino citizens who live in Manila. While still students in MNS State, they got married although they are first cousins. It appears that both in HK and in MNS State first cousins could marry legally. They plan to reside and set up business in the Philippines. But they have been informed, however, that the marriage of first cousins here is considered void from the beginning by reason of public policy. They are in a dilemma. They don’t want to break Philippine law, much less their marriage vow. They seek your advice on whether their civil status will be adversely affected by Philippine domestic law? What is your advice? (5%) SUGGESTED ANSWER: My advise is as follows: The civil status of' PH and LV will not be adversely affected by Philippine law because they are nationals of Hong Kong and not Filipino citizens.Being foreigners, their status, conditions and legal capacity in the Philippines are governed by the law of Hong Kong, the

country of which they are citizens. Since their marriage is valid under Hong Kong law, it shall be valid and respected in the Philippines.

Naturalization (2003) Miss Universe, from Finland, came to the Philippines on a tourist visa. While in this country, she fell in love with and married a Filipino doctor. Her tourist visa having been expired and after the maximum extension allowed therefore, the Bureau of Immigration and Deportation (BID) is presently demanding that she immediately leave the country but she refuses to do so, claiming that she is already a Filipino Citizen by her marriage to a Filipino citizen. Can the BID still order the deportation of Miss Universe? Explain. 5% SUGGESTED ANSWER:

Yes, the BID can order the deportation of Miss Universe. The marriage of an alien woman to a Filipino does not automatically make her a Filipino Citizen. She must first prove in an appropriate proceeding that she does not have any disqualification for Philippine citizenship. (Yung Uan Chu v. Republic of the Philippines, 158 SCRA 593 [1988]). Since Miss Universe is still a foreigner, despite her marriage to a Filipino doctor, she can be deported upon expiry of her allowable stay in the Philippines. ANOTHER SUGGESTED ANSWER:

No, the Bureau of Immigration cannot order her deportation. An alien woman marrying a Filipino, native-born or naturalized, becomes ipso facto a Filipino if she is not disqualified to be a citizen of the Philippines (Mo Ya Lim v Commission of Immigration, 41 SCRA 292 [1971]), (Sec 4, Naturalization Law). All that she has to do is prove in the

deportation proceeding the fact of her marriage and that she is not disqualified to become a Filipino Citizen. ANOTHER SUGGESTED ANSWER:

It depends. If she is disqualified to be a Filipino citizen, she may be deported. If she is not disqualified to be a Filipino citizen, she may not be deported. An alien woman who marries a Filipino citizen becomes one. The marriage of Miss Universe to the Filipino doctor did not automatically make her a Filipino citizen. She still has to prove that she is not disqualified to become a citizen. Theory; significant relationships theory (1994) Able, a corporation domiciled in State A, but, doing business in the Philippines, hired Eric, a Filipino engineer, for its project in State B. In the contract of employment executed by the parties in State B, it was stipulated that the contract could be terminated at the company's will, which stipulation is allowed in State B. When Eric was summarily dismissed by Able, he sued Able for damages in the Philippines. Will the Philippine court apply the contractual stipulation? SUGGESTED ANSWER:

a) Using the "SIGNIFICANT RELATIONSHIPS THEORY", there are contacts significant to the Philippines. Among these are that the place of business is the Philippines, the employee concerned is a Filipino and the suit was filed in the Philippines, thereby justifying the application of Philippine law. In the American Airlines case the Court held that when what is involved is PARAMOUNT STATE INTEREST such as the

CIVIL LAW Answers to the BAR as Arranged by Topics (Year 1990-2006)

protection of the rights of Filipino laborers, the court can disregard choice of forum and choice of law. Therefore the Philippine Court should not apply the stipulation in question. ALTERNATIVE ANSWER:

b) No, lex fori should be applied because the suit is filed in Philippine courts and Eric was hired in the Philippines. The Philippine Constitution affords full protection to labor and the stipulation as to summary dismissal runs counter to our fundamental and statutory laws. Torts; Prescriptive Period (2004) In a class suit for damages, plaintiffs claimed they suffered injuries from torture during martial law. The suit was filed upon President EM’s arrival on exile in HI, a U.S. state. The court in HI awarded plaintiffs the equivalent of P100 billion under the U.S. law on alien tort claims. On appeal, EM’s Estate raised the issue of prescription. It argued that since said U.S. law is silent on the matter, the court should apply: (1) HI’s law setting a two-year limitation on tort claims; or (2) the Philippine law which appears to require that claims for personal injury arising from martial law be brought within one year.

natural mother as her middle name. The Court has ruled that there is no law prohibiting an illegitimate child adopted by her natural father to use, as middle name, her mother's surname. What is not prohibited is allowed. After all, the use of the maternal name as the middle name is in accord with Filipino culture and customs and adoption is intended for the benefit of the adopted [In re: Adoption of Stephanie Nathy Astorga Garcia, G.R. No. 148311, March 31, 2005; Rabuya, The Law on Persons and Family Relations, p. 613].

Inter-Country Adoption; Formalities (2005) Hans Berber, a German national, and his Filipino wife, Rhoda, are permanent residents of Canada. They desire so much to adopt Magno, an 8-year old orphaned boy and a baptismal godson of Rhoda. Since the accidental death of Magno's parents in 2004, he has been staying with his aunt who, however, could hardly afford to feed her own family. Unfortunately, Hans and Rhoda cannot come to the Philippines to adopt Magno although they possess all the qualifications as adoptive parents. Is there a possibility for them to adopt Magno? How should they go about it? (5%) SUGGESTED ANSWER:

Plaintiffs countered that provisions of the most analogous federal statute, the Torture Victims Protection Act, should be applied. It sets ten years as the period for prescription. Moreover, they argued that equity could toll the statute of limitations. For it appeared that EM had procured Constitutional amendments granting himself and those acting under his direction immunity from suit during his tenure. In this case, has prescription set in or not? Considering the differences in the cited laws, which prescriptive period should be applied: one year under Philippine law, two years under HI’s law, ten years under U.S. federal law, or none of the above? Explain. (5%) SUGGESTED ANSWER:

The US Court will apply US law, the law of the Jorum, in determining the applicable prescriptive period. While US law is silent on this matter, the US Court will not apply Philippine law in determining the prescriptive period. It is generally affirmed as a principle in private international law that procedural law is one of the exceptions to the application of foreign law by the forum. Since prescription is a matter of procedural law even in Philippine jurisprudence, (Codaltn v. POEA/ JVLRC/Broum and Root International, 238 SCRA 721 [1994]), the US Court will apply either HI or Federal law in determining the applicable prescriptive period and not Philippine law. The Restatement of American law affirms this principle.

ADOPTION

Adoption; Use of Surname of her Natural Mother (2006) May an illegitimate child, upon adoption by her natural father, use the surname of her natural mother as the middle name? (2.5%) SUGGESTED ANSWER: Yes,

an illegitimate child, upon adoption by her natural father, can use the surname of her

Yes, it is possible for Hans and Rhoda to adopt Magno. Republic Act No. 8043 or the Inter-Country Adoption Act, allows aliens or Filipinos permanently residing abroad to apply for inter-country adoption of a Filipino child. The law however requires that only legally free child, or one who has been voluntarily or involuntarily committed to the DSWD or any of its accredited agencies, may be subject of intercountry adoption. The law further requires that aside from possessing all the qualifications, the adoptive parents must come from a country where the Philippines has diplomatic relations and that the government maintains a similarly accredited agency and that adoption is allowed under the national law of the alien. Moreover, it must be further shown that all possibilities for a domestic adoption have been exhausted and the inter-country adoption is best for the interest of the child. Hans and Rhoda have to file an application to adopt Magno, either with the Regional Trial Court having jurisdiction over Magno or with the Inter-Country Adoption Board in Canada. Hans and Rhoda will then undergo a trial custody for six (6) months from the time of placement. It is only after the lapse of the trial custody that the decree of adoption can be issued. Parental Authority; Rescission of Adoption (1994) In 1975, Carol begot a daughter Bing, out of wedlock. When Bing was ten years old, Carol gave her consent for Bing's legal adoption by Norma and Manuel, which was granted by the court in 1990. In 1991, Carol learned that Norma and Manuel were engaged in a call-girl-ring that catered to tourists. Some of the girls lived with Norma and Manuel. Carol got Bing back, who in the first place wanted to return to her natural mother. 1) Who has a better right to the custody of Bing, Carol or Norma? 2) Aside from taking physical custody of Bing, what legal actions can Carol take to protect Bing?

CIVIL LAW Answers to the BAR as Arranged by Topics (Year 1990-2006) SUGGESTED ANSWER:

1) a) It depends on whether or not Bing was at least 18 years old at the time Carol asserts the prerogative to take custody of Bing. If she was at least 18 years old, then she is no longer under parental authority and neither Carol nor Norma can assert the prerogative to take custody. However, if she was less than 18 years old, then Norma has a better right since the adoption by Norma of Bing terminates the parental authority of Carol over Bing. b) The natural mother, Carol, should have the better right in light of the principle that the child's welfare is the paramount consideration in custody rights. Obviously, Bing's continued stay in her adopting parents' house, where interaction with the call girls is inevitable, would be detrimental to her moral and spiritual development. This could be the reason for Bing's expressed desire to return to her natural mother. It should be noted, however, that Bing is no longer a minor, being 19 years of age now. It is doubtfu1 that a court can still resolve the question of custody over one who is sui juris and not otherwise incapacitated. SUGGESTED ANSWER:

2) a) On the assumption that Bing is still a minor or otherwise incapacitated, Carol may petition the proper court for resolution or rescission of the decree of adoption on the ground that the adopting parents have exposed, or are exposing, the child to corrupt influence, tantamount to giving her corrupting orders or examples. She can also ask for the revesting in her of parental authority over Bing. If However, Bing is already 19 years of age and therefore no longer a minor, it is not Carol but Bing herself who can petition the court for judicial rescission of the adoption, provided she can show a ground for disinheritance of an ascendant. b) Carol may file an action to deprive Norma of parental authority under Article 231 of the Family Code or file an action for the rescission of the adoption under Article 191 in relation to Article 231 (2) of the Family Code.

sister. Thus, under the above-cited provision, Eva is qualified to adopt Vicky.

b) Would your answer be the same if they sought to adopt Eva's illegitimate daughter? Explain. (2%) SUGGESTED ANSWER:

My answer will still be the same. Paragraph 3(a) of Article 184 of the Family Code does not make any distinction. The provision states that an alien who is a former Filipino citizen is qualified to adopt a relative by consanguinity. c) Supposing that they filed the petition to adopt Vicky in the year 2000, will your answer be the same? Explain. (2%) SUGGESTED ANSWER:

Yes, my answer will still be the same. Under Sec. 7(b), Art. III of the New Domestic Adoption Act, an alien who possesses all the qualifications of a Filipino national who is qualified to adopt may already adopt provided that his country has diplomatic relations with the Philippines, that he has been living in the Philippines for at least three (3) continuous years prior to the filing of the application for adoption and maintains such residence until the adoption decree is entered, that he has been certified by his diplomatic or consular office or any appropriate government agency that he has the legal capacity to adopt in his country, and that his government allows the adoptee to enter his country as his adopted child. Qualification of Adopter; Applicable Law (2001) A German couple filed a petition for adoption of a minor Filipino child with the Regional Trial Court of Makati under the provisions of the Child and Youth Welfare Code which allowed aliens to adopt. Before the petition could be heard, the Family Code, which repealed the Child and Youth Welfare Code, came into effect. Consequently, the Solicitor General filed a motion to dismiss the petition, on the ground that the Family Code prohibits aliens from adopting. If you were the judge, how will you rule on the motion? (5%) SUGGESTED ANSWER:

Qualification of Adopter (2005) In 1984, Eva, a Filipina, went to work as a nurse in the USA. There, she met and fell in love with Paul, an American citizen, and they got married in 1985. Eva acquired American citizenship in 1987. During their sojourn in the Philippines in 1990, they filed a joint petition for the adoption of Vicky, a 7-year old daughter of Eva's sister. The government, through the Office of the Solicitor General, opposed the petition on the ground that the petitioners, being both foreigners, are disqualified to adopt Vicky. a) Is the government's opposition tenable? Explain. (2%) SUGGESTED ANSWER:

The government's position is untenable. Under paragraph 3, Article 184 of the Family Code, an alien, as a general rule cannot adopt. However, an alien who is a former Filipino citizen and who seeks to adopt a relative by consanguinity is qualified to adopt, (par. 3[a], Art. 184, Family Code) In the given problem, Eva, a naturalized American citizen would like to adopt Vicky, a 7-year old daughter of her

The motion to dismiss the petition for adoption should be denied. The law that should govern the action is the law in force at the time of filing of the petition. At that time, it was the Child and Youth Welfare Code that was in effect, not the Family Code. Petitioners have already acquired a vested right on their qualification to adopt which cannot be taken away by the Family Code. (Republic v. Miller G.R. No. 125932, April 21, 1999, citing Republic v. Court of Appeals, 205 SCRA 356) ALTERNATIVE ANSWER:

The motion has to be granted. The new law shall govern their qualification to adopt and under the new law, the German couple is disqualified from adopting. They cannot claim that they have already acquired a vested right because adoption is not a right but a mere privilege. No one acquires a vested right on a privilege. [Note: If the examinee based his answer on the current law, RA 8552, his answer should be considered correct. This question is based on the repealed provision of the Family Code on Adoption.]

Qualifications of Adopter (2000)

CIVIL LAW Answers to the BAR as Arranged by Topics (Year 1990-2006)

Sometime in 1990, Sarah, born a Filipino but by then a naturalized American citizen, and her American husband Tom, filed a petition in the Regional Trial Court of Makati, for the adoption of the minor child of her sister, a Filipina. Can the petition be granted? (5%) SUGGESTED ANSWER:

(per dondee) It depends. Rules on Adoption effective August 22, 2002 provides the following; SEC. 4. Who may adopt. –

under Sec. 7(b) of RA8552. The Supreme Court has held in several cases that when husband and wife are required to adopt jointly, each one of them must be qualified to adopt in his or her own right (Republic v. Toledano, 233 SCRA 9 (1994). However, the American husband must comply with the requirements of the law including the residency requirement of three (3) years. Otherwise, the adoption will not be allowed.

The following may adopt: Any Filipino Citizen

Successional Rights of Adopted Child (2004) A Filipino couple, Mr. and Mrs. BM, Jr., decided to adopt YV, an orphan from St. Claire’s orphanage in New York City. They loved and treated her like a legitimate child for they have none of their very own. However, BM, Jr., died in an turpitude; who is emotionally and psychologically capable of caring accident at sea, followed to the grave a year later by his sick for children, father, BM, Sr. Each left a sizable estate consisting of bank at least sixteen (16) years older than the adoptee, deposits, lands and buildings in Manila. May the adopted and who is in a position to support and care for his children child, YV, inherit from BM, Jr.? May she also inherit from in keeping with the means of the family. BM, Sr.? Is there a difference? Why? Explain. (5%) of legal age, in possession of full civil capacity and legal rights, of good moral character, has not been convicted of any crime involving moral

• The requirement of a 16-year difference between the age of the adopter and adoptee may be waived when the adopter is the biological parent of the adoptee or is the spouse of the adoptee’s parent; Any Alien possessing the same qualifications as above-stated for Filipino nationals: Provided, a) That his country has diplomatic relations with the Republic of the Philippines, b) that he has been living in the Philippines for at least three (3) continuous years prior to the filing of the petition for adoption and maintains such residence until the adoption decree is entered, c) that he has been certified by his diplomatic or consular office or any appropriate government agency to have the legal capacity to adopt in his country, d) and that his government allows the adoptee to enter his country as his adopted child.

Provided, further, That the requirements on residency and certification of the alien’s qualification to adopt in his country may be waived for the following: a) a former Filipino citizen who seeks to adopt a relative within the fourth (4th) degree of consanguinity or affinity; or b) one who seeks to adopt the legitimate child of his Filipino spouse; or

c)

one who is married to a Filipino citizen and seeks to adopt jointly with his spouse a relative within the fourth (4th) degree of consanguinity or affinity of the Filipino spouse.

Qualifications of Adopter (2003) Lina, a former Filipina who became an American citizen shortly after her marriage to an American husband, would like to adopt in the Philippines, jointly with her husband, one of her minor brothers. Assuming that all the required consents have been obtained, could the contemplated joint adoption in the Philippine prosper? Explain. SUGGESTED ANSWER: Yes, Lina and her American husband can jointly adopt a minor brother of Lina because she and her husband are both qualified to adopt. Lina, as a former Filipino citizen, can adopt her minor brother under Sec. 7(b)(i) of RA 8552 (Domestic Adoption Act of 1998), or under Art. 184 (3)(1) of the Family Code. The alien husband can now adopt

SUGGESTED ANSWER:

YV can inherit from BM, Jr. The succession to the estate of BM, Jr. is governed by Philippine law because he was a Filipino when he died (Article 16, Civil Code). Under Article 1039 of the Civil Code, the capacity of the heir to succeed is governed by the national law of the decedent and not by the national law of the heir. Hence, whether or not YV can inherit from BM, Jr. is determined by Philippine law. Under Philippine law, the adopted inherits from the adopter as a legitimate child of the adopter. YV, however, cannot inherit, in his own right, from the father of the adopter, BM, Sr., because he is not a legal heir of BM, Sr. The legal fiction of adoption exists only between the adopted and the adopter. (Teotico v. Del Val 13 SCRA 406 [1965]). Neither may he inherit from BM, Sr. by representing BM, Jr. because in representation, the representative must be a legal heir not only of the person he is representing but also of the decedent from whom the represented was supposed to inherit (Article 973, Civil Code).

FAMILY CODE Emancipation (1993) Julio and Lea, both 18 years old, were sweethearts. At a party at the house of a mutual friend. Lea met Jake, also 18 years old, who showed interest in her. Lea seemed to entertain Jake because she danced with him many times. In a fit of jealousy, Julio shot Jake with his father's 38 caliber revolver which, before going to the party he was able to get from the unlocked drawer inside his father's bedroom. Jake died as a result of the lone gunshot wound he sustained. His parents sued Julio's parents for damages arising from quasi-delict. At the time of the incident, Julio was 18 years old living with his parents. Julio's parents moved to dismiss the complaint against them claiming that since Julio was already of majority age, they were no longer liable for his acts. 1) Should the motion to dismiss be granted? Why? 2) What is the liability of Julio's parents to Jake's parents? Explain your answer.

Page 23 of 119

CIVIL LAW Answers to the BAR as Arranged by Topics (Year 1990-2006) SUGGESTED ANSWER:

1) No, the Motion to Dismiss should not be granted. Article 236 of the Family Code as amended by Republic Act 6809, provides in the third paragraph that "nothing in this Code shall be construed to derogate from the duty or responsibility of parents and guardians for children and wards below twenty-one years of age mentioned in the second and third paragraphs of Article 2180 of the Civil Code". 2) The liability of Julio's parents to Jake's parents arises from quasi-delict (Arts. 2176 and 2180 Civil Code) and shall cover specifically the following: a) P50,000.00 for the death of the son; b) such amount as would correspond to lost earning capacity; and c) moral damages. Family Code; Retroactive Application; Vested Rights (2000) On April 15, 1980, Rene and Angelina were married to each other without a marriage settlement. In 1985, they acquired a parcel of land in Quezon City. On June 1, 1990, when Angelina was away in Baguio, Rene sold the said lot to Marcelo. Is the sale void or voidable? (2%) SUGGESTED ANSWER:

The sale is void. Since the sale was executed in 1990, the Family Code is the law applicable. Under Article 124 of the FC, the sale of a conjugal property by a spouse without the consent of the other is void. ALTERNATIVE ANSWER:

The sale is voidable. The provisions of the Family Code may apply retroactively but only if such application will not impair vested rights. When Rene and Angelina got married in 1980, the law that governed their property relations was the New Civil Code. Under the NCC, as interpreted by the Supreme Court in Heirs of Felipe v. Aldon, 100 SCRA 628 and reiterated in Heirs of Ayuste v. Malabonga, G.R No, 118784, 2 September 1999, the sale executed by the husband without the consent of

the wife is voidable. The husband has already acquired a vested right on the voidable nature of dispositions made without the consent of the wife. Hence, Article 124 of the Family Code which makes the sale void does not apply. Family Home; Dwelling House (1994) In 1991, Victor established judicially out of conjugal property, a family home in Manila worth P200.000.00 and extrajudicially a second family home in Tagaytay worth P50.000.00. Victor leased the family home in Manila to a foreigner. Victor and his family transferred to another house of his in Pasig. Can the two family homes be the subject of execution on a judgment against Victor's wife for non-payment of the purchase in 1992 of household appliances? SUGGESTED ANSWER:

The two (2) so-called family homes can be the subject of execution. Neither of the abodes are considered family homes because for purposes of availing the benefits under the Family Code, there can only be one (1) family home which is defined as the "dwelling house" where the husband and the wife and their family actually "reside" and the land on which it is situated. (Arts. 152 and 161, Family Code)

Family; Constitutional Mandates; Divorce (1991) A. How does the 1987 Constitution strengthen the family as an Institution? B. Do the Constitutional policy on the family and the provision that marriage is the foundation of the family and shall be protected by the State bar Congress from enacting a law allowing divorce in the Philippines?

SUGGESTED ANSWER:

A. Sec, 2, Article II of the Constitution provides that: The State recognizes the sanctity of family life and shall protect and strengthen the family as a basic autonomous social institution. It shall equally protect the life of the mother and the life of the unborn from conception. The natural and primary right and duty of parents in the rearing of the youth for civic efficiency and the development of moral character shall receive the support of the Government. Section I, Article XV, further provides that: The State recognizes the Filipino family as the foundation of the nation. Accordingly, it shall strengthen its solidarity and actively promote its total development. (Note: The Committee recommends that a citation of either one of the provisions be credited as a complete answer).

SUGGESTED ANSWER:

B, No, the Constitutional policy, as well as the supporting provision, does not amount to a prohibition to Congress to enact a law on divorce. The Constitution only meant to help the marriage endure, to "strengthen its solidarity and actively promote its total development." ALTERNATIVE ANSWER:

B. Yes. Congress is barred from enacting a law allowing divorce, since Section 2 of Article XV provides: "Sec. 2. Marriage, as an inviolable social institution, is the foundation of the family and shall be protected by the State." Since marriage is "Inviolable", it cannot be dissolved by an absolute divorce. Marriage; Annulment; Effects; Requisites Before Remarriage (1990) The marriage of H and W was annulled by the competent court. Upon finality of the judgment of nullity. H began looking for his prospective second mate. He fell in love with a sexy woman S who wanted to be married as soon as possible, i.e., after a few months of courtship. As a young lawyer, you were consulted by H, (a) How soon can H be joined in lawful wedlock to his girlfriend S? Under existing laws, are there certain requisites that must be complied with before he can remarry? What advice would you give H? (b) Suppose that children were born from the union of H and W, what would be the status of said children? Explain your answer. (c) If the subsequent marriage of H to S was contracted before compliance with the statutory condition for its validity, what are the rights of the children of the first marriage (i.e., of H and W) and of the children of the subsequent marriage (of H and S)? SUGGESTED ANSWER:

(a) H, or either spouse for that matter, can marry again after complying with the provisions of Article 52 of the Family Code, namely, there must be a partition and distribution, of the properties of the spouses, and the delivery of the

CIVIL LAW Answers to the BAR as Arranged by Topics (Year 1990-2006)

children's presumptive legitimes which should be recorded in the appropriate civil registry and registries of property. H should be so advised. ALTERNATIVE ANSWER: for (a) The following are the requisites prescribed by law and I advice to H is to comply with them, namely: 1) If either spouse contracted the marriage in bad faith, his or her share of the net profits of the community property : or conjugal partnership property shall be forfeited in favor of the common children or, if there are none, the children of the guilty spouse by a previous marriage or, in default of children, the innocent spouse; 2) Donations by reason of marriage shall remain valid except that if the donee contracted the marriage in bad faith, such donations made to said donee are revoked by operation of law; 3) The spouse who contracted the subsequent marriage in bad faith shall be disqualified to inherit from the innocent spouse by testate and intestate succession; 4) If both spouses of the subsequent marriage acted in bad faith all donations by reason of marriage and testamentary dispositions made by one in favor of the other are revoked by operation of law. 5) The judgment of annulment of the marriage, the partition and distribution of the properties of the spouses, and the delivery of the children's presumptive legitimes shall be recorded in the appropriate civil registry and registers of property, (Articles 53. 52, 43. 44. Family Code). SUGGESTED ANSWER:

(b) The children born from the union of H and W would be legitimate children if conceived or born before the decree of annulment of the marriage (under Art. 45 of the Family Code) has become final and executory (Art. 54, Family Code}. SUGGESTED ANSWER:

(c) The children of the first marriage shall be considered legitimate children if conceived or born before the Judgment of annulment of the marriage of H and W has become final and executory. Children conceived or born of the subsequent marriage shall likewise be legitimate even if the marriage of H and S be null and void for failure to comply with the requisites of Article 52 of the Family Code (Article 53, Family Code). As legitimate children, they have the following rights; a) To bear the surnames of the father and the mother in conformity with the provisions of the Civil Code on Surnames; b) To receive support from their parents, their ascendants, and in proper cases, their brothers and sisters, in conformity with the provisions of this Code on Support; and c) To be entitled to the legitime and other successional rights granted to them by the Civil Code (Article 174, Family Code). Marriage; Annulment; Grounds (1991) One of the grounds for annulment of marriage is that either party, at the time of their marriage was afflicted with a

sexually-transmissible disease, found to be serious and appears incurable. Two (2) years after their marriage, which took place on 10 October 1988, Bethel discovered that her husband James has a sexually-transmissible disease which he contracted even prior to their marriage although James did not know it himself until he was examined two [2) years later when a child was already born to them. Bethel sues James for annulment of their marriage. James opposes the annulment on the ground that he did not even know that he had such a disease so that there was no fraud or bad faith on his part. Decide. B. Suppose that both parties at the time of their marriage were similarly afflicted with sexually-transmissible diseases, serious and incurable, and both knew of their respective infirmities, can Bethel or James sue for annulment of their marriage?

SUGGESTED ANSWER:

A. The marriage can be annulled, because good faith is not a defense when the ground is based upon sexually-transmissible disease on the part of either party. SUGGESTED ANSWER:

B. Yes, the marriage can still be annulled because the fact that both of them are afflicted with sexually-transmissible diseases does not efface or nullity the ground. Alternative Answer:

B. No, the marriage can no longer be annulled, because the fact that both were afflicted and that both knew of their respective infirmities constitutes a waiver of that ground. Marriage; Annulment; Judicial Declaration (1993) Maria and Luis, both Filipinos, were married by a Catholic priest in Lourdes Church, Quezon City in 1976, Luis was drunk on the day of his wedding. In fact, he slumped at the altar soon after the ceremony. After marriage, Luis never had a steady job because he was drunk most of the time. Finally, he could not get employed at all because of drunkenness. Hence, it was Maria who had to earn a living to support herself and her child begotten with Luis. In 1986, Maria filed a petition in the church matrimonial court in Quezon City to annul her marriage with Luis on the ground of psychological incapacity to comply with his marital obligation. Her petition was granted by the church matrimonial court. 1) Can Maria now get married legally to another man under Philippine laws after her marriage to Luis was annulled by the church matrimonial court? Explain. 2) What must Maria do to enable her to get married lawfully to another man under Philippine laws? SUGGESTED ANSWER:

1) No, Maria cannot validly contract a subsequent marriage without a court declaration of nullity of the first marriage. The law does not recognize the church declaration of nullity of a marriage. 2) To enable Maria to get married lawfully to another man. she must obtain a judicial declaration of nullity of the prior marriage under Article 36 Family Code. Marriage; Annulment; Legal Separation; Prescription of Actions (1996) 2) Bert and Baby were married to each other on December 23, 1988. Six months later, she discovered that he was a

CIVIL LAW Answers to the BAR as Arranged by Topics (Year 1990-2006)

drug addict. Efforts to have him rehabilitated were unsuccessful. Can Baby ask for annulment of marriage, or legal separation? Explain. SUGGESTED ANSWER:

No, Baby cannot ask for annulment of her marriage or for legal separation because both these actions had already prescribed. While concealment of drug addiction existing at the time of marriage constitutes fraud under Art. 46 of the FC which makes the marriage voidable under Art. 45 of the FC, the action must, however, be brought within 5 years from the discovery thereof under Article 47(3), FC, Since the drug addiction of Bert was discovered by Baby in June 1989, the action had already prescribed in June of 1994. Although drug addiction is a ground for legal separation under Art. 55(5) and Art. 57 of the FC requires that the action must be brought within 5 years from the occurrence of the cause. Since Bert had been a drug addict from the time of the celebration of the marriage, the action for legal separation must have been brought not later than 23 December 1993. Hence, Baby cannot, now, bring the action for legal separation. Marriage; Annulment; Proper Party (1990) D and G, age 20 and 19, respectively, and both single, eloped and got married to each other without parental consent in the case of G, a teenaged student of an exclusive college for girls. Three years later, her parents wanted to seek judicial annulment on that ground. You were consulted and asked to prepare the proper complaint. What advice would you give G's parents? Explain your answer. SUGGESTED ANSWER:

G himself should file the complaint under Article 45 of the Family Code, and no longer the parents because G is already 22 years of age. Marriage; Annulment; Proper Party (1995) Yvette was found to be positive for HIV virus, considered sexually transmissible, serious and incurable. Her boyfriend Joseph was aware of her condition and yet married her. After two (2) years of cohabiting with Yvette, and in his belief that she would probably never be able to bear him a healthy child, Joseph now wants to have his marriage with Yvette annulled. Yvette opposes the suit contending that Joseph is estopped from seeking annulment of their marriage since he knew even before their marriage that she was afflicted with HIV virus. Can the action of Joseph for annulment of his marriage with Yvette prosper? Discuss fully.

In 1989, Maris, a Filipino citizen, married her boss Johnson, an American citizen, in Tokyo in a wedding ceremony celebrated according to Japanese laws. One year later, Johnson returned to his native Nevada, and he validly obtained in that state an absolute divorce from his wife Maris. After Maris received the final judgment of divorce, she married her childhood sweetheart Pedro, also a Filipino citizen, in a religious ceremony in Cebu City, celebrated according to the formalities of Philippine law. Pedro later left for the United States and became naturalized as an American citizen. Maris followed Pedro to the United States, and after a serious quarrel, Marts filed a suit and obtained a divorce decree issued by the court in the state of Maryland. Maris then returned to the Philippines and in a civil ceremony celebrated in Cebu City according to the formalities of Philippine law, she married her former classmate Vincent likewise a Filipino citizen. b) Was the marriage of Maris and Pedro valid when celebrated? Is their marriage still valid existing now? Reasons. c) Was the marriage of Marts and Vincent valid when celebrated? Is their marriage still validly existing now? Reasons. d) At this point in time, who is the lawful husband of Marts? Reasons.

SUGGESTED ANSWER:

(b) The marriage of Maris and Pedro was valid when celebrated because the divorce validly obtained by Johnson in Manila capacitated Maris to marry Pedro. The marriage of Maris and Pedro is still validly existing, because the marriage has not been validly dissolved by the Maryland divorce [Art. 26, Family Code). (c) The marriage of Maris and Vincent is void ab initio because it is a bigamous marriage contracted by Maris during the subsistence of her marriage with Pedro (Art 25 and 41, Family Code). The marriage of Maris and Vincent does not validly exist because Article 26 does not apply. Pedro was not a foreigner at the time of his marriage with marts and the divorce abroad (in Maryland) was initiated and obtained not by the alien spouse, but by the Filipino spouse. Hence, the Maryland divorce did not capacitate Marts to marry Vincent.

(d) At this point in time, Pedro is still the lawful husband of Maris because their valid marriage has not been dissolved by any valid cause (Art. 26. Family Code)

SUGGESTED ANSWER:

No, Joseph knew that Yvette was HIV positive at the time of the marriage. He is, therefore, not an injured party. The FC gives the right to annul the marriage only to an injured party. [Art. 47 (5), FC] ALTERNATIVE ANSWER:

The action for annulment can prosper because the prescriptive period of five (5) years has not yet lapsed. [Art. 45 (6), FC]. Marriage; Divorce Decree; Void Marriages (1992)

Marriage; Divorce Decrees; Filiation of Children (2005) In 1985, Sonny and Lulu, both Filipino citizens, were married in the Philippines. In 1987, they separated, and Sonny went to Canada, where he obtained a divorce in the same year. He then married another Filipina, Auring, in Canada on January 1,1988. They had two sons, James and John. In 1990, after failing to hear from Sonny, Lulu married Tirso, by whom she had a daughter, Verna. In 1991, Sonny visited the Philippines where he succumbed to heart attack..

CIVIL LAW Answers to the BAR as Arranged by Topics (Year 1990-2006)

a) Discuss the effect of the divorce obtained by Sonny and Lulu in Canada. (2%) SUGGESTED ANSWER:

The divorce is not valid. Philippine law does not provide for absolute divorce. Philippine courts cannot grant it. A marriage between two (2) Filipinos cannot be dissolved by a divorce obtained abroad. (Garcia v. Redo, G.R. No. 138322, October 2, 2001). Philippine laws apply to Sonny and Lulu. Under Article 15 of the New Civil Code, laws relating to family rights and duties, status, and capacity of persons are binding upon citizens of the Philippines wherever they may be. Thus, the marriage of Sonny and Lulu is still valid and subsisting. b) Explain the status of the marriage between Sonny and Auring. (2%) SUGGESTED ANSWER:

Since the decree of divorce obtained by Lulu and Sony in Canada is not recognized here in the Philippines, the marriage between Sonny and Auring is void. (Art. 35, Family Code) Any marriage subsequently contracted during the lifetime of the first spouse shall be illegal and void, subject only to the exception in the cases of absence or where the prior marriage was dissolved or annulled. (Ninal v. Bayadog, G.R. No. 133778, March 14, 2000) The marriage of Sonny and Auring does not fall within the exception. c) Explain the status of the marriage between Lulu and Tirso. (2%) SUGGESTED ANSWER:

The marriage of Lulu and Tirso is also void. Mere absence of the spouse does not give rise to a right of the present spouse to remarry. Article 41 of the Family Code provides for a valid bigamous marriage only where a spouse has been absent for four consecutive years before the second marriage and the present spouse had a well-founded belief that the absent spouse is already dead. (Republic v. Nolasco, G.R. No. 94053, March 17, 1993) d) Explain the respective filiation of James, John and Verna. (2%) SUGGESTED ANSWER:

James, John and Verna are illegitimate children since their parents are not validly married. Under Article 165 of the Family Code, children conceived and born outside a valid marriage are illegitimate, unless otherwise provided in this Code. e) Who are the heirs of Sonny? Explain. (2%) Suggested answer: Sonny's heirs include James, John, and Lulu. Article 887 of the Civil Code provides that the compulsory heirs of the deceased are among others, his widow and his illegitimate children. The widow referred to in Article 887 is the legal wife of the deceased. Lulu is still a compulsory heir of Sonny because the divorce obtained by Sonny in Canada cannot be recognized in the Philippines. The legitime of each illegitimate child shall consist of one-half of the legitime of a legitimate child. (Art. 176, Family Code) Marriage; Divorce Decrees; Filipino Spouses becoming Alien (1996)

Flor and Virgillo were married to each other in Roxas City in 198O. In 1984, Flor was offered a teaching Job in Canada, which she accepted. In 1989, she applied for and was granted Canadian citizenship. The following year, she sued for divorce from Virgilio in a Canadian court. After Virgilio was served with summons, the Canadian court tried the case and decreed the divorce. Shortly thereafter, Flor married a Canadian. Can Virgilio marry again in the Philippines? Explain.

SUGGESTED ANSWER:

No, Virgilio cannot validly remarry. His case is not covered by Article 26 of the Family Code, For said Article to be applicable, the spouse who filed for divorce must be a foreigner at the time of the marriage. Since both of them were Filipinos at the time of the marriage, the divorce obtained by Flor did not capacitate Virgilio to remarry. The fact that Flor was already an alien at the time she obtained the divorce does not give Virgilio the capacity to remarry under Philippine Law. ALTERNATIVE ANSWERS:

a) Yes, Virgilio can validly remarry. Art. 26 of the FC, merely States the alien spouse without taking into consideration his or her nationality at the time of the marriage. While his case is not covered by the letter of Article 26 FC, it is, however, covered by the spirit of said Article, the injustice to the Filipino spouse sought to be cured by said Article is present in this case. (Department of Justice Opinion No. 134 Series of 1993). b) Although the marriage originally involved Filipino citizens, it eventually became a marriage between an alien and a Filipino after Flor became a Canadian citizen. Thus, the divorce decree was one obtained by an alien spouse married to a Filipino. Although nothing is said about whether such divorce did capacitate Flor to remarry, that fact may as well be assumed since the problem states that she married a Canadian shortly after obtaining the divorce. Hence, Virgillo can marry again under Philippine law, pursuant to Art. 26. FC which applies because Flor was already an alien at the time of the divorce. Marriage; Divorce Decrees; Filipino Spouses becoming Alien (1999) Ben and Eva were both Filipino citizens at the time of their marriage in 1967, When their marriage turned sour, Ben went to a small country in Europe, got himself naturalized there, and then divorced Eva in accordance with the law of that country, Later, he returned to the Philippines with his new wife. Eva now wants to know what action or actions she can file against Ben. She also wants to know if she can likewise marry again. What advice can you give her? {5%) SUGGESTED ANSWER:

Considering that Art. 26(2nd par.) contemplates a divorce between a foreigner and a Filipino, who had such respective nationalities at the time of their marriage, the divorce in Europe will not capacitate the Filipino wife to remarry. The advice we can give her is either to file a petition for legal separation, on the ground of sexual infidelity and of contracting a bigamous marriage abroad, or to file a petition to dissolve the conjugal partnership or absolute community of property as the case maybe.

CIVIL LAW Answers to the BAR as Arranged by Topics (Year 1990-2006) ALTERNATIVE ANSWER:

Eva may file an action for legal separation on the grounds of sexual infidelity of her husband and the contracting by her husband of a bigamous marriage abroad. She may remarry. While a strict interpretation of Article 26 of the Family Code would capacitate a Filipino spouse to remarry only when the other spouse was a foreigner at the time of the marriage, the DOJ has issued an opinion (Opinion 134 s. of 1993) that the same injustice sought to be cured by Article 26 is present in the case of spouses who were both Filipino at the time of the marriage but one became an alien subsequently. Said injustice is the anomaly of Eva remaining married to her husband who is no longer married to her. Hence, said Opinion makes Article 26 applicable to her case and the divorce obtained abroad by her former Filipino husband would capacitate her to remarry. To contract a subsequent marriage, all she needs to do is present to the civil registrar the decree of divorce when she applies for a marriage license under Article 13 of the Family Code. Marriage; Donations by Reason of Marriage; Effect of Declaration of Nullity (1996) 1) On the occasion of Digna's marriage to George, her father gave her a donation propter nuptias of a car. Subsequently, the marriage was annulled because of the psychological immaturity of George. May Digna's father revoke the donation and get back the car? Explain. SUGGESTED ANSWER:

No, Digna's father may not revoke the donation because Digna was not in bad faith, applying Art. 86(3) of the Family Code. ALTERNATIVE ANSWER:

a) Yes, the donation is revocable. Since the ground for the annulment of the marriage is the psychological immaturity of George, the judgment was in the nature of a declaration of nullity under Art. 36 of the FC and, therefore, the donation may be revoked under Art. 86( 1) of the FC for the reason that the marriage has been judicially declared void ab initio. ALTERNATIVE ANSWER:

b) No, the donation cannot be revoked. The law provides that a donation by reason of marriage may be revoked by the donor if among other cases, the marriage is judicially declared void ab initio [par. (1) Art. 86. Family Code], or when the marriage is annulled and the donee acted in bad faith [par. (3), Id.]. Since the problem states that the marriage was annulled and there is no intimation of bad faith on the part of the donee Digna, the conclusion is that the donor cannot revoke the donation. ALTERNATIVE ANSWER: c) Yes, the donation can be revoked. The ground used in dissolving the marriage was the psychological immaturity of George, which is not a ground for annulment of marriage. If this term is equated with psychological incapacity as used in Art. 36 of the Family Code, then it is a ground for declaration of nullity of the marriage. Consequently, par. (1) of Art. 86, FC, is the applicable law. Since Art. 86 of the FC makes no qualification as to who furnished the ground or who was in bad faith in connection with the nullification of

the marriage, the conclusion is that Digna's father may revoke the donation and get back the car.

Marriage; Grounds; Declaration of Nullity: Annulment: Legal Separation: Separation of Property (2003) Which of the following remedies, i.e., (a) declaration of nullity of marriage, (b) annulment of marriage, (c) legal separation, and/or (d) separation of property, can an aggrieved spouse avail himself/herself of(i) If the wife discovers after the marriage that her husband has (ii) If the ―AIDS‖. wife goes (to) abroad to work as a nurse and refuses to come home after the expiration of her three-year contract there. (iii) If the husband discovers after the marriage that his wife has been a prostitute before they got married. (iv) If the husband has a serious affair with his secretary and refuses to stop notwithstanding advice from relatives and friends. (v) If the husband beats up his wife every time he comes home drunk. 5% SUGGESTED ANSWER:

(i) Since AIDS is a serious and incurable sexually-transmissible disease, the wife may file an action for annulment of the marriage on this ground whether such fact was concealed or not from the wife, provided that the disease was present at the time of the marriage. The marriage is voidable even though the husband was not aware that he had the disease at the time of marriage. (ii) If the wife refuses to come home for three (3) months from the expiration of her contract, she is presumed to have abandoned the husband and he may file an action for judicial separation of property. If the refusal continues for more than one year from the expiration of her contract, the husband may file the action for legal separation under Art. 55 (10) of the Family Code on the ground of abandonment of petitioner by respondent without justifiable cause for more than one year. The wife is deemed to have abandoned the husband when she leaves the conjugal dwelling without any intention of returning (Article 101, FC). The intention not to return cannot be presumed during the 30year period of her contract.

(iii) If the husband discovers after the marriage that his wife was a prostitute before they got married, he has no remedy. No misrepresentation or deceit as to character, health, rank, fortune or chastity shall constitute fraud as legal ground for an action for the annulment of marriage (Article 46 FC).

(iv) The wife may file an action for legal separation. The husband’s sexual infidelity is a ground for legal separation 9Article 55, FC). She may also file an action for judicial separation of property for failure of her husband to comply with his martial duty of fidelity (Article 135 (4), 101, FC). (v) The wife may file an action for legal separation on the ground of repeated physical violence on her person (Article 55 (1), FC). She may also file an action for judicial

CIVIL LAW Answers to the BAR as Arranged by Topics (Year 1990-2006)

separation of property for failure of the husband to comply with his marital duty of mutual respect (Article 135 (4), Article 101, FC). She may also file an action for declaration of nullity of the marriage if the husband’s behavior constitute psychological incapacity existing at the time of the celebration of marriage.

Saudi Arabia to work. There, after being converted into Islam, Ariel married Mystica, Rosa learned of the second marriage of Ariel on January 1, 1992 when Ariel returned to the Philippines with Mystica. Rosa filed an action for legal separation on February 5, 1994, 1) Does Rosa have legal grounds to ask for legal separation? 2) Has the action prescribed?

Marriage; Grounds; Nullity; Annulment; Legal Separation (1997) Under what conditions, respectively, may drug addiction be a ground, if at all, (a) for a declaration of nullity of marriage, (b) for an annulment of the marriage contract, and (c) for legal separation between the spouses?

SUGGESTED ANSWER:

SUGGESTED ANSWER:

(a) Declaration of nullity of marriage: 1) The drug addiction must amount to psychological incapacity to comply with the essential obligations of marriage; 2) It must be antecedent (existing at the time of marriage), grave and incurable: 3) The case must be filed before August 1, 1998. Because if they got married before August 3, 1998, it must be filed before August 1, 1998. (b) Annulment of the Marriage Contract: 1) The drug addiction must be concealed; 2) It must exist at the time of marriage; 3) There should be no cohabitation with full knowledge of the drug addiction; 4) The case is filed within five (5) years from discovery. (c) Legal Separation; 1) There should be no condonation or consent to the drug addiction; 2) The action must be filed within five (5) years from the occurrence of the cause. 3) Drug addiction arises during the marriage and not at the time of marriage. Marriage; Legal Separation; Declaration of Nullity (2002) If drug addiction, habitual alcoholism, lesbianism or homosexuality should occur only during the marriage, would this constitute grounds for a declaration of nullity or for legal separation, or would they render the marriage voidable? (1%). SUGGESTED ANSWER:

In accordance with law, if drug addiction, habitual alcoholism, lesbianism or homosexuality should occur only during the marriage, they: a) Will not constitute as ground for declaration of nullity (Art. 36, Family Code); b) Will constitute as grounds for legal separation (Art. 56, FC) and c) will not constitute as grounds to render the marriage voidable (Art.45and 46, FC) Marriage; Legal Separation; Grounds; Prescriptive Period (1994) Rosa and Ariel were married in the Catholic Church of Tarlac, Tarlac on January 5. 1988. In 1990, Ariel went to

1) a) Yes, the abandonment of Rosa by Ariel for more than one (1) year is a ground for legal separation unless upon returning to the Philippines, Rosa agrees to cohabit with Ariel which is allowed under the Muslim Code. In this case, there is condonation. b) Yes. The contracting of a subsequent bigamous marriage whether in the Philippines or abroad is a ground for legal separation under Article 55(7) of the Family Code. Whether the second marriage is valid or not, Ariel having converted into Islam, is immaterial. SUGGESTED ANSWER:

2) No. Under Article 57 of the Family Code, the aggrieved spouse must file the action within five (5) years from the occurrence of the cause. The subsequent marriage of Ariel could not have occurred earlier than 1990, the time he went to Saudi Arabia. Hence, Rosa has until 1995 to bring the action under the Family Code. Marriage; Legal Separation; Mutual guilt (2006) Saul, a married man, had an adulterous relation with Tessie. In one of the trysts, Saul's wife, Cecile, caught them in flagrante. Armed with a gun, Cecile shot Saul in a fit of extreme jealousy, nearly killing him. Four (4) years after the incident, Saul filed an action for legal separation against Cecile on the ground that she attempted to kill him. (1) If you were Saul's counsel, how will you argue his case? (2.5%) SUGGESTED ANSWER:

As the counsel of Saul, I will argue that an attempt by the wife against the life of the husband is one of the grounds enumerated by the Family Code for legal separation and there is no need for criminal conviction for the ground to be invoked (Art. 55, par. 9, Family Code). (2) If you were the lawyer of Cecile, what will be your defense? (2.5%) SUGGESTED ANSWER:

As the counsel of Cecile, I will invoke the adultery of Saul. Mutual guilt is a ground for the dismissal of an action for legal separation (Art. 56, par. 4, Family Code). The rule is anchored on a well-established principle that one must come to court with clean hands. (3) If you were the judge, how will you decide the case? (5%) SUGGESTED ANSWER:

If I were the judge, I will dismiss the action on the ground of mutual guilt of the parties. The Philippine Constitution protects marriage as an inviolable social institution (Art. XV, Sec. 2, 1987 Constitution). An action for legal separation involves public interest and no such decree should be issued if any legal obstacle thereto appears on record. This is in line with the policy that in case of doubt,

CIVIL LAW Answers to the BAR as Arranged by Topics (Year 1990-2006)

the court shall uphold the validity and sanctity of marriage

the preceding Article, only the properties acquired by both

(Brown v. Yambao, G.R. No. L-10699, October 18, 1957).

of the parties through their actual joint contribution of money, property, or industry shall be owned by them in common in proportion to their respective contributions. In the absence, of proof to the contrary, their contributions and corresponding shares are presumed to be equal. The same rule and presumption shall apply to joint deposits of money and evidences of credit.

Marriage; Non-Bigamous Marriages (2006) Marvin, a Filipino, and Shelley, an American, both residents of California, decided to get married in their local parish. Two years after their marriage, Shelley obtained a divorce in California. While in Boracay, Marvin met Manel, a Filipina, who was vacationing there. Marvin fell in love with her. After a brief courtship and complying with all the requirements, they got married in Hongkong to avoid publicity, it being Marvin's second marriage. Is his marriage to Manel valid? Explain. (5%) SUGGESTED ANSWER:

Yes. The marriage will not fall under Art. 35(4) of the Family Code on bigamous marriages, provided that Shelley obtained an absolute divorce, capacitating her to remarry under her national law. Consequently, the marriage between Marvin and Manel may be valid as long as it was solemnized and valid in accordance with the laws of Hongkong [Art. 26, paragraphs 1 and 2, Family Code]. Marriage; Property Relations; Void Marriages (1991) In June 1985, James married Mary. In September 1988, he also married Ophelia with whom he begot two (2) children, A and B. In July 1989, Mary died. In July 1990, he married Shirley and abandoned Ophelia, During their union. James and Ophelia acquired a residential lot worth P300,000.00. Ophelia sues James for bigamy and prays that his marriage with Shirley be declared null and void. James, on the other hand, claims that since his marriage to Ophelia was contracted during the existence of his marriage with Mary, the former is not binding upon him, the same being void ab initio he further claims that his marriage to Shirley is valid and binding as he was already legally capacitated at the time he married her. a) Is the contention of James correct? b) What property Relations governed the union of James and Ophelia? c) Is the estate of Mary entitled to a share in the residential lot acquired by James and Ophelia? SUGGESTED ANSWER:

A. Yes. His marriage to Ophelia is void ab initio because of his subsisting prior marriage to Mary. His marriage to Shirley, after Mary's death, is valid and binding. ALTERNATIVE ANSWER:

A. No. The contention of James is not correct. Art. 40, Family Code, provides that the "absolute nullity of a previous marriage may be invoked for purposes of remarriage on the basis solely of a final judgment declaring such previous marriage void." It can be said, therefore, that the marriage of James to Shirley is void since his previous marriage to Ophelia, although itself void, had not yet been judicially declared void, ALTERNATIVE ANSWER:

A. No. The contention of James is not correct. He cannot set up as a defense his own criminal act or wrongdoingSUGGESTED ANSWER: B. The provisions of Art 148 of the Family Code, shall govern: Art. 148. In cases of cohabitation not falling under

SUGGESTED ANSWER:

C. It should be distinguished when the property was acquired. • If it was acquired before Mary's death, the estate of Mary is entitled to 1/2 of the share of James. • If it was acquired after Mary's death, there will be no share at all for the estate of Mary. Marriage; Psychological Incapacity (1996) On April 15, 1983, Jose, an engineer, and Marina, a nurse, were married to each other in a civil ceremony in Boac. Marinduque. Six months after their marriage, Jose was employed in an oil refinery in Saudi Arabia for a period of three years. When he returned to the Philippines, Marina was no longer living in their house, but in Zamboanga City, working in a hospital. He asked her to come home, but she refused to do so, unless he agreed not to work overseas anymore because she cannot stand living alone. He could not agree as in fact, he had signed another three year contract. When he returned in 1989, he could not locate Marina anymore. In 1992, Jose filed an action served by publication in a newspaper of general circulation. Marina did not file any answer, a possible collusion between the parties was ruled out by the Public Prosecutor. Trial was conducted and Marina neither appeared nor presented evidence in her favor. If you were the judge, will you grant the annulment. Explain. SUGGESTED ANSWER:

As judge, I will not grant the annulment. The facts do not show any taint of personality disorder on the part of the wife Marina so as to lend substance to her husband's averment of psychological incapacity within the meaning of Art 36 of the Family Code. In Santos vs. CA (240 SCRA 20), this particular ground for nullity of marriage was held to be limited only to the most serious cases of personality disorders (clearly demonstrative of utter sensitivity or inability to give meaning and significance to the marriage. Marina's refusal to come home to her husband unless he agreed not to work overseas, far from being indicative of an insensitivity to the meaning of marriage, or of a personality disorder, actually shows a sensitive awareness on her part of the marital duty to live together as husband and wife. Mere refusal to rejoin her husband when he did not accept the condition imposed by her does not furnish any basis for concluding that she was suffering from psychological incapacity to discharge the essential marital obligations. Mere intention to live apart does not fall under Art. 36, FC. Furthermore, there is no proof that the alleged psychological incapacity existed at the time of the marriage.

Page 30 of 119

CIVIL LAW Answers to the BAR as Arranged by Topics (Year 1990-2006)

Marriage; Psychological Incapacity (2006) Gemma filed a petition for the declaration of nullity of her marriage with Arnell on the ground of psychological incapacity. She alleged that after 2 months of their marriage, Arnell showed signs of disinterest in her, neglected her and went abroad. He returned to the Philippines after 3 years but did not even get in touch with her. Worse, they met several times in social functions but he snubbed her. When she got sick, he did not visit her even if he knew of her confinement in the hospital. Meanwhile, Arnell met an accident which disabled him from reporting for work and earning a living to support himself. Will Gemma's suit prosper? Explain. (5%) SUGGESTED ANSWER:

No, Gemma's suit will not prosper. Even if taken as true, the grounds, singly or collectively, do not constitute "psychological incapacity." In Santos v. CA, G.R. No. 112019, January 4, 1995, the Supreme Court clearly explained that "psychological incapacity must be characterized by (a) gravity, (b) juridical antecedence, and (c) incurability" (Ferraris v. Ferraris, G.R. No. 162368, July 17, 2006; Choa v. Choa, G.R. No. 143376, November 26, 2002). The illness must be shown

as downright incapacity or inability to perform one's marital obligations, not a mere refusal, neglect, difficulty or much less, ill will. Moreover, as ruled in Republic v. Molina, GR No. 108763, February 13, 1997, it is essential that the husband is capable of meeting his marital responsibilities due to psychological and not physical illness (Antonio v. Reyes, G.R. No. 155800, March 10, 2006; Republic v. Quintero-Hamano, G.R. No. 149498, May 20, 2004).

Furthermore, the condition complained of did not exist at the time of the celebration of marriage. Marriage; Psychological Incapacity (2006) Article 36 of the Family Code provides that a marriage contracted by any party who, at the time of the celebration, was psychologically incapacitated to comply with the essential marital obligations of marriage, shall be void. Choose the spouse listed below who is psychologically incapacitated. Explain. (2.5%) a) Nagger b) Gay or Lesbian c) Congenital sexual pervert d) Gambler e) Alcoholic SUGGESTED ANSWER: The best answers are B and C. To be sure, the existence and concealment of these conditions at the inception of marriage renders the marriage contract voidable (Art. 46, Family Code). They may serve as indicia of psychological incapacity, depending on the degree and severity of the disorder (Santos v. CA, G.R. No. 112019, Jan. 4, 1995). Hence, if the condition of homosexuality, lesbianism or sexual perversion, existing at the inception of the marriage, is of such a degree as to prevent any form of sexual intimacy, any of them may qualify as a ground for psychological incapacity. The law provides that the husband and wife are obliged to live together, observe mutual love, respect and fidelity (Art. 68, Family Code). The mandate is actually the spontaneous, mutual affection between the spouses. In the natural order it is sexual intimacy which brings the spouses wholeness and oneness (Chi Ming Tsoi

ALTERNATIVE ANSWER:

None of them are necessarily psychologically incapacitated. Being a nagger, etc. are at best only physical manifestations indicative of psychological incapacity. More than just showing the manifestations of incapacity, the petitioner must show that the respondent is incapacitated to comply with the essential marital obligations of marriage and that it is also essential that he must be shown to be incapable of doing so due to some psychological, not physical illness (Republic v. Quintero-Hamano, G.R. No. 149498, May 20, 2004). ALTERNATIVE ANSWER:

A congenital sexual pervert may be psychologically incapacitated if his perversion incapacitates him from discharging his marital obligations. For instance, if his perversion is of such a nature as to preclude any normal sexual activity with his spouse. Marriage; Requisites (1995) Isidro and Irma, Filipinos, both 18 years of age, were passengers of Flight No. 317 of Oriental Airlines. The plane they boarded was of Philippine registry. While en route from Manila to Greece some passengers hijacked the plane, held the chief pilot hostage at the cockpit and ordered him to fly instead to Libya. During the hijacking Isidro suffered a heart attack and was on the verge of death. Since Irma was already eight months pregnant by Isidro, she pleaded to the hijackers to allow the assistant pilot to solemnize her marriage with Isidro. Soon after the marriage, Isidro expired. As the plane landed in Libya Irma gave birth. However, the baby died a few minutes after complete delivery. Back in the Philippines Irma immediately filed a claim for inheritance. The parents of Isidro opposed her claim contending that the marriage between her and Isidro was void ab initio on the following grounds: (a) they had not given their consent to the marriage of their son; (b) there was no marriage license; (c) the solemnizing officer had no authority to perform the marriage; and, (d) the solemnizing officer did not file an affidavit of marriage with the proper civil registrar. 1. Resolve each of the contentions ([a] to [d]) raised by the parents of Isidro. Discuss fully. SUGGESTED ANSWER:

1. (a) The fact that the parents of Isidro and of Irma did not give their consent to the marriage did not make the marriage void ab initio. The marriage is merely voidable under Art 45 of the FC. (b) Absence of marriage license did not make the marriage void ab initio. Since the marriage was solemnized in articulo mortis, it was exempt from the license requirement under Art. 31 of the FC. (c) On the assumption that the assistant pilot was acting for and in behalf of the airplane chief who was under disability, and by reason of the extraordinary and exceptional circumstances of the case [ie. hostage situation), the marriage was solemnized by an authorized officer under Art. 7 (3) and Art. 31. of the FC.

v. CA, G.R. No. 119190, January 16,1997).

CIVIL LAW Answers to the BAR as Arranged by Topics (Year 1990-2006)

(d) Failure of the solemnizing officer to file the affidavit of marriage did not affect the validity of the marriage. It is merely an irregularity which may subject the solemnizing officer to sanctions.

ALTERNATIVE ANSWER:

If the two Filipinos believed in good faith that the Notary Public is authorized to solemnize marriage, then the marriage is valid.

ALTERNATIVE ANSWER:

Considering that the solemnizing officer has no authority to perform the marriage because under Art. 7 the law authorizes only the airplane chief, the marriage is void, hence, a, c, and d are immaterial. Marriage; Requisites (1999) What is the status of the following marriages and why? (a) A marriage between two 19-year olds without parental consent, (2%) (b) A marriage between two 21-year olds without parental advice. (2%) (c) A marriage between two Filipino first cousins in Spain where such marriage is valid. (2%) (d) A marriage between two Filipinos in Hongkong before a notary public. (2%) (e) A marriage solemnized by a town mayor three towns away from his jurisdiction, (2%) SUGGESTED ANSWER:

(a) The marriage is voidable. The consent of the parties to the marriage was defective. Being below 21 years old, the consent of the parties is not full without the consent of their parents. The consent of the parents of the parties to the marriage is indispensable for its validity. SUGGESTED ANSWER:

(b) Between 21-year olds, the marriage is valid despite the absence of parental advice, because such absence is merely an irregularity affecting a formal requisite i.e., the marriage license and does not affect the validity of the marriage itself. This is without prejudice to the civil, criminal, or administrative liability of the party responsible therefor. SUGGESTED ANSWER:

(c) By reason of public policy, the marriage between Filipino first cousins is void [Art. 38, par. (1), Family Code], and the fact that it is considered a valid marriage in a foreign country in this case, Spain— does not validate it, being an exception to the general rule in Art. 96 of said Code which accords validity to all marriage solemnized outside the Philippine x x x and valid there as such. ALTERNATIVE ANSWER

The marriage it void. Under Article 96 of the Family Code, a marriage valid where celebrated is valid in the Philippines except those marriages enumerated in said Article which marriages will remain void even though valid where solemnized. The marriage between first cousins is one of those marriages enumerated therein, hence, it is void even though valid in Spain where it was celebrated. By reason of Art. 15 in relation to Article 38 of the Civil Code, which applies to Filipinos wherever they are, the marriage is void. SUGGESTED ANSWER:

(d) It depends. If the marriage before the notary public is valid under Hongkong Law, the marriage is valid in the Philippines. Otherwise, the marriage that is invalid in Hongkong will be invalid in the Philippines.

SUGGESTED ANSWER:

(e) Under the Local Government Code, a town mayor may validly solemnize a marriage but said law is silent as to the territorial limits for the exercise by a town mayor of such authority. However, by analogy, with the authority of members of the Judiciary to solemnize a marriage, it would seem that the mayor did not have the requisite authority to solemnize a marriage outside of his territorial jurisdiction. Hence, the marriage is void, unless it was contracted with either or both parties believing in good faith that the mayor had the legal authority to solemnize this particular marriage (Art 35, par 2 Family Code). ALTERNATIVE ANSWER:

The marriage is valid. Under the Local Government Code, the authority of a mayor to solemnize marriages is not restricted within his municipality implying that he has the authority even outside the territory thereof. Hence, the marriage he solemnized outside his municipality is valid. And even assuming that his authority is restricted within his municipality, such marriage will nevertheless, be valid because solemnizing the marriage outside said municipality is a mere irregularity applying by analogy the case of Navarro v Domagtoy, 259 Scra 129. In this case, the Supreme Court held that the celebration by a judge of a marriage outside the jurisdiction of his court is a mere irregularity that did not affect the validity of the marriage notwithstanding Article 7 of the Family Code which provides that an incumbent member of the judiciary is authorized to solemnize marriages only within the court’s jurisdiction. Marriage; Requisites; Marriage License (1996) On Valentine's Day 1996, Ellas and Fely, both single and 25 years of age, went to the city hall where they sought out a fixer to help them obtain a quickie marriage. For a fee, the fixer produced an ante-dated marriage license for them, Issued by the Civil Registrar of a small remote municipality. He then brought them to a licensed minister in a restaurant behind the city hall, and the latter solemnized their marriage right there and then. 1) Is their marriage valid, void or voidable? Explain. SUGGESTED ANSWER:

The marriage is valid. The irregularity in the issuance of a valid license does not adversely affect the validity of the marriage. The marriage license is valid because it was in fact issued by a Civil Registrar (Arts. 3 and 4. FC). ALTERNATIVE ANSWER:

It depends. If both or one of the parties was a member of the religious sect of the solemnizing officer, the marriage is valid. If none of the parties is a member of the sect and both of them were aware of the fact, the marriage is void. They cannot claim good faith in believing that the solemnizing officer was authorized because the scope of the authority of the solemnizing officer is a matter of law. If, however, one of the parties believed in good faith that the other was a member of the sect, then the marriage is valid

CIVIL LAW Answers to the BAR as Arranged by Topics (Year 1990-2006)

under Article 35 (2), FC. In that case, the party in good faith is acting under a mistake of fact, not a mistake of law, 2) Would your answer be the same if it should turn out that the marriage license was spurious? Explain. SUGGESTED ANSWER:

No, the answer would not be the same. The marriage would be void because of the absence of a formal requisite. In such a case, there was actually no valid marriage license. Marriage; Requisites; Marriage License (2002) On May 1, 1978 Facundo married Petra, by whom he had a son Sotero. Petra died on July 1, 1996, while Facundo died on January 1, 2002. Before his demise, Facundo had married, on July 1, 2002, Quercia. Having lived together as husband and wife since July 1, 1990, Facundo and Quercia did not secure a marriage license but executed the requisite affidavit for the purpose. To ensure that his inheritance rights are not adversely affected by his father second marriage, Sotero now brings a suit to seek a declaration of the nullity of the marriage of Facundo and Quercia, grounded on the absence of a valid marriage license. Quercia contends that there was no need for a marriage license in view for her having lived continuously with Facundo for five years before their marriage and that has Sotero has no legal personality to seek a declaration of nullity of the marriage since Facundo is now deceased. A. Is the marriage of Facundo and Quercia valid, despite the absence of a marriage license? Explain. (2%) SUGGESTED ANSWER:

A. The marriage with Quercia is void. The exemption from the requirement of a marriage license under Art, 34, Family Code, requires that the man and woman must have lived together as husband and wife for at least five years and without any legal impediment to marry each other during those five years. The cohabitation of Facundo and Quercia for six years from 1990 to July 1, 1996 when Petra died was one with a legal impediment hence, not in compliance with the requirement of law. On other hand, the cohabitation thereafter until the marriage on July 1, 2000, although free from legal impediment, did not meet the 5-year cohabitation requirement.

This is different from the case of Nināl V. Bayadog, (328 SCRA 122 [2000]). In the said case, the situation occurred during the Relations of the new Civil Code where Article 76 thereof clearly provides that during the five-year cohabitation, the parties must be unmarried. This is not so anymore in the Family Code. The Change in the Family Code is significant. If the second marriage occurred before the effectivity of the Family Code, the answer would that be that the marriage is void. B. Does Sotero have the personality to seek the declaration of nullity of the marriage, especially now that Facundo is already deceased? Explain. (3%) SUGGESTED ANSWER:

B. A void marriage may be questioned by any interested party in any proceeding where the resolution of the issue is material. Being a compulsory heir, Soterro has the personality to question the validity of the marriage of Facundo and Quercia. Otherwise, his participation in the estate on Facundo would be affected. (Ninãl V. Bayadog, 328 SCRA 122 [2000] ). Marriage; Requisites; Solemnizing Officers (1994) 1} The complete publication of the Family Code was made on August 4, 1987. On September 4, 1987, Junior Cruz and Gemma Reyes were married before a municipal mayor. Was the marriage valid? 2) Suppose the couple got married on September 1, 1994 at the Manila Hotel before the Philippine Consul General to Hongkong, who was on vacation in Manila. The couple executed an affidavit consenting to the celebration of the marriage at the Manila Hotel. Is the marriage valid? SUGGESTED ANSWER:

1) a) Yes, the marriage is valid. The Family Code took effect on August 3, 1988. At the time of the marriage on September 4, 1987, municipal mayors were empowered to solemnize marriage under the Civil Code of 1950. 2) a) The marriage is not valid. Consuls and vice-consuls are empowered to solemnize marriages between Philippine citizens abroad in the consular office of the foreign country to which they were assigned and have no power to solemnize marriage on Philippine soil.

ALTERNATIVE ANSWER:

The marriage of Facundo and Quercia is VALID. The second marriage was solemnized on July 1, 2000, when the Family code was already affective. The family code took effect on August 3, 1988. Under the Family Code, no marriage license is required if the parties have been cohabiting for the period of five years and there is no legal impediment. There must no legal impediment ONLY AT THE TIME OF THE SOLEMNIZATION OF THE MARRIAGE, and not the whole five years period. This is clearly the intent of the code framers (see Minutes of the 150th joint Civil Code of the Family Law Committees held on August 9, 1986). Also, in Manzano V. Sanchez, AM NO. MT –00-129, March 8, 2001, the Supreme Court said that, as one of the requisites for the exception to apply, there must be no legal impediment at the time of the marriage. The Supreme Court did not say that the legal impediment must exist all throughout the five-year period.

b) A Philippine consul is authorized by law to solemnize marriages abroad between Filipino citizens. He has no authority to solemnize a marriage in the Philippines. Consequently, the marriage in question is void, unless either or both of the contracting parties believed in good faith that the consul general had authority to solemnize their marriage in which case the marriage is valid. Marriage; Requisites; Void Marriage (1993) A and B, both 18 years old, were sweethearts studying in Manila. On August 3, 1988, while in first year college, they eloped. They stayed in the house of a mutual friend in town X, where they were able to obtain a marriage license. On August 30, 1988, their marriage was solemnized by the town mayor of X in his office. Thereafter, they returned to Manila and continued to live separately in their respective boarding houses, concealing from their parents, who were living in the province what they had done. In 1992, after graduation

CIVIL LAW Answers to the BAR as Arranged by Topics (Year 1990-2006)

from college, A and B decided to break their relation and parted ways. Both went home to their respective towns to live and work. 1) Was the marriage of A and B solemnized on August 30, 1988 by the town mayor of X in his office a valid marriage? Explain your answer. 2) Can either or both of them contract marriage with another person without committing bigamy? Explain your answer. SUGGESTED ANSWER:

1) The marriage of A and B is void because the solemnizing officer had no legal authority to solemnize the marriage. But if either or both parties believed in good faith that the solemnizing officer had the legal authority to do so, the marriage is voidable because the marriage between the parties, both below 21 years of age, was solemnized without the consent of the parents. (Art. 35, par. (2) and Art. 45 par. (1), Family Code) 2) Either or both of the parties cannot contract marriage in the Philippines with another person without committing bigamy, unless there is compliance with the requirements of Article 52 Family Code, namely: there must be a judgment of annulment or absolute nullity of the marriage, partition and distribution of the properties of the spouses and the delivery of their children's presumptive legitimes, which shall be recorded in the appropriate Civil Registry and Registry of Property, otherwise the same shall not affect third persons and the subsequent marriage shall be null and void. (Arts. 52 and 53. Family Code) ALTERNATIVE ANSWER:

2) Yes, they can. The subsequent marriage contracted by one of the parties will not give rise to bigamy even in the absence of a court declaration of nullity of the first marriage. The subsistence of a prior valid marriage is an indispensable element of the crime of bigamy. The prior court declaration of nullity of the first marriage is required by the Family Code only for the purpose of the validity of the subsequent marriage, not as an element of the crime of bigamy. Marriage; Void Marriages (2004) A. BONI and ANNE met while working overseas. They became sweethearts and got engaged to be married on New Year’s Eve aboard a cruise ship in the Caribbean. They took the proper license to marry in New York City, where there is a Filipino consulate. But as planned the wedding ceremony was officiated by the captain of the Norwegian-registered vessel in a private suite among selected friends. Back in Manila, Anne discovered that Boni had been married in Bacolod City 5 years earlier but divorced in Oslo only last year. His first wife was also a Filipina but now based in Sweden. Boni himself is a resident of Norway where he and Anne plan to live permanently. Anne retains your services to advise her on whether her marriage to Boni is valid under Philippine law? Is there anything else she should do under the circumstances? (5%) SUGGESTED ANSWER: If Boni is still a Filipino citizen, his legal capacity is governed by Philippine Law (Art. 15 Civil Code). Under

Philippine Law, his marriage to Anne is void because of a

prior existing marriage which was not dissolved by the divorce decreed in Oslo. Divorce obtained abroad by a Filipino is not recognized.

If Boni was no longer a Filipino citizen, the divorce is valid. Hence, his marriage to Anne is valid if celebrated in accordance with the law of the place where it was celebrated. Since the marriage was celebrated aboard a vessel of Norwegian registry, Norwegian law applies. If the Ship Captain has authority to solemnize the marriage aboard his ship, the marriage is valid and shall be recognized in the Philippines. As to the second question, if Boni is still a Filipino, Anne can file an action for declaration of nullity of her marriage to him. Marriage; Void Marriages (2006) Gigi and Ric, Catholics, got married when they were 18 years old. Their marriage was solemnized on August 2, 1989 by Ric's uncle, a Baptist Minister, in Calamba, Laguna. He overlooked the fact that his license to solemnize marriage expired the month before and that the parties do not belong to his congregation. After 5 years of married life and blessed with 2 children, the spouses developed irreconcilable differences, so they parted ways. While separated, Ric fell in love with Juliet, a 16 year-old sophomore in a local college and a Seventh-Day Adventist. They decided to get married with the consent of Juliet's parents. She presented to him a birth certificate showing she is 18 years old. Ric never doubted her age much less the authenticity of her birth certificate. They got married in a Catholic church in Manila. A year after, Juliet gave birth to twins, Aissa and Aretha. (1) What is the status of the marriage between Gigi and Ric — valid, voidable or void? Explain. (2.5%) SUGGESTED ANSWER: Even

if the Minister's license expired, the marriage is valid if either or both Gigi and Ric believed in good faith that he had the legal authority to solemnize marriage. While the authority of the solemnizing officer is a formal requisite of marriage, and at least one of the parties must belong to the solemnizing officer's church, the law provides that the good faith of the parties cures the defect in the lack of authority of the solemnizing officer

(Art. 35 par. 2, Family Code; Sempio-Diy, p. 34; Rabuya, The Law on Persons and Family Relations, p. 208).

The absence of parental consent despite their having married at the age of 18 is deemed cured by their continued cohabitation beyond the age of 21. At this point, their marriage is valid (See Art. 45, Family Code). (2) What is the status of the marriage between Ric and Juliet — valid, voidable or void? (2.5%) SUGGESTED ANSWER: The marriage between Juliet and Ric is void. First of all, the marriage is a bigamous marriage not falling under Article 41 [Art. 35(4)Family Code], A subsisting marriage constitutes a legal impediment to remarriage. Secondly, Juliet is below eighteen years of age. The marriage is void even if consented to by her parents

CIVIL LAW Answers to the BAR as Arranged by Topics (Year 1990-2006)

[Art. 35(1), Family Code]. The fact that Ric was not aware of her real age is immaterial.

(3) Suppose Ric himself procured the falsified birth certificate to persuade Juliet to marry him despite her minority and assured her that everything is in order. He did not divulge to her his prior marriage with Gigi. What action, if any, can Juliet take against him? Explain. (2.5%) SUGGESTED ANSWER: Juliet can file an action for the declaration of nullity of the marriage on the ground that he willfully caused loss or injury to her in a manner that is contrary to morals, good customs and public policy [Art. 21, New Civil Code]. She may also bring criminal actions for seduction, falsification, illegal marriage and bigamy against Ric. (4) If you were the counsel for Gigi, what action/s will you take to enforce and protect her interests? Explain. (2.5%) SUGGESTED ANSWER: I would file an action to declare the marriage between Juliet and Ric null and void ab initio and for Ric's share in the co-ownership of that marriage to be forfeited in favor and considered part of the absolute community in the marriage between Gigi and Ric [Arts. 148 & 147, Family Code]. I would also file an action for damages against Ric on the grounds that his acts constitute an abuse of right and they are contrary to law and morals, causing damages to Gigi (See Arts 19, 20, 21, New Civil Code).

Marriage; Void Marriages; Psychological Incapacity (2002) A. Give a brief definition or explanation of the term ―psychological incapacity‖ as a ground for the declaration of nullity of a marriage. (2%) B. If existing at the inception of marriage, would the state of being of unsound mind or the concealment of drug addiction, habitual alcoholism, homosexuality or lesbianism be considered indicia of psychological incapacity? Explain. (2%). SUGGESTED ANSWER:

A.‖ PSYCHOLOGICAL INCAPACITY‖ is a mental disorder of the most serious type showing the incapability of one or both spouses to comply the essential marital obligations of love, respect, cohabitation, mutual help and support, trust and commitment. It must be characterized by Juridical antecedence, gravity and incurability and its root causes must be clinically identified or examined. (Santos v. CA, 240 SCRA 20 [1995]). B. In the case of Santos v. Court of Appeals, 240 SCRA 20 (1995), the Supreme Court held that being of unsound mind, drug addiction, habitual alcoholism, lesbianism or homosexuality may be indicia of psychological incapacity, depending on the degree of severity of the disorder. However, the concealment of drug addiction, habitual alcoholism, lesbianism or homosexuality is a ground of annulment of marriage. Parental Authority; Child under 7 years of age (2006)

Under Article 213 of the Family Code, no child under 7 years of age shall be separated from the mother unless the court finds compelling reasons to order otherwise. (1) Explain the rationale of this provision. (2.5%) SUGGESTED ANSWER:

The rationale of the 2nd paragraph of Article 213 of the Family Code is to avoid the tragedy of a mother who sees her baby torn away from her. It is said that the maternal affection and care during the early years of the child are generally needed by the child more than paternal care (Hontiveros v. IAC, G.R. No. 64982, October 23, 1984; Tolentino, Commentaries and Jurisprudence on the Civil Code, Volume One, pp. 718-719). The general rule is that a child

below 7 years old shall not be separated from his mother due to his basic need for her loving care (Espiritu v. C.A., G.R. No. 115640, March 15,1995).

(2) Give at least 3 examples of "compelling reasons" which justify the taking away from the mother's custody of her child under 7 years of age. (2.5%) SUGGESTED ANSWER:

a.

The mother is insane (Sempio-Diy, Handbook on the Family Code of the Philippines, pp. 296-297);

• The mother is sick with a disease that is communicable and might endanger the health and life of the child; • The mother has been maltreating the child; • The mother is engaged in prostitution; • The mother is engaged in adulterous relationship; • The mother is a drug addict; • The mother is a habitual drunk or an alcoholic; • Authority; The mother is in jail or serving sentence. Parental Special Parental Authority; Liability of Teachers (2003) If during class hours, while the teacher was chatting with other teachers in the school corridor, a 7 year old male pupil stabs the eye of another boy with a ball pen during a fight, causing permanent blindness to the victim, who could be liable for damages for the boy’s injury: the teacher, the school authorities, or the guilty boy’s parents? Explain. SUGGESTED ANSWER:

The school, its administrators, and teachers have special parental authority and responsibility over the minor child while under their supervision, instruction or custody (Article 218, FC). They are principally and solidarily liable for the damages caused by the acts or omissions of the unemancipated minor unless they exercised the proper diligence required under the circumstances (Article 219, FC). In the problem, the TEACHER and the SCHOOL AUTHORITIES are liable for the blindness of the victim, because the student who cause it was under their special parental authority and they were negligent. They were negligent because they were chatting in the corridor during the class period when the stabbing incident occurred. The incident could have been prevented had the teacher been inside the classroom at that time. The guilty boy’s PARENTS are subsidiarily liable under Article 219 of the Family Code. Parental Authority; Substitute vs. Special (2004)

CIVIL LAW Answers to the BAR as Arranged by Topics (Year 1990-2006)

Distinguish briefly but clearly between: Substitute parental authority and special parental authority. SUGGESTED ANSWER:

In substitute parental authority, the parents lose their parental authority in favor of the substitute who acquires it to the exclusion of the parents. In special parental authority, the parents or anyone exercising parental authority does not lose parental authority. Those who are charged with special parental authority exercise such authority only during the time that the child is in their custody or supervision. Substitute parental authority displaces parental authority while special parental authority concurs with parental authority. Paternity & Filiation (1999) (a) Two (2) months after the death of her husband who was shot by unknown criminal elements on his way home from office, Rose married her childhood boyfriend, and seven (7) months after said marriage, she delivered a baby. In the absence of any evidence from Rose as to who is her child's father, what status does the law give to said child? Explain. (2%) SUGGESTED ANSWER:

(a) The child is legitimate of the second marriage under Article 168(2) of the Family Code which provides that a "child born after one hundred eighty days following the celebration of the subsequent marriage is considered to have been conceived during such marriage, even though it be born within three hundred days after the termination of the former marriage." Paternity & Filiation; Proofs (1999) (b) Nestor is the illegitimate son of Dr. Perez. When Dr. Perez died, Nestor intervened in the settlement of his father's estate, claiming that he is the illegitimate son of said deceased, but the legitimate family of Dr. Perez is denying Nestor's claim. What evidence or evidences should Nestor present so that he may receive his rightful share in his father's estate? (3%)

sperm. After a series of test, Andy's sperm was medically introduced into Beth's ovary. She became pregnant and 9 months later, gave birth to a baby boy, named Alvin. (1) Who is the Father of Alvin? Explain. (2.5%) SUGGESTED ANSWER:

Andy is the biological father of Alvin being the source of the sperm. Andy is the legal father of Alvin because there was neither consent nor ratification to the artificial insemination. Under the law, children conceived by artificial insemination are legitimate children of the spouses, provided, that both of them authorized or ratified the insemination in a written instrument executed and signed by both of them before the birth of the child (Art. 164, Family Code). (2) What are the requirements, if any, in order for Ed to establish his paternity over Alvin. (2.5%) SUGGESTED ANSWER:

The following are the requirements for Ed to establish his paternity over Alvin: • The artificial insemination has been authorized or ratified by the spouses in a written instrument executed and signed by them before the birth of the child; and • The written instrument is recorded in the civil registry together with the birth certificate of the child (Art. 164, 2nd paragraph, Family Code). Paternity & Filiation; Common-Law Union (2004) A. RN and DM, without any impediment to marry each other, had been living together without benefit of church blessings. Their common-law union resulted in the birth of ZMN. Two years later, they got married in a civil ceremony. Could ZMN be legitimated? Reason. (5%) SUGGESTED ANSWER:

ZMN was legitimated by the subsequent marriage of RN and DM because at the time he was conceived, RN and DM could have validly married each other. Under the Family Code children conceived and born outside of wedlock of parents who, at the time of the former's conception, were not disqualified by any impediment to marry each other are legitimated by the subsequent marriage of the parents.

SUGGESTED ANSWER:

(b) To be able to inherit, the illegitimate filiation of Nestor must have been admitted by his father in any of the following: (1) the record of birth appearing in the civil register, (2) a final judgment, (3) a public document signed by the father, or (4) a private handwritten document signed by the lather (Article 17S in relation to Article 172 of the Family Code).

Paternity & Filiation; Proofs; Limitations; Adopted Child (1995) Abraham died intestate on 7 January 1994 survived by his son Braulio. Abraham's older son Carlos died on 14 February 1990. Danilo who claims to be an adulterous child of Carlos intervenes in the proceedings for the settlement of the estate of Abraham in representation of Carlos. Danilo was legally adopted on 17 March 1970 by Carlos with the consent of the " latter's wife.

Paternity & Filiation; Artificial Insemination; Formalities(2006) Ed and Beth have been married for 20 years without children. Desirous to have a baby, they consulted Dr. Jun Canlas, a , prominent medical specialist on human fertility. He advised Beth to undergo artificial insemination. It was found that Ed’s sperm count was inadequate to induce pregnancy Hence, the couple looked for a willing donor. Andy the brother of Ed, readily consented to donate his

1. Under the Family Code, how may an illegitimate filiation be proved? Explain. 2. As lawyer for Danilo, do you have to prove Danilo's illegitimate filiation? Explain. 3. Can Danilo inherit from Abraham in representation of his father Carlos? Explain. SUGGESTED ANSWER:

1. Under Art. 172 in relation to Art. 173 andArt. 175 of the FC, the filiation of illegitimate children may be established

CIVIL LAW Answers to the BAR as Arranged by Topics (Year 1990-2006)

in the same way and by the same evidence as legitimate children. Art. 172 provides that the filiation of legitimate children is established by any of the following: (1) the record of birth appearing in the civil register or a final Judgment; or (2) an admission of legitimate filiation in a public document or a private handwritten instrument and signed by the parent concerned. In the absence of the foregoing evidence, the legitimate filiation shall be proved by: (1) the open and continuous possession of the status of a legitimate child; or (2) any other means allowed by the Rules of Court and special laws. SUGGESTED ANSWER:

2. No. Since Danilo has already been adopted by Carlos, he ceased to be an illegitimate child. An adopted child acquires all the rights of a legitimate child under Art, 189 of the FC. SUGGESTED ANSWER:

3. No, he cannot. Danilo cannot represent Carlos as the latter's adopted child in the inheritance of Abraham because adoption did not make Danilo a legitimate grandchild of Abraham. Adoption is personal between Carlos and Danilo. He cannot also represent Carlos as the latter's illegitimate child because in such case he is barred by Art. 992 of the NCC from inheriting from his illegitimate grandfather Abraham. ALTERNATIVE ANSWER:

An adopted child's successional rights do not include the right to represent his deceased adopter in the inheritance of the latter's legitimate parent, in view of Art. 973 which provides that in order that representation may take place, the representative must himself be capable of succeeding the decedent. Adoption by itself did not render Danilo an heir of the adopter's legitimate parent. Neither does his being a grandchild of Abraham render him an heir of the latter because as an illegitimate child of Carlos, who was a legitimate child of Abraham, Danilo is incapable of succeeding Abraham under Art. 992 of the Code. Paternity & Filiation; Recognition of illegitimate Child (2005) Steve was married to Linda, with whom he had a daughter, Tintin. Steve fathered a son with Dina, his secretary of 20 years, whom Dina named Joey, born on September 20, 1981. Joey's birth certificate did not indicate the father's name. Steve died on August 13, 1993, while Linda died on December 3, 1993, leaving their legitimate daughter, Tintin, as sole heir. On May 16, 1994, Dina filed a case on behalf of Joey, praying that the latter be declared an acknowledged illegitimate son of Steve and that Joey be given his share in Steve's estate, which is now being solely held by Tintin. Tintin put up the defense that an action for recognition shall only be filed during the lifetime of the presumed parents and that the exceptions under Article 285 of the Civil Code do not apply to him since the said article has been repealed by the Family Code. In any case, according to Tintin, Joey's birth certificate does not show that Steve is his father. a) Does Joey have a cause of action against Tintin for recognition and partition? Explain. (2%) SUGGESTED ANSWER: No, Joey does not have a cause of action against Tintin for recognition and partition. Under Article 175 of the Family Code, as a general rule, an action for compulsory

recognition of an illegitimate child can be brought at any time during the lifetime of the child. However, if the action is based on "open and continuous possession of the status of an illegitimate child, the same can be filed during the lifetime of the putative father."

In the present case, the action for compulsory recognition was filed by Joey's mother, Dina, on May 16,1994, after the death of Steve, the putative father. The action will prosper if Joey can present his birth certificate that bears the signature of his putative father. However, the facts clearly state that the birth certificate of Joey did not indicate the father's name. A birth certificate not signed by the alleged father cannot be taken as a record of birth to prove recognition of the child, nor can said birth certificate be taken as a recognition in a public instrument. (Reyes v. Court of Appeals, G.R. No. 39537, March 19, 1985) Consequently, the action filed by Joey's mother has already prescribed. b) Are the defenses set up by Tintin tenable? Explain. (2%) SUGGESTED ANSWER:

Yes, the defenses of Tintin are tenable. In Tayag v. Court of Appeals (G.R. No. 95229, June 9,1992), a complaint to compel recognition of an illegitimate child was brought before effectivity of the Family Code by the mother of a minor child based on "open and continuous possession of the status of an illegitimate child." The Supreme Court held that the right of action of the minor child has been vested by the filing of the complaint in court under the regime of the Civil Code and prior to the effectivity of the Family Code. The ruling in Tayag v. Court of Appeals finds no application in the instant case. Although the child was born before the effectivity of the Family Code, the complaint was filed after its effectivity. Hence, Article 175 of the Family Code should apply and not Article 285 of the Civil Code. c) Supposing that Joey died during the pendency of the action, should the action be dismissed? Explain. (2%) SUGGESTED ANSWER:

If Joey died during the pendency of the action, the action should still be dismissed because the right of Joey or his heirs to file the action has already prescribed. (Art. 175, Family Code) Paternity & Filiation; Rights of Legitimate Children (1990) B and G (college students, both single and not disqualified to marry each other) had a romantic affair, G was seven months in the family way as of the graduation of B. Right after graduation B went home to Cebu City. Unknown to G, B had a commitment to C (his childhood sweetheart) to marry her after getting his college degree. Two weeks after B marriage in Cebu City, G gave birth to a son E in Metro Manila. After ten years of married life in Cebu, B became a widower by the sudden death of C in a plane crash. Out of the union of B and C, two children, X and Y were born. Unknown to C while on weekend trips to Manila during the last 5 years of their marriage, B invariably visited G and lived at her residence and as a result of which, they renewed their relationship. A baby girl F was born to B and G two years

CIVIL LAW Answers to the BAR as Arranged by Topics (Year 1990-2006)

before the death of C. Bringing his family later to Manila, B finally married G. Recently. G died. What are the rights of B's four children: X and Y of his first marriage; and E and F, his children with G? Explain your answer. SUGGESTED ANSWER:

Under the facts stated, X and Y are legitimate children of B and C. E is the legitimate children of B and G. E is the legitimated child of B&G. F is the illegitimate child of B and C. As legitimate children of B and C, X and Y have the following rights: 1) To bear the surnames of the father and the mother, in conformity with the provisions of the Civil Code on Surnames; 2) To receive support from their parents, their ascendants, and in proper cases, their brothers and sisters, in conformity with the provisions of the Family Code on Support; and 3) To be entitled to the legitime and other successional rights granted to them by the Civil Code. (Article 174, Family Code). E is the legitimated child of B and G. Under Art. 177 of the Family Code, only children conceived and born outside of wedlock of parents who, at the time of the conception of the former, were not disqualified by any impediment to marry each other may be legitimated. E will have the same rights as X and Y. F is the illegitimate child of B and G. F has the right to use the surname of G, her mother, and is entitled to support as well as the legitime consisting of 1/2 of that of each of X, Y and E. (Article 176, Family Code) Presumptive Legitime (1999) What do you understand by "presumptive legitime", in what case or cases must the parent deliver such legitime to the children, and what are the legal effects in each case if the parent fails to do so? (5%) SUGGESTED ANSWER:

PRESUMPTIVE LEGITIME is not defined in the law. Its definition must have been taken from Act 2710, the Old Divorce Law, which required the delivery to the legitimate children of "the equivalent of what would have been due to them as their legal portion if said spouse had died intestate immediately after the dissolution of the community of property." As used in the Family Code, presumptive legitime is understood as the equivalent of the legitimate children's legitimes assuming that the spouses had died immediately after the dissolution of the community of property. Presumptive legitime is required to be delivered to the common children of the spouses when the marriage is annulled or declared void ab initio and possibly, when the conjugal partnership or absolute community is dissolved as in the case of legal separation. Failure of the parents to deliver the presumptive legitime will make their subsequent marriage null and void under Article 53 of the Family Code. Property Relations; Absolute Community (1994)

Paulita left the conjugal home because of the excessive drinking of her husband, Alberto. Paulita, out of her own endeavor, was able to buy a parcel of land which she was able to register under her name with the addendum "widow." She also acquired stocks in a listed corporation registered in her name. Paulita sold the parcel of land to Rafael, who first examined the original of the transfer certificate of title. 1) Has Alberto the right to share in the shares of stock acquired by Paulita? 2) Can Alberto recover the land from Rafael? SUGGESTED ANSWER:

1. a) Yes. The Family Code provides that all property acquired during the marriage, whether the acquisition appears to have been made, contracted or registered in the name of one or both spouses, is presumed to be absolute community property unless the contrary is proved. b) Yes. The shares are presumed to be absolute community property having been acquired during the marriage despite the fact that those shares were registered only in her name. Alberto's right to claim his share will only arise, however, at dissolution. c) The presumption is still that the shares of stock are owned in common. Hence, they will form part of the absolute community or the conjugal partnership depending on what the property Relations is. d) Since Paulita acquired the shares of stock by onerous title during the marriage, these are part of the conjugal or absolute community property, as the case maybe (depending on whether the marriage was celebrated prior to. or after, the effectivity of the Family Code). Her physical separation from her husband did not dissolve the community of property. Hence, the husband has a right to share in the shares of stock. SUGGESTED ANSWER:

2) a) Under a community of property, whether absolute or relative, the disposition of property belonging to such community is void if done by just one spouse without the consent of the other or authority of the proper court. However, the land was registered in the name of Paulita as "widow". Hence, the buyer has the right to rely upon what appears in the record of the Register of Deeds and should, consequently, be protected. Alberto cannot recover the land from Rafael but would have the right of recourse against his wife b) The parcel of land is absolute community property having been acquired during the marriage and through Paulita's industry despite the registration being only in the name of Paulita. The land being community property, its sale to Rafael without the consent of Alberto is void. However, since the land is registered in the name of Paulita as widow, there is nothing in the title which would raise a suspicion for Rafael to make inquiry. He, therefore, is an innocent purchaser for value from whom the land may no longer be recovered.

CIVIL LAW Answers to the BAR as Arranged by Topics (Year 1990-2006)

c) No. Rafael is an innocent purchaser in good faith who, upon relying on the correctness of the certificate of title, acquires rights which are to be protected by the courts.

Under the established principles of land registration law, the presumption is that the transferee of registered land is not aware of any defect in the title of the property he purchased. (See Tojonera v. Court of Appeals, 103 SCRA 467). Moreover, the person dealing with registered land may safely rely on the correctness of its certificate of title and the law will in no way oblige him to go behind the certificate to determine the condition of the property. [Director of Lands v. Abache, et al. 73 Phil. 606). No strong considerations of public policy have been presented which would lead the Court to reverse the established and sound doctrine that the buyer in good faith of a registered parcel of land does not have to look beyond the Torrens Title and search for any hidden defect or inchoate right which may later invalidate or diminish his right to what he purchased. (Lopez v. Court of Appeals. 189 SCRA 271) d) The parcel of land is absolute community property having been acquired during the marriage and through Paulita's industry despite registration only in the name of Paulita. The land being community property, its sale to Rafael without the consent of Alberto is void. Property Relations; Ante Nuptial Agreement (1995) Suppose Tirso and Tessie were married on 2 August 1988 without executing any ante nuptial agreement. One year after their marriage, Tirso while supervising the clearing of Tessie's inherited land upon the latter's request, accidentally found the treasure not in the new river bed but on the property of Tessie. To whom shall the treasure belong? Explain. SUGGESTED ANSWER:

Since Tirso and Tessie were married before the effectivity of the Family Code, their property relation is governed by conjugal partnership of gains. Under Art. 54 of the Civil Code, the share of the hidden treasure which the law awards to the finder or the proprietor belongs to the conjugal partnership of gains. The one-half share pertaining to Tessie as owner of the land, and the one-half share pertaining to Tirso as finder of the treasure, belong to the conjugal partnership of gains. Property Relations; Conjugal Partnership of Gains (1998) In 1970, Bob and Issa got married without executing a marriage settlement. In 1975, Bob inherited from his father a residential lot upon which, in 1981, he constructed a two-room bungalow with savings from his own earnings. At that time, the lot was worth P800.000.00 while the house, when finished cost P600,000.00. In 1989 Bob died, survived only by his wife, Issa and his mother, Sofia. Assuming that the relative values of both assets remained at the same proportion: 1. State whether Sofia can rightfully claim that the house and lot are not conjugal but exclusive property of her deceased son. [3%] 2. Will your answer be the same if Bob died before August 3, 1988? [2%] SUGGESTED ANSWER:

1. Since Bob and Sofia got married In 1970, then the law that governs is the New Civil Code (Persons), in which case, the property relations that should be applied as regards the property of the spouses is the system of relative community or conjugal partnership of gains (Article 119, Civil Code). By conjugal partnership of gains, the husband and the wife place in a common fund the fruits of their separate property and the income from their work or Industry (Article 142, Civil Code). In this instance, the lot inherited by Bob in 1975 is his own separate property, he having acquired the same by lucrative title (par. 2, Art. 148, Civil Code). However, the house constructed from his own savings in 1981 during the subsistence of his marriage with Issa is conjugal property and not exclusive property in accordance with the principle of "reverse accession" provided for in Art. 158, Civil Code. ANOTHER ANSWER:

1. Sofia, being her deceased son's legal heir concurring with his surviving spouse (Arts. 985, 986 and 997, Civil Code), may rightfully claim that the house and lot are not conjugal but belong to the hereditary estate of Bob. The value of the land being more than the cost of the improvement (Art. 120, Family Code). SUGGESTED ANSWER:

2. Yes, the answer would still be the same. Since Bob and Issa contracted their marriage way back in 1970, then the property relations that will govern is still the relative community or conjugal partnership of gains (Article 119, Civil Code). It will not matter if Bob died before or after August 3. 1988 (effectivity date of the Family Code], what matters is the date when the marriage was contracted. As Bob and Issa contracted their marriage way back in 1970. the property relation that governs them is still the conjugal partnership of gains. (Art. 158, Civil Code) ANOTHER ANSWER:

2. If Bob died be fore August 3, 1988. which is the date the Family Code took effect, the answer will not be the same. Art. 158. Civil Code, would then apply. The land would then be deemed conjugal, along with the house, since conjugal funds were used in constructing it. The husband's estate would be entitled to a reimbursement of the value of the land from conjugal partnership funds. Property Relations; Marriage Settlement; Conjugal Partnership of Gains (2005) Gabby and Mila got married at Lourdes Church in Quezon City on July 10, 1990. Prior thereto, they executed a marriage settlement whereby they agreed on the regime of conjugal partnership of gains. The marriage settlement was registered in the Register of Deeds of Manila, where Mila is a resident. In 1992, they jointly acquired a residential house and lot, as well as a condominium unit in Makati. In 1995, they decided to change their property relations to the regime of complete separation of property. Mila consented, as she was then engaged in a lucrative business. The spouses then signed a private document dissolving their conjugal partnership and agreeing on a complete separation of property.

CIVIL LAW Answers to the BAR as Arranged by Topics (Year 1990-2006)

Thereafter, Gabby acquired a mansion in Baguio City, and a 5-hectare agricultural land in Oriental Mindoro, which he registered exclusively in his name. In the year 2000, Mila's business venture failed, and her creditors sued her for P10,000,000.00. After obtaining a favorable judgment, the creditors sought to execute on the spouses' house and lot and condominium unit, as well as Gabby's mansion and agricultural land. a) Discuss the status of the first and the amended marriage settlements. (2%) SUGGESTED ANSWER:

The marriage settlement between Gabby and Mila adopting the regime of conjugal partnership of gains still subsists. It is not dissolved by the mere agreement of the spouses during the marriage. It is clear from Article 134 of the Family Code that in the absence of an express declaration in the marriage settlement, the separation of property between the spouses during the marriage shall not take place except by judicial order.

Bar Candidates Patricio Mahigugmaon and Rowena Amor decided to marry each other before the last day of the 1991 Bar Examinations. They agreed to execute a Marriage Settlement. Rowena herself prepared the document in her own handwriting. They agreed on the following: (1) a conjugal partnership of gains; (2) each donates to the other fifty percent (50%) of his/her present property, (3) Rowena shall administer the conjugal partnership property; and (4) neither may bring an action for the annulment or declaration of nullity of their marriage. Both signed the agreement in the presence of two (2) witnesses. They did not, however, acknowledge it before a notary public. A. As to form, is the Marriage Settlement valid? May it be registered in the registry of property? If not, what steps must be taken to make it registerable? B. Are the stipulations valid? C. If the Marriage Settlement is valid as to form and the above stipulations are likewise valid, does it now follow that said Marriage Settlement is valid and enforceable?

SUGGESTED ANSWER:

b) Discuss the effects of the said settlements on the properties acquired by the spouses. (2%) SUGGESTED ANSWER:

The regime of conjugal partnership of gains governs the properties acquired by the spouses. All the properties acquired by the spouses after the marriage belong to the conjugal partnership. Under Article 116 of the Family Code, even if Gabby registered the mansion and 5-hectare agricultural land exclusively in his name, still they are presumed to be conjugal properties, unless the contrary is proved. c) What properties may be held answerable for Mila's obligations? Explain. (2%) ALTERNATIVE ANSWER:

Since all the properties are conjugal, they can be held answerable for Mila's obligation if the obligation redounded to the benefit of the family. (Art. 121 [3], Family Code) However, the burden of proof lies with the creditor claiming against the properties. (Ayala Investment v. Court of Appeals, G.R. No. 118305, February 12,1998, reiterated in Homeowners Savings & Loan Bank v. Dailo, G.R. No. 153802, March 11, 2005) ALTERNATIVE ANSWER:

Except for the residential house which is the family home, all other properties of Gabby and Mila may be held answerable for Mila's obligation. Since the said properties are conjugal in nature, they can be held liable for debts and obligations contracted during the marriage to the extent that the family was benefited or where the debts were contracted by both spouses, or by one of them, with the consent of the other. A family home is a dwelling place of a person and his family. It confers upon a family the right to enjoy such property, which must remain with the person constituting it as a family home and his heirs. It cannot be seized by creditors except in special cases. (Taneo, Jr. v. Court of Appeals, G.R. No. 108532, March 9, 1999)

Property Relations; Marriage Settlements (1991)

A. Yes, it is valid as to form because it is in writing. No, it cannot be registered in the registry of property because it is not a public document. To make it registerable, it must be reformed and has to be notarized. SUGGESTED ANSWER:

B. Stipulations (1) and (3) are valid because they are not contrary to law. Stipulation (4) is void because it is contrary to law. Stipulation (2) is valid up to 1/5 of their respective present properties but void as to the excess (Art 84, Family Code). SUGGESTED ANSWER:

C. No. on September 15, 1991, the marriage settlement is not yet valid and enforceable until the celebration of the marriage, to take place before the last day of the 1991 bar Examinations. Property Relations; Marriage Settlements (1995) On 10 September 1988 Kevin, a 26-year old businessman, married Karla, a winsome lass of 18. Without the knowledge of their parents or legal guardians, Kevin and Karla entered into an ante-nuptial contract the day before their marriage stipulating that conjugal partnership of gains shall govern their marriage. At the time of their marriage Kevin's estate was worth 50 Million while Karla's was valued at 2 Million. A month after their marriage Kevin died in a freak helicopter accident. He left no will, no debts, no obligations. Surviving Kevin, aside from Karla, are his only relatives: his brother Luis and first cousin Lilia. 1) What property Relations governed the marriage of Kevin and Karla? Explain. 2) Determine the value of the estate of Kevin, 3) Who are Kevin's heirs? 4) How much is each of Kevin's heirs entitled to inherit? SUGGESTED ANSWER:

1. Since the marriage settlement was entered into without the consent and without the participation of the parents (they did not sign the document), the marriage settlement is invalid applying Art. 78, F.C. which provides that a minor

CIVIL LAW Answers to the BAR as Arranged by Topics (Year 1990-2006)

who according to law may contract marriage may also enter into marriage settlements but they shall be valid only if the person who may give consent to the marriage are made parties to the agreement. (Karla was still a minor at the time the marriage settlement was executed in September 1988 because the law, R.A. 6809, reducing the age of majority to 18 years took effect on 18 December 1989). The marriage settlement being void, the property Relations governing the marriage is, therefore, absolute community of property, under Art. 75 of the FC. 2. All the properties which Kevin and Karla owned at the time of marriage became community property which shall be divided equally between them at dissolution. Since Kevin owned 50 Million and Karla. 2 Million, at the time of the marriage, 52 Million constituted their community property. Upon the death of Kevin, the community was dissolved and half of the 52 Million or 26 Million is his share in the community. This 26 Million therefore is his estate. 3. Karla and Luis are the Intestate heirs of Kevin. 4. They are entitled to share the estate equally under Article 1001 of the NCC. Therefore. Karla gets 13 Million and Luis gets 13 Million. Property Relations; Obligations; Benefit of the Family (2000)

As finance officer of K and Co., Victorino arranged a loan of P5 Million from PNB for the corporation. However, he was required by the bank to sign a Continuing Surety Agreement to secure the repayment of the loan. The corporation failed to pay the loan, and the bank obtained a judgment against it and Victorino, jointly and severally. To enforce the judgment, the sheriff levied on a farm owned by the conjugal partnership of Victorino and his wife Elsa. Is the levy proper or not? (3%) SUGGESTED ANSWER:

The levy is not proper there being no showing that the surety agreement executed by the husband redounded to the benefit of the family. An obligation contracted by the husband alone is chargeable against the conjugal partnership only when it was contracted for the benefit of the family. When the obligation was contracted on behalf of the family business the law presumes that such obligation will redound to the benefit of the family. However, when the obligation was to guarantee the debt of a third party, as in the problem, the obligation is presumed for the benefit of the third party, not the family. Hence, for the obligation under the surety agreement to be chargeable against the partnership it must be proven that the family was benefited and that the benefit was a direct result of such agreement, (Ayala Investment v. Ching, 286 SCRA 272)

Property Relations; Unions without Marriage (1992) In 1989, Rico, then a widower forty (40) years of age, cohabited with Cora, a widow thirty (30) years of age. While living together, they acquired from their combined earnings a parcel of riceland. After Rico and Cora separated, Rico lived together with Mabel, a maiden sixteen (16) years of age. While living

together, Rico was a salaried employee and Mabel kept house for Rico and did full-time household chores for him. During their cohabitation, a parcel of coconut land was acquired by Rico from his savings. After living together for one (1) year, Rico and Mabel separated. Rico then met and married Letty, a single woman twenty-six (26) years of age. During the marriage of Rico and Letty, Letty bought a mango orchard out of her own personal earnings. a) Who would own the riceland, and what property Relations governs the ownership? Explain. b) Who would own the coconut land, and what property Relations governs the ownership? Explain. c) Who would own the mango orchard, and what property Relations governs the ownership? Explain.

SUGGESTED ANSWER:

(a) Rico and Cora are the co-owners of the riceland. The Relations is that of co-ownership (Art. 147, Family Code, first paragraph).

(Optional Addendum: However, after Rico's marriage to Letty, the half interest of Rico in the riceland will then become absolute community property of Rico and Letty.)

(b) Rico is the exclusive owner of the coconut land. The Relations is a sole/single proprietorship (Art. 148. Family Code, first paragraph is applicable, and not Art. 147 Family Code).

(Optional Addendum: However, after Rico's marriage to Letty, the coconut land of Rico will then become absolute community property of Rico and Letty.)

(c) Rico and Letty are the co-owners. The Relations is the Absolute Community of Property (Arts, 75,90and9l, Family Code). Property Relations; Unions without Marriage (1997) Luis and Rizza, both 26 years of age and single, live exclusively with each other as husband and wife without the benefit of marriage, Luis is gainfully employed, Rizza is not employed, stays at home, and takes charge of the household chores. After living together for a little over twenty years, Luis was able to save from his salary earnings during that period the amount of P200,000.00 presently deposited in a bank. A house and lot worth P500,000.00 was recently purchased for the same amount by the couple. Of the P500.000.00 used by the common-law spouses to purchase the property, P200.000.00 had come from the sale of palay harvested from the hacienda owned by Luis and P300,000.00 from the rentals of a building belonging to Rizza. In fine, the sum of P500.000.00 had been part of the fruits received during the period of cohabitation from their separate property, a car worth P100.000.00. being used by the common-law spouses, was donated Just months ago to Rizza by her parents. Luis and Rizza now decide to terminate their cohabitation, and they ask you to give them your legal advice on the following:

(a) How, under the law should the bank deposit of P200,000.00 the house and lot valued at P500.000.00 and the car worth P100.000.00 be allocated to them?

Page 41 of 119

CIVIL LAW Answers to the BAR as Arranged by Topics (Year 1990-2006)

(b) What would your answer be (to the above question) had Luis and Rizza been living together all the time, ie., since twenty years ago, under a valid marriage? SUGGESTED ANSWER:

a) Art. 147 of the Family Code provides in part that when a man and a woman who are capacitated to marry each other, live exclusively with each other as husband and wife without the benefit of marriage or under a void marriage, their wages and salaries shall be owned by them in equal shares and the property acquired by both of them through their work or industry shall be governed by the rules of coownership. In the absence of proof to the contrary, properties acquired while they lived together shall be presumed to have been obtained by their Joint efforts, worker Industry, and shall be owned by them in equal shares. A party who did not participate in the acquisition by the other party of any property shall be deemed to have contributed jointly in the acquisition thereof if the former's efforts consisted in the care and maintenance of the family and of the household. Thus: 1) the wages and salaries of Luis in the amount of P200,000.00 shall be divided equally between Luis and Rizza. 2) the house and lot valued at P500.000.00 having been acquired by both of them through work or industry shall be divided between them in proportion to their respective contribution, in consonance with the rules on co-ownership. Hence, Luis gets 2\5 while Rizza gets 3\5 of P500.000.00. 3) the car worth P100,000.00 shall be exclusively owned by Rizza, the same having been donated to her by her parents.

SUGGESTED ANSWER:

(b) The property relations between Luis and Rizza, their marriage having been celebrated 20 years ago (under the Civil Code) shall be governed by the conjugal partnership of gains, under which the husband and wife place in a common fund the proceeds, products, fruits and income from their separate properties and those acquired by either or both spouses through their efforts or by chance, and upon dissolution of the marriage or of the partnership, the net gains or benefits obtained by either or both spouse shall be divided equally between them (Art. 142. Civil Code). Thus: 1) The salary of Luis deposited in the bank in the amount of P200.000.00 and the house and lot valued at P500,000.00 shall be divided equally between Luis and Rizza. 2) However, the car worth P100.000,00 donated to Rizza by her parents shall be considered to her own paraphernal property, having been acquired by lucrative title (par. 2, Art. 148, Civil Code).

Property Relations; Unions without Marriage (2000) For five years since 1989, Tony, a bank Vice-president, and Susan, an entertainer, lived together as husband and wife without the benefit of marriage although they were capacitated to many each other. Since Tony's salary was more than enough for their needs, Susan stopped working and merely "kept house". During that period, Tony was able to buy a lot and house in a plush subdivision. However, after five years, Tony and Susan decided to separate.

a) Who will be entitled to the house and lot? (3%) SUGGESTED ANSWER:

Tony and Susan are entitled to the house and lot as coowners in equal shares. Under Article 147 of the Family Code, when a man and a woman who are capacitated to marry each other lived exclusively with each other as husband and wife, the property acquired during their cohabitation are presumed to have been obtained by their joint efforts, work or industry and shall be owned by them in equal shares. This is true even though the efforts of one of them consisted merely in his or her care and maintenance of the family and of the household. b) Would it make any difference if Tony could not marry Susan because he was previously married to Alice from whom he is legally separated? (2%) SUGGESTED ANSWER:

Yes, it would make a difference. Under Article 148 of the Family Code, when the parties to the cohabitation could not marry each other because of an impediment, only those properties acquired by both of them through their actual joint contribution of money, property, or Industry shall be owned by them in common in proportion to their respective contributions. The efforts of one of the parties in maintaining the family and household are not considered adequate contribution in the acquisition of the properties. Since Susan did not contribute to the acquisition of the house and lot, she has no share therein. If Tony cohabited with Susan after his legal separation from Alice, the house and lot is his exclusive property. If he cohabited with Susan before his legal separation from Alice, the house and lot belongs to his community or partnership with Alice.

SUCCESSION Amount of Successional Rights (2004) Mr. XT and Mrs. YT have been married for 20 years. Suppose the wife, YT, died childless, survived only by her husband, XT. What would be the share of XT from her estate as inheritance? Why? Explain. (5%) SUGGESTED ANSWER:

Under the Civil Code, the widow or widower is a legal and compulsory heir of the deceased spouse. If the widow is the only surviving heir, there being no legitimate ascendants, descendants, brothers, and sisters, nephews and nieces, she gets the entire estate. Barrier between illegitimate & legitimate relatives (1993) A is the acknowledged natural child of B who died when A was already 22 years old. When B's full blood brother, C, died he (C) was survived by his widow and four children of his other brother D. Claiming that he is entitled to inherit from his father's brother C. A brought suit to obtain his share in the estate of C. Will his action prosper? SUGGESTED ANSWER:

No, the action of A will not prosper. On the premise that B, C and D are legitimate brothers, as an illegitimate child of B, A cannot inherit in intestacy from C who is a legitimate brother of B. Only the wife of C in her own right and the

CIVIL LAW Answers to the BAR as Arranged by Topics (Year 1990-2006)

legitimate relatives of C (i.e. the children of D as C's legitimate nephews inheriting as collateral relatives) can inherit in intestacy. (Arts. 992, 1001, 1OO5 and 975, Civil Code) ALTERNATIVE ANSWER:

The action of A will not prosper. Being an illegitimate, he is barred by Article 992 of the Civil Code from inheriting ab intestato from the legitimate relatives of his father. Barrier between illegitimate & legitimate relatives (1996) Cristina the illegitimate daughter of Jose and Maria, died intestate, without any descendant or ascendant. Her valuable estate is being claimed by Ana, the legitimate daughter of Jose, and Eduardo, the legitimate son of Maria. Is either, both, or neither of them entitled to inherit? Explain. SUGGESTED ANSWER:

Neither Ana nor Eduardo is entitled to inherit of ab intestato from Cristina. Both are legitimate relatives of Cristina's illegitimate parents and therefore they fall under the prohibition prescribed by Art. 992, NCC (Manuel v. Ferrer, 242 SCRA 477; Diaz v. Court of Appeals, 182 SCRA 427).

Collation (1993) Joaquin Reyes bought from Julio Cruz a residential lot of 300 square meters in Quezon City for which Joaquin paid Julio the amount of P300,000.00, When the deed was about to be prepared Joaquin told Julio that it be drawn in the name of Joaquina Roxas, his acknowledged natural child. Thus, the deed was so prepared and executed by Julio. Joaquina then built a house on the lot where she, her husband and children resided. Upon Joaquin's death, his legitimate children sought to recover possession and ownership of the lot, claiming that Joaquina Roxas was but a trustee of their father. Will the action against Joaquina Roxas prosper? SUGGESTED ANSWER:

Yes, because there is a presumed donation in favor of Joaquina under Art. 1448 of the Civil Code (De los Santos v. Reyes, 27 January 1992, 206 SCRA 437). However, the donation should be collated to the hereditary estate and the legitime of the other heirs should be preserved. ALTERNATIVE ANSWER:

Yes, the action against Joaquina Roxas will prosper, but only to the extent of the aliquot hereditary rights of the legitimate children as heirs. Joaquina will be entitled to retain her own share as an illegitimate child, (Arts. 1440 and 1453. Civil Code; Art. 176, F. C.) Disinheritance vs. Preterition (1993) Maria, to spite her husband Jorge, whom she suspected was having an affair with another woman, executed a will, unknown to him, bequeathing all the properties she inherited from her parents, to her sister Miguela. Upon her death, the will was presented for probate. Jorge opposed probate of the will on the ground that the will was executed by his wife without his knowledge, much less consent, and that it deprived him of his legitime. After all, he had given her no cause for disinheritance, added Jorge in his opposition.

How will you rule on Jorge's opposition to the probate of Maria's will. If you were the Judge?

SUGGESTED ANSWER:

As Judge, I shall rule as follows: Jorge's opposition should be sustained in part and denied in part. Jorge's omission as spouse of Maria is not preterition of a compulsory heir in the direct line. Hence, Art. 854 of the Civil Code does not apply, and the institution of Miguela as heir is valid, but only to the extent of the free portion of one-half. Jorge is still entitled to one-half of the estate as his legitime. (Art. 1001, Civil Code) ALTERNATIVE ANSWERS:

a) As Judge, I shall rule as follows: Jorge's opposition should be sustained in part and denied in part. This is a case of ineffective disinheritance under Art, 918 of the Civil Code, because the omission of the compulsory heir Jorge by Maria was intentional. Consequently, the institution of Miguela as heir is void only insofar as the legitime of Jorge is prejudiced. Accordingly, Jorge is entitled to his legitime of one-half of the estate, and Miguela gets the other half. b) As Judge, I shall rule as follows: Jorge's opposition should be sustained. This is a case of preterition under Article 854 Civil Code, the result of the omission of Jorge as compulsory heir having the same right equivalent to a legitimate child "in the direct line" is that total intestacy will arise, and Jorge will inherit the entire estate. c) As Judge, I shall rule as follows: the opposition should be denied since it is predicated upon causes not recognized by law as grounds for disallowance of a wll, to wit: 1 that the will was made without his knowledge; 2 that the will was made without his consent; and 3 that it has the effect of depriving him of his legitime, which is a ground that goes into the intrinsic validity of the will and need not be resolved during the probate proceedings. However, the opposition may be entertained for, the purpose of securing to the husband his right to the legitime on the theory that the will constitutes an ineffective disinheritance under Art. 918 of the Civil Code, d) As Judge, I shall rule as follows: Jorge is entitled to receive his legitime from the estate of his wife. He was not disinherited in the will even assuming that he gave ground for disinheritance, hence, he is still entitled to his legitime. Jorge, however, cannot receive anything from the free portion. He cannot claim preterition as he is not a compulsory heir in the direct line. There being no preterition, the institution of the sister was valid and the only right of Jorge is to claim his legitime. Disinheritance; Ineffective (1999) Mr. Palma, widower, has three daughters D, D-l and D-2. He executes a Will disinheriting D because she married a man he did not like, and instituting daughters D-1 and D-2 as his heirs to his entire estate of P 1,000,000.00, Upon Mr, Palma's death, how should his estate be divided? Explain. (5%) SUGGESTED ANSWER:

CIVIL LAW Answers to the BAR as Arranged by Topics (Year 1990-2006)

This is a case of ineffective disinheritance because marrying a man that the father did not approve of is not a ground for disinheriting D. Therefore, the institution of D-l and D-2 shall be annulled insofar as it prejudices the legitime of D, and the institution of D-l and D-2 shall only apply on the free portion in the amount of P500,000.00. Therefore, D, D-l and D-2 will get their legitimes of P500.000.00 divided into three equal parts and D-l and D-2 will get a reduced testamentary disposition of P250,000.00 each. Hence, the shares will be: D P166,666.66 D-l P166,666.66 + P250.000.00 D-2 P166,666.66 + P250,000.00

Disinheritance; Ineffective; Preterition (2000) In his last will and testament, Lamberto 1) disinherits his daughter Wilma because "she is disrespectful towards me and raises her voice talking to me", 2) omits entirely his spouse Elvira, 3) leaves a legacy of P100,000.00 to his mistress Rosa and P50,000.00 to his driver Ernie and 4) institutes his son Baldo as his sole heir. How will you distribute his estate of P1,000,000.00? (5%) SUGGESTED ANSWER:

The disinheritance of Wilma was ineffective because the ground relied upon by the testator does not constitute maltreatment under Article 919(6) of the New Civil Code. Hence, the testamentary provisions in the will shall be annulled but only to the extent that her legitime was impaired. The total omission of Elvira does not constitute preterition because she is not a compulsory heir in the direct line. Only compulsory heirs in the direct line may be the subject of preterition. Not having been preterited, she will be entitled only to her legitime. The legacy in favor of Rosa is void under Article 1028 for being in consideration of her adulterous relation with the testator. She is, therefore, disqualified to receive the legacy of 100,000 pesos. The legacy of 50,000 pesos in favor of Ernie is not inofficious not having exceeded the free portion. Hence, he shall be entitled to receive it. The institution of Baldo, which applies only to the free portion, shall be respected. In sum, the estate of Lamberto will be distributed as follows: Baldo-----------------450,000 Wilma---------------250,000 Elvira-----------------250,000 Ernie-----------------50,000 1,000,000 ALTERNATIVE ANSWER: The disinheritance of Wilma was effective because disrespect of, and raising of voice to, her father constitute maltreatment under Article 919(6) of the New Civil Code. She is, therefore, not entitled to inherit anything. Her inheritance will go to the other legal heirs. The total omission of Elvira is not preterition because she is not a compulsory heir in the direct line. She will receive only her legitime. The legacy in favor of Rosa is void under Article

1028 for being in consideration of her adulterous relation with the testator. She is, therefore, disqualified to receive the legacy. Ernie will receive the legacy in his favor because it is not inofficious. The institution of Baldo, which applies only to the free portion, will be respected. In sum, the estate of Lamberto shall be distributed as follows:

Heir Legitime Legacy Institution TOTAL Baldo 500,000 200.000 700,000 Elvira 250,000 250,000 Ernie 50,000 50,000 TOTAL 750,000 50,000 200,000 1,000,000

ANOTHER ALTERNATIVE ANSWER:

Same answer as the first Alternative Answer except as to distribution. Justice Jurado solved this problem differently. In his opinion, the legitime of the heir who was disinherited is distributed among the other compulsory heirs in proportion to their respective legitimes, while his share in the intestate portion. If any, is distributed among the other legal heirs by accretion under Article 1018 of the NCC in proportion to their respective intestate shares. In sum the distribution shall be as follows: Heir

Distribution Legitime of Wilma’s Legitime

Legacy Institution

TOTAL

Bald o 250,0000 125,000 200,000 575,000 Wil ma (250.000) Elvi 250,000 125.000 375.000 ra Heirs; Intestate Heirs; Reserva Troncal (1995) Erni and Irma, Filipinos, both50,000 Isidro 18 years of age, 50.000 were e passengers of Flight No. 317 of Oriental Airlines. The plane TO 50,0 200,00 they was of Philippine registry. route from TALboarded 500,000 250,000 00 0 While en 1,000,000

Manila to Greece some passengers hijacked the plane, held the chief pilot hostage at the cockpit and ordered him to fly instead to Libya. During the hijacking Isidro suffered a heart attack and was on the verge of death. Since Irma was already eight months pregnant by Isidro, she pleaded to the hijackers to allow the assistant pilot to solemnize her marriage with Isidro. Soon after the marriage, Isidro expired. As the plane landed in Libya Irma gave birth. However, the baby died a few minutes after complete delivery. Back in the Philippines Irma Immediately filed a claim for inheritance. The parents of Isidro opposed her claim contending that the marriage between her and Isidro was void ab initio on the following grounds: (a) they had not given their consent to the marriage of their son; (b) there was no marriage license; (c) the solemnizing officer had no authority to perform the marriage; and, (d) the solemnizing officer did not file an affidavit of marriage with the proper civil registrar. 2. Does Irma have any successional rights at all? Discuss fully. SUGGESTED ANSWER:

2. Irma succeeded to the estate of Isidro as his surviving spouse to the estate of her legitimate child. When Isidro

CIVIL LAW Answers to the BAR as Arranged by Topics (Year 1990-2006) (c) X = 1/2 by representation of B representation of C

died, he was succeeded by his surviving wife Irma, and his legitimate unborn child. They divided the estate equally between them, the child excluding the parents of Isidro. An unborn child is considered born for all purposes favorable to it provided it is born later. The child was considered born because, having an intra-uterine life of more than seven months, it lived for a few minutes after its complete delivery. It was legitimate because it was born within the valid marriage of the parents. Succession is favorable to it. When the child died, Irma inherited the share of the child. However, the share of the child in the hands of Irma is subject to reserva troncal for the benefit of the relatives of the child within the third degree of consanguinity and who belong to the line of Isidro. ALTERNATIVE ANSWER:

If the marriage is void. Irma has no successional rights with respect to Isidro but she would have successional rights with respect to the child. Heirs; Intestate Heirs; Shares (2003) Luis was survived by two legitimate children, two illegitimate children, his parents, and two brothers. He left an estate of P1 million. Luis died intestate. Who are his intestate heirs, and how much is the share of each in his estate? SUGGESTED ANSWER:

The intestate heirs are the two (2) legitimate children and the two (2) illegitimate children. In intestacy the estate of the decedent is divided among the legitimate and illegitimate children such that the share of each illegitimate child is one -half the share of each legitimate child. Their share are : For each legitimate child – P333,333.33 For each illegitimate child – P166,666.66 (Article 983, New Civil Code; Article 176, Family Code)

Intestate Succession (1992) F had three (3) legitimate children: A, B, and C. B has one (1) legitimate child X. C has two (2) legitimate children: Y and Z. F and A rode together in a car and perished together at the same time in a vehicular accident, F and A died, each of them leaving substantial estates in intestacy. a) Who are the intestate heirs of F? What are their respective fractional shares? b) Who are the intestate heirs of A? What are their respective fractional shares? c) If B and C both predeceased F, who are F’s intestate heirs? What are their respective fractional shares? Do they inherit in their own right or by representation? Explain your answer. d) If B and C both repudiated their shares in the estate of F who are F's intestate heirs? What are their respective fractional shares? Do they inherit in their own right or by representation? Explain your answer, SUGGESTED ANSWER:

(a) B = 1/2 (b) B = 1/2 Z = 1/4 by representation of C C= 1/2 Article 982

of the Civil Code provides that grandchildren inherit by right of representation.

C=l/2 Y = 1/4 by

(d) X - 1/3 in his own right Y- 1/3 in his own right 2 - 1/3 in his own right Article 977 of the Civil Code provides that heirs who repudiate their share cannot be represented. Intestate Succession (1997) "T" died intestate on 1 September 1997.He was survived by M (his mother), W (his widow), A and B (his legitimate children), C (his grandson, being the legitimate son of B), D (his other grandson, being the son of E who was a legitimate son of, and who predeceased, "T"), and F (his grandson, being the son of G, a legitimate son who repudiated the inheritance from "T"). His distributable net estate is P120.000.00. How should this amount be shared in intestacy among the surviving heirs? SUGGESTED ANSWER:

The legal heirs are A, B, D, and W. C is excluded by B who is still alive. D inherits in representation of E who predeceased. F is excluded because of the repudiation of G, the predecessor. M is excluded by the legitimate children of T. The answer may be premised on two theories: the Theory of Exclusion and the Theory of Concurrence. Under the Theory of Exclusion the legitimes of the heirs are accorded them and the free portion will be given exclusively to the legitimate descendants. Hence under the Exclusion Theory: A will get P20.000.00. and P 13.333.33 (1/3 of the free portion) B will get P 20,000.00. and P13. 333.33 (1/3 of the free portion) D will get P20.000.00. and P13. 333.33 (1/3 of the free portion)

W, the widow is limited to the legitime of P20.000.00 Under the Theory of Concurrence. In addition to their legitimes, the heirs of A, B, D and W will be given equal shares in the free portions: A: P20.000.00 plus P10.000.00 (1 /4 of the free portion) B: P20,000.00 plus P10.000.00 (l/4 of the free portlon) C: P20,000.00 plus P10.000.00 (1/4 of the free portion) W: P20,000.00 plus P10,000.00 (l/4 of the free portion) Alternative Answer: Shares in Intestacy T - decedent Estate: P120.000.00 Survived by: M - Mother............................None W Widow.............................P 30,000.00 A - Son.................................P 30,000.00 B - Son.................................P30.000.00 C - Grandson (son of B).............None D Grandson (son of E who predeceased T)................P 30,000.00 F - Grandson (son of G who repudiated the Inheritance from"T").......................None

Explanation: a) The mother (M) cannot inherit from T because under Art. 985 the ascendants shall inherit in default of legitimate children and descendants of the deceased. b) The widow's share is P30.000.00 because under Art, 996 it states that if the widow or widower and legitimate children or descendants are left, the surviving

CIVIL LAW Answers to the BAR as Arranged by Topics (Year 1990-2006)

spouse has in the succession the same share as that of each of the children, c) C has no share because his father is still alive hence succession by representation shall not apply (Art. 975). d) D inherits P30.000 which is the share of his father E who predeceased T by virtue of Art. 981 on the right of representation. e) F has no share because his father G repudiated the inheritance. Under Article 977 heirs who repudiate their share may not be represented.

be set aside as Mario's conjugal share from the community

Intestate Succession (1998) Enrique died, leaving a net hereditary estate of P1.2 million. He is survived by his widow, three legitimate children, two legitimate grandchildren sired by a legitimate child who predeceased him, and two recognized illegitimate children. Distribute the estate in intestacy. [5%]

Intestate Succession (1999) Mr. and Mrs. Cruz, who are childless, met with a serious motor vehicle accident with Mr. Cruz at the wheel and Mrs. Cruz seated beside him, resulting in the instant death of Mr. Cruz. Mrs. Cruz was still alive when help came but she also died on the way to the hospital. The couple acquired properties worth One Million (P1,000,000.00) Pesos during their marriage, which are being claimed by the parents of both spouses in equal shares. Is the claim of both sets of parents valid and why? (3%)

SUGGESTED ANSWER:

Under the theory of Concurrence, the shares are as follows: A (legitimate child) = P200,000 B (legitimate child) = P200,000 C (legitimate child) = P200,000 D (legitimate child) = O (predeceased] E (legitimate child of D) = P100,000 by right of representation F (legitimate child of D) = P100,000 - by right of representation G (illegitimate child) = P100,000 - 1/2 share of the legitimate child H (illegitimate child) = P100,000 - 1/2 share of the legitimate child W (Widow) = P200.000 - same share as legitimate child ANOTHER ANSWER:

Under the theory of Exclusion the free portion (P300,000) is distributed only among the legitimate children and is given to them in addition to their legitime. All other Intestate heirs are entitled only to their respective legitimes. The distribution is as follows: Legitime Free Portion Total

A [legitimate child) P150.000 + P 75,000 - P225.000 B {legitimate child) P150.000 + P150.000 - P225.000 C (legitimate child) P150.000 + P 75.000 - P225.000 D (legitimate child) 0 0 0 E (legitimate child of D) P 75,000 + P35.500 - P112,500 F (legitimate child of D) P 75.000 + P 37.500 - P112,500 G (illegitimate child) P 75.000 0 -P 75,500 H (illegitimate child) P 75.000 0 - P 75,500 W (Widow) P150,000 0 -P150.000

Intestate Succession (1998) Tessie died survived by her husband Mario, and two nieces, Michelle and Jorelle, who are the legitimate children of an elder sister who had predeceased her. The only property she left behind was a house and lot worth two million pesos, which Tessie and her husband had acquired with the use of Mario's savings from his income as a doctor. How much of the property or its value, if any, may Michelle and Jorelle claim as their hereditary shares? [5%] SUGGESTED ANSWER: Article 1001 of the Civil Code provides, "Should brothers and sisters or their children survive with the widow or widower, the latter shall be entitled to one-half of the inheritance and the brothers and sisters or their children to the other half." Tessie's gross estate consists of a house and lot acquired during her marriage, making it part of the community property. Thus, one-half of the said property would have to

property. The other half, amounting to one million pesos, is her conjugal share (net estate), and should be distributed to her intestate heirs. Applying the above provision of law, Michelle and Jorelle, Tessie's nieces, are entitled to one-half of her conjugal share worth one million pesos, or 500,000 pesos, while the other one-half amounting to P500,000 will go to Mario, Tessie's surviving spouse. Michelle and Jorelle are then entitled to P250,000 pesos each as their hereditary share.

SUGGESTED ANSWER:

(a) No, the claim of both parents is not valid. When Mr. Cruz died, he was succeeded by his wife and his parents as his intestate heirs who will share his estate equally. His estate was 0.5 Million pesos which is his half share in the absolute community amounting to 1 Million Pesos. His wife, will, therefore, inherit O.25 Million Pesos and his parents will inherit 0.25 Million Pesos. When Mrs. Cruz died, she was succeeded by her parents as her intestate heirs. They will inherit all of her estate consisting of her 0.5 Million half share in the absolute community and her 0.25 Million inheritance from her husband, or a total of 0.750 Million Pesos. In sum, the parents of Mr. Cruz will inherit 250,000 Pesos while the parents of Mrs. Cruz will inherit 750,000 Pesos. Intestate Succession (2000) Eugenio died without issue, leaving several parcels of land in Bataan. He was survived by Antonio, his legitimate brother; Martina, the only daughter of his predeceased sister Mercedes; and five legitimate children of Joaquin, another predeceased brother. Shortly after Eugenio's death, Antonio also died, leaving three legitimate children. Subsequently, Martina, the children of Joaquin and the children of Antonio executed an extrajudicial settlement of the estate of Eugenio, dividing it among themselves. The succeeding year, a petition to annul the extrajudicial settlement was filed by Antero, an illegitimate son of Antonio, who claims he is entitled to share in the estate of Eugenio. The defendants filed a motion to dismiss on the ground that Antero is barred by Article 992 of the Civil Code from inheriting from the legitimate brother of his father. How will you resolve the motion? (5%) SUGGESTED ANSWER:

The motion to dismiss should be granted. Article 992 does not apply. Antero is not claiming any inheritance from Eugenio. He is claiming his share in the inheritance of his father consisting of his father's share in the inheritance of

CIVIL LAW Answers to the BAR as Arranged by Topics (Year 1990-2006) Eugenio (Dela Merced v. Dela Merced, Gr No. 126707, 25 5M inherited by Mrs. Luna from Mr. Luna will be inherited from her by her parents. February 1999). ALTERNATIVE ANSWER:

It depends. If Antero was not acknowledged by Antonio, the motion to dismiss should be granted because Antero is not a legal heir of Antonio. If Antero was acknowledged, the motion should be denied because Article 992 is not applicable. This is because Antero is claiming his inheritance from his illegitimate father, not from Eugenio.

However, if the child had intra-uterine life of less than 7 months, half of the estate of Mr. Luna, or 5M, will be inherited by the widow (Mrs. Luna), while the other half, or 5M, will be inherited by the parents of Mr. Luna. Upon the death of Mrs. Luna, her estate of 5M will be inherited by her own parents.

Intestate Succession; Reserva Troncal (1999) Mr. Luna died, leaving an estate of Ten Million (P1 0,000,000.00) Pesos. His widow gave birth to a child four months after Mr, Luna's death, but the child died five hours after birth. Two days after the child's death, the widow of Mr. Luna also died because she had suffered from difficult childbirth. The estate of Mr. Luna is now being claimed by his parents, and the parents of his widow. Who is entitled to Mr. Luna'a estate and why? (5%)

Legitime (1997) "X", the decedent, was survived by W (his widow). A (his son), B (a granddaughter, being the daughter of A) and C and D (the two acknowledged illegitimate children of the decedent). "X" died this year (1997) leaving a net estate of P180,000.00. All were willing to succeed, except A who repudiated the inheritance from his father, and they seek your legal advice on how much each can expect to receive as their respective shares in the distribution of the estate. Give your answer.

SUGGESTED ANSWER:

Half of the estate of Mr. Luna will go to the parents of Mrs. Luna as their inheritance from Mrs. Luna, while the other half will be inherited by the parents of Mr. Luna as the reservatarios of the reserved property inherited by Mrs. Luna from her child. When Mr. Luna died, his heirs were his wife and the unborn child. The unborn child inherited because the inheritance was favorable to it and it was born alive later though it lived only for five hours. Mrs. Luna inherited half of the 10 Million estate while the unborn child inherited the other half. When the child died, it was survived by its mother, Mrs. Luna. As the only heir, Mrs. Luna inherited, by operation of law, the estate of the child consisting of its 5 Million inheritance from Mr. Luna. In the hands of Mrs. Luna, what she inherited from her child was subject to reserva troncal for the benefit of the relatives of the child within the third degree of consanguinity and who belong to the family of Mr. Luna, the line where the property came from. When Mrs. Luna died, she was survived by her parents as her only heirs. Her parents will inherit her estate consisting of the 5 Million she inherited from Mr. Luna. The other 5 Million she inherited from her child will be delivered to the parents of Mr. Luna as beneficiaries of the reserved property. In sum, 5 Million Pesos of Mr. Luna's estate will go to the parents of Mrs. Luna, while the other 5 Million Pesos will go to the parents of Mr. Luna as reservatarios. ALTERNATIVE ANSWER: If the child had an intra-uterine life of not less than 7 months, it inherited from the father. In which case, the estate of 10M will be divided equally between the child and the widow as legal heirs. Upon the death of the child, its share of 5M shall go by operation of law to the mother, which shall be subject to reserva troncal. Under Art. 891, the reserva is in favor of relatives belonging to the paternal line and who are within 3 degrees from the child. The parents of Mr, Luna are entitled to the reserved portion which is 5M as they are 2 degrees related from child. The

SUGGESTED ANSWER:

The heirs are B, W, C and D. A inherits nothing because of his renunciation. B inherits a legitime of P90.000.00 as the nearest and only legitimate descendant, inheriting in his own right not by representation because of A's renunciation. W gets a legitime equivalent to one-half (1 / 2) that of B amounting to P45.000. C and D each gets a legitime equivalent to one-half (1/2) that of B amounting to P45.000.00 each. But since the total exceeds the entire estate, their legitimes would have to be reduced corresponding to P22.500.00 each (Art. 895. CC). The total of all of these amounts to P180.000.00. ALTERNATIVE ANSWER:

INTESTATE SUCCESSION ESTATE: P180,000.00

W- (widow gets 1/2 share) P90.000.00 (Art. 998) A- (son who repudiated his inheritance) None Art. 977) B - (Granddaughter) None C - (Acknowledged illegitimate child) P45.000.00 (Art.998) D - (Acknowledged illegitimate child) P45,000.00 (Art. 998) The acknowledged illegitimate child gets 1/2 of the share of each legitimate child.

Legitime; Compulsory Heirs (2003) Luis was survived by two legitimate children, two illegitimate children, his parents, and two brothers. He left an estate of P1 million. Who are the compulsory heirs of Luis, how much is the legitime of each, and how much is the free portion of his estate, if any? SUGGESTED ANSWER:

The compulsory heirs are the two legitimate children and the two illegitimate children. The parents are excluded by the legitimate children, while the brothers are not compulsory heirs at all. Their respective legitimate are: a) The legitime of the two (2) legitimate children is one half (1/2) of the estate (P500,000.00) to be divided between them equally, or P250,000.00 each. b) The legitimate of each illegitimate child is one-half (1/2) the legitime of each legitimate child or P125,000.00.

CIVIL LAW Answers to the BAR as Arranged by Topics (Year 1990-2006)

c) Since the total legitime of the compulsory heirs is P750,000.00, the balance of P250,000.00 is the free portion.

Legitime; Compulsory Heirs vs. Secondary Compulsory Heirs (2005) Emil, the testator, has three legitimate children, Tom, Henry and Warlito; a wife named Adette; parents named Pepe and Pilar; an illegitimate child, Ramon; brother, Mark; and a sister, Nanette. Since his wife Adette is well-off, he wants to leave to his illegitimate child as much of his estate as he can legally do. His estate has an aggregate net amount of Pl,200,000.00, and all the above-named relatives are still living. Emil now comes to you for advice in making a will. How will you distribute his estate according to his wishes without violating the law on testamentary succession? (5%) SUGGESTED ANSWER:

P600,000.00 — legitime to be divided equally between Tom, Henry and Warlito as the legitimate children. Each will be entitled to P200,000.00. (Art. 888, Civil Code) P100,000.00 -share of Ramon the illegitimate child. Equivalent to 1/2 of the share of each legitimate child. (Art. 176, Family Code) P200,000.00 — Adette the wife. Her share is equivalent to the share of one legitimate child. (Art. 892, par. 2, Civil Code)

Pepe and Pilar, the parents are only secondary compulsory heirs and they cannot inherit if the primary compulsory heirs (legitimate children) are alive. (Art. 887, par. 2, Civil Code) Brother Mark and sister Nanette are not compulsory heirs since they are not included in the enumeration under Article 887 of the Civil Code. The remaining balance of P300,000.00 is the free portion which can be given to the illegitimate child Ramon as an instituted heir. (Art. 914, Civil Code) If so given by the decedent, Ramon would receive a total of P400,000.00. Preterition (2001) Because her eldest son Juan had been pestering her for capital to start a business, Josefa gave him P100,000. Five years later, Josefa died, leaving a last will and testament in which she instituted only her four younger children as her sole heirs. At the time of her death, her only properly left was P900,000.00 in a bank. Juan opposed the will on the ground of preterition. How should Josefa's estate be divided among her heirs? State briefly the reason(s) for your answer. (5%) SUGGESTED ANSWER:

There was no preterition of the oldest son because the testatrix donated 100,000 pesos to him. This donation is considered an advance on the son's inheritance. There being no preterition, the institutions in the will shall be respected but the legitime of the oldest son has to be completed if he received less. After collating the donation of P100.000 to the remaining property of P900,000, the estate of the testatrix is P1,000,000. Of this amount, one-half or P500,000, is the

legitime of the legitimate children and it follows that the legitime of one legitimate child is P100,000. The legitime, therefore, of the oldest son is P100,000. However, since the donation given him was P100,000, he has already received in full his legitime and he will not receive anything anymore from the decedent. The remaining P900,000, therefore, shall go to the four younger children by institution in the will, to be divided equally among them. Each will receive P225,000. ALTERNATIVE ANSWER:

Assuming that the donation is valid as to form and substance, Juan cannot invoke preterition because he actually had received a donation inter vivos from the testatrix (III Tolentino 188,1992 ed.). He would only have a right to a completion of his legitime under Art. 906 of the Civil Code. The estate should be divided equally among the five children who will each receive P225,000.00 because the total hereditary estate, after collating the donation to Juan (Art. 1061, CC), would be P1 million. In the actual distribution of the net estate, Juan gets nothing while his siblings will get P225,000.00 each. Preterition; Compulsory Heir (1999) (a) Mr, Cruz, widower, has three legitimate children, A, B and C. He executed a Will instituting as his heirs to his estate of One Million (P1,000,000.00) Pesos his two children A and B, and his friend F. Upon his death, how should Mr. Cruz's estate be divided? Explain. (3%) (b) In the preceding question, suppose Mr. Cruz instituted his two children A and B as his heirs in his Will, but gave a legacy of P 100,000.00 to his friend F. How should the estate of Mr, Cruz be divided upon his death? Explain, (2%) SUGGESTED ANSWER:

(a) Assuming that the institution of A, B and F were to the entire estate, there was preterition of C since C is a compulsory heir in the direct line. The preterition will result in the total annulment of the institution of heirs. Therefore, the institution of A, B and F will be set aside and Mr. Cuz's estate will be divided, as in intestacy, equally among A, B and C as follows: A - P333,333.33; B - P333.333.33; and C P333,333.33. (b) On the same assumption as letter (a), there was preterition of C. Therefore, the institution of A and B is annulled but the legacy of P100.000.00 to F shall be respected for not being inofficious. Therefore, the remainder of P900.000.00 will be divided equally among A, B and C. Proceedings; Intestate Proceedings; Jurisdiction (2004) In his lifetime, a Pakistani citizen, ADIL, married three times under Pakistani law. When he died an old widower, he left behind six children, two sisters, three homes, and an estate worth at least 30 million pesos in the Philippines. He was born in Lahore but last resided in Cebu City, where he had a mansion and where two of his youngest children now live and work. Two of his oldest children are farmers in Sulu, while the two middle-aged children are employees in Zamboanga City. Finding that the deceased left no will, the youngest son wanted to file intestate proceedings before the Regional Trial Court of Cebu City. Two other siblings

CIVIL LAW Answers to the BAR as Arranged by Topics (Year 1990-2006)

objected, arguing that it should be in Jolo before a Shari’a court since his lands are in Sulu. But Adil’s sisters in Pakistan want the proceedings held in Lahore before a Pakistani court. Which court has jurisdiction and is the proper venue for the intestate proceedings? The law of which country shall govern succession to his estate? (5%)

mother, in favor of another sister, with their mother not only giving her authority thereto but even signing said deeds, there is a valid partition inter vivos between the mother and her children which cannot be revoked by the mother. Said deeds of sale are not contracts entered into with respect to future inheritance.

SUGGESTED ANSWER:

"It would be unjust for the mother to revoke the sales to a son and to execute a simulated sale in favor of a daughter who already benefited by the partition."

In so far as the properties of the decedent located in the Philippines are concerned, they are governed by Philippine law (Article 16, Civil Code). Under Philippine law, the proper venue for the settlement of the estate is the domicile of the decedent at the time of his death. Since the decedent last resided in Cebu City, that is the proper venue for the intestate settlement of his estate. However, the successional rights to the estate of ADIL are governed by Pakistani law, his national law, under Article 16 of the Civil Code. Succession; Death; Presumptive Legitime (1991) a) For purposes of succession, when is death deemed to occur or take place? b) May succession be conferred by contracts or acts inter vivos? Illustrate. c) Is there any law which allows the delivery to compulsory heirs of their presumptive legitimes during the lifetime of their parents? If so, in what instances? SUGGESTED ANSWER:

SUGGESTED ANSWER:

C. Yes, under Arts. 51 and 52 of the New Family Code. In case of legal separation, annulment of marriage, declaration of nullity of marriage and the automatic termination of a subsequent marriage by the reappearance of the absent spouse, the common or community property of the spouses shall be dissolved and liquidated. Art, 51. In said partition, the value of the presumptive legitimes of all common children, computed as of the date of the final judgment of the trial court, shall be delivered in cash, property or sound securities, unless the parties, by mutual agreement, judicially approved, had already provided for such matters. The children of their guardian, or the trustee of their property, may ask for the enforcement of the judgment.

A. Death as a fact is deemed to occur when it actually takes place. Death is presumed to take place in the circumstances under Arts. 390-391 of the Civil Code. The time of death is presumed to be at the expiration of the 10year period as prescribed by Article 390 and at the moment of disappearance under Article 391.

The delivery of the presumptive legitimes herein prescribed shall in no way prejudice the ultimate successional rights of the children accruing upon the death of either or both of the parents; but the value of the properties already received under the decree of annulment or absolute nullity shall be considered as advances on their legitime.

B. Under Art. 84 of the Family Code amending Art 130 of the Civil Code, contractual succession is no longer possible since the law now requires that donations of future property be governed by the provisions on the testamentary succession and formalities of wills.

Art. 52. The judgment of annulment or of absolute nullity of the marriage, the partition and distribution of the properties of the spouses, and the delivery of the children's presumptive legitimes shall be recorded in the appropriate civil registry and registries of property; otherwise, the same shall not affect third persons.

ALTERNATIVE ANSWER:

B. In the case of Coronado vs.CA(l91 SCRA81), it was ruled that no property passes under a will without its being probated, but may under Article 1058 of the Civil Code of 1898, be sustained as a partition by an act inter vivos [Many-Oy vs. CA 144SCRA33).

And in the case of Chavez vs, IAC 1191 SCRA211), it was ruled that while the law prohibits contracts upon future inheritance, the partition by the parent, as provided in Art. 1080 is a case expressly authorized by law. A person has two options in making a partition of his estate: either by an act inter vivos or by will. If the partition is by will, it is imperative that such partition must be executed in accordance with the provisions of the law on wills; if by an act inter vivos, such partition may even be oral or written, and need not be in the form of a will, provided the legitime is not prejudiced. "Where several sisters execute deeds of sale over their 1 /6 undivided share of the paraphernal property of their

Wills; Codicil; Institution of Heirs; Substitution of Heirs (2002) By virtue of a Codicil appended to his will, Theodore devised to Divino a tract of sugar land, with the obligation on the part of Divino or his heirs to deliver to Betina a specified volume of sugar per harvest during Betina’s lifetime. It is also stated in the Codicil that in the event the obligation is not fulfilled, Betina should immediately seize the property from Divino or latter’s heirs and turn it over to Theodore’s compulsory heirs. Divino failed to fulfill the obligation under the Codicil. Betina brings suit against Divino for the reversion of the tract of land. a) Distinguish between modal institution and substation of heirs. (3%) b) Distinguish between simple and fideicommissary substitution of heirs. (2%) c) Does Betina have a cause of action against Divino? Explain (5%) SUGGESTED ANSWER:

CIVIL LAW Answers to the BAR as Arranged by Topics (Year 1990-2006)

A. A MODAL INSTITUTION is the institution of an heir made for a certain purpose or cause (Arts. 871 and 882, NCC). SUBSTITUTION is the appointment of another heir so that he may enter into the inheritance in default of the heir originality instituted. (Art. 857, NCC).

B. In a SIMPLE SUBSTITUTION of heirs, the testator designates one or more persons to substitute the heirs instituted in case such heir or heirs should die before him, or should not wish or should be incapacitated to accept the inheritance. In a FIDEICOMMISSARY SUBSTITUTION, the testator institutes a first heir and charges him to preserve and transmit the whole or part of the inheritance to a second heir. In a simple substitution, only one heir inherits. In a fideicommissary substitution, both the first and second heirs inherit. (Art. 859 and 869, NCC) C. Betina has a cause of action against Divino. This is a case of a testamentary disposition subject to a mode and the will itself provides for the consequence if the mode is not complied with. To enforce the mode, the will itself gives Betina the right to compel the return of the property to the heirs of Theodore. (Rabadilla v. Conscoluella, 334 SCRA 522 [2000] GR 113725, 29 June 2000).

Wills; Formalities (1990) (1) If a will is executed by a testator who is a Filipino citizen, what law will govern if the will is executed in the Philippines? What law will govern if the will is executed in another country? Explain your answers. (2) If a will is executed by a foreigner, for instance, a Japanese, residing in the Philippines, what law will govern if the will is executed in the Philippines? And what law will govern if the will is executed in Japan, or some other country, for instance, the U.S.A.? Explain your answers. SUGGESTED ANSWER:

(1) a. If the testator who is a Filipino citizen executes his will in the Philippines, Philippine law will govern the formalities. b. If said Filipino testator executes his will in another country, the law of the country where he maybe or Philippine law will govern the formalities. (Article 815, Civil Code}

b. In the case of a foreigner, his national law shall govern substantive validity whether he executes his will in the Philippines or in a foreign country. Wills; Holographic Wills; Insertions & Cancellations (1996) Vanessa died on April 14, 1980, leaving behind a holographic will which is entirely written, dated and signed in her own handwriting. However, it contains insertions and cancellations which are not authenticated by her signature. For this reason, the probate of Vanessa's will was opposed by her relatives who stood to inherit by her intestacy. May Vanessa's holographic will be probated? Explain. SUGGESTED ANSWER:

Yes, the will as originally written may be probated. The insertions and alterations were void since they were not authenticated by the full signature of Vanessa, under Art. 814, NCC. The original will, however, remains valid because a holographic will is not invalidated by the unauthenticated insertions or alterations (Ajero v. CA, 236 SCRA 468]. ALTERNATIVE ANSWER:

It depends. As a rule, a holographic will is not adversely affected by Insertions or cancellations which were not authenticated by the full signature of the testator (Ajero v. CA, 236 SCRA 468). However, when the insertion or cancellation amounts to revocation of the will, Art.814 of the NCC does not apply but Art. 830. NCC. Art. 830 of the NCC does not require the testator to authenticate his cancellation for the effectivity of a revocation effected through such cancellation (Kalaw v. Relova, 132 SCRA 237). In the Kalaw case, the original holographic will designated only one heir as the only substantial provision which was altered by substituting the original heir with another heir. Hence, if the unauthenticated cancellation amounted to a revocation of the will, the will may not be probated because it had already been revoked. Wills; Holographic Wills; Witnesses (1994) On his deathbed, Vicente was executing a will. In the room were Carissa, Carmela, Comelio and Atty. Cimpo, a notary public. Suddenly, there was a street brawl which caught Comelio's attention, prompting him to look out the window. Cornelio did not see Vicente sign a will. Is the will valid? SUGGESTED ANSWERS:

SUGGESTED ANSWER:

(2) a. If the testator is a foreigner residing in the Philippines and he executes his will in the Philippines, the law of the country of which he is a citizen or Philippine law will govern the formalities. b. If the testator is a foreigner and executes his will in a foreign country, the law of his place of residence or the law of the country of which he is a citizen or the law of the place of execution, or Philippine law will govern the formalities (Articles 17. 816. 817. Civil Code). POSSIBLE ADDITIONAL ANSWERS:

a. In the case of a Filipino citizen, Philippine law shall govern substantive validity whether he executes his will in the Philippines or in a foreign country.

a) Yes, The will is valid. The law does not require a witness to actually see the testator sign the will. It is sufficient if the witness could have seen the act of signing had he chosen to do so by casting his eyes to the proper direction. b) Yes, the will is valid. Applying the "test of position", although Comelio did not actually see Vicente sign the will, Cornelio was in the proper position to see Vicente sign if Cornelio so wished. Wills; Joint Wills (2000) Manuel, a Filipino, and his American wife Eleanor, executed a Joint Will in Boston, Massachusetts when they were residing in said city. The law of Massachusetts allows the execution of joint wills. Shortly thereafter, Eleanor died. Can the said Will be probated in the Philippines for the settlement of her estate? (3%)

CIVIL LAW Answers to the BAR as Arranged by Topics (Year 1990-2006) SUGGESTED ANSWER:

Yes, the will may be probated in the Philippines insofar as the estate of Eleanor is concerned. While the Civil Code prohibits the execution of Joint wills here and abroad, such prohibition applies only to Filipinos. Hence, the joint will which is valid where executed is valid in the Philippines but only with respect to Eleanor. Under Article 819, it is void with respect to Manuel whose joint will remains void in the Philippines despite being valid where executed. ALTERNATIVE ANSWER:

The will cannot be probated in the Philippines, even though valid where executed, because it is prohibited under Article 818 of the Civil Code and declared void under Article 819, The prohibition should apply even to the American wife because the Joint will is offensive to public policy. Moreover, it is a single juridical act which cannot be valid as to one testator and void as to the other. Wills; Probate; Intrinsic Validity (1990) H died leaving a last will and testament wherein it is stated that he was legally married to W by whom he had two legitimate children A and B. H devised to his said forced heirs the entire estate except the free portion which he gave to X who was living with him at the time of his death. In said will he explained that he had been estranged from his wife W for more than 20 years and he has been living with X as man and wife since his separation from his legitimate family. In the probate proceedings, X asked for the issuance of letters testamentary in accordance with the will wherein she is named sole executor. This was opposed by W and her children. (a) Should the will be admitted in said probate proceedings? (b) Is the said devise to X valid? (c) Was it proper for the trial court to consider the intrinsic validity of the provisions of said will? Explain your answers, SUGGESTED ANSWER:

(a) Yes, the will may be probated if executed according to the formalities prescribed by law. (b) The institution giving X the free portion is not valid, because the prohibitions under Art. 739 of the Civil Code on donations also apply to testamentary dispositions (Article 1028, Civil Code), Among donations which are considered void are those made between persons who were guilty of adultery or concubinage at the time of the donation. (c) As a general rule, the will should be admitted in probate proceedings if all the necessary requirements for its extrinsic validity have been met and the court should not consider the intrinsic validity of the provisions of said will. However, the exception arises when the will in effect contains only one testamentary disposition. In effect, the only testamentary disposition under the will is the giving of the free portion to X, since legitimes are provided by law. Hence, the trial court may consider the intrinsic validity of the provisions of said will. (Nuguid v. Nuguid, etal.. No. L23445, June 23, 1966, 17 SCRA; Nepomuceno v. CA, L-62952, 9 October 1985. 139 SCRA 206).

Wills; Probate; Notarial and Holographic Wills (1997) Johnny, with no known living relatives, executed a notarial will giving all his estate to his sweetheart. One day, he had a serious altercation with his sweetheart. A few days later, he was introduced to a charming lady who later became a dear friend. Soon after, he executed a holographic will expressly revoking the notarial will and so designating his new friend as sole heir. One day when he was clearing up his desk, Johnny mistakenly burned, along with other papers, the only copy of his holographic will. His business associate, Eduardo knew well the contents of the will which was shown to him by Johnny the day it was executed. A few days after the burning incident, Johnny died. Both wills were sought to be probated in two separate petitions. Will either or both petitions prosper? SUGGESTED ANSWER:

The probate of the notarial will will prosper. The holographic will cannot be admitted to probate because a holographic will can only be probated upon evidence of the will itself unless there is a photographic copy. But since the holographic will was lost and there was no other copy, it cannot be probated and therefore the notarial will will be admitted to probate because there is no revoking will. ADDITIONAL ANSWERS:

1.

In the case of Gan vs. Yap (104 Phil 509), the execution and the contents of a lost or destroyed holographic will may not be proved by the bare testimony of witnesses who have seen or read such will. The will itself must be presented otherwise it shall produce no effect. The law regards the document itself as material proof of authenticity. Moreover, in order that a will may be revoked by a subsequent will, it is necessary that the latter will be valid and executed with the formalities required for the making of a will. The latter should possess all the requisites of a valid will whether it be ordinary or a holographic will, and should be probated in order that the revocatory clause thereof may produce effect. In the case at bar, since the holographic will itself cannot be presented, it cannot therefore be probated. Since it cannot be probated, it cannot revoke the notarial will previously written by the decedent. 2. On the basis of the Rules of Court, Rule 76, Sec. 6, provides that no will shall be proved as a lost or destroyed will unless its provisions are clearly and distinctly proved by at least two (2) credible witnesses. Hence, if we abide strictly by the two-witness rule to prove a lost or destroyed will, the holographic will which Johnny allegedly mistakenly burned, cannot be probated, since there is only one witness, Eduardo, who can be called to testify as to the existence of the will. If the holographic will, which purportedly, revoked the earlier notarial will cannot be proved because of the absence of the required witness, then the petition for the probate of the notarial will should prosper. Wills; Revocation of Wills; Dependent Relative Revocation (2003) Mr. Reyes executed a will completely valid as to form. A week later, however, he executed another will which expressly revoked his first will, which he tore his first will to pieces. Upon the death of Mr. Reyes, his second will was presented for probate by his heirs, but it was denied probate

CIVIL LAW Answers to the BAR as Arranged by Topics (Year 1990-2006)

due to formal defects. Assuming that a copy of the first will is available, may it now be admitted to probate and given effect? Why?

excluded by a legitimate son of the decedent [Art. 887, New Civil Code]. This follows the principle that the descendants exclude the ascendants from inheritance.

SUGGESTED ANSWER:

Yes, the first will may be admitted to probate and given effect. When the testator tore first will, he was under the mistaken belief that the second will was perfectly valid and he would not have destroyed the first will had he known that the second will is not valid. The revocation by destruction therefore is dependent on the validity of the second will. Since it turned out that the second will was invalid, the tearing of the first will did not produce the effect of revocation. This is known as the doctrine of dependent relative revocation (Molo v. Molo, 90 Phil 37.) ALTERNATIVE ANSWERS:

No, the first will cannot be admitted to probate. While it is true that the first will was successfully revoked by the second will because the second will was later denied probate, the first will was, nevertheless, revoked when the testator destroyed it after executing the second invalid will. (Diaz v. De Leon, 43 Phil 413 [1922]).

Wills; Testamentary Disposition (2006) Don died after executing a Last Will and Testament leaving his estate valued at P12 Million to his common-law wife Roshelle. He is survived by his brother Ronie and his half-sister Michelle. (1) Was Don's testamentary disposition of his estate in accordance with the law on succession? Whether you agree or not, explain your answer. Explain.

Yes, Don's testamentary disposition of his estate is in accordance with the law on succession. Don has no compulsory heirs not having ascendants, descendants nor a spouse [Art. 887, New Civil Code]. Brothers and sisters are not compulsory heirs. Thus, he can bequeath his entire estate to anyone who is not otherwise incapacitated to inherit from him. A common-law wife is not incapacitated under the law, as Don is not married to anyone. SUGGESTED ANSWER:

(2) If Don failed to execute a will during his lifetime, as his lawyer, how will you distribute his estate? Explain. (2.5%)

After paying the legal obligations of the estate, I will give Ronie, as full-blood brother of Don, 2/3 of the net estate, twice the share of Michelle, the half-sister who shall receive 1/3. Roshelle will not receive anything as she is not a legal heir [Art. 1006 New Civil Code]. SUGGESTED ANSWER:

(3) Assuming he died intestate survived by his brother Ronie, his half-sister Michelle, and his legitimate son Jayson, how will you distribute his estate? Explain. (2.5%)

Jayson will be entitled to the entire P12 Million as the brother and sister will be excluded by a legitimate son of the decedent. This follows the principle of proximity, where "the nearer excludes the farther." SUGGESTED ANSWER:

(4) Assuming further he died intestate, survived by his father Juan, his brother Ronie, his half-sister Michelle, and his legitimate son Jayson, how will you distribute his estate? Explain. (2.5%) SUGGESTED ANSWER: Jayson

will still be entitled to the entire P12 Million as the father, brother and sister will be

Wills; Testamentary Intent (1996) Alfonso, a bachelor without any descendant or ascendant, wrote a last will and testament in which he devised." all the properties of which I may be possessed at the time of my death" to his favorite brother Manuel. At the time he wrote the will, he owned only one parcel of land. But by the time he died, he owned twenty parcels of land. His other brothers and sisters insist that his will should pass only the parcel of land he owned at the time it was written, and did not cover his properties acquired, which should be by intestate succession. Manuel claims otherwise. Who is correct? Explain. SUGGESTED ANSWER:

Manuel is correct because under Art. 793, NCC, property acquired after the making of a will shall only pass thereby, as if the testator had possessed it at the time of making the will, should it expressly appear by the will that such was his intention. Since Alfonso's intention to devise all properties he owned at the time of his death expressly appears on the will, then all the 20 parcels of land are included in the devise.

DONATION

Donation vs. Sale (2003) a) May a person sell something that does not belong to him? Explain. b) May a person donate something that does not belong to him? Explain. 5% SUGGESTED ANSWER:

(a) Yes, a person may sell something which does not belong to him. For the sale to be valid, the law does not require the seller to be the owner of the property at the time of the sale. (Article 1434, NCC). If the seller cannot transfer ownership over the thing sold at the time of delivery because he was not the owner thereof, he shall be liable for breach of contact. (b) As a general rule, a person cannot donate something which he cannot dispose of at the time of the donation (Article 751, New Civil Code). Donations; Condition; Capacity to Sue (1996) Sometime in 1955, Tomas donated a parcel of land to his stepdaughter Irene, subject to the condition that she may not sell, transfer or cede the same for twenty years. Shortly thereafter, he died. In 1965, because she needed money for medical expenses, Irene sold the land to Conrado. The following year, Irene died, leaving as her sole heir a son by the name of Armando. When Armando learned that the land which he expected to inherit had been sold by Irene to Conrado, he filed an action against the latter for annulment of the sale, on the ground that it violated the restriction imposed by Tomas. Conrado filed a motion to dismiss, on the ground that Armando did not have the legal capacity to sue. If you were the Judge, how will you rule on this motion to dismiss? Explain.

CIVIL LAW Answers to the BAR as Arranged by Topics (Year 1990-2006) SUGGESTED ANSWER:

As judge, I will grant the motion to dismiss. Armando has no personality to bring the action for annulment of the sale to Conrado. Only an aggrieved party to the contract may bring the action for annulment thereof (Art. 1397. NCC). While Armando is heir and successor-in-interest of his mother (Art. 1311, NCC), he [standing in place of his mother) has no personality to annul the contract. Both are not aggrieved parties on account of their own violation of the condition of, or restriction on, their ownership imposed by the donation. Only the donor or his heirs would have the personality to bring an action to revoke a donation for violation of a condition thereof or a restriction thereon. (Garrido u. CA, 236 SCRA 450). Consequently, while the donor or his heirs were not parties to the sale, they have the right to annul the contract of sale because their rights are prejudiced by one of the contracting parties thereof [DBP v. CA, 96 SCRA 342; Teves vs. PHHC. 23 SCRA 114]. Since Armando is neither the donor nor heir of the donor, he has no personality to bring the action for annulment. ALTERNATIVE ANSWER:

As judge, I will grant the motion to dismiss. Compliance with a condition imposed by a donor gives rise to an action to revoke the donation under Art. 764, NCC. However, the right of action belongs to the donor. Is transmissible to his heirs, and may be exercised against the donee's heirs. Since Armando is an heir of the donee, not of the donor, he has no legal capacity to sue for revocation of the donation. Although he is not seeking such revocation but an annulment of the sale which his mother, the donee, had executed in violation of the condition imposed by the donor, an action for annulment of a contract may be brought only by those who are principally or subsidiarily obliged thereby (Art. 1397, NCC). As an exception to the rule, it has been held that a person not so obliged may nevertheless ask for annulment if he is prejudiced in his rights regarding one of the contracting parties (DBP us. CA. 96 SCRA 342 and other cases) and can show the detriment which would result to him from the contract in which he had no intervention, (Teves vs. PHHC, 23 SCRA 1141).

property to Ferdinand who then sued to recover the land from the city government. Will the suit prosper?

SUGGESTED ANSWER:

Ferdinand has no right to recover the land. It is true that the donation was revocable because of breach of the conditions. But until and unless the donation was revoked, it remained valid. Hence, Spouses Michael and Linda had no right to sell the land to Ferdinand. One cannot give what he does not have. What the donors should have done first was to have the donation annulled or revoked. And after that was done, they could validly have disposed of the land in favor of Ferdinand. ALTERNATIVE ANSWER:

A. Until the contract of donation has been resolved or rescinded under Article 1191 of the Civil Code or revoked under Art. 764 of the Civil Code, the donation stands effective and valid. Accordingly, the sale made by the donor to Ferdinand cannot be said to have conveyed title to Ferdinand, who, thereby, has no cause of action for recovery of the land acting for and in his behalf. B. The donation is onerous, And being onerous, what applies is the law on contracts, and not the law on donation (De Luna us. Abrigo, 81 SCRA 150). Accordingly, the prescriptive period for the filing of such an action would be the ordinary prescriptive period for contacts which may either be six or ten depending upon whether it is verbal or written. The filing of the case five years later is within the prescriptive period and, therefore, the action can prosper, Alternative Answer:

The law on donation lays down a special prescriptive period in the case of breach of condition, which is four years from non-compliance thereof (Article 764 Civil Code). Since the action has prescribed, the suit will not prosper, Donations; Effect; illegal & immoral conditions (1997) Are the effects of illegal and immoral conditions on simple donations the same as those effects that would follow when such conditions are imposed on donations con causa onerosa?

Such detriment or prejudice cannot be shown by Armando. As a forced heir, Armando's interest in the property was, at best, a mere expectancy. The sale of the land by his mother did not impair any vested right. The fact remains that the premature sale made by his mother (premature because only half of the period of the ban had elapsed) was not voidable at all, none of the vices of consent under Art. 139 of the NCC being present. Hence, the motion to dismiss should be granted.

SUGGESTED ANSWER:

Donations; Conditions; Revocation (1991) Spouses Michael and Linda donated a 3-hectare residential land to the City of Baguio on the condition that the city government would build thereon a public park with a boxing arena, the construction of which shall commence within six (6) months from the date the parties ratify the donation. The donee accepted the donation and the title to the property was transferred in its name. Five years elapsed but the public park with the boxing arena was never started. Considering the failure of the donee to comply with the condition of the donation, the donor-spouses sold the

Donations con causa onerosa is governed by law on obligations and contracts, under which an impossible or Illicit condition annuls the obligation dependent upon the condition where the condition is positive and suspensive. If the impossible or illicit condition is negative, it is simply considered as not written, and the obligation is converted into a pure and simple one. However, in order that an illegal condition may annul a contract, the impossibility must exist at the time of the creation of the obligation; a supervening impossibility does not affect the existence of the obligation.

No, they don't have the same effect. Illegal or impossible conditions in simple and remuneratory donations shall be considered as not imposed. Hence the donation is valid. The donation will be considered as simple or pure. The condition or mode is merely an accessory disposition, and its nullity does not affect the donation, unless it clearly appears that the donor would not have made the donation without the mode or condition.

ADDITIONAL ANSWER:

CIVIL LAW Answers to the BAR as Arranged by Topics (Year 1990-2006)

No. In simple or pure donation, only the illegal or impossible condition is considered not written but the donation remains valid and becomes free from conditions. The condition or mode being a mere accessory disposition. Its nullity does not affect the donation unless it clearly appears that the donor would not have made the donation without the mode or condition. On the other hand, onerous donation is governed by the rules on contracts. Under Article 1183, Impossible or illegal conditions shall annul the obligation which depends upon them. In these cases, both the obligation and the condition are void.

Donations; Formalities; Mortis Causa (1990) B donated to M a parcel of land in 1980. B made the deed of donation, entitled ―Donation Inter Vivos,‖ in a public instrument and M accepted the donation in the same document. It was provided in the deed that the land donated shall be immediately delivered to M and that M shall have the right to enjoy the fruits fully. The deed also provided that B was reserving the right to dispose of said land during his (B’s) lifetime, and that M shall not register the deed of donation until after B’s death. Upon B’s death, W, B’s widow and sole heir, filed an action for the recovery of the donated land, contending that the donation made by B is a donation mortis causa and not a donation inter vivos. Will said action prosper? Explain your answer. SUGGESTED ANSWER:

Yes, the action will prosper. The donation is a donation mortis causa because the reservation is to dispose of all the property donated and, therefore, the donation is revocable at will. Accordingly, the donation requires the execution of a valid will, either notarial or holographic (Arts 755, 728 NCC). Donations; Formalities; Mortis Causa (1998) Ernesto donated in a public instrument a parcel of land to Demetrio, who accepted it in the same document. It is there declared that the donation shall take effect immediately, with the donee having the right to take possession of the land and receive its fruits but not to dispose of the land while Ernesto is alive as well as for ten years following his death. Moreover, Ernesto also reserved in the same deed his right to sell the property should he decide to dispose of it at any time - a right which he did not exercise at all. After his death, Ernesto's heirs seasonably brought an action to recover the property, alleging that the donation was void as it did not comply with the formalities of a will. Will the suit prosper? [5%] SUGGESTED ANSWER:

Yes, the suit will prosper as the donation did not comply with the formalities of a will. In this instance, the fact that the donor did not intend to transfer ownership or possession of the donated property to the donee until the donor's death, would result in a donation mortis causa and in this kind of disposition, the formalities of a will should be complied with, otherwise, the donation is void. In this Instance, donation mortis causa embodied only in a public instrument without the formalities of a will could not have transferred ownership of disputed property to another. ALTERNATIVE ANSWER: One of the essential distinctions between a donation inter vivos and a donation mortis causa is that while the former is

irrevocable, the latter is revocable. In the problem given, all the clauses or conditions mentioned in the deed of donation, except one, are consistent with the rule of irrevocability and would have sustained the view that the donation is inter vivos and therefore valid. The lone exception is the clause which reserves the donor's right to sell the property at any time before his death. Such a reservation has been held to render the donation revocable and, therefore, becomes a donation mortis causa (Puig vs. Penqflorida, 15 SCRA 276, at p. 286). That the right was not exercised is immaterial; its reservation was an implied recognition of the donor's power to nullify the donation anytime he wished to do so. Consequently, it should have been embodied in a last will and testament. The suit for nullity will thus prosper. Donations; Inter Vivos; Acceptance (1993) On January 21, 1986, A executed a deed of donation inter vivos of a parcel of land to Dr. B who had earlier constructed thereon a building in which researches on the dreaded disease AIDS were being conducted. The deed, acknowledged before a notary public, was handed over by A to Dr. B who received it. A few days after, A flew to Davao City. Unfortunately, the airplane he was riding crashed on landing killing him. Two days after the unfortunate accident. Dr. B, upon advice of a lawyer, executed a deed acknowledged before a notary public accepting the donation. Is the donation effective? Explain your answer. SUGGESTED ANSWER:

No, the donation is not effective. The law requires that the separate acceptance of the donee of an immovable must be done in a public document during the lifetime of the donor (Art. 746 & 749, Civil Code) In this case, B executed the deed of acceptance before a notary public after the donor had already died. Donations; Perfection (1998) On July 27, 1997, Pedro mailed in Manila a letter to his brother, Jose, a resident of Ilollo City, offering to donate a vintage sports car which the latter had long been wanting to buy from the former. On August 5, 1997, Jose called Pedro by cellular phone to thank him for his generosity and to inform him that he was sending by mail his letter of acceptance. Pedro never received that letter because it was never mailed. On August 14, 1997, Pedro received a telegram from Iloilo informing him that Jose had been killed in a road accident the day before (August 13, 1997) 1. Is there a perfected donation? [2%] 2. Will your answer be the same if Jose did mail his acceptance letter but it was received by Pedro in Manila days after Jose's death? [3%] SUGGESTED ANSWER:

1. None. There is no perfected donation. Under Article 748 of the Civil Code, the donation of a movable may be made orally or in writing. If the value of the personal property donated exceeds five thousand pesos, the donation and the acceptance shall be made in writing. Assuming that the value of the thing donated, a vintage sports car, exceeds P5,000.00 then the donation and the acceptance must be in writing. In this instance, the acceptance of Jose was not in writing, therefore, the donation is void. Upon the other

CIVIL LAW Answers to the BAR as Arranged by Topics (Year 1990-2006)

hand, assuming that the sports car costs less than P5,000.00 then the donation maybe oral, but still, the simultaneous delivery of the car is needed and there being none, the donation was never perfected.

Code which requires the donation and the acceptance thereof to be in a public instrument in order to be valid. The acceptance not being in a public instrument, the part which is not onerous is void and Rosa may recover it from Amanda.

SUGGESTED ANSWER:

2. Yes, the answer is the same. If Jose's mail containing his acceptance of the donation was received by Pedro after the former's death, then the donation is still void because under Article 734 of the Civil Code, the donation is perfected the moment the donor knows of the acceptance by the donee. The death of Jose before Pedro could receive the acceptance indicates that the donation was never perfected. Under Article 746 acceptance must be made during the lifetime of both the donor and the donee. Donations; Requisites; Immovable Property Anastacia purchased a house and lot on installments at a housing project in Quezon City. Subsequently, she was employed in California and a year later, she executed a deed of donation, duly authenticated by the Philippine Consulate in Los Angeles, California, donating the house and lot to her friend Amanda. The latter brought the deed of donation to the owner of the project and discovered that Anastacia left unpaid installments and real estate taxes. Amanda paid these so that the donation in her favor can be registered in the project owner's office. Two months later, Anastacia died, leaving her mother Rosa as her sole heir. Rosa filed an action to annul the donation on the ground that Amanda did not give her consent in the deed of donation or in a separate public instrument. Amanda replied that the donation was an onerous one because she had to pay unpaid installments and taxes; hence her acceptance may be implied. Who is correct? (2%) SUGGESTED ANSWER:

Donations; Unregistered; Effects; Non-Compliance; Resolutory Condition (2006) Spouses Alfredo and Racquel were active members of a religious congregation. They donated a parcel of land in favor of that congregation in a duly notarized Deed of Donation, subject to the condition that the Minister shall construct thereon a place of worship within 1 year from the acceptance of the donation. In an affidavit he executed on behalf of the congregation, the Minister accepted the donation. The Deed of Donation was not registered with the Registry of Deeds. However, instead of constructing a place of worship, the Minister constructed a bungalow on the property he used as his residence. Disappointed with the Minister, the spouses revoked the donation and demanded that he vacate the premises immediately. But the Minister refused to leave, claiming that aside from using the bungalow as his residence, he is also using it as a place for worship on special occasions. Under the circumstances, can Alfredo and Racquel evict the Minister and recover possession of the property? If you were the couple's counsel, what action you take to protect the interest of your clients? (5%)

ALTERNATIVE ANSWER:

Yes, Alfredo and Racquel can bring an action for ejectment against the Minister for recovery of possession of the property evict the Minister and recover possession of the property. An action for annulment of the donation, reconveyance and damages should be filed to protect the interests of my client. The donation is an onerous donation and therefore shall be governed by the rules on contracts. Because there was no fulfillment or compliance with the condition which is resolutory in character, the donation may now be revoked and all rights which the donee may have acquired under it shall be deemed lost and extinguished

Rosa is correct because the donation is void. The property donated was an immovable. For such donation to be valid, Article 749 of the New Civil Code requires both the donation and the acceptance to be in a public instrument. There being no showing that Amanda's acceptance was made in a public instrument, the donation is void. The contention that the donation is onerous and, therefore, need not comply with Article 749 for validity is without merit. The donation is not onerous because it did not impose on Amanda the obligation to pay the balance on the purchase price or the arrears in real estate taxes. Amanda took it upon herself to pay those amounts voluntarily. For a donation to be onerous, the burden must be imposed by the donor on the donee. In the problem, there is no such burden imposed by the donor on the donee. The donation not being onerous, it must comply with the formalities of Article 749.

(Central Philippine University, G.R. No. 112127, July 17,1995).

ALTERNATIVE ANSWER:

Neither Rosa nor Amanda is correct. The donation is onerous only as to the portion of the property corresponding to the value of the installments and taxes paid by Amanda.

& Sons, Inc. v. Roman Catholic Bishop, G.R. No. 133705, March 31, 2005; Heirs ofRozendo Sevilla v. De Leon, G.R. No. 149570, March 12, 2004).

The portion in excess thereof is not onerous. The onerous portion is governed by the rules on contracts which do not require the acceptance by the donee to be in any form. The onerous part, therefore, is valid. The portion which is not onerous must comply with Article 749 of the New Civil

Donations; Validity; Effectivity; for Unborn Child (1999) Elated that her sister who had been married for five years was pregnant for the first time, Alma donated P100,000.00 to the unborn child. Unfortunately, the baby died one hour after delivery. May Alma recover the P100.000.00 that she

ALTERNATIVE ANSWER:

No, an action for ejectment will not prosper. I would advice Alfredo and Racquel that the Minister, by constructing a structure which also serves as a place of worship, has pursued the objective of the donation. His taking up residence in the bungalow may be regarded as a casual breach and will not warrant revocation of the donation. Similarily, therefore, an action for revocation of the donation will be denied (C. J. Yulo

CIVIL LAW Answers to the BAR as Arranged by Topics (Year 1990-2006)

had donated to said baby before it was born considering that the baby died? Stated otherwise, is the donation valid and binding? Explain. (5%) SUGGESTED ANSWER:

The donation is valid and binding, being an act favorable to the unborn child, but only if the baby had an intra-uterine life of not less than seven months and provided there was due acceptance of the donation by the proper person representing said child. If the child had less than seven months of intra-uterine life, it is not deemed born since it died less than 24 hours following its delivery, in which ease the donation never became effective since the donee never became a person, birth being determinative of personality. ALTERNATIVE ANSWER:

Even if the baby had an intra-uterine life of more than seven months and the donation was properly accepted, it would be void for not having conformed with the proper form. In order to be valid, the donation and acceptance of personal property exceeding five thousand pesos should be in writing. (Article 748, par. 3) Donations; with Resolutory Condition (2003) In 1950, Dr. Alba donated a parcel of land to Central University on condition that the latter must establish a medical college on the land to be named after him. In the year 2000, the heirs of Dr. Alba filed an action to annul the donation and for the reconveyance of the property donated to them for the failure, after 50 years, of the University to established on the property a medical school named after their father. The University opposed the action on the ground of prescription and also because it had not used the property for some purpose other than that stated in the donation. Should the opposition of the University to the action of Dr. Alba’s heirs be sustained? Explain. SUGGESTED ANSWER:

The donation may be revoked. The non-established of the medical college on the donated property was a resolutory condition imposed on the donation by the donor. Although the Deed of Donation did not fix the time for the established of the medical college, the failure of the donee to establish the medical college after fifty (50) years from the making of the donation should be considered as occurrence of the resolutory condition, and the donation may now be revoked. While the general rule is that in case the period is not fixed in the agreement of the parties, the period must be fixed first by the court before the obligation may be demanded, the period of fifty (50) years was more than enough time for the donee to comply with the condition. Hence, in this case, there is no more need for the court to fix the period because such procedure with the condition. (Central Philippine University v. CA. 246 SCRA 511).

ANOTHER SUGGESTED ANSWER:

The donation may not as yet revoked. The establishment of a medical college is not a resolutory or suspensive condition but a ―charge‖, obligation‖, or a ―mode‖. The noncompliance with the charge or mode will give the donor the right to revoke the donation within four (4) years from the time the charge was supposed to have been complied with, or to enforce the charge by specific performance within ten (10) years from the time the cause of action accrued. Inasmuch as the time to established the medical college has

not been fixed in the Deed of Donation, the donee is not yet default in his obligation until the period is fixed by order of the court under Article 1197 of the New Civil Code. Since the period has not been fixed as yet, the donee is not yet default, and therefore the donor has no cause of action to revoke the donation. (Dissenting opinion of Davide, CJ, Central Philippine University v. Court of Appeals, 246 SCRA 511 [1995])

PROPERTY Accretion; Alluvion (2001) For many years, the Rio Grande river deposited soil along its bank, beside the titled land of Jose. In time, such deposit reached an area of one thousand square meters. With the permission of Jose, Vicente cultivated the said area. Ten years later, a big flood occurred in the river and transferred the 1000 square meters to the opposite bank, beside the land of Agustin. The land transferred is now contested by Jose and Agustin as riparian owners and by Vicente who claims ownership by prescription. Who should prevail,? Why? (5%) SUGGESTED ANSWER:

Jose should prevail. The disputed area, which is an alluvion, belongs by right of accretion to Jose, the riparian owner (Art. 457 CC). When, as given in the problem, the very same area" was "transferred" by flood waters to the opposite bank, it became an avulsion and ownership thereof is retained by Jose who has two years to remove it (Art. 459, CC). Vicente's claim based on prescription is baseless since his possession was by mere tolerance of Jose and, therefore, did not adversely affect Jose's possession and ownership (Art. 537, CC). Inasmuch as his possession is merely that of a holder, he cannot acquire the disputed area by prescription. Accretion; Avulsion (2003) Andres is a riparian owner of a parcel of registered land. His land, however, has gradually diminished in area due to the current of the river, while the registered land of Mario on the opposite bank has gradually increased in area by 200square meters. (a) Who has the better right over the 200-square meter area that has been added to Mario’s registered land, Mario or Andres? (b) May a third person acquire said 200-square meter land by prescription? SUGGESTED ANSWER:

a. Mario has a better right over the 200 square meters increase in area by reason of accretion, applying Article 457 of the New Civil Code, which provides that ―to the owners of lands adjoining the banks of rivers belong the accretion which they gradually received from the effects of the current of the waters‖. Andres cannot claim that the increase in Mario’s land is his own, because such is an accretion and not result of the sudden detachment of a known portion of his land and its attachment to Mario’s land, a process called ―avulsion‖. He can no longer claim ownership of the portion of his registered land which was gradually and naturally eroded due to the current of the river, because he

CIVIL LAW Answers to the BAR as Arranged by Topics (Year 1990-2006)

had lost it by operation of law. That portion of the land has become part of the public domain. SUGGESTED ANSWER:

b. Yes, a third party may acquire by prescription the 200 square meters, increase in area, because it is not included in the Torrens Title of the riparian owner. Hence, this does not involve the imprescriptibility conferred by Section 47, P.D. No. 1529. The fact that the riparian land is registered does not automatically make the accretion thereto a registered land. (Grande v. CA, 115 521 (1962); Jagualing v. CA, 194 SCRA 607 (1991).

Builder; Good Faith (1992) A owns a parcel of residential land worth P500,000.00 unknown to A, a residential house costing P 100,000.00 is built on the entire parcel by B who claims ownership of the land. Answer all the following questions based on the premise that B is a builder in good faith and A is a landowner in good faith. a) May A acquire the house built by B? If so, how? b) If the land increased in value to P500,000.00 by reason of the building of the house thereon, what amount should be paid by A in order to acquire the house from B? c) Assuming that the cost of the house was P90,000.00 and not P100,000.00, may A require B to buy the land? d) If B voluntarily buys the land as desired by A, under what circumstances may A nevertheless be entitled to have the house removed? e) In what situation may a "forced lease" arise between A and B. and what terms and conditions would govern the lease? Give reasons for your answers. SUGGESTED ANSWER:

(a) Yes, A may acquire the house build by B by paying indemnity to B. Article 448 of the Civil Code provides that the owner of the land on which anything has been built, sown or planted in good faith, shall have the right to appropriate as his own the works, sowing or planting, after payment of the indemnity provided for in Articles 546 and 546 of the Civil Code. (b) A should pay B the sum of P50,000. Article 548 of the Civil Code provides that useful expenses shall be refunded to the possessor in good faith with the right of retention, the person who has defeated him in the possession having the option of refunding the amount of the expenses or of paying the increase in value which the thing may have acquired by reason thereof. The increase in value amounts to P50,000.00. (c) Yes, A may require B to buy the land. Article 448 of the Civil Code provides that the owner of the land on which anything has been built in good faith shall have the right to oblige the one who built to pay the price of the land if its value is not considerably more than that of the building, (d) If B agrees to buy land but fails to pay, A can have the house removed ( Depra vs. Dumlao, 136 SCRA 475). (e) Article 448 of the Civil Code provides that the builder cannot be obliged to buy the land if its value is considerably more than that of the building. In such case, he shall pay

reasonable rent, if the owner of the land does not choose to appropriate the building after proper indemnity. The parties shall agree upon the terms of the lease and in case of disagreement, the court fix the terms thereof. Builder; Good Faith vs. Bad Faith (1999) (a) Because of confusion as to the boundaries of the adjoining lots that they bought from the same subdivision company, X constructed a house on the adjoining lot of Y in the honest belief that it is the land that he bought from the subdivision company. What are the respective rights of X and Y with respect to X's house? (3%) (b) Suppose X was in good faith but Y knew that X was constructing on his (Y's) land but simply kept quiet about it, thinking perhaps that he could get X's house later. What are the respective rights of the parties over X's house in this case? (2%)

SUGGESTED ANSWER:

(a) The rights of Y, as owner of the lot, and of X, as builder of a house thereon, are governed by Art. 448 of the Civil Code which grants to Y the right to choose between two remedies: (a) appropriate the house by indemnifying X for its value plus whatever necessary expenses the latter may have incurred for the preservation of the land, or (b) compel X to buy the land if the price of the land is not considerably more than the value of the house. If it is, then X cannot be obliged to buy the land but he shall pay reasonable rent, and in case of disagreement, the court shall fix the terms of the lease. SUGGESTED ANSWER:

(b) Since the lot owner Y is deemed to be in bad faith (Art 453), X as the party in good faith may (a) remove the house and demand indemnification for damages suffered by him, or (b) demand payment of the value of the house plus reparation for damages (Art 447, in relation to Art 454). Y continues as owner of the lot and becomes, under the second option, owner of the house as well, after he pays the sums demanded. Builder; Good Faith vs. Bad Faith (2000) In good faith, Pedro constructed a five-door commercial building on the land of Pablo who was also in good faith. When Pablo discovered the construction, he opted to appropriate the building by paying Pedro the cost thereof. However, Pedro insists that he should be paid the current market value of the building, which was much higher because of inflation. 1) Who is correct Pedro or Pablo?(1%) 2) In the meantime that Pedro is not yet paid, who is entitled to the rentals of the building, Pedro or Pablo? (1%) SUGGESTED ANSWER:

Pablo is correct. Under Article 448 of the New Civil Code in relation to Article 546, the builder in good faith is entitled to a refund of the necessary and useful expenses incurred by him, or the increase in value which the land may have acquired by reason of the improvement, at the option of the landowner. The builder is entitled to a refund of the expenses he incurred, and not to the market value of the improvement

CIVIL LAW Answers to the BAR as Arranged by Topics (Year 1990-2006) The case of Pecson v. CA, 244 SCRA 407, is not applicable to square meters. Jose claims that Mike is a builder in bad faith

the problem. In the Pecson case, the builder was the owner of the land who later lost the property at a public sale due to non-payment of taxes. The Court ruled that Article 448 does not apply to the case where the owner of the land is the builder but who later lost the land; not being applicable, the indemnity that should be paid to the buyer must be the fair market value of the building and not just the cost of construction thereof. The Court opined in that case that to do otherwise would unjustly enrich the new owner of the land. ALTERNATIVE ANSWER:

Pedro is correct. In Pecson vs. CA, it was held that Article 546 of the New Civil Code does not specifically state how the value of useful improvements should be determined in fixing the amount of indemnity that the owner of the land should pay to the builder in good faith. Since the objective of the law is to adjust the rights of the parties in such manner as "to administer complete justice to both of them in such a way as neither one nor the other may enrich himself of that which does not belong to him", the Court ruled that the basis of reimbursement should be the fair market value of the building. SUGGESTED ANSWER:

2) Pablo is entitled to the rentals of the building. As the owner of the land, Pablo is also the owner of the building being an accession thereto. However, Pedro who is entitled to retain the building is also entitled to retain the rentals. He, however, shall apply the rentals to the indemnity payable to him after deducting reasonable cost of repair and maintenance. ALTERNATIVE ANSWER:

Pablo is entitled to the rentals. Pedro became a possessor in bad faith from the time he learned that the land belongs to Pablo. As such, he loses his right to the building, including the fruits thereof, except the right of retention. Builder; Good Faith vs. Bad Faith; Accession (2000) a) Demetrio knew that a piece of land bordering the beach belonged to Ernesto. However, since the latter was studying in Europe and no one was taking care of the land, Demetrio occupied the same and constructed thereon nipa sheds with tables and benches which he rented out to people who want to have a picnic by the beach. When Ernesto returned, he demanded the return of the land. Demetrio agreed to do so after he has removed the nipa sheds. Ernesto refused to let Demetrio remove the nipa sheds on the ground that these already belonged to him by right of accession. Who is correct? (3%) SUGGESTED ANSWER:

Ernesto is correct, Demetrio is a builder in bad faith because he knew beforehand that the land belonged to Ernesto, under Article 449 of the New Civil Code, one who builds on the land of another loses what is built without right to indemnity. Ernesto becomes the owner of the nipa sheds by right of accession. Hence, Ernesto is well within his right in refusing to allow the removal of the nipa sheds. Builder; Good Faith vs. Bad Faith; Presumption (2001) Mike built a house on his lot in Pasay City. Two years later, a survey disclosed that a portion of the building actually stood on the neighboring land of Jose, to the extent of 40

because he should know the boundaries of his lot, and demands that the portion of the house which encroached on his land should be destroyed or removed. Mike replies that he is a builder in good faith and offers to buy the land occupied by the building instead. 1) Is Mike a builder in good faith or bad faith? Why? (3%) 2) Whose preference should be followed? Why? (2%) SUGGESTED ANSWER:

1) Yes, Mike is a builder in good faith. There is no showing that when he built his house, he knew that a portion thereof encroached on Jose's lot. Unless one is versed in the science of surveying, he cannot determine the precise boundaries or location of his property by merely examining his title. In the absence of contrary proof, the law presumes that the encroachment was done in good faith [Technogas Phils, v. CA, 268 SCRA 5, 15 (1997)].

2} None of the preferences shall be followed. The preference of Mike cannot prevail because under Article 448 of the Civil Code, it is the owner of the land who has the option or choice, not the builder. On the other hand, the option belongs to Jose, he cannot demand that the portion of the house encroaching on his land be destroyed or removed because this is not one of the options given by law to the owner of the land. The owner may choose between the appropriation of what was built after payment of indemnity, or to compel the builder to pay for the land if the value of the land is not considerably more than that of the building. Otherwise, the builder shall pay rent for the portion of the land encroached. ALTERNATIVE ANSWER:

1) Mike cannot be considered a builder in good faith because he built his house without first determining the corners and boundaries of his lot to make sure that his construction was within the perimeter of his property. He could have done this with the help of a geodetic engineer as an ordinary prudent and reasonable man would do under the circumstances. 2) Jose's preference should be followed. He may have the building removed at the expense of Mike, appropriate the building as his own, oblige Mike to buy the land and ask for damages in addition to any of the three options. (Articles 449, 450, 451, CC) Chattel Mortgage vs. Pledge (1999) Distinguish a contract of chattel mortgage from a contract of pledge. (2%) SUGGESTED ANSWER:

In a contract of CHATTEL MORTGAGE possession belongs to the creditor, while in a contract of PLEDGE possession belongs to the debtor. A chattel mortgage is a formal contract while a pledge is a real contract. A contract of chattel mortgage must be recorded in a public instrument to bind third persons while a contract of pledge must be in a public instrument containing description of the thing pledged and the date thereof to bind third persons.

CIVIL LAW Answers to the BAR as Arranged by Topics (Year 1990-2006)

Chattel Mortgage; Immovables (1994) Vini constructed a building on a parcel of land he leased from Andrea. He chattel mortgaged the land to Felicia. When he could not pay Felicia. Felicia initiated foreclosure proceedings. Vini claimed that the building he had constructed on the leased land cannot be validly foreclosed because the building was, by law, an immovable. Is Vini correct? SUGGESTED ANSWERS:

a) The Chattel Mortgage is void and cannot be foreclosed because the building is an immovable and cannot be an object of a chattel mortgage. b) It depends. If the building was intended and is built of light materials, the chattel mortgage may be considered as valid as between the parties and it may be considered in respect to them as movable property, since it can be removed from one place to another. But if the building is of strong material and is not capable of being removed or transferred without being destroyed, the chattel mortgage is void and cannot be foreclosed.

foreclosure sale, foreclosed the mortgage and acquired X’s house and lot. Learning of the proceedings conducted by the bank, Z is now demanding that the bank reconvey to him X’s house or pay X’s loan to him plus interests. Is Z’s demand against the bank valid and sustainable? Why? 5% SUGGESTED ANSWER:

No, Z’s demand is not valid. A building is immovable or real property whether it is erected by the owner of the land, by a usufructuary, or by a lessee. It may be treated as a movable by the parties to chattel mortgage but such is binding only between them and not on third parties (Evangelista v. Alto Surety Col, inc. 103 Phil. 401 [1958]). In this case, since the bank is not a party to the chattel mortgage, it is not bound by it, as far as the Bank is concerned, the chattel mortgage, does not exist. Moreover, the chattel mortgage does not exist. Moreover, the chattel mortgage is void because it was not registered. Assuming that it is valid, it does not bind the Bank because it was not annotated on the title of the land mortgaged to the bank. Z cannot demand that the Bank pay him the loan Z extended to X, because the Bank was not privy to such loan transaction.

c) If it was the land which Vini chattel mortgaged, such mortgage would be void, or at least unenforceable, since he was not the owner of the land. If what was mortgaged as a chattel is the building, the chattel mortgage is valid as between the parties only, on grounds of estoppel which would preclude the mortgagor from assailing the contract on the ground that its subject-matter is an immovable. Therefore Vini's defense is untenable, and Felicia can foreclose the mortgage over the building, observing, however, the procedure prescribed for the execution of sale of a judgment debtor's immovable under Rule 39, Rules of Court, specifically, that the notice of auction sale should be published in a newspaper of general circulation.

ANOTHER SUGGESTED ANSWER:

d) The problem that Vini mortgaged the land by way of a chattel mortgage is untenable. Land can only be the subject matter of a real estate mortgage and only an absolute owner of real property may mortgage a parcel of land. (Article 2085 (2) Civil Code). Hence, there can be no foreclosure.

ALTERNATIVE ANSWER:

But on the assumption that what was mortgaged by way of chattel mortgage was the building on leased land, then the parties are treating the building as chattel. A building that is not merely superimposed on the ground is an immovable property and a chattel mortgage on said building is legally void but the parties cannot be allowed to disavow their contract on account of estoppel by deed. However, if third parties are involved such chattel mortgage is void and has no effect. Chattel Mortgage; Immovables (2003) X constructed a house on a lot which he was leasing from Y. Later, X executed a chattel mortgage over said house in favor of Z as security for a loan obtained from the latter. Still later, X acquired ownership of the land where his house was constructed, after which he mortgaged both house and land in favor of a bank, which mortgage was annotated on the Torrens Certificate of Title. When X failed to pay his loan to the bank, the latter, being the highest bidder at the

No, Z’s demand against the bank is not valid. His demand that the bank reconvey to him X’s house presupposes that he has a real right over the house. All that Z has is a personal right against X for damages for breach of the contract of loan. The treatment of a house, even if built on rented land, as movable property is void insofar as third persons, such as the bank, are concerned. On the other hand, the Bank already had a real right over the house and lot when the mortgage was annotated at the back of the Torrens title. The bank later became the owner in the foreclosure sale. Z cannot ask the bank to pay for X’s loan plus interest. There is no privity of contract between Z and the bank. The answer hinges on whether or not the bank is an innocent mortgagee in good faith or a mortgagee in bad faith. In the former case, Z’s demand is not valid. In the latter case, Z’s demand against the bank is valid and sustainable. Under the Torrens system of land registration, every person dealing with registered land may rely on the correctness of the certificate of title and the law will not in any way oblige to him to look behind or beyond the certificate in order to determine the condition of the title. He is not bound by anything not annotated or reflected in the certificate. If he proceeds to buy the land or accept it as a collateral relying on the certificate, he is considered a buyer or a mortgagee in good faith. On this ground, the Bank acquires a clean title to the land and the house. However, a bank is not an ordinary mortgagee. Unlike private individuals, a bank is expected to exercise greater care and prudence in its dealings. The ascertainment of the condition of a property offered as collateral for a loan must be a standard and indispensable part of its operation. The bank should have conducted further inquiry regarding the house standing on the land considering that it was already

Page 59 of 119

CIVIL LAW Answers to the BAR as Arranged by Topics (Year 1990-2006)

standing there before X acquired the title to the land. The bank cannot be considered as a mortgagee in good faith. On this ground, Z’s demand against the Bank is valid and sustainable.

Chattel Mortgage; Possession (1993) A, about to leave the country on a foreign assignment, entrusted to B his brand new car and its certificate of registration. Falsifying A's signature. B sold A's car to C for P200,000.00. C then registered the car in his name. To complete the needed amount, C borrowed P100.000.00 from the savings and loan association in his office, constituting a chattel mortgage on the car. For failure of C to pay the amount owed, the savings and loan association filed in the RTC a complaint for collection with application for issuance of a writ of replevin to obtain possession of the vehicle so that the chattel mortgage could be foreclosed. The RTC issued the writ of replevin. The car was then seized from C and sold by the sheriff at public auction at which the savings and loan association was the lone bidder. Accordingly, the car was sold to it. A few days later, A arrived from his foreign assignment. Learning of what happened to his car, A sought to recover possession and ownership of it from the savings and loan association. Can A recover his car from the savings and loan association? Explain your answer. SUGGESTED ANSWER:

Under the prevailing rulings of the Supreme Court, A can recover the car from the Savings and Loan Association provided he pays the price at which the Association bought the car at a public auction. Under that doctrine, there has been an unlawful deprivation by B of A of his car and, therefore, A can recover it from any person in possession thereof. But since it was bought at a public auction in good faith by the Savings and Loan Association, he must reimburse the Association at the price for which the car was bought. ALTERNATIVE ANSWER:

Yes, A can recover his car from the Savings and Loan Association. In a Chattel Mortgage, the mortgagor must be the absolute owner of the thing mortgaged. Furthermore, the person constituting the mortgage must have the free disposal of the property, and in the absence thereof, must be legally authorized for the purpose. In the case at bar, these essential requisites did not apply to the mortgagor B, hence the Chattel Mortgage was not valid. Chattel Mortgage; Preference of Creditors (1995) Lawrence, a retired air force captain, decided to go into the air transport business. He purchased an aircraft in cash except for an outstanding balance of P500,000.00. He incurred an indebtedness of P300,000.00 for repairs with an aircraft repair company. He also borrowed P1 Million from a bank for additional capital and constituted a chattel mortgage on the aircraft to secure the loan. While on a test flight the aircraft crashed causing physical injuries to a third party who was awarded damages of P200,000.00. Lawrence's insurance claim for damage to the aircraft was denied thus leaving him nothing else but the aircraft which

was then valued only at P1 Million. Lawrence was declared insolvent. Assuming that the aircraft was sold for Pl Million, give the order of preference of the creditors of Lawrence and distribute the amount of P1 Million.

SUGGESTED ANSWER:

Assuming that the aircraft was sold for P1 Million, there is no order of preference. The P1 Million will all go to the bank as a chattel mortgagee because a chattel mortgage under Art. 2241 (4) NCC defeats Art. 2244 (12) and (14}. Art. 2241 (3) and (5) are not applicable because the aircraft is no longer in the possession of the creditor. Easement vs. Usufruct (1995) 1. What is easement? Distinguish easement from usufruct. 2. Can there be (a) an easement over a usufruct? (b) a usufruct over an easement? (c) an easement over another easement? Explain. SUGGESTED ANSWER:

1. An EASEMENT or servitude is an encumbrance imposed upon an immovable for the benefit of another immovable belonging to a different owner. (Art. 613, NCC) USUFRUCT gives a right to enjoy the property of another with the obligation of preserving its form and substance, unless the title constituting it or the law otherwise provides. (Art. 562, NCC). ALTERNATIVE ANSWER:

Easement is an encumbrance imposed upon an immovable for the benefit of another immovable belonging to a different owner in which case it is called real or predial easement, or for the benefit of a community or group of persons in which case it is known as a personal easement. The distinctions between usufruct and easement are: a) Usufruct includes all uses of the property and for all purposes, including jus fruendi. Easement is limited to a specific use. b) Usufruct may be constituted on immovable or movable property. Easement may be constituted only on an immovable property. c) Easement is not extinguished by the death of the owner of the dominant estate while usufruct is extinguished by the death of the usufructuary unless a contrary intention appears. d) An easement contemplates two (2) estates belonging to two (2) different owners; a usufruct contemplates only one property (real or personal) whereby the usufructuary uses and enjoys the property as well as its fruits, while another owns the naked title during the period of the usufruct. e) A usufruct may be alienated separately from the property to which it attaches, while an easement cannot be alienated separately from the property to which it attaches. NOTE: It is recommended by the Committee that any two (2) distinctions should be given full credit. SUGGESTED ANSWER:

CIVIL LAW Answers to the BAR as Arranged by Topics (Year 1990-2006)

2. (a) There can be no easement over a usufruct. Since an easement may be constituted only on a corporeal immovable property, no easement may be constituted on a usufruct which is not a corporeal right (b) There can be no usufruct over an easement. While a usufruct maybe created over a right, such right must have an existence of its own independent of the property. A servitude cannot be the object of a usufruct because it has no existence independent of the property to which It attaches. ALTERNATIVE ANSWERS:

There cannot be a usufruct over an easement since an easement presupposes two (2) tenements belonging to different persons and the right attaches to the tenement and not to the owner. While a usufruct gives the usufructuary a right to use, right to enjoy, right to the fruits, and right to possess, an easement gives only a limited use of the servient estate. However, a usufruct can be constituted over a property that has in its favor an easement or one burdened with servitude. The usufructuary will exercise the easement during the period of usufruct. (c) There can be no easement over another easement for the same reason as in (a). An easement, although it is a real right over an immovable, is not a corporeal right. There is a Roman maxim which says that: There can be no servitude over another servitude. Easement; Effects; Discontinuous Easements; Permissive Use (2005) Don was the owner of an agricultural land with no access to a public road. He had been passing through the land of Ernie with the latter's acquiescence for over 20 years. Subsequently, Don subdivided his property into 20 residential lots and sold them to different persons. Ernie blocked the pathway and refused to let the buyers pass through his land. a) Did Don acquire an easement of right of way? Explain. (2%) ALTERNATIVE ANSWER:

No, Don did not acquire an easement of right of way. An easement of right of way is discontinuous in nature — it is exercised only if a man passes over somebody's land. Under Article 622 of the Civil Code, discontinuous easements, whether apparent or not, may only be acquired by virtue of a title. The Supreme Court, in Abellana, Sr. v. Court of Appeals (G.R. No. 97039, April 24, 1992), ruled that an easement of right of way being discontinuous in nature is not acquirable by prescription. Further, possession of the easement by Don is only permissive, tolerated or with the acquiescence of Ernie. It is settled in the case of Cuaycong v. Benedicto (G.R. No. 9989, March 13, 1918) that a permissive use of a road over the land of another, no matter how long continued, will not create an easement of way by prescription. ALTERNATIVE ANSWER: Yes, Don acquired an easement of right of way. An easement that is continuous and apparent can be acquired by prescription and title. According to Professor Tolentino, an easement of right of way may have a continuous nature if

there is a degree of regularity to indicate continuity of possession and that if coupled with an apparent sign, such easement of way may be acquired by prescription.

ALTERNATIVE ANSWER:

Yes, Ernie could close the pathway on his land. Don has not acquired an easement of right of way either by agreement or by judicial grant. Neither did the buyers. Thus, establishment of a road or unlawful use of the land of Ernie would constitute an invasion of possessory rights of the owner, which under Article 429 of the Civil Code may be repelled or prevented. Ernie has the right to exclude any person from the enjoyment and disposal of the land. This is an attribute of ownership that Ernie enjoys. ALTERNATIVE ANSWER:

Yes, Ernie may close the pathway, subject however, to the rights of the lot buyers. Since there is no access to the public road, this results in the creation of a legal easement. The lot buyers have the right to demand that Ernie grant them a right of way. In turn, they have the obligation to pay the value of the portion used as a right of way, plus damages. c) What are the rights of the lot buyers, if any? Explain. (2%) SUGGESTED ANSWER:

Prior to the grant of an easement, the buyers of the dominant estate have no other right than to compel grant of easement of right of way. Since the properties of the buyers are surrounded by other immovables and has no adequate outlet to a public highway and the isolation is not due to their acts, buyers may demand an easement of a right of way provided proper indemnity is paid and the right of way demanded is the shortest and least prejudicial to Ernie. (Villanueva v. Velasco, G.R. No. 130845, November 27, 2000).

Easement; Nuisance; Abatement (2002) Lauro owns an agricultural land planted mostly with fruit trees. Hernando owns an adjacent land devoted to his piggery business, which is two (2) meters higher in elevation. Although Hernando has constructed a waste disposal lagoon for his piggery, it is inadequate to contain the waste water containing pig manure, and it often overflows and inundates Lauro’s plantation. This has increased the acidity of the soil in the plantation, causing the trees to wither and die. Lauro sues for damages caused to his plantation. Hernando invokes his right to the benefit of a natural easement in favor of his higher estate, which imposes upon the lower estate of Lauro the obligation to receive the waters descending from the higher estate. Is Hernando correct? (5%) SUGGESTED ANSWER:

Hernando is wrong. It is true that Lauro’s land is burdened with the natural easement to accept or receive the water which naturally and without interruption of man descends from a higher estate to a lower estate. However, Hernando has constructed a waste disposal lagoon for his piggery and it is this waste water that flows downward to Lauro’s land. Hernando has, thus, interrupted the flow of water and has created and is maintaining a nuisance. Under Act. 697 NCC, abatement of a nuisance does not preclude recovery of damages by Lauro even for the past existence of a nuisance.

CIVIL LAW Answers to the BAR as Arranged by Topics (Year 1990-2006)

The claim for damages may also be premised in Art. 2191 (4) NCC. ANOTHER ANSWER:

Hernando is not correct. Article 637 of the New Civil Code provides that the owner of the higher estate cannot make works which will increase the burden on the servient estate. (Remman Enterprises, Inc. v. CA, 330 SCRA 145 [2000]). The owner of the higher estate may be compelled to pay damages to the owner of the lower estate. Easements; Classification (1998) Distinguish between: 1. Continuous and discontinuous easements; |2%] 2. Apparent and non-apparent easements; and [2%] 3. Positive and negative easements. [1%] SUGGESTED ANSWER:

1. CONTINUOUS EASEMENTS are those the use of which is or may be incessant, without the intervention of any act of man, while DISCONTINUOUS EASEMENTS are those which are used at intervals and depend upon the acts of man. (Art. 615, Civil Code) SUGGESTED ANSWER:

2. APPARENT EASEMENTS are those which are made known and are continually kept in view by external signs that reveal the use and enjoyment of the same, while NONAPPARENT EASEMENTS are those which show no external indication of their existence. (Art. 615, Civil Code) SUGGESTED ANSWER:

3. POSITIVE EASEMENTS are those which impose upon the owner of the servient estate the obligation of allowing something to be done or of doing it himself, while NEGATIVE EASEMENTS are those which prohibit the owner of the servient estate from doing something which he could lawfully do if the easement did not exist. (Art. 615. Civil Code) Easements; Right of Way (1993) Tomas Encarnacion's 3,000 square meter parcel of land, where he has a plant nursery, is located just behind Aniceta Magsino's two hectare parcel land. To enable Tomas to have access to the highway, Aniceta agreed to grant him a road right of way a meter wide through which he could pass. Through the years Tomas' business flourished which enabled him to buy another portion which enlarged the area of his plant nursery. But he was still landlocked. He could not bring in and out of his plant nursery a jeep or delivery panel much less a truck that he needed to transport his seedlings. He now asked Aniceta to grant him a wider portion of her property, the price of which he was willing to pay, to enable him to construct a road to have access to his plant nursery. Aniceta refused claiming that she had already allowed him a previous road right of way. Is Tomas entitled to the easement he now demands from Aniceta? SUGGESTED ANSWER: Art. 651 of the Civil Code provides that the width of the easement must be sufficient to meet the needs of the dominant estate, and may accordingly change from time to time. It is the need of the dominant estate which determines the width of the passage. These needs may vary from time

to time. As Tomas' business grows, the need for use of modern conveyances requires widening of the easement. ALTERNATIVE ANSWER:

The facts show that the need for a wider right of way arose from the increased production owing to the acquisition by Tomas of an additional area. Under Art. 626 of the Civil Code, the easement can be used only for the immovable originally contemplated. Hence, the increase in width is justified and should have been granted. Easements; Right of Way (2000) The coconut farm of Federico is surrounded by the lands of Romulo. Federico seeks a right of way through a portion of the land of Romulo to bring his coconut products to the market. He has chosen a point where he will pass through a housing project of Romulo. The latter wants him to pass another way which is one kilometer longer. Who should prevail? (5%) SUGGESTED ANSWER:

Romulo will prevail. Under Article 650 of the New Civil Code, the easement of right of way shall be established at the point least prejudicial to the servient estate and where the distance from the dominant estate to a public highway is the shortest. In case of conflict, the criterion of least prejudice prevails over the criterion of shortest distance. Since the route chosen by Federico will prejudice the housing project of Romulo, Romulo has the right to demand that Federico pass another way even though it will be longer. Easements; Right of Way; Inseparability (2001) Emma bought a parcel of land from Equitable-PCI Bank, which acquired the same from Felisa, the original owner. Thereafter, Emma discovered that Felisa had granted a right of way over the land in favor of the land of Georgina, which had no outlet to a public highway, but the easement was not annotated when the servient estate was registered under the Torrens system. Emma then filed a complaint for cancellation of the right of way, on the ground that it had been extinguished by such failure to annotate. How would you decide the controversy? (5%) SUGGESTED ANSWER:

The complaint for cancellation of easement of right of way must fail. The failure to annotate the easement upon the title of the servient estate is not among the grounds for extinguishing an easement under Art. 631 of the Civil Code. Under Article 617, easements are inseparable from the estate to which they actively or passively belong. Once it attaches, it can only be extinguished under Art. 631, and they exist even if they are not stated or annotated as an encumbrance on the Torrens title of the servient estate. (II Tolentino 326, 1987 ed.) ALTERNATIVE ANSWER:

Under Section 44, PD No. 1529, every registered owner receiving a certificate of title pursuant to a decree of registration, and every subsequent innocent purchaser for value, shall hold the same free from all encumbrances except those noted on said certificate. This rule, however, admits of exceptions. Under Act 496, as amended by Act No. 2011, and Section 4, Act 3621, an easement if not registered shall remain and shall be held to pass with the land until cutoff or

CIVIL LAW Answers to the BAR as Arranged by Topics (Year 1990-2006)

extinguished by the registration of the servient estate. However, this provision has been suppressed in Section 44, PD No. 1529. In other words, the registration of the servient estate did not operate to cut-off or extinguish the right of way. Therefore, the complaint for the cancellation of the right of way should be dismissed.

Easements; Right of Way; Requisites (1996) David is the owner of the subdivision in Sta. Rosa, Laguna, without an access to the highway. When he applied for a license to establish the subdivision, David represented that he will purchase a rice field located between his land and the highway, and develop it into an access road. But. when the license was already granted, he did not bother to buy the rice field, which remains unutilized until the present. Instead, he chose to connect his subdivision with the neighboring subdivision of Nestor, which has an access to the highway. Nestor allowed him to do this, pending negotiations on the compensation to be paid. When they failed to arrive at an agreement, Nestor built a wall across the road connecting with David's subdivision. David filed a complaint in court, for the establishment of an easement of right of way through the subdivision of Nestor which he claims to be the most adequate and practical outlet to the highway. 1) What are the requisites for the establishment of a compulsory easement of a right of way? SUGGESTED ANSWER:

Art, 649, NCC. The owner, or any person who by virtue of a real right may cultivate or use any immovable which is surrounded by other immovables pertaining to other persons and without adequate outlet to a public highway, is entitled to demand a right of way through the neighboring estates, after payment of the property indemnity. Should this easement be established in such a manner that its use may be continuous for all the needs of the dominant estate, establishing a permanent passage, the indemnity shall consist of the value of the land occupied and the amount of the damage caused to the servient estate. In case the right of way is limited to the necessary passage for the cultivation of the estate surrounded by others and for the gathering of its crops through the servient estate without a permanent way, the indemnity shall consist in the payment of the damage cause by such encumbrance. This easement is not compulsory if the isolation of the immovable is due to the proprietor's own acts. (564a). The easement of right of way shall be established at the point least prejudicial to the servient estate, and insofar as consistent with this rule, where the distance from the dominant estate to a public highway may be the shortest (Art. 650, NCC: Vda. de Baltazar v. CA. 245 SCRA 333} ALTERNATIVE ANSWER: The requisites for a compulsory easement of right of way are: (a) the dominant estate is surrounded by other immovables and is without an adequate outlet to a public street or highway; (b) proper indemnity must be paid; (c) the isolation must not be due to the acts of the owner of the dominant estate; and (d) the right of way claimed is at a point least prejudicial to the servient estate and, insofar as is

consistent with this rule, where the distance to the street or highway is shortest.

2) Is David entitled to a right of way in this case? Why or why not? SUGGESTED ANSWER:

No, David is not entitled to the right of way being claimed. The isolation of his subdivision was due to his own act or omission because he did not develop into an access road the rice field which he was supposed to purchase according to his own representation when he applied for a license to establish the subdivision (Floro us. Llenado, 244 SCRA713). Ejectment Suit vs. Cancellation of Title (2005) In an ejectment case filed by Don against Cesar, can the latter ask for the cancellation of Don's title considering that he (Cesar) is the rightful owner of the lot? Explain. (2%) SUGGESTED ANSWER:

Cesar cannot ask for the cancellation of Don's title even if he is the rightful owner of the lot. In an action for ejectment, the only issue involved is one of possession de facto, the purpose of which is merely to protect the owner from any physical encroachment from without. The title of the land or its ownership is not involved, for if a person is in actual possession thereof, he is entitled to be maintained and respected in it even against the owner himself. (Garcia v. Anas, G.R. No. L-20617, May 31, 1965)

Since the case filed by Don against Cesar is an ejectment case, the latter cannot ask for the cancellation of Don's title. He has to file the proper action where the issue of ownership over the property can be raised. Ejectment Suit; Commodatum (2006) Alberto and Janine migrated to the United States of America, leaving behind their 4 children, one of whom is Manny. They own a duplex apartment and allowed Manny to live in one of the units. While in the United States, Alberto died. His widow and all his children executed an Extrajudicial Settlement of Alberto's estate wherein the 2door apartment was assigned by all the children to their mother, Janine. Subsequently, she sold the property to George. The latter required Manny to sign a prepared Lease Contract so that he and his family could continue occupying the unit. Manny refused to sign the contract alleging that his parents allowed him and his family to continue occupying the premises. If you were George's counsel, what legal steps will you take? Explain. (5%) SUGGESTED ANSWER:

If I were George's counsel, I would first demand that Manny vacate the apartment. If Manny refuses, I will file an ejectment suit. When Manny was allowed by his parents to occupy the premises, without compensation, the contract of commodatum was created. Upon the death of the father, the contract was extinguished as it is a purely personal contract. As the new owner of the apartment George is entitled to exercise his right of possession over the same. Extra-Judicial Partition; Fraud (1990) X was the owner of a 10,000 square meter property. X married Y and out of their union. A, B and C were born.

CIVIL LAW Answers to the BAR as Arranged by Topics (Year 1990-2006)

After the death of Y, X married Z and they begot as children, D, E and F. After the death of X, the children of the first and second marriages executed an extrajudicial partition of the aforestated property on May 1, 1970. D, E and F were given a one thousand square meter portion of the property. They were minors at the time of the execution of the document. D was 17 years old, E was 14 and F was 12; and they were made to believe by A, B and C that unless they sign the document they will not get any share. Z was not present then. In January 1974, D, E and F filed an action in court to nullify the suit alleging they discovered the fraud only in 1973. (a) Can the minority of D, E and F be a basis to nullify the partition? Explain your answer. (b) How about fraud? Explain your answer.

share allotted by law to the finder since the phrase "by chance" means "by accident", meaning an unexpected discovery. The liberal view, however, would sustain Tim's right to the allocated share interpreting the phrase in question as meaning "by a stroke of good fortune", which does not rule out deliberate or intentional search. It is submitted that the liberal view should prevail since in practical reality, hidden treasure is hardly ever found without conscious effort to find it, and the strict view would tend to render the codal provision in question illusory.

SUGGESTED ANSWER:

Hidden Treasures (1997) Marcelino, a treasure hunter as just a hobby, has found a map which appears to indicate the location of hidden treasure. He has an idea of the land where the treasure might possibly be found. Upon inquiry, Marcelino learns that the owner of the land, Leopoldo, is a permanent resident of Canada, Nobody, however, could give him Leopoldo's exact address. Ultimately, anyway, he enters the land and conducts a search. He succeeds.

(b) In the case of fraud, when through insidious words or machinations of one party the other is induced to enter into the contract without which he would not have agreed to, the action still prosper because under Art, 1391 of the Civil Code, in case of fraud, the action for annulment may be brought within four years from the discovery of the fraud.

Leopoldo learning of Marcelino's "find", seeks to recover the treasure from Marcelino but the latter is not willing to part with it. Failing to reach an agreement, Leopoldo sues Marcelino for the recovery of the property. Marcelino contests the action. How would you decide the case?

(a) Yes, minority can be a basis to nullify the partition because D, E and F were not properly represented by their parents or guardians at the time they contracted the extrajudicial partition. (Articles 1327. 1391, Civil Code).

SUGGESTED ANSWER:

Hidden Treasure (1995) Tim came into possession of an old map showing where a purported cache of gold bullion was hidden. Without any authority from the government Tim conducted a relentless search and finally found the treasure buried in a new river bed formerly part of a parcel of land owned by spouses Tirso and Tessie. The old river which used to cut through the land of spouses Ursula and Urbito changed its course through natural causes. To whom shall the treasure belong? Explain. SUGGESTED ANSWER:

The treasure was found in a property of public dominion, the new river bed. Since Tim did not have authority from the government and, therefore, was a trespasser, he is not entitled to the one-half share allotted to a finder of hidden treasure. All of it will go to the State. In addition, under Art. 438 of the NCC in order that the finder be entitled to the 1/2 share, the treasure must be found by chance, that is by sheer luck. In this case, since Tim found the treasure not by chance but because he relentlessly searched for it, he is not entitled to any share in the hidden treasure. ALTERNATIVE ANSWER: The law grants a one-half share to a finder of hidden treasure provided he is not a trespasser and the finding is by chance. It is submitted that Tim is not a trespasser despite his not getting authority from the government, because the new river bed where he found the treasure is property for public use (Art. 420 NCC), to which the public has legitimate access. The question, therefore, boils down to whether or not the finding was by chance in view of the fact that Tim "conducted a relentless search" before finding the treasure. The strict or literal view holds that deliberate or intentional search precludes entitlement to the one-half

I would decide in favor of Marcelino since he is considered a finder by chance of the hidden treasure, hence, he is entitled to one-half (1/2) of the hidden treasure. While Marcelino may have had the intention to look for the hidden treasure, still he is a finder by chance since it is enough that he tried to look for it. By chance in the law does not mean sheer luck such that the finder should have no intention at all to look for the treasure. By chance means good luck, implying that one who intentionally looks for the treasure is embraced in the provision. The reason is that it is extremely difficult to find hidden treasure without looking for it deliberately. Marcelino is not a trespasser since there is no prohibition for him to enter the premises, hence, he is entitled to half of the treasure. ALTERNATIVE ANSWERS:

1. Marcelino did not find the treasure by chance because he had a map, he knew the location of the hidden treasure and he intentionally looked for the treasure, hence, he is not entitled to any part of the treasure. 2. Marcelino appears to be a trespasser and although there may be a question of whether he found it by chance or not, as he has found the hidden treasure by means of a treasure map, he will not be entitled to a finder's share. The hidden treasure shall belong to the owner. 3. The main rule is that hidden treasure belongs to the owner of the land, building or other property on which it is found. If it is found by chance by a third person and he is not a trespasser, he is entitled to one-half (1/2). If he is a trespasser, he loses everything. Mortgage; Pactum Commissorium (1999) Page 64 of 119

CIVIL LAW Answers to the BAR as Arranged by Topics (Year 1990-2006)

(a)

X borrowed money from Y and gave a piece of land as security by way of mortgage. It was expressly agreed between the parties in the mortgage contract that upon nonpayment of the debt on time by X, the mortgaged land would already belong to Y. If X defaulted in paying, would Y now become the owner of the mortgaged land? Why? (3%)

(b) Suppose in the preceding question, the agreement between X and Y was that if X failed to pay the mortgage debt on time, the debt shall be paid with the land mortgaged by X to Y. Would your answer be the same as in the preceding question? Explain. (3%) SUGGESTED ANSWER:

(a) No, Y would not become the owner of the land. The stipulation is in the nature of pactum commissorium which is prohibited by law. The property should be sold at public auction and the proceeds thereof applied to the indebtedness. Any excess shall be given to the mortgagor. SUGGESTED ANSWER:

(d) No, the answer would not be the same. This is a valid stipulation and does not constitute pactum commissorium. In pactum commissorium, the acquisition is automatic without need of any further action. In the instant problem another act is required to be performed, namely, the conveyance of the property as payment (dacion en pago). Mortgage; Pactum Commissorium (2001) To secure a loan obtained from a rural bank, Purita assigned her leasehold rights over a stall in the public market in favor of the bank. The deed of assignment provides that in case of default in the payment of the loan, the bank shall have the right to sell Purita's rights over the market stall as her attorney-in-fact, and to apply the proceeds to the payment of the loan. 1) Was the assignment of leasehold rights a mortgage or a cession? Why? (3%) 2) Assuming the assignment to be a mortgage, does the provision giving the bank the power to sell Purita's rights constitute pactum commissorium or not? Why? (2%) SUGGESTED ANSWER:

1) The assignment was a mortgage, not a cession, of the leasehold rights. A cession would have transferred ownership to the bank. However, the grant of authority to the bank to sell the leasehold rights in case of default is proof that no such ownership was transferred and that a mere encumbrance was constituted. There would have been no need for such authority had there been a cession. SUGGESTED ANSWER:

2) No, the clause in question is not a pactum commissorium. It is pactum commissorium when default in the payment of the loan automatically vests ownership of the encumbered property in the bank. In the problem given, the bank does not automatically become owner of the property upon default of the mortgagor. The bank has to sell the property and apply the proceeds to the indebtedness. Mortgage; Right of Redemption vs. Equity of Redemption (1999)

Are the right of redemption and the equity of redemption given by law to a mortgagor the same? Explain. (2%) SUGGESTED ANSWER:

The equity of redemption is different from the right of redemption. EQUITY OF REDEMPTION is the right of the mortgagor after judgment in a judicial foreclosure to redeem the property by paying to the court the amount of the judgment debt before the sale or confirmation of the sale. On the other hand, RIGHT OF REDEMPTION is the right of the mortgagor to redeem the property sold at an extra-judicial foreclosure by paying to the buyer in the foreclosure sale the amount paid by the buyer within one year from such sale. Nuisance; Family House; Not Nuisance per se (2006) A drug lord and his family reside in a small bungalow where they sell shabu and other prohibited drugs. When the police found the illegal trade, they immediately demolished the house because according to them, it was a nuisance per se that should be abated. Can this demolition be sustained? Explain. (5%) SUGGESTED ANSWER:

No, the demolition cannot be sustained. The house is not a nuisance per se or at law as it is not an act, occupation, or structure which is a nuisance at all times and under any circumstances, regardless of location or surroundings. A nuisance per se is a nuisance in and of itself, without regard to circumstances [Tolentino, p. 695, citing Wheeler v. River Falls Power Co., 215 Ala. 655, 111 So. 907].

Nuisance; Public Nuisance vs. Private Nuisance (2005) State with reason whether each of the following is a nuisance, and if so, give its classification, whether public or private: Article 694 of the Civil Code defines nuisance as any act, omission, establishment, business, condition or property, or anything else which injures or endangers the health or safety of others, or annoys or offends the senses, or shocks, defies or disregards decency or morality or obstructs or interferes with the free passage of any public highway or street or any body of water or hinders or impairs the use of property. It is a public nuisance if it affects a community or neighborhood or any considerable number of persons. It is a direct encroachment upon public rights or property which results injuriously to the public. It is a private nuisance, if it affects only a person or small number of persons. It violates only private rights. a) A squatter's hut (1%) If constructed on public streets or riverbeds, it is a public nuisance because it obstructs the free use by the public of said places. (City of Manila v. Garcia, G.R. No. L-26053, February 21,1967) If constructed on private land, it is a private nuisance because it hinders or impairs the use of the property by the owner. b) A swimming pool (1%) This is not a nuisance in the absence of any unusual condition or artificial feature other than the mere water. In Hidalgo Enterprises v. Balandan (G.R. No. L-3422, June 13, 1952), the Supreme Court ruled that a swimming pool is but

CIVIL LAW Answers to the BAR as Arranged by Topics (Year 1990-2006)

a duplication of nature — thus, could not be considered as a nuisance.

c) A house of prostitution (1%) Irrespective of its location and how its business is conducted, it is a nuisance since it defies, shocks and disregards decency and morality. It is a public nuisance because of its injury to the public. d) A noisy or dangerous factory in a private land (1%) If the noise injuriously affects the health and comfort of ordinary people in the vicinity to an unreasonable extent, it is a nuisance. It is a public nuisance because there is a tendency to annoy the public. (Velasco v. Manila Electric Co., G.R. No. L-18390, August 6, 1971)

e) Uncollected garbage (1%) It will become a nuisance if it substantially impairs the comfort and enjoyment of the adjacent occupants. The annoyance and the smell must be substantial as to interfere sensibly with the use and enjoyment by persons of ordinary sensibilities. It is a public nuisance because of its injury to the public. Ownership; Co-Ownership (1992) A, B and C are the co-owners in equal shares of a residential house and lot. During their co-ownership, the following acts were respectively done by the co-owners: 1) A undertook the repair of the foundation of the house, then tilting to one side, to prevent the house from collapsing. 2) B and C mortgaged the house and lot to secure a loan. 3) B engaged a contractor to build a concrete fence all around the lot. 4) C built a beautiful grotto in the garden. 5) A and C sold the land to X for a very good price. (a) Is A's sole decision to repair the foundation of the house binding on B and C? May A require B and C to contribute their 2/3 share of the expense? Reasons. (b) What is the legal effect of the mortgage contract executed by B and C? Reasons. (c) Is B's sole decision to build the fence binding upon A and C? May B require A and C to contribute their 2/ 3 share of the expense? Reasons. (d) Is C's sole decision to build the grotto binding upon A and B? May C require A and B to contribute their 2/ 3 share of the expense? Reasons. (e) What are the legal effects of the contract of sale executed by A. C and X? Reasons. SUGGESTED ANSWER:

(a) Yes. A's sole decision to repair the foundation is binding upon B and C. B and C must contribute 2/3 of the expense. Each co-owner has the right to compel the other co-owners to contribute to the expense of preservation of the thing (the house) owned in common in proportion to their respective interests (Arts. 485 and 488, Civil Code). SUGGESTED ANSWER:

(b) The mortgage shall not bind the 1/3 right and interest of A and shall be deemed to cover only the rights and interests of B and C in the house and lot. The mortgage shall be limited to the portion (2/3) which may be allotted to B and C in the partition (Art. 493, Civil Code). SUGGESTED ANSWER:

(c) B's sole decision to build the concrete fence is not binding upon A and C. Expenses to improve the thing owned in common must be decided upon by a majority of the co-owners who represent the controlling interest (Arts. 489 and 492. Civil Code). SUGGESTED ANSWER:

(d) C's sole decision to build the grotto is not binding upon A and B who cannot be required to contribute to the expenses for the embellishment of the thing owned in common if not decided upon by the majority of the coowners who represent the controlling interest (Arts. 489 and 492, Civil Code). SUGGESTED ANSWER:

(e) The sale to X shall not bind the 1/3 share of B and shall be deemed to cover only the 2/3 share of A and C in the land (Art. 493, Civil Code). B shall have the right to redeem the 2/3 share sold to X by A and C since X is a third person (Art. 1620, Civil Code). Ownership; Co-Ownership; Prescription (2000) In 1955, Ramon and his sister Rosario inherited a parcel of land in Albay from their parents. Since Rosario was gainfully employed in Manila, she left Ramon alone to possess and cultivate the land. However, Ramon never shared the harvest with Rosario and was even able to sell one-half of the land in 1985 by claiming to be the sole heir of his parents. Having reached retirement age in 1990 Rosario returned to the province and upon learning what had transpired, demanded that the remaining half of the land be given to her as her share. Ramon opposed, asserting that he has already acquired ownership of the land by prescription, and that Rosario is barred by laches from demanding partition and reconveyance. Decide the conflicting claims. (5%) SUGGESTED ANSWER:

Ramon is wrong on both counts: prescription and laches. His possession as co-owner did not give rise to acquisitive prescription. Possession by a co-owner is deemed not adverse to the other co-owners but is, on the contrary, deemed beneficial to them (Pongon v. GA, 166 SCRA 375). Ramon's possession will become adverse only when he has repudiated the co-ownership and such repudiation was made known to Rosario. Assuming that the sale in 1985 where Ramon claimed he was the sole heir of his parents amounted to a repudiation of the co-ownership, the prescriptive period began to run only from that time. Not more than 30 years having lapsed since then, the claim of Rosario has not as yet prescribed. The claim of laches is not also meritorious. Until the repudiation of the co-ownership was made known to the other co-owners, no right has been violated for the said co-owners to vindicate. Mere delay in vindicating the right, standing alone, does not constitute laches.

CIVIL LAW Answers to the BAR as Arranged by Topics (Year 1990-2006) ALTERNATIVE ANSWER:

Ramon has acquired the land by acquisitive prescription, and because of laches on the part of Rosario. Ramon's possession of the land was adverse because he asserted sole ownership thereof and never shared the harvest therefrom. His adverse possession having been continuous and uninterrupted for more than 30 years, Ramon has acquired the land by prescription. Rosario is also guilty of laches not having asserted her right to the harvest for more than 40 years. Ownership; Co-Ownership; Prescription (2002) Senen and Peter are brothers. Senen migrated to Canada early while still a teenager. Peter stayed in Bulacan to take care of their widowed mother and continued to work on the Family farm even after her death. Returning to the country some thirty years after he had left, Senen seeks a partition of the farm to get his share as the only co-heir of Peter. Peter interposes his opposition, contending that acquisitive prescription has already set in and that estoppel lies to bar the action for partition, citing his continuous possession of the property for at least 10 years, for almost 30 years in fact. It is undisputed that Peter has never openly claimed sole ownership of the property. If he ever had the intention to do so, Senen was completely ignorant of it. Will Senen’s action prosper? Explain. (5%). SUGGESTED ANSWER:

Senen’s action will prosper. Article 494 of the New Civil Code provides that ―no prescription shall run in favor of a co-owner or co-heir against his co-owners or co-heirs so long as he expressly or impliedly recognizes the coownership nor notified Senen of his having repudiated the same. ALTERNATIVE ANSWER:

Senen’s action will prosper. This is a case of implied trust. (Art 1441, NCC) For purposes of prescription under the concept of an owner (Art. 540, NCC). There is no such concept here. Peter was a co-owner, he never claimed sole ownership of the property. He is therefore estopped under Art. 1431, NCC. Ownership; Co-Ownership; Redemption (1993) In 1937, A obtained a loan of P20,000.00 from the National City Bank of New York, an American-owned bank doing business in the Philippines. To guarantee payment of his obligation, A constituted a real estate mortgage on his 30hectare parcel of agricultural land. In 1939, before he could pay his obligation. A died intestate leaving three children. B, a son by a first marriage, and C and D, daughters by a second marriage. In 1940, the bank foreclosed the mortgage for non-payment of the principal obligation. As the only bidder at the extrajudicial foreclosure sale, the bank bought the property and was later issued a certificate of sale. The war supervened in 1941 without the bank having been able to obtain actual possession of the property which remained with A's three children who appropriated for themselves the income from it. In 1948, B bought the property from the bank using the money he received as back pay from the U. S. Government, and utilized the same in agribusiness. In 1960, as B's business flourished, C and D sued B for partition and accounting of the income of the property, claiming that as heirs of their father they were co-owners

thereof and offering to reimburse B for whatever he had paid in purchasing the property from the bank. In brief, how will you answer the complaint of C and D, if you were engaged by D as his counsel?

SUGGESTED ANSWER:

As counsel of B, I shall answer the complaint as follows: When B bought the property, it was not by a right of redemption since the period therefore had already expired. Hence, B bought the property in an independent unconditional sale. C and D are not co-owners with B of the property. Therefore, the suit of C and D cannot prosper. ALTERNATIVE ANSWER:

As counsel of B, I shall answer the complaint as follows: From the facts described, it would appear that the Certificate of sale has not been registered. The one-year period of redemption begins to run from registration. In this case, it has not yet even commenced. Under the Rules of Court, the property may be released by the Judgment debtor or his successor in interest. (Sec. 29, Rule 27). It has been held that this includes a joint owner. (Ref. Magno vs.Ciola, 61 Phil. 80). Ownership; Co-Ownership; Redemption (2000) Ambrosio died, leaving his three daughters, Belen, Rosario and Sylvia a hacienda which was mortgaged to the Philippine National Bank due to the failure of the daughters to pay the bank, the latter foreclosed the mortgage and the hacienda was sold to it as the highest bidder. Six months later, Sylvia won the grand prize at the lotto and used part of it to redeem the hacienda from the bank. Thereafter, she took possession of the hacienda and refused to share its fruits with her sisters, contending that it was owned exclusively by her, having bought it from the bank with her own money. Is she correct or not? (3%) SUGGESTED ANSWER:

Sylvia is not correct. The 3 daughters are the co-owners of the hacienda being the only heirs of Ambrosio. When the property was foreclosed, the right of redemption belongs also to the 3 daughters. When Sylvia redeemed the entire property before the lapse of the redemption period, she also exercised the right of redemption of her co-owners on their behalf. As such she is holding the shares of her two sisters in the property, and all the fruits corresponding thereto, in trust for them. Redemption by one co-owner inures to the benefit of all (Adille v. CA.157 SCRA 455). Sylvia, however, is entitled to be reimbursed the shares of her two sisters in the redemption price. Ownership; Co-Ownership; Redemption (2002) Antonio, Bart, and Carlos are brothers. They purchased from their parents specific portions of a parcel of land as evidenced by three separates deeds of sale, each deed referring to a particular lot in meter and bounds. When the deeds were presented for registration, the Register of Deeds could not issue separate certificates of Title had to be issued, therefore, in the names of three brothers as coowners of the entire property. The situation has not changed up to now, but each of the brothers has been receiving rentals exclusively from the lot actually purchased by him. Antonio sells his lot to a third person, with notice to his brothers. To enable the buyer to secure a new title in

CIVIL LAW Answers to the BAR as Arranged by Topics (Year 1990-2006)

his name, the deed of sale was made to refer to undivided interest in the property of the seller (Antonio), with the metes and bounds of the lot sold being stated. Bart and Carlos reacted by signifying their exercise of their right of redemption as co owners. Antonio in his behalf and in behalf of his buyer, contends that they are no longer coowners, although the title covering the property has remained in their names as such. May Bart and Carlos still redeem the lot sold by Antonio? Explain. (5%) SUGGESTED ANSWER:

No, they may not redeem because there was no Coownership among Antonio, Bart, and Carlos to start with. Their parents already partitioned the land in selling separate portions to them. The situation is the same as in the case Si v. Court of Appeals, (342 SCRA 653 [2000]).

Possession (1998) Using a falsified manager's check, Justine, as the buyer, was able to take delivery of a second hand car which she had just bought from United Car Sales Inc. The sale was registered with the Land Transportation Office. A week later, the seller learned that the check had been dishonored, but by that time, Justine was nowhere to be seen. It turned out that Justine had sold the car to Jerico, the present possessor who knew nothing about the falsified check. In a suit by United Car Sales, Inc. against Jerico for recovery of the car, plaintiff alleges it had been unlawfully deprived of its property through fraud and should, consequently, be allowed to recover it without having to reimburse the defendant for the price the latter had paid. Should the suit prosper? [5%] SUGGESTED ANSWER:

The suit should prosper as to the recovery of the car. However, since Jerico was not guilty of any fraud and appears to be an innocent purchaser for value, he should be reimbursed for the price he paid. This is without prejudice to United Car Sales, Inc. right of action against Justine. As between two innocent parties, the party causing the injury should suffer the loss. Therefore, United Car Sales, Inc. should suffer the loss. ALTERNATIVE ANSWER:

Yes, the suit will prosper because the criminal act of estafa should be deemed to come within the meaning of unlawful deprivation under Art. 559, Civil Code, as without it plaintiff would not have parted with the possession of its car. ANOTHER ANSWER:

No, the suit will not prosper. The sale is valid and Jerico is a buyer in good faith. ANOTHER ANSWER:

Under the law on Sales, when the thing sold is delivered by the seller to the buyer without reservation of ownership, the ownership is transferred to the buyer. Therefore in the suit of United Car Sales, Inc. against Jerico for the recovery of the car, the plaintiff should not be allowed to recover the car without reimbursing the defendant for the price that the latter paid. (EDCA Publishing and Distributing Corp. vs. Santos, 184 SCRA 614, April 26, 1990)

Property; Real vs. Personal Property (1995)

Salvador, a timber concessionaire, built on his lot a warehouse where he processes and stores his timber for shipment. Adjoining the warehouse is a furniture factory owned by NARRAMIX of which Salvador is a majority stockholder. NARRAMIX leased space in the warehouse where it placed its furniture-making machinery. 1. How would you classify the furniture-making machinery as property under the Civil Code? Explain. 2. Suppose the lease contract between Salvador and NARRAMIX stipulates that at the end of the lease the machinery shall become the property of the lessor, will your answer be the same? Explain.

SUGGESTED ANSWER:

1. The furniture-making machinery is movable property because it was not installed by the owner of the tenement. To become immovable under Art. 415 (5) of the NCC, the machinery must be installed by the owner of the tenement. ALTERNATIVE ANSWER:

It depends on the circumstances of the case. If the machinery was attached in a fixed manner, in such a way that it cannot be separated from the tenement without breaking the material or causing deterioration thereof, it is immovable property [Art. 415 (3), NCC]. However, if the machinery can be transported from place to place without impairment of the tenement to which they were fixed, then it is movable property. [Art. 416 (4), NCC] SUGGESTED ANSWER:

2. It is immovable property. When there is a provision in the lease contract making the lessor, at the end of the lease, owner of the machinery installed by the lessee, the said machinery is considered to have been installed by the lessor through the lessee who acted merely as his agent. Having been installed by the owner of the tenement, the machinery became immovable .under Art. 415 of the NCC. (Davao Sawmill v. Castillo 61 Phil. 709)

Property; Real vs. Personal Property (1997) Pedro is the registered owner of a parcel of land situated in Malolos, Bulacan. In 1973, he mortgaged the land to the Philippine National Bank (PNB) to secure a loan of P100.000.00. For Pedro's failure to pay the loan, the PNB foreclosed on the mortgage in 1980, and the land was sold at public auction to PNB for being the highest bidder. PNB secured title thereto in 1987. In the meanwhile, Pedro, who was still in possession of the land, constructed a warehouse on the property. In 1988, the PNB sold the land to Pablo, the Deed of Sale was amended in 1989 to include the warehouse. Pedro, claiming ownership of the warehouse, files a complaint to annul the amended Deed of Sale before the Regional Trial Court of Quezon City, where he resides, against both the PNB and Pablo. The PNB filed a motion to dismiss the complaint for improper venue contending that the warehouse is real property under Article 415(1) of the Civil Code and therefore the action should have instead been filed in Malolos, Bulacan. Pedro claims otherwise. The question arose as to whether the warehouse should be considered as real or as personal property.

CIVIL LAW Answers to the BAR as Arranged by Topics (Year 1990-2006)

If consulted, what would your legal advice be? SUGGESTED ANSWER:

The warehouse which is a construction adhered to the soil is an immovable by nature under Art. 415 (1) and the proper venue of any case to recover ownership of the same, which is what the purpose of the complaint to annul the amended Deed of Sale amounts to, should be the place where the property is located, or the RTC of Bulacan. ADDITIONAL ANSWERS:

1. Buildings are always immovable property, and even in the instances where the parties to a contract seem to have dealt with it separate and apart from the land on which it stood in no wise does it change its character as immovable property. A building is an immovable even if not erected by the owner of the land. The only criterion is union or incorporation with the soil. (Ladera vs. Hodges (CA) 48

O.G. 4374) (Reyes and Puno, Outline of Philippine Civil Law, Vol. 2. p.7)

2. The warehouse built by Pedro on the mortgaged property is real property within the context of Article 415 of the New Civil Code, although it was built by Pedro after the foreclosure sale without the knowledge and consent of the new owner which makes him a builder in bad faith, this does not alter the character of the warehouse as a real property by incorporation. It is a structure which cannot be removed without causing injury to the land. So, my advice to Pedro is to file the case with the RTC of Bulacan, the situs of the property, (Note: If the examinee does not mention that the structure was built by a builder in bad faith, it should be given full credit).

Sower; Good Faith/ Bad Faith (2000) Felix cultivated a parcel of land and planted it to sugar cane, believing it to be his own. When the crop was eight months old, and harvestable after two more months, a resurvey of the land showed that it really belonged to Fred. What are the options available to Fred? (2%) SUGGESTED ANSWER:

As to the pending crops planted by Felix in good faith, Fred has the option of allowing Felix to continue the cultivation and to harvest the crops, or to continue the cultivation and harvest the crops himself. In the latter option, however, Felix shall have the right to a part of the expenses of cultivation and to a part of the net harvest, both in proportion to the time of possession. (Art. 545 NCC), ALTERNATIVE ANSWER:

Since sugarcane is not a perennial crop. Felix is considered a sower in good faith. Being so, Art. 448 applies. The options available to Fred are: (a) to appropriate the crop after paying Felix the indemnity under Art. 546, or (b) to require Felix to pay rent. Usufruct (1997) On 1 January 1980, Minerva, the owner of a building, granted Petronila a usufruct over the property until 01 June 1998 when Manuel, a son of Petronila, would have reached his 30th birthday. Manuel, however, died on 1 June 1990 when he was only 26 years old. Minerva notified Petronila that the usufruct had been extinguished by the death of Manuel and demanded that the

latter vacate the premises and deliver the same to the former. Petronila refused to vacate the place on the ground that the usufruct in her favor would expire only on 1 June 1998 when Manuel would have reached his 30th birthday and that the death of Manuel before his 30th birthday did not extinguish the usufruct. Whose contention should be accepted?

SUGGESTED ANSWER:

Petronila's contention is correct. Under Article 606 of the Civil Code, a usufruct granted for the time that may elapse before a third person reaches a certain age shall subsist for the number of years specified even if the third person should die unless there is an express stipulation in the contract that states otherwise. In the case at bar, there is no express stipulation that the consideration for the usufruct is the existence of Petronila's son. Thus, the general rule and not the exception should apply in this case. ALTERNATIVE ANSWER:

This is a usufruct which is clearly intended for the benefit of Manuel until he reaches 30 yrs. of age with Petronila serving only as a conduit, holding the property in trust for his benefit. The death of Manuel at the age of 26 therefore, terminated the usufruct.

LAND TRANSFER & DEEDS Acquisition of Lands; Citizenship Requirement (2003) In 1970, the spouses Juan and Juana de la Cruz, then Filipinos, bought the parcel of unregistered land in the Philippines on which they built a house which became their residence. In 1986, they migrated to Canada and became Canadian citizens. Thereafter, in 1990, they applied, opposed by the Republic, for the registration of the aforesaid land in their names. Should the application of the spouses de la Cruz be granted over the Republic’s opposition? Why? 5% SUGGESTED ANSWER:

Yes, the application should be granted. As a rule, the Constitution prohibits aliens from owning private lands in the Philippines. This rule, however, does not apply to the spouses Juan and Juana de la Cruz because at the time they acquired ownership over the land, albeit imperfect, they were still Filipino citizens. The application for registration is a mere confirmation of the imperfect title which the spouses have already acquired before they became Canadian citizens. (Republic v. CA, 235 SCRA 567 [1994]).

Adverse Claims; Notice of Levy (1998) Section 70 of Presidential Decree No. 1529, concerning adverse claims on registered land, provides a 30-day period of effectivity of an adverse claim, counted from the date of its registration. Suppose a notice of adverse claim based upon a contract to sell was registered on March 1, 1997 at the instance of the BUYER, but on June 1, 1997, or after the lapse of the 30-day period, a notice of levy on execution in favor of a JUDGMENT CREDITOR was also registered to enforce a final judgment for money against the registered owner. Then, on June 15, 1997 there having been no formal cancellation of his notice of adverse claim, the BUYER pays

CIVIL LAW Answers to the BAR as Arranged by Topics (Year 1990-2006)

to the seller-owner the agreed purchase price in full and registers the corresponding deed of sale. Because the annotation of the notice of levy is carried over to the new title in his name, the BUYER brings an action against the JUDGMENT CREDITOR to cancel such annotation, but the latter claims that his lien is superior because it was annotated after the adverse claim of the BUYER had ipso facto ceased to be effective. Will the suit prosper? [5%]

residential, commercial, industrial, or similar productive purposes, and only by lease when not needed by the government for public service. (2) If the land is suited or actually used for fishpond or aquaculture purposes, it comes under the Jurisdiction of the Bureau of Fisheries and Aquatic Resources (BFAR) and can only be acquired by lease. (P.D. 705)

SUGGESTED ANSWER:

The suit will prosper. While an adverse claim duly annotated at the back of a title under Section 7O of P.D. 1529 is good only for 30 days, cancellation thereof is still necessary to render it ineffective, otherwise, the inscription thereof will remain annotated as a lien on the property. While the life of adverse claim is 3O days under P.D. 1529, it continuous to be effective until it is canceled by formal petition filed with the Register of Deeds. The cancellation of the notice of levy is justified under Section 108 of P.D. 1529 considering that the levy on execution can not be enforced against the buyer whose adverse claim against the registered owner was recorded ahead of the notice of levy on execution. Annotation of Lis Pendens; When Proper (2001) Mario sold his house and lot to Carmen for P1 million payable in five (5) equal annual installments. The sale was registered and title was issued in Carmen's name. Carmen failed to pay the last three installments and Mario filed an. action for collection, damages and attorneys fees against her. Upon filing of the complaint, he caused a notice of lis pendens to be annotated on Carmen's title. Is the notice of lis pendens proper or not? Why? (5%)

(3) Free Patent is a mode of concession under Section 41, Chapter VII of the Public Land Act, which is applicable only for agricultural lands. (4) The certificate of the district forester that the land is already "alienable and disposable" simply means that the land is no longer needed for forest purposes, but the Bureau of Lands could no longer dispose of it by free patent because it is already covered by a lease contract between BFAR and Regina. That contract must be respected. (5) The free patent of Jorge is highly irregular and void ab initio, not only because the Bureau has no statutory authority to issue a free patent over a foreshore area, but also because of the false statements made in his sworn application that he has occupied and cultivated the land since July 4, 1945, as required by the free patent law. Under Section 91 of the Public Land Act, any patent concession or title obtained thru false representation is void ab initio. In cases of this nature, it is the government that shall institute annulment proceedings considering that the suit carries with it a prayer for the reversion of the land to the state. However, Regina is a party in interest and the case will prosper because she has a lease contract for the same land with the government.

SUGGESTED ANSWER:

The notice of lis pendens is not proper for the reason that the case filed by Mario against Carmen is only for collection, damages, and attorney's fees. Annotation of a lis pendens can only be done in cases involving recovery of possession of real property, or to quiet title or to remove cloud thereon, or for partition or any other proceeding affecting title to the land or the use or occupation thereof. The action filed by Mario does not fall on anyone of these. Foreshore Lands (2000) Regina has been leasing foreshore land from the Bureau of Fisheries and Aquatic Resources for the past 15 years. Recently, she learned that Jorge was able to obtain a free patent from the Bureau of Agriculture, covering the same land, on the basis of a certification by the District Forester that the same is already "alienable and disposable". Moreover, Jorge had already registered the patent with the Register of Deeds of the province, and he was issued an Original Certificate of Title for the same. Regina filed an action for annulment of Jorge's title on the ground that it was obtained fraudulently. Will the action prosper? (2%) SUGGESTED ANSWER:

An action for the annulment of Jorge's Original Certificate of Title will prosper on the following grounds: (1) Under Chapter IX of C .A, No. 141, otherwise known as the Public Land Act, foreshore lands are disposable for

Forgery; Innocent Purchaser; Holder in Bad Faith (2005) Rod, the owner of an FX taxi, found in his vehicle an envelope containing TCT No. 65432 over a lot registered in Cesar's name. Posing as Cesar, Rod forged Cesar's signature on a Deed of Sale in Rod's favor. Rod registered the said document with the Register of Deeds, and obtained a new title in his name. After a year, he sold the lot to Don, a buyer in good faith and for value, who also registered the lot in his name. a) Did Rod acquire title to the land? Explain. (2%) SUGGESTED ANSWER:

No, Rod did not acquire title to the land. The inscription in the registry, to be effective, must be made in good faith. The defense of indefeasibility of a Torrens Title does not extend to a transferee who takes the certificate of title with notice of a flaw. A holder in bad faith of a certificate of title is not entitled to the protection of the law, for the law cannot be used as a shield for frauds. (Samonte v. Court of Appeals, G.R. No. 104223, July 12, 2001)

In the case at bar, Rod only forged Cesar's signature on the -Deed of Sale. It is very apparent that there was bad faith on the part of Rod from the very beginning. As such, he is not entitled to the protection of the Land Registration Act. b) Discuss the rights of Don, if any, over the property. (2%) SUGGESTED ANSWER:

CIVIL LAW Answers to the BAR as Arranged by Topics (Year 1990-2006)

It is a well-known rule in this jurisdiction that persons dealing with registered land have the legal right to rely on the face of the Torrens Certificate of Title and to dispense with the need to inquire further, except when the party concerned has actual knowledge of facts and circumstances that would impel a reasonably cautious man to make such inquiry.

(Naawan Community Rural Bank v. Court of Appeals, G.R. No. 128573, January 13, 2003)

In the given problem, the property was already registered in the name of Rod when he bought the same from the latter. Thus, Don could be considered as a buyer in good faith and for value. However, since Rod did not actually sell any property to him, Don has no right to retain ownership over the property. He has only the right to recover the purchase price plus damages. Forgery; Innocent Purchaser; Mirror Principle (1991) Bruce is the registered owner, of a parcel of land with a building thereon and is in peaceful possession thereof. He pays the real estate taxes and collects the rentals therefrom. Later, Catalino, the only brother of Bruce, filed a petition where he, misrepresenting to be the attorney-in-fact of Bruce and falsely alleging that the certificate of title was lost, succeeded in obtaining a second owner's duplicate copy of the title and then had the same transferred in his name through a simulated deed of sale in his favor. Catalino then mortgaged the property to Desiderio who had the mortgage annotated on the title. Upon learning of the fraudulent transaction, Bruce filed a complaint against Catalino and Desiderio to have the title of Catalino and the mortgage in favor of Desiderio declared null and void. Will the complaint prosper, or will the title of Catalino and the mortgage to Desiderio be sustained? SUGGESTED ANSWER:

The complaint for the annulment of Catalino's Title will prosper. In the first place, the second owner's copy of the title secured by him from the Land Registration Court is void ab initio, the owner's copy thereof having never been lost, let alone the fact that said second owner's copy of the title was fraudulently procured and improvidently issued by the Court. In the second place, the Transfer Certificate of Title procured by Catalino is equally null and void, it having been issued on the basis of a simulated or forged Deed of Sale. A forged deed is an absolute nullity and conveys no title. The mortgage in favor of Desiderio is likewise null and void because the mortgagor is not the owner of the mortgaged property. While it may be true that under the "Mirror Principle" of the Torrens System of Land Registration, a buyer or mortgagee has the right to rely on what appears on the Certificate of Title, and in the absence of anything to excite suspicion, is under no obligation to look beyond the certificate and investigate the mortgagor's title, this rule does not find application in the case at hand because here. Catalino's title suffers from two fatal infirmities, namely: a) The fact that it emanated from a forged deed of a simulated sale; b) The fact that it was derived from a fraudulently procured or improvidently issued second owner's copy, the real owner's copy being still intact and in the possession of the true owner, Bruce.

The mortgage to Desiderio should be cancelled without prejudice to his right to go after Catalino and/or the government for compensation from the assurance fund. Fraud; Procurement of Patent; Effect (2000) In 1979, Nestor applied for and was granted a Free Patent over a parcel of agricultural land with an area of 30 hectares, located in General Santos City. He presented the Free Patent to the Register of Deeds, and he was issued a corresponding Original Certificate of Title (OCT) No. 375, Subsequently, Nestor sold the land to Eddie. The deed of sale was submitted to the Register of Deeds and on the basis thereof, OCT No, 375 was cancelled and Transfer Certificate of Title (TCT) No. 4576 was issued in the name of Eddie. In 1986, the Director of Lands filed a complaint for annulment of OCT No, 375 and TCT No. 4576 on the ground that Nestor obtained the Free Patent through fraud. Eddie filed a motion to dismiss on the ground that he was an innocent purchaser for value and in good faith and as such, he has acquired a title to the property which is valid, unassailable and indefeasible. Decide the motion. (5%)

SUGGESTED ANSWER:

The motion of Nestor to dismiss the complaint for annulment of O.C.T. No. 375 and T.C.T. No. 4576 should be denied for the following reasons: 1) Eddie cannot claim protection as an innocent purchaser for value nor can he interpose the defense of indefeasibility of his title, because his TCT is rooted on a void title. Under Section 91 of CA No. 141, as amended, otherwise known as the Public Land Act, statements of material facts in the applications for public land must be under oath. Section 91 of the same act provides that such statements shall be considered as essential conditions and parts of the concession, title, or permit issued, any false statement therein, or omission of facts shall ipso facto produce the cancellation of the concession. The patent issued to Nestor in this case is void ab initio not only because it was obtained by fraud but also because it covers 30 hectares which is far beyond the maximum of 24 hectares provided by the free patent law. 2) The government can seek annulment of the original and transfer certificates of title and the reversion of the land to the state. Eddie's defense is untenable. The protection afforded by the Torrens System to an innocent purchaser for value can be availed of only if the land has been titled thru judicial proceedings where the issue of fraud becomes academic after the lapse of one (1) year from the issuance of the decree of registration. In public land grants, the action of the government to annul a title fraudulently obtained does not prescribe such action and will not be barred by the transfer of the title to an innocent purchaser for value. Homestead Patents; Void Sale (1999) In 1950, the Bureau of Lands issued a Homestead patent to A. Three years later, A sold the homestead to B. A died in 1990, and his heirs filed an action to recover the homestead from B on the ground that its sale by their father to the latter is void under Section 118 of the Public Land Law. B contends, however, that the heirs of A cannot recover the Page 71 of 119

CIVIL LAW Answers to the BAR as Arranged by Topics (Year 1990-2006)

homestead from him anymore because their action has prescribed and that furthermore, A was in pari delicto. Decide. (5%) SUGGESTED ANSWER:

The sale of the land by A to B 3 years after issuance of the homestead patent, being in violation of Section 118 of the Public Land Act, is void from its inception. The action filed by the heirs of B to declare the nullity or inexistence of the contract and to recover the land should be given due course. B's defense of prescription is untenable because an action which seeks to declare the nullity or inexistence of A contract does not prescribe. (Article 1410; Banaga vs. Soler, 2 8CRA 765)

On the other hand, B's defense of pari delicto is equally untenable. While as a rule, parties who are in pari delicto have no recourse against each other on the principle that a transgressor cannot profit from his own wrongdoing, such rule does not apply to violations of Section 118 of the Public Land Act because of the underlying public policy in the said Act "to conserve the land which a homesteader has acquired by gratuitous grant from the government for himself and his family". In keeping with this policy, it has been held that one who purchases a homestead within the five-year prohibitory period can only recover the price which he has paid by filing a claim against the estate of the deceased seller (Labrador vs. Delos Santos 66 Phil. 579) under the principle that no one shall enrich himself at the expense of another. Applying the pari delicto rule to violation of Section 118 of the Public Land Act, the Court of Appeals has ruled that "the homesteader suffers the loss of the fruits realized by the vendee who in turn forfeits the improvement that he has introduced into the land." (Obot vs. SandadiUas, 69 OG, April 35, 1966} FIRST ALTERNATIVE ANSWER:

The action to declare the nullity of the sale did not prescribe (Art. 1410}, such sale being one expressly prohibited and declared void by the Public Lands Act [Art. 1409, par. (7)]. The prohibition of the law is clearly for the protection of the heirs of A such that their recovering the property would enhance the public policy regarding ownership of lands acquired by homestead patent (Art. 1416). The defense of pari delicto is not applicable either, since the law itself allows the homesteader to reacquire the land even if it has been sold. SECOND ALTERNATIVE ANSWER:

Prescription does not arise with respect to actions to declare a void contract a nullity (Article 1410). Neither is the doctrine of pari delicto applicable because of public policy. The law is designed for the protection of the plaintiff so as to enhance the public policy of the Public Land Act to give land to the landless. If the heirs are not allowed to recover, it could be on the ground of laches inasmuch as 40 years had elapsed and the owner had not brought any action against B especially if the latter had improved the land. It would be detrimental to B if the plaintiff is allowed to recover. Innocent Purchaser for Value (2001)

Cesar bought a residential condominium unit from High Rise Co. and paid the price in full. He moved into the unit, but somehow he was not given the Condominium Certificate of Title covering the property. Unknown to him, High Rise Co. subsequently mortgaged the entire condominium building to Metrobank as security for a loan of P500 million. High Rise Co. failed to pay the loan and the bank foreclosed the mortgage. At the foreclosure sale, the bank acquired the building, being the highest bidder. When Cesar learned about this, he filed an action to annul the foreclosure sale insofar as his unit was concerned. The bank put up the defense that it relied on the condominium certificates of title presented by High Rise Co., which were clean. Hence, it was a mortgagee and buyer in good faith. Is this defense tenable or not? Why? (5%.)

SUGGESTED ANSWER:

Metrobank's defense is untenable. As a rule, an innocent purchaser for value acquires a good and a clean title to the property. However, it is settled that one who closes his eyes to facts that should put a reasonable man on guard is not an innocent purchaser for value. In the present problem the bank is expected, as a matter of standard operating procedure, to have conducted an ocular inspection, of the promises before granting any loan. Apparently, Metrobank did not follow this procedure. Otherwise, it should have discovered that the condominium unit in question was occupied by Cesar and that fact should have led it to make further inquiry. Under the circumstances, Metrobank cannot be considered a mortgagee and buyer in good faith. Mirror Principle (1990) In 1950's, the Government acquired a big landed estate in Central Luzon from the registered owner for subdivision into small farms and redistribution of bonafide occupants, F was a former lessee of a parcel of land, five hectares in area. After completion of the resurvey and subdivision, F applied to buy the said land in accordance with the guidelines of the implementing agency. Upon full payment of the price in 1957, the corresponding deed of absolute sale was executed in his favor and was registered, and in 1961, a new title was issued in his name. In 1963, F sold the said land to X; and in 1965 X sold it to Y, new titles were successively issued in the names of the said purchasers. In 1977, C filed an action to annul the deeds of sale to F, X and Y and their titles, on the ground that he (C) had been in actual physical possession of the land, and that the sale to F and the subsequent sales should be set aside on the ground of fraud. Upon motion of defendants, the trial court dismissed the complaint, upholding their defenses of their being innocent purchasers for value, prescription and laches. Plaintiff appealed. (a) Is the said appeal meritorious? Explain your (b) Suppose the government agency concerned joined C in answer filing the said action against the defendants, would that change the result of the litigation? Explain. SUGGESTED ANSWER:

(a) The appeal is not meritorious. The trial court ruled correctly in granting defendant's motion to dismiss for the following reasons: 1. While there is the possibility that F, a former lessee of the land was aware of the fact that C was the bona fide

CIVIL LAW Answers to the BAR as Arranged by Topics (Year 1990-2006)

occupant thereof and for this reason his transfer certificate of title may be vulnerable, the transfer of the same land and the issuance of new TCTs to X and Y who are innocent purchasers for value render the latter's titles indefeasible. A person dealing with registered land may safely rely on the correctness of the certificate of title and the law will not in any way oblige him to go behind the certificate to determine the condition of the property in search for any hidden defect or inchoate right which may later invalidate or diminish the right to the land. This is the mirror principle of the Torrens System of land registration. 1. 2.

required to explore beyond what the record in the registry indicates on its face in quest for any hidden defect or inchoate right which may subsequently defeat his right thereto. This is the "mirror principle' of the Torrens system which makes it possible for a forged deed to be the root of a good title. Besides, it appears that spouses X and Y are guilty of contributory negligence when they delivered this OCT to the mortgagee without annotating the mortgage thereon. Between them and the innocent purchaser for value, they should bear the loss.

The action to annul the sale was instituted in 1977 or more ALTERNATIVE ANSWER: than (10) years from the date of execution thereof in 1957, If the buyer B, who relied on the teller A's title, was not aware of the adverse possession of the land by the spouses X hence, it has long prescribed. Under Sec 45 of Act 496, ―the entry of a certificate of title and Y, then the latter cannot recover the property from shall be regarded as an agreement running with the land, and B. B has in his favor the presumption of good faith which binding upon the applicant and all his successors in title that can only be overthrown by adequate proof of bad faith. the land shall be and always remain registered land. A title However, nobody buys land without seeing the property, hence, B could not have been unaware of such adverse under Act 496 is indefeasible and to preserve that character, the title is cleansed anew with every transfer for value (De Jesus v possession. If after learning of such possession, B simply City of Manila; 29 Phil. 73; Laperal v City of Manila, 62 Phil 313;closed his eyes and did nothing about it, then the suit for reconveyance will prosper as the buyer's bad faith will have Penullar v PNB 120 S 111). become evident.

SUGGESTED ANSWER:

(b) Even if the government joins C, this will not alter the outcome of the case so much because of estoppel as an express provision in Sec 45 of Act 496 and Sec 31 of PD 1529 that a decree of registration and the certificate of title issued in pursuance thereof ―shall be conclusive upon and against all persons, including the national government and all branches thereof, whether mentioned by name in the application or not.‖ Mirror Principle; Forgery; Innocent Purchaser (1999) The spouses X and Y mortgaged a piece of registered land to A, delivering as well the OCT to the latter, but they continued to possess and cultivate the land, giving 1/2 of each harvest to A in partial payment of their loan to the latter, A, however, without the knowledge of X and Y, forged a deed of sale of the aforesaid land in favor of himself, got a TCT in his name, and then sold the land to B, who bought the land relying on A's title, and who thereafter also got a TCT in his name. It was only then that the spouses X and Y learned that their land had been titled in B's name. May said spouses file an action for reconveyance of the land in question against b? Reason. (5%) SUGGESTED ANSWER:

The action of X and Y against B for reconveyance of the land will not prosper because B has acquired a clean title to the property being an innocent purchaser for value. A forged deed is an absolute nullity and conveys no title. The fact that the forged deed was registered and a certificate of title was issued in his name, did not operate to vest upon an ownership over the property of X and Y. The registration of the forged deed will not cure the infirmity. However, once the title to the land is registered in the name of the forger and title to the land thereafter falls into the hands of an innocent purchaser for value, the latter acquires a clean title thereto. A buyer of a registered land is not

Notice of Lis Pendens (1995) Rommel was issued a certificate of title over a parcel of land in Quezon City. One year later Rachelle, the legitimate owner of the land, discovered the fraudulent registration obtained by Rommel. She filed a complaint against Rommel for reconveyance and caused the annotation of a notice of lis pendens on the certificate of title issued to Rommel. Rommel now invokes the indefeasibility of his title considering that one year has already elapsed from its issuance. He also seeks the cancellation of the notice of Lis pendens. May the court cancel the notice of lis pendens even before final judgment is rendered? Explain. SUGGESTED ANSWER:

A Notice of Lis Pendens may be canceled even before final Judgment upon proper showing that the notice is for the purpose of molesting or harassing the adverse party or that the notice of lis pendens is not necessary to protect the right of the party who caused it to be registered. (Section 77, P.D. No. 1529) In this case, it is given that Rachelle is the legitimate owner of the land in question. It can be said, therefore, that when she filed her notice of lis pendens her purpose was to protect her interest in the land and not just to molest Rommel. It is necessary to record the Lis pendens to protect her interest because if she did not do it, there is a possibility that the land will fall into the hands of an innocent purchaser for value and in that event, the court loses control over the land making any favorable judgment thereon moot and academic. For these reasons, the notice of lis pendens may not be canceled. Notice of Lis Pendens; Transferee Pendente Lite (2002) Sancho and Pacifico are co-owners of a parcel of land. Sancho sold the property to Bart. Pacifico sued Sancho and Bart for annulment of the sale and reconveyance of the

CIVIL LAW Answers to the BAR as Arranged by Topics (Year 1990-2006)

property based on the fact that the sale included his onehalf pro-indiviso share. Pacifico had a notice of lis pendens annotated on the title covering the property and ordered the cancellation of the notice of lis pendens. The notice of lis pendens could not be cancelled immediately because the title over the property was with a bank to which the property had been mortgaged by Bart. Pacifico appealed the case. While the appeal was pending and with the notice of lis pendens still uncancelled, Bart sold the property to Carlos, who immediately caused the cancellation of the notice of lis pendens, as well as the issuance of a new title in his name. Is Carlos (a) a purchaser in good faith, or (b) a transferee pendente lite? If your answer is (a), how can the right of Pacifico as co-owner be protected? Explain. (5%) SUGGESTED ANSWER:

A. Carlos is a buyer in bad faith. The notice of lis pendens was still annotated at the back of the title at the time he bought the land from Bart. The uncancelled notice of lis pendens operates as constructive notice of its contents as well as interests, legal or equitable, included therein. All persons are charged with the knowledge of what it contains. In an earlier case, it was held that a notice of an adverse claim remains effective and binding notwithstanding the lapse of the 30 days from its inscription in the registry. This ruling is even more applicable in a lis pendens. Carlos is a transferee pendente lite insofar as Sancho’s share in the co-ownership in the land is concerned because the land was transferred to him during the pendency of the appeal. B. Pacifico can protect his right as a co-owner by pursuing his appeal; asking the Court of Appeals to order the re-annotation of the lis pendens on the title of Carlos; and by invoking his right of redemption of Bart’s share under Articles 1620 of the New Civil Code. ALTERNATIVE ANSWER:

A. Carlos is a purchaser in good faith. A possessor in good faith has been defined as ―one who is unaware that there exists a flaw which invalidates his acquisition of the thing‖ (Art. 526, NCC). Good faith consists in the possessor’s belief that the person from whom he received the thing was the owner of the same and could convey his title. In the case [at bar], in question, while Carlos bought the subject property from Bart while a notice of lis pendens was still annotated thereon, there was also an existing court order canceling the same. Hence, Carlos cannot be considered as being ―aware of a flaw which invalidates [their] the acquisition of the thing‖ since the alleged flaw, the notice of lis pendens, was already being ordered cancelled at the time of the purchase. On this ground alone, Carlos can already be considered a buyer in good faith. (Po Lam v. Court of Appeals, 347 SCRA 86, [2000]).

B. To protect his right over the subject property, Pacifico should have timely filed an action for reconveyance and reinstated the notice of lis pendens. Prescription & Laches; Elements of Laches (2000) In an action brought to collect a sum of money based on a surety agreement, the defense of laches was raised as the claim was filed more than seven years from the maturity of

the obligation. However, the action was brought within the ten-year prescriptive period provided by law wherein actions based on written contracts can be instituted. a) Will the defense prosper? Reason. (3%) b) What are the essential elements of laches? (2%)

SUGGESTED ANSWER:

No, the defense will not prosper. The problem did not give facts from which laches may be inferred. Mere delay in filing an action, standing alone, does not constitute laches (Agra v. PNB. 309 SCRA 509). SUGGESTED ANSWER:

b) The four basic elements of laches are; (1) conduct on the part of the defendant or of one under whom he claims, giving rise to the situation of which complainant seeks a remedy; (2) delay in asserting the complainant's rights, the complainant having had knowledge or notice of the defendant's conduct and having been afforded an opportunity to institute suit; (3) lack of knowledge on the part of the defendant that the complainant would assert the right on which he bases his suit; and (4) injury or prejudice to the defendant in the event relief is accorded to the complainant, or the suit is not held to be barred. Prescription & Laches; Indefeasibility Rule of Torrens Title (2002) Way back in 1948, Winda’s husband sold in favor of Verde Sports Center Corp. (Verde) a 10-hectare property belonging to their conjugal partnership. The sale was made without Winda’s knowledge, much less consent. In 1950, Winda learned of the sale, when she discovered the deed of sale among the documents in her husband’s vault after his demise. Soon after, she noticed that the construction of the sports complex had started. Upon completion of the construction in 1952, she tried but failed to get free membership privileges in Verde. Winda now files a suit against Verde for the annulment of the sale on the ground that she did not consent to the sale. In answer, Verde contends that, in accordance with the Spanish Civil Code which was then in force, the sale in 1948 of the property did not need her concurrence. Verde contends that in any case the action has prescribed or is barred by laches. Winda rejoins that her Torrens title covering the property is indefeasible, and imprescriptible. A. Define or explain the term ―laches‖. (2%) B. Decide the case, stating your reasons for your decision. (3%) SUGGESTED ANSWER:

A. LACHES means failure or neglect, for an unreasonable and unexplained length of time, to do what, by exercising due diligence, could or should have been done earlier. It is negligence or omission to assert a right within a reasonable time. (De Vera v. CA, 305 SCRA 624 [1999]) B. While Article 1413 of the Spanish Civil Code did not require the consent of the wife for the validity of the sale, an alienation by the husband in fraud of the wife is void as held in Uy Coque v. Navas, 45 Phil. 430 (1923). Assuming that the alienation in 1948 was in fraud of Winda and, therefore, makes the sale to Verde void, the action to set aside the sale, nonetheless, is already barred by

CIVIL LAW Answers to the BAR as Arranged by Topics (Year 1990-2006)

prescription and laches. More than 52 years have already elapsed from her discovery of the sale in 1950. ALTERNATIVE ANSWER:

B. Winda’s claim that her Torrens Title covering the property is indefeasible and imprescriptible [does not hold water] is not tenable. The rule of indefeasibility of a Torrens Title means that after one year from the date of issue of the decree of registration or if the land has fallen into the hands of an innocent purchaser for value, the title becomes incontestable and incontrovertible. IMPRESCRIPTIBILITY, on the other hand, means that no title to the land in derogation of that of the registered owner may be acquired by adverse possession or acquisitive prescription or that the registered owner does not lose by extinctive prescription his right to recover ownership and possession of the land. The action in this case is for annulment of the sale executed by the husband over a conjugal partnership property covered by a Torrens Title. Action on contracts are subject to prescription. Prescription (1990) In 1960, an unregistered parcel of land was mortgaged by owner O to M, a family friend, as collateral for a loan. O acted through his attorney-in-fact, son S, who was duly authorized by way of a special power of attorney, wherein O declared that he was the absolute owner of the land, that the tax declarations/receipts were all issued in his name, and that he has been in open, continuous and adverse possession in the concept of owner. As O was unable to pay back the loan plus interest for the past five [5) years, M had to foreclose the mortgage. At the foreclosure sale, M was the highest bidder. Upon issuance of the sheriff’s final deed of sale and registration in January, 1966, the mortgage property was turned over to M's possession and control M has since then developed the said property. In 1967, O died, survived by sons S and P. In 1977, after the tenth (10th) death anniversary of his father O. son P filed a suit to annul the mortgage deed and subsequent sale of the property, etc., on the ground of fraud. He asserted that the property in question was conjugal in nature actually belonging, at the time of the mortgage, to O and his wife, W, whose conjugal share went to their sons (S and P) and to O. (a) Is the suit filed by P barred by prescription? Explain your answer. (b) After the issuance of the sheriff's final deed of sale in 1966 in this case, assuming that M applied for registration under the Torrens System and was issued a Torrens Title to the said property in question, would that added fact have any significant effect on your conclusion? State your reason. SUGGESTED ANSWER: (a) Under Art. 173 of the

Civil Code, the action is barred by prescription because the wife had only ten (10) years from the transaction and during the marriage to file a suit for the annulment of the mortgage deed. Alternative Answers to (a) first Alternative Answer:

(a) The mortgage contract executed by O, if at all, is only a voidable contract since it involves a conjugal partnership property. The action to annul the same instituted in 1977, or eleven years after the execution of the sheriff's final sale, has obviously prescribed because: 1) An action to annul a contract on the ground of fraud must be brought within four (4) years from the date of discovery of the fraud. Since this is in essence an action to recover ownership, it must be reckoned from the date of execution of the contract or from the registration of the alleged fraudulent document with the assessor's office for the purpose of transferring the tax declaration, this being unregistered land, (Bael u. Intermediate Appellate Court G. R. L-74423 Jan.30, 1989 169 SCRA 617).

2) If the action is to be treated as an action to recover ownership of land, it would have prescribed just the same because more than 10 years have already elapsed since the date of the execution of the sale. SECOND ALTERNATIVE ANSWER:

(a) The action to recover has been barred by acquisitive prescription in favor of M considering that M has possessed the land under a claim of ownership for ten (10) years with a just title. (b) If M had secured a Torrens Title to the land, all the more S and P could not recover because if at all their remedies would be: 1. A Petition to Review the Decree of Registration. This can be availed of within one (1) year from-the entry thereof, but only upon the basis of "actual fraud." There is no showing that M committed actual fraud in securing his title to the land; or 2. An action in personam against M for the reconveyance of the title in their favor. Again, this remedy is available within four years from the date of the discovery of the fraud but not later than ten (10) years from the date of registration of the title in the name of M. Prescription; Real Rights (1992) A owned a parcel of unregistered land located on the Tarlac side of the boundary between Tarlac and Pangasinan. His brother B owned the adjoining parcel of unregistered land on the Pangasinan side. A sold the Tarlac parcel to X in a deed of sale executed as a public instrument by A and X. After X paid in full the, price of the sale, X took possession of the Pangasinan parcel in the belief that it was the Tarlac parcel covered by the deed of sale executed by A and X. After twelve (12) years, a controversy arose between B and X on the issue of the ownership of the Pangasinan parcel, B claims a vested right of ownership over the Pangasinan parcel because B never sold that parcel to X or to anyone else. On the other hand, X claims a vested right of ownership over the Pangasinan parcel by acquisitive prescription, because X possessed this parcel for over ten (10] years under claim of ownership.

CIVIL LAW Answers to the BAR as Arranged by Topics (Year 1990-2006)

Decide on these claims, giving your reasons.

The right to recover possession of registered land likewise does not prescribe because possession is just a necessary At this point in time, X cannot claim the right of vested incident of ownership. ownership over the Pangasinan parcel by acquisitive prescription. In addition to the requisites common to ordinary SUGGESTED ANSWER: and extraordinary acquisitive prescription consisting of b) Mikaelo's defense of laches, however, appears to be more uninterrupted, peaceful, public, adverse and actual possession sustainable. Renren bought the land and had the sale in the concept of owner, ordinary acquisitive prescription for registered way back in 1965. From the facts, it appears that it ten (10) years requires (1) possession in good faith and (2) just was only in 1998 or after an inexplicable delay of 33 years that title. "Just title" means that the adverse claimant came into he took the first step asserting his right to the land. It was not possession of the property through one of the modes even an action to recover ownership but only possession of recognized by law for the acquisition of ownership but the the land. By ordinary standards, 33 years of neglect or grantor was not the owner or could not transmit any right (Art. inaction is too long and maybe considered unreasonable. As 1129. Civil Code). In this case, there is no "just title" and no often held by the Supreme Court, the principle of "mode" that can be invoked by X for the acquisition of the imprescriptibility sometimes has to yield to the equitable Pangasinan parcel. There was no constructive delivery of the principle of laches which can convert even a registered land Pangasinan parcel because it was not the subject-matter of the owner's claim into a stale demand. deed of sale. Hence, B retains ownership of the Pangasinan Mikaelo's claim of laches, however, is weak insofar as the parcel of land. element of equity is concerned, there being no showing in the facts how he entered into the ownership and possession Primary Entry Book; Acquisitive Prescription; Laches (1998) of the land. SUGGESTED ANSWER:

In 1965, Renren bought from Robyn a parcel of registered land evidenced by a duly executed deed of sale. The owner presented the deed of sale and the owner's certificate of title to the Register of Deeds. The entry was made in the daybook and corresponding fees were paid as evidenced by official receipt. However, no transfer of certificate of title was issued to Renren because the original certificate of title in Robyn's name was temporarily misplaced after fire partly gutted the Office of the Register of Deeds. Meanwhile, the land had been possessed by Robyn's distant cousin, Mikaelo, openly, adversely and continuously in the concept of owner since 1960. It was only in April 1998 that Renren sued Mikaelo to recover possession. Mikaelo invoked a) acquisitive prescription and b) laches, asking that he be declared owner of the land. Decide the case by evaluating these defenses, [5%] SUGGESTED ANSWER:

a) Renren's action to recover possession of the land will prosper. In 1965, after buying the land from Robyn, he submitted the Deed of Sale to the Registry of Deeds for registration together with the owner's duplicate copy of the title, and paid the corresponding registration fees. Under Section 56 of PD No. 1529, the Deed of Sale to Renren is considered registered from the time the sale was entered in the Day Book (now called the Primary Entry Book). For all legal intents and purposes, Renren is considered the registered owner of the land. After all, it was not his fault that the Registry of Deeds could not issue the corresponding transfer certificate of title. Mikaelo's defense of prescription can not be sustained. A Torrens title is imprescriptible. No title to registered land in derogation of the title of the registered owner shall be acquired by prescription or adverse possession. (Section 47, P.D. No, 1529)

Reclamation of Foreshore Lands; Limitations (2000) Republic Act 1899 authorizes municipalities and chartered cities to reclaim foreshore lands bordering them and to construct thereon adequate docking and harbor facilities. Pursuant thereto, the City of Cavite entered into an agreement with the Fil-Estate Realty Company, authorizing the latter to reclaim 300 hectares of land from the sea bordering the city, with 30% of the land to be reclaimed to be owned by Fil-Estate as compensation for its services. The Solicitor General questioned the validity of the agreement on the ground that it will mean reclaiming land under the sea which is beyond the commerce of man. The City replies that this is authorized by RA. 1899 because it authorizes the construction of docks and harbors. Who is correct? (3%) SUGGESTED ANSWER:

The Solicitor General is correct. The authority of the City of Cavite under RA 1899 to reclaim land is limited to foreshore lands. The Act did not authorize it to reclaim land from the sea. "The reclamation being unauthorized, the City of Cavite did not acquire ownership over the reclaimed land. Not being the owner, it could not have conveyed any portion thereof to the contractor. ALTERNATIVE ANSWER:

It depends. If the reclamation of the land from the sea is necessary in the construction of the docks and the harbors, the City of Cavite is correct. Otherwise, it is not. Since RA 1899 authorized the city to construct docks and harbors, all works that are necessary for such construction are deemed authorized. Including the reclamation of land from the sea. The reclamation being authorized, the city is the owner of the reclaimed land and it may convey a portion thereof as payment for the services of the contractor. ANOTHER ALTERNATIVE ANSWER:

On the assumption that the reclamation contract was entered into before RA 1899 was repealed by PD 3-A, the City of Cavite is correct. Lands under the sea are "beyond the commerce of man" in the sense that they are not susceptible of private appropriation, ownership or

CIVIL LAW Answers to the BAR as Arranged by Topics (Year 1990-2006)

alienation. The contract in question merely calls for the reclamation of 300 hectares of land within the coastal waters of the city. Per se, it does not vest, alienate or transfer ownership of land under the sea. The city merely engaged the services of Fil-Estate to reclaim the land for the city. Registration; Deed of Mortgage (1994) How do you register now a deed of mortgage of a parcel of land originally registered under the Spanish Mortgage Law? SUGGESTED ANSWER:

a) After the Spanish Mortgage Law was abrogated by P.D. 892 on February 16, 1976, all lands covered by Spanish titles that were not brought under the Torrens system within six 16] months from the date thereof have been considered as "unregistered private lands." Thus, a deed of mortgage affecting land originally registered under the Spanish Mortgage Law is now governed by the system of registration of transactions or instruments affecting unregistered land under Section 194 of the Revised Administrative Code as amended by Act No. 3344. Under this law, the instrument or transaction affecting unregistered land is entered in a book provided for the purpose but the registration thereof is purely voluntary and does not adversely affect third persons who have a better right. b) By recording and registering with the Register of Deeds of the place where the land is located, in accordance with Act 3344. However, P.D. 892 required holders of Spanish title to bring the same under the Torrens System within 6 months from its effectivity on February 16, 1976. Remedies; Judicial Confirmation; Imperfect Title (1993) On June 30, 1986, A filed in the RTC of Abra an application for registration of title to a parcel of land under P. D. No. 1529, claiming that since June 12, 1945, he has been in open, continuous, exclusive and notorious possession and occupation of said parcel of land of the public domain which was alienable and disposable, under a bona fide claim of ownership. After issuance of the notice of initial hearing and publication, as required by law, the petition was heard on July 29, 1987. On the day of the hearing nobody but the applicant appeared. Neither was there anyone who opposed the application. Thereupon, on motion of the applicant, the RTC issued an order of general default and allowed the applicant to present his evidence. That he did. On September 30, 1989, the RTC dismissed A's application for lack of sufficient evidence. A appealed to the Court of Appeals. The appellant urged that the RTC erred in dismissing his application for registration and in not ordering registration of his title to the parcel of land in question despite the fact that there was no opposition filed by anybody to his application. Did the RTC commit the error attributed to it? SUGGESTED ANSWER: No, the RTC did not commit the error attributed to it. In an application for Judicial confirmation of imperfect or incomplete title to public agricultural land under Section 48 of the Public Land Act, the lack of opposition and the consequent order of default against those who did not

answer or show up on the date of initial hearing, does not

guarantee the success of the application. It is still incumbent upon the applicant to prove with well nigh incontrovertible evidence that he has acquired a title to the land that is fit for registration. Absent such registrable title, it is the clear duty of the Land Registration Court to dismiss the application and declare the land as public land.

An application for land registration is a proceeding in rem. Its main objective is to establish the status of the res whether it is still part of our public domain as presumed under the Regalian doctrine or has acquired the character of a private property. It is the duty of the applicant to overcome that presumption with sufficient evidence. Remedies; Judicial Reconstitution of Title (1996) In 1989, the heirs of Gavino, who died on August 10, 1987, filed a petition for reconstitution of his lost or destroyed Torrens Title to a parcel of land in Ermita, Manila. This was opposed by Marilou who claimed ownership of the said land by a series of sales. She claimed that Gavino had sold the property to Bernardo way back in 1941 and as evidence thereof, she presented a Tax Declaration in 1948 in the name of Bernardo, which cancelled the previous Tax Declaration in the name of Gavino. Then she presented two deeds of sale duly registered with the Register of Deeds, the first one executed by Bernardo in 1954 selling the same property to Carlos, and the second one executed by Carlos in 1963, selling the same property to her. She also claimed that she and her predecessors in interest have been in possession of the property since 1948. If you were the judge, how will you decide the petition? Explain. SUGGESTED ANSWER:

If I were the judge, I will give due course to the petition of the heirs of Gavino despite the opposition of Marilou for the following reasons: a) Judicial reconstitution of a certificate of title under RA. No. 26 partakes of a land registration proceeding and is perforce a proceeding in rem. It denotes restoration of an existing instrument which has been lost or destroyed in its original form and condition. The purpose of reconstitution of title or any document is to have the same reproduced, after proceedings. In the same form they were when the loss or destruction occurred. b) If the Court goes beyond that purpose, it acts without or in excess of jurisdiction. Thus, where the Torrens Title sought to be reconstituted is in the name of Gavino, the court cannot receive evidence proving that Marilou is the owner of the land. Marilou's dominical claim to the land should be ventilated in a separate civil action before the Regional Trial Court in its capacity as a court of general jurisdiction. REFERENCES: Heirs of Pedro Pinate vs. Dulay. 187 SCRA 12-20 (1990); Bunagan vs. CF1 Cebu Branch VI. 97 SCRA 72 (1980); Republic vs. IAC. 157 SCRA 62,66 (1988); Margolles vs. CA, 230 SCRA 709; Republic us, Feliciano, 148 SCRA 924.

Remedies; Procedure; Consulta (1994) What is the procedure of consulta when an instrument is denied registration? SUGGESTED ANSWER:

CIVIL LAW Answers to the BAR as Arranged by Topics (Year 1990-2006)

1) The Register of Deeds shall notify the interested party in writing, setting forth the defects of the instrument or the legal ground relied upon for denying the registration, and advising that if he is not agreeable to such ruling, he may, without withdrawing the documents from the Registry, elevate the matter by Consulta to the Administrator of the Land Registration Authority (LRA). 2) Within five {5) days from receipt of notice of denial, the party-in-interest shall file his Consulta with the Register of Deeds concerned and pay the consulta fee. 3) After receipt of the Consulta and payment of the corresponding fee the Register of Deeds makes an annotation of the pending consulta at the back of the certificate of title. 4) The Register of Deeds then elevates the case to the LRA Administrator with certified records thereof and a summary of the facts and issues involved. 5) The LRA Administrator then conducts hearings after due notice or may just require parties to submit their memoranda. 6) After hearing, the LRA Administrator issues an order prescribing the step to be taken or the memorandum to be made. His resolution in consulta shall be conclusive and binding upon all Registers of Deeds unless reversed on appeal by the Court of Appeals or by the Supreme Court. (Section 117, P.D. 1529).

(a) An action for reconveyance against Huey is not the proper remedy, because Huey is an innocent purchaser for value. The proper recourse is for Louie to go after Dewey for damages by reason of the fraudulent registration and subsequent sale of the land. If Dewey is insolvent, Louie may file a claim against the Assurance Fund (Heirs of Pedro Lopez v. De Castro 324 SCRA 591 [2000] citing Sps. Eduarte v. CA, 323 Phil. 462, 467 [1996]).

(b) Yes, the remedy will prosper because the action prescribes in ten (10) years, not within one (1) year when a petition for the reopening of the registration decree may be filed. The action for reconveyance is distinct from the petition to reopen the decree of registration (Grey Alba v. De la Cruz, 17 Phil. 49 [1910}). There is no need to reopen the registration proceedings, but the property should just be reconveyed to the real owner. The action for reconveyance is based on implied or constructive trust, which prescribes in ten (10) years from the date of issuance of the original certificate of title. This rule assumes that the defendant is in possession of the land. Where it is the plaintiff who is in possession of the land, the action for reconveyance would be in the nature of a suit for quieting for the title which action is imprescriptible (David v. Malay, 318 SCRA 711 [1999]).

Remedies; Reconveyance; Elements (1995) Rommel was issued a certificate of title over a parcel of land • The procedure of consulta is a mode of appeal from denial in Quezon City. One year later Rachelle, the legitimate owner by the Register of Deeds of the registration of the instrument to the of the land, discovered the fraudulent registration obtained by Commissioner of Land Registration. • Within five days from receipt of the notice of denial, the Rommel. She filed a complaint against Rommel for reconveyance and caused the annotation of a notice of lis interested party may elevate the matter by consulta to the pendens on the certificate of title issued to Rommel. Rommel Commissioner of Land Registration who shall enter an order now invokes the indefeasibility of his title considering that prescribing the step to be taken or memorandum to be made. Resolution in consulta shall be binding upon all Registers of Deedsone year has already elapsed from its issuance. He also seeks the cancellation of the notice of Lis pendens. Will Rachelle's provided that the party in interest may appeal to the Court of suit for reconveyance prosper? Explain. Appeals within the period prescribed (Sec. 117, P.D. 1529).

Remedies; Reconveyance vs. Reopening of a Decree; Prescriptive Period (2003) Louie, before leaving the country to train as a chef in a five-star hotel in New York, U.S.A., entrusted to his first-degree cousin Dewey an application for registration, under the Land Registration Act, of a parcel of land located in Bacolod City. A year later, Louie returned to the Philippines and discovered that Dewey registered the land and obtained an Original Certificate of Title over the property in his Dewey’s name. Compounding the matter, Dewey sold the land to Huey, an innocent purchaser for value. Louie promptly filed an action for reconveyance of the parcel of land against Huey. (a) Is the action pursued by Louie the proper remedy? (b) Assuming that reconveyance is the proper remedy, will the action prosper if the case was filed beyond one year, but within ten years, from the entry of the decree of registration? 5% SUGGESTED ANSWER:

SUGGESTED ANSWER:

Yes, Rachelle's suit will prosper because all elements for an action for reconveyance are present, namely: a) Rachelle is claiming dominical rights over the same land. b) Rommel procured his title to the land by fraud. c) The action was brought within the statutory period of four (4) years from discovery of the fraud and not later than ten (10} years from the date of registration of Rommel's title. d) Title to the land has not passed into the hands of an innocent purchaser for value. Rommel can invoke the indefeasibility of his title if Rachelle had filed a petition to reopen or review the decree of registration. But Rachelle instead filed an ordinary action in personam for reconveyance. In the latter action, indefeasibility is not a valid defense because, in filing such action, Rachelle is not seeking to nullify nor to impugn the indefeasibility of Rommel's title. She is only asking the court to compel Rommel to reconvey the title to her as the legitimate owner of the land. ALTERNATIVE ANSWER:

CIVIL LAW Answers to the BAR as Arranged by Topics (Year 1990-2006)

Yes. The property registered is deemed to be held in trust for the real owner by the person in whose name it is registered. The Torrens system was not designed to shield one who had committed fraud or misrepresentation and thus holds the title in bad faith. (Walstrom v. Mapa Jr., (G .R 38387, 29 Jan. 1990) as cited in Martinez, D., Summary of SC Decisions, January to June, 1990, p. 359],

Remedies; Reconveyance; Prescriptive Period (1997) On 10 September 1965, Melvin applied for a free patent covering two lots - Lot A and Lot B - situated in Santiago, Isabela. Upon certification by the Public Land Inspector that Melvin had been in actual, continuous, open, notorious, exclusive and adverse possession of the lots since 1925, the Director of Land approved Melvin's application on 04 June 1967. On 26 December 1967, Original Certificate of Title (OCT) No. P-2277 was issued in the name of Melvln. On 7 September 1971, Percival filed a protest alleging that Lot B which he had been occupying and cultivating since 1947 was included in the Free Patent issued in the name of Melvin. The Director of Lands ordered the investigation of Percival's protest. The Special Investigator who conducted the investigation found that Percival had been in actual cultivation of Lot B since 1947. On 28 November 1986, the Solicitor General filed in behalf of the Republic of the Philippines a complaint for cancellation of the free patent and the OCT issued in the name of Melvin and the reversion of the land to public domain on the ground of fraud and misrepresentation in obtaining the free patent. On the same date, Percival sued Martin for the reconveyance of Lot B. Melvin filed his answers interposing the sole defense in both cases that the Certificate of Title issued in his name became incontrovertible and indefeasible upon the lapse of one year from the issuance of the free patent. Given the circumstances, can the action of the Solicitor General and the case for reconveyance filed by Percival possibly prosper? SUGGESTED ANSWER:

"If fraud be discovered in the application which led to the issuance of the patent and Certificate of Title, this Title becomes ipso facto null and void. Thus, in a case where a person who obtained a free patent, knowingly made a false statement of material and essential facts in his application for the same, by stating therein that the lot in question was part of the public domain not occupied or claimed by any other person, his title becomes ipso facto canceled and consequently rendered null and void." "It is to the public interest that one who succeeds In fraudulently acquiring title to public land should not be allowed to benefit therefrom and the State, through the Solicitor General, may file the corresponding action for annulment of the patent and the reversion of the land involved to the public domain" (Dinero us. Director of Lands; Kayaban vs. Republic L-33307,8-20-73; Director of Lands us. Hon. Pedro Samson Animas, L-37682, 3-29-74.)

This action does not prescribe. With respect to Percival's

action for reconveyance, it would have prescribed, having been filed more than ten (10) years after registration and issuance of an O.C.T. in the name of Melvin, were it not for the inherent infirmity of the latter's title. Under the facts, the statute of limitations will not apply to Percival because Melvin knew that a part of the land covered by his title actually belonged to Percival. So, instead of nullifying in toto the title of Melvin, the court, in the exercise of equity and jurisdiction, may grant prayer for the reconveyance of Lot B to Percival who has actually possessed the land under a claim of ownership since 1947. After all, if Melvin's title is declared void ab initio and the land is reverted to the public domain, Percival would just the same be entitled to preference right to acquire the land from the government. Besides, well settled is the rule that once public land has been in open, continuous, exclusive and notorious possession under a bonafide claim of acquisition of ownership for the period prescribed by Section 48 of the Public Land Act, the same ipso jure ceases to be public and in contemplation of law acquired the character of private land. Thus, reconveyance of the land from Melvin to Percival would be the better procedure, (Vitale vs. Anore, 90 Phil. 855; Pena, Land Titles and Deeds, 1982, Page 427)

ALTERNATIVE ANSWER:

The action of the Solicitor General should prosper, considering that the doctrine of indefeasibility of title does not apply to free patent secured through fraud. A certificate of title cannot be used as shield to perpetuate fraud. The State is not bound by the period of prescription stated in Sec. 38 of Act 496. (Director of Lands vs. Abanilla, 124 SCRA 358) The action for reconveyance filed by Percival may still prosper provided that the property has not passed to an innocent third party for value (Dablo us. Court of Appeals. 226 SCRA 618), and provided that the action is filed within the prescriptive period of ten years (Tale vs. Court of Appeals. 208 SCRA 266). Since the action was filed by Percival 19 years after the issuance of Melvin's title, it is submitted that the same is already barred by prescription. ALTERNATIVE ANSWER (to second part of question) The action for reconveyance filed by Percival will prosper, because the land has ceased to be public land and has become private land by open, continuous, public, exclusive possession under a bona fide claim of ownership for more than thirty years, and Percival is still in possession of the property at present. His action for reconveyance can be considered as an action to quiet title, which does not prescribe if the plaintiff is in possession of the property.

(Olviga v. CA. GR 1048013. October 21, 1993)

Remedies; Reopening of a Decree; Elements (1992) What are the essential requisites or elements for the allowance of the reopening or review of a decree of registration? SUGGESTED ANSWER:

The essential elements are: (1) that the petitioner has a real or dominical right; (2) that he has been deprived thereof through fraud; (3) that the petition is filed within one (1) year from the issuance of the decree; and (4) that the property has not yet been transferred to an innocent

CIVIL LAW Answers to the BAR as Arranged by Topics (Year 1990-2006) purchaser {Rublico vs. Orellana 30 SCRA 511; Ubudan vs. Gil Administrative Code of 1987 which prohibits officers and 45 SCRA 17). employees of the government from purchasing directly or indirectly OPTIONAL EXTENDED ANSWER:

Petition for review of the Decree of Registration. A remedy expressly provided in Section 32 of P. D. No. 1529 (formerly Section 38. Act 496), this remedy has the following elements: a) The petition must be filed by a person claiming dominical or other real rights to the land registered in the name of respondent. b) The registration of the land in the name of respondent was procured by means of actual, (not just constructive) fraud, which must be extrinsic. Fraud is actual if the registration was made through deceit or any other intentional act of downright dishonesty to enrich oneself at the expense of another. It is extrinsic when it is something that was not raised, litigated and passed upon in the main proceedings. c) The petition must be filed within one (1) year from the date of the issuance of the decree. d) Title to the land has not passed to an Innocent purchaser for value (Libudan vs. Gil, 45_ SCRA 27, 1972), Rublico vs. Orrelana. 30 SCRA 511, 1969); RP vs. CA, 57 G. R No. 40402. March 16, 1987).

any property sold by the government for nonpayment of any tax, fee or other public charge. (a) Is the sale to Juan valid? If so, what is the effect of the Issuance of the Certificate of Title to Maria? (b) If the sale is void, may Juan recover the P10,000.00? If not, why not? (c) If the sale is void, did it not nevertheless, operate to divert Maria of her ownership? If it did, who then is the owner of the property? SUGGESTED ANSWER:

A. The sale of the land to Juan is not valid, being contrary to law. Therefore, no transfer of ownership of the land was effected from the delinquent taxpayer to him. The original certificates of title obtained by Maria thru a free patent grant from the Bureau of Lands under Chapter VII, CA 141 is valid but in view of her delinquency, the said title is subject to the right of the City Government to sell the land at public auction. The issuance of the OCT did not exempt the land from the tax sales. Section 44 of P.O. No. 1529 provides that every registered owner receiving a Certificate of Title shall hold the same free from an encumbrances, subject to certain exemptions.

Torrens System vs. Recording of Evidence of Title (1994) Distinguish the Torrens system of land registration from the system of recording of evidence of title.

B. Juan may recover because he was not a party to the violation of the law.

SUGGESTED ANSWER:

C. No, the sale did not divest Maria of her title precisely because the sale is void. It is as good as if no sale ever took place. In tax sales, the owner is divested of his land initially upon award and issuance of a Certificate of Sale, and finally after the lapse of the 1 year period from date of registration, to redeem, upon execution by the treasurer of an instrument sufficient in form and effects to convey the property. Maria remained owner of the land until another tax sale is to be performed in favor of a qualified buyer.

a) The TORRENS SYSTEM OF LAND REGISTRATION is a system for the registration of title to the land. Thus, under this system what is entered in the Registry of Deeds, is a record of the owner's estate or interest in the land, unlike the system under the Spanish Mortgage Law or the system under Section 194 of the Revised Administrative Code as amended by Act 3344 where only the evidence of such title is recorded. In the latter system, what is recorded is the deed of conveyance from hence the owner's title emanated—and not the title itself.

b) Torrens system of land registration is that which is prescribed in Act 496 (now PD 1529), which is either Judicial or quasi-judicial. System or recording of evidence of title is merely the registration of evidence of acquisitions of land with the Register of Deeds, who annotates the same on the existing title, cancels the old one and issues a new title based on the document presented for registration. Unregistered Land (1991) Maria Enriquez failed to pay the realty taxes on her unregistered agricultural land located in Magdugo, Toledo City. In 1989, to satisfy the taxes due, the City sold it at public auction to Juan Miranda, an employee at the Treasurer's Office of said City, whose bid at P10,000.00 was the highest. In due time, a final bill of sale was executed in his favor. Maria refused to turn-over the possession of the property to Juan alleging that (1) she had been, in the meantime, granted a free patent and on the basis thereof an Original Certificate of Title was issued to her, and (2) the sale in favor of Juan is void from the beginning in view of the provision in the

CONTRACTS Consensual vs. Real Contracts; Kinds of Real Contracts (1998) Distinguish consensual from real contracts and name at least four (4) kinds of real contracts under the present law. [3%] SUGGESTED ANSWER:

CONSENSUAL CONTRACTS are those which are perfected by mere consent (Art. 1315. Civil Code). REAL CONTRACTS are those which are perfected by the delivery of the object of the obligation. (Art. 1316, Civil Code) Examples of real contracts are deposit, pledge, commodatum and simple loan (mutuum). Consideration; Validity (2000) Lolita was employed in a finance company. Because she could not account for the funds entrusted to her, she was charged with estafa and ordered arrested. In order to secure her release from jail, her parents executed a promissory note to pay the finance company the amount allegedly misappropriated by their daughter. The finance company

CIVIL LAW Answers to the BAR as Arranged by Topics (Year 1990-2006)

then executed an affidavit of desistance which led to the withdrawal of the information against Lolita and her release from jail. The parents failed to comply with their promissory note and the finance company sued them for specific performance. Will the action prosper or not? (3%) SUGGESTED ANSWER:

The action will prosper. The promissory note executed by Lolita's parents is valid and binding, the consideration being the extinguishment of Lolita's civil liability and not the stifling of the criminal prosecution. ALTERNATIVE ANSWER:

The action will not prosper because the consideration for the promissory note was the non-prosecution of the criminal case for estafa. This cannot be done anymore because the information has already been filed in court and to do it is illegal. That the consideration for the promissory note is the stifling of the criminal prosecution is evident from the execution by the finance company of the affidavit of desistance immediately after the execution by Lolita's parents of the promissory note. The consideration being illegal, the promissory note is invalid and may not be enforced by court action. Contract of Option; Elements (2005) Marvin offered to construct the house of Carlos for a very reasonable price of P900,000.00, giving the latter 10 days within which to accept or reject the offer. On the fifth day, before Carlos could make up his mind, Marvin withdrew his offer. a) What is the effect of the withdrawal of Marvin's offer? (2%) SUGGESTED ANSWER:

The withdrawal of Marvin's offer will cause the offer to cease in law. Hence, even if subsequently accepted, there could be no concurrence of the offer and the acceptance. In the absence of concurrence of offer and acceptance, there can be no consent. (Laudico v. Arias Rodriguez, G.R. No. 16530, March 31, 1922) Without consent, there is no perfected contract for the construction of the house of Carlos. (Salonga v. Farrales, G.R. No. L-47088, July 10, 1981) Article 1318 of the Civil Code provides that there can be no contract unless the following requisites concur: (1) consent of the parties; (2) object certain which is the subject matter of the contract; and (3) cause of the obligation. Marvin will not be liable to pay Carlos any damages for withdrawing the offer before the lapse of the period granted. In this case, no consideration was given by Carlos for the option given, thus there is no perfected contract of option for lack of cause of obligation. Marvin cannot be held to have breached the contract. Thus, he cannot be held liable for damages. b) Will your answer be the same if Carlos paid Marvin P10,000.00 as consideration for that option? Explain. (2%) ALTERNATIVE ANSWER:

My answer will be the same as to the perfection of the contract for the construction of the house of Carlos. No perfected contract arises because of lack of consent. With the withdrawal of the offer, there could be no concurrence of offer and acceptance.

My answer will not be the same as to damages. Marvin will be liable for damages for breach of contract of option. With the payment of the consideration for the option given, and with the consent of the parties and the object of contract being present, a perfected contract of option was created.

(San Miguel, Inc. v. Huang, G.R. No. 137290, July 31, 2000) Under Article 1170 of the Civil Code, those who in the performance of their obligation are guilty of contravention thereof, as in this case, when Marvin did not give Carlos the agreed period of ten days, are liable for damages. ALTERNATIVE ANSWER:

My answer will not be the same if Carlos paid Marvin P10,000.00 because an option contract was perfected. Thus, if Marvin withdrew the offer prior to the expiration of the 10-day period, he breached the option contract. (Article 1324, Civil Code) c) Supposing that Carlos accepted the offer before Marvin could communicate his withdrawal thereof? Discuss the legal consequences. (2%) SUGGESTED ANSWER:

A contract to construct the house of Carlos is perfected. Contracts are perfected by mere consent manifested by the meeting of the offer and the acceptance upon the thing and the cause which are to constitute the contract. (Gomez v. Court of Appeals, G.R. No. 120747, September 21, 2000) Under Article 1315 of the Civil Code, Carlos and Marvin are bound to fulfill what has been expressly stipulated and all consequences thereof. Under Article 1167, if Marvin would refuse to construct the house, Carlos is entitled to have the construction be done by a third person at the expense of Marvin. Marvin in that case will be liable for damages under Article 1170. Inexistent Contracts vs. Annullable Contracts (2004) Distinguish briefly but clearly between Inexistent contracts and annullable contracts. SUGGESTED ANSWER:

INEXISTENT CONTRACTS are considered as not having been entered into and, therefore, void ob initio. They do not create any obligation and cannot be ratified or validated, as there is no agreement to ratify or validate. On the other hand, ANNULLABLE or VOIDABLE CONTRACTS are valid until invalidated by the court but may be ratified. In inexistent contracts, one or more requisites of a valid contract are absent. In anullable contracts, all the elements of a contract are present except that the consent of one of the contracting parties was vitiated or one of them has no capacity to give consent. Nature of Contracts; Obligatoriness (1991) Roland, a basketball star, was under contract for one year to play-for-play exclusively for Lady Love, Inc. However, even before the basketball season could open, he was offered a more attractive pay plus fringes benefits by Sweet Taste, Inc. Roland accepted the offer and transferred to Sweet Taste. Lady Love sues Roland and Sweet Taste for breach of contract. Defendants claim that the restriction to play for Lady Love alone is void, hence, unenforceable, as it

CIVIL LAW Answers to the BAR as Arranged by Topics (Year 1990-2006)

constitutes an undue interference with the right of Roland to enter into contracts and the impairment of his freedom to play and enjoy basketball. Can Roland be bound by the contract he entered into with Lady Love or can he disregard the same? Is he liable at all? How about Sweet Taste? Is it liable to Lady Love? SUGGESTED ANSWER:

Roland is bound by the contract he entered into with Lady Love and he cannot disregard the same, under the principles of obligatoriness of contracts. Obligations arising from contracts have the force of law between the parties. SUGGESTED ANSWER:

Yes, Roland is liable under the contract as far as Lady Love is concerned. He is liable for damages under Article 1170 of the Civil Code since he contravened the tenor of his obligation. Not being a contracting party, Sweet Taste is not bound by the contract but it can be held liable under Art. 1314. The basis of its liability is not prescribed by contract but is founded on quasi-delict, assuming that Sweet Taste knew of the contract. Article 1314 of the Civil Code provides that any third person who induces another to violate his contract shall be liable for damages to the other contracting party. ALTERNATIVE ANSWER:

It is assumed that Lady Love knew of the contract. Neither Roland nor Sweet Taste would be liable, because the restriction in the contract is violative of Article 1306 as being contrary to law morals, good customs, public order or public policy. Nature of Contracts; Privity of Contract (1996) Baldomero leased his house with a telephone to Jose. The lease contract provided that Jose shall pay for all electricity, water and telephone services in the leased premises during the period of the lease. Six months later. Jose surreptitiously vacated the premises. He left behind unpaid telephone bills for overseas telephone calls amounting to over P20,000.00. Baldomero refused to pay the said bills on the ground that Jose had already substituted him as the customer of the telephone company. The latter maintained that Baldomero remained as his customer as far as their service contract was concerned, notwithstanding the lease contract between Baldomero and Jose. Who is correct, Baldomero or the telephone company? Explain. SUGGESTED ANSWER:

The telephone company is correct because as far as it is concerned, the only person it contracted with was Baldomero. The telephone company has no contract with Jose. Baldomero cannot substitute Jose in his stead without the consent of the telephone company (Art. 1293, NCC). Baldomero is, therefore, liable under the contract. Nature of Contracts; Relativity of Contracts (2002) Printado is engaged in the printing business. Suplico supplies printing paper to Printado pursuant to an order agreement under which Suplico binds himself to deliver the same volume of paper every month for a period of 18 months, with Printado in turn agreeing to pay within 60 days after each delivery. Suplico has been faithfully delivering under the order agreement for 10 months but thereafter stopped doing so, because Printado has not made

any payment at all. Printado has also a standing contract with publisher Publico for the printing of 10,000 volumes of school textbooks. Suplico was aware of said printing contract. After printing 1,000 volumes, Printado also fails to perform under its printing contract with Publico. Suplico sues Printado for the value of the unpaid deliveries under their order agreement. At the same time Publico sues Printado for damages for breach of contract with respect to their own printing agreement. In the suit filed by Suplico, Printado counters that: (a) Suplico cannot demand payment for deliveries made under their order agreement until Suplico has completed performance under said contract; (b) Suplico should pay damages for breach of contract; and (c) with Publico should be liable for Printado’s breach of his contract with Publico because the order agreement between Suplico and Printado was for the benefit of Publico. Are the contentions of Printado tenable? Explain your answers as to each contention. (5%) SUGGESTED ANSWER:

No, the contentions of Printado are untenable. Printado having failed to pay for the printing paper covered by the delivery invoices on time, Suplico has the right to cease making further delivery. And the latter did not violate the order agreement (Integrated Packaging Corporation v. Court of Appeals, (333 SCRA 170, G.R. No. 115117, June 8, [2000]). Suplico cannot be held liable for damages, for breach of contract, as it was not he who violated the order agreement, but Printado. Suplico cannot be held liable for Printado’s breach of contract with Publico. He is not a party to the agreement entered into by and between Printado and Publico. Theirs is not a stipulation pour atrui. [Aforesaid] Such contracts do could not affect third persons like Suplico because of the basic civil law principle of relativity of contracts which provides that contracts can only bind the parties who entered into it, and it cannot favor or prejudice a third person, even if he is aware of such contract and has acted with knowledge thereof. (Integrated Packaging Corporation

v. CA, supra.)

Rescission of Contracts; Proper Party (1996) In December 1985, Salvador and the Star Semiconductor Company (SSC) executed a Deed of Conditional Sale wherein the former agreed to sell his 2,000 square meter lot in Cainta, Rizal, to the latter for the price of P1,000,000.00, payable P100,000.00 down, and the balance 60 days after the squatters in the property have been removed. If the squatters are not removed within six months, the P100,000.00 down payment shall be returned by the vendor to the vendee, Salvador filed ejectment suits against the squatters, but in spite of the decisions in his favor, the squatters still would not leave. In August, 1986, Salvador offered to return the P100,000.00 down payment to the vendee, on the ground that he is unable to remove the squatters on the property. SSC refused to accept the money and demanded that Salvador execute a deed of absolute sale of the property in its favor, at which time it will pay the balance of the price. Incidentally, the value of the land had doubled by that time.

CIVIL LAW Answers to the BAR as Arranged by Topics (Year 1990-2006)

Salvador consigned the P 100,000.00 in court, and filed an action for rescission of the deed of conditional sale, plus damages. Will the action prosper? Explain.

property of ZY, his wife may also sue to recover it under Article 2016 of the Civil Code if she and the family needed the money for support.

SUGGESTED ANSWER:

ALTERNATIVE ANSWER (2):

No, the action will not prosper. The action for rescission may be brought only by the aggrieved party to the contract. Since it was Salvador who failed to comply with his conditional obligation, he is not the aggrieved party who may file the action for rescission but the Star Semiconductor Company. The company, however, is not opting to rescind the contract but has chosen to waive Salvador's compliance with the condition which it can do under Art. 1545, NCC. ALTERNATIVE ANSWER:

The action for rescission will not prosper. The buyer has not committed any breach, let alone a substantial or serious one, to warrant the rescission/resolution sought by the vendor. On the contrary, it is the vendor who appears to have failed to comply with the condition imposed by the contract the fulfillment of which would have rendered the obligation to pay the balance of the purchase price demandable. Further, far from being unable to comply with what is incumbent upon it, ie., pay the balance of the price the buyer has offered to pay it even without the vendor having complied with the suspensive condition attached to the payment of the price, thus waiving such condition as well as the 60-day term in its favor The stipulation that the P100,000.00 down payment shall be returned by the vendor to the vendee if the squatters are not removed within six months, is also a covenant for the benefit of the vendee, which the latter has validly waived by implication when it offered to pay the balance of the purchase price upon the execution of a deed of absolute sale by the vendor. (Art. 1545, NCC)

OBLIGATIONS Aleatory Contracts; Gambling (2004) A. Mr. ZY lost P100,000 in a card game called Russian poker, but he had no more cash to pay in full the winner at the time the session ended. He promised to pay PX, the winner, two weeks thereafter. But he failed to do so despite the lapse of two months, so PX filed in court a suit to collect the amount of P50,000 that he won but remained unpaid. Will the collection suit against ZY prosper? Could Mrs. ZY file in turn a suit against PX to recover the P100,000 that her husband lost? Reason. (5%) SUGGESTED ANSWER:

A. 1. The suit by PX to collect the balance of what he won from ZY will not prosper. Under Article 2014 of the Civil Code, no action can be maintained by the winner for the collection of what he has won in a game of chance. Although poker may depend in part on ability, it is fundamentally a game of chance. 2) If the money paid by ZY to PX was conjugal or community property, the wife of ZY could sue to recover it because Article 117(7) of the Family Code provides that losses in gambling or betting are borne exclusively by the loser-spouse. Hence, conjugal or community funds may not be used to pay for such losses. If the money were exclusive

A. (2). Mrs. ZY cannot file a suit to recover what her husband lost. Art 2014 of the Civil Code provides that any loser in a game of chance may recover his loss from the winner, with legal interest from the time he paid the amount lost. This means that only he can file the suit. Mrs. ZY cannot recover as a spouse who has interest in the absolute community property or conjugal partnership of gains, because under Art. 117(7} of the Family Code, losses are borne exclusively by the loser-spouse. Therefore, these cannot be charged against absolute community property or conjugal partnership of gains. This being so, Mrs. ZY has no interest in law to prosecute and recover as she has no legal standing in court to do so. Conditional Obligations (2000) Pedro promised to give his grandson a car if the latter will pass the bar examinations. When his grandson passed the said examinations, Pedro refused to give the car on the ground that the condition was a purely potestative one. Is he correct or not? (2%) SUGGESTED ANSWER:

No, he is not correct. First of all, the condition is not purely potestative, because it does not depend on the sole will of one of the parties. Secondly, even if it were, it would be valid because it depends on the sole will of the creditor (the donee) and not of the debtor (the donor). Conditional Obligations (2003) Are the following obligations valid, why, and if they are valid, when is the obligation demandable in each case? a) If the debtor promises to pay as soon as he has the means to pay; b) If the debtor promises to pay when he likes; c) If the debtor promises to pay when he becomes a lawyer; d) If the debtor promises to pay if his son, who is sick with cancer, does not die within one year. 5% SUGGESTED ANSWER:

(a) The obligation is valid. It is an obligation subject to an indefinite period because the debtor binds himself to pay when his means permit him to do so (Article 1180, NCC). When the creditor knows that the debtor already has the means to pay, he must file an action in court to fix the period, and when the definite period as set by the court arrives, the obligation to pay becomes demandable 9Article 1197, NCC). SUGGESTED ANSWER:

(b) The obligation ―to pay when he likes‖ is a suspensive condition the fulfillment of which is subject to the sole will of the debtor and, therefore the conditional obligation is void. (Article 1182, NCC). SUGGESTED ANSWER:

(c) The obligation is valid. It is subject to a suspensive condition, i.e. the future and uncertain event of his becoming a lawyer. The performance of this obligation does

CIVIL LAW Answers to the BAR as Arranged by Topics (Year 1990-2006)

not depend solely on the will of the debtor but also on other factors outside the debtor’s control.

condition of Eva passing the 1998 Bar Examinations. Hence, upon Eva's passing the Bar, the rights of the other buyer terminated and Eva acquired ownership of the property.

SUGGESTED ANSWER:

(d) The obligation is valid. The death of the son of cancer within one year is made a negative suspensive condition to his making the payment. The obligation is demandable if the son does not die within one year (Article 1185, NCC). Conditional Obligations; Promise (1997) In two separate documents signed by him, Juan Valentino "obligated" himself each to Maria and to Perla, thus - 'To Maria, my true love, I obligate myself to give you my one and only horse when I feel like It." - and -'To Perla, my true sweetheart, I obligate myself to pay you the P500.00 I owe you when I feel like it." Months passed but Juan never bothered to make good his promises. Maria and Perla came to consult you on whether or not they could recover on the basis of the foregoing settings. What would your legal advice be? SUGGESTED ANSWER:

I would advise Maria not to bother running after Juan for the latter to make good his promise. [This is because a promise is not an actionable wrong that allows a party to recover especially when she has not suffered damages resulting from such promise. A promise does not create an obligation on the part of Juan because it is not something which arises from a contract, law, quasi-contracts or quasidelicts (Art, 1157)]. Under Art. 1182, Juan's promise to Maria is void because a conditional obligation depends upon the sole will of the obligor. As regards Perla, the document is an express acknowledgment of a debt, and the promise to pay what he owes her when he feels like it is equivalent to a promise to pay when his means permits him to do so, and is deemed to be one with an indefinite period under Art. 1180. Hence the amount is recoverable after Perla asks the court to set the period as provided by Art. 1197, par. 2. Conditional Obligations; Resolutory Condition (1999) In 1997, Manuel bound himself to sell Eva a house and lot which is being rented by another person, if Eva passes the 1998 bar examinations. Luckily for Eva, she passed said examinations. (a) Suppose Manuel had sold the same house and lot to another before Eva passed the 1998 bar examinations, is such sale valid? Why? (2%) (b) Assuming that it is Eva who is entitled to buy said house and lot, is she entitled to the rentals collected by Manuel before she passed the 1998 bar examinations? Why? (3%) SUGGESTED ANSWER:

(a) Yes, the sale to the other person is valid as a sale with a resolutory condition because what operates as a suspensive condition for Eva operates a resolutory condition for the buyer. FIRST ALTERNATIVE ANS WER: Yes, the sale to the other person is valid. However, the buyer acquired the property subject to a resolutory

SECOND ALTERNATIVE ANSWER:

The sale to another person before Eva could buy it from Manuel is valid, as the contract between Manuel and Eva is a mere promise to sell and Eva has not acquired a real right over the land assuming that there is a price stipulated in the contract for the contract to be considered a sale and there was delivery or tradition of the thing sold. SUGGESTED ANSWER:

(b) No, she is not entitled to the rentals collected by Manuel because at the time they accrued and were collected, Eva was not yet the owner of the property. FIRST ALTERNATIVE ANSWER:

Assuming that Eva is the one entitled to buy the house and lot, she is not entitled to the rentals collected by Manuel before she passed the bar examinations. Whether it is a contract of sale or a contract to sell, reciprocal prestations are deemed imposed A for the seller to deliver the object sold and for the buyer to pay the price. Before the happening of the condition, the fruits of the thing and the interests on the money are deemed to have been mutually compensated under Article 1187. SECOND ALTERNATIVE ANSWER:

Under Art. 1164, there is no obligation on the part of Manuel to deliver the fruits (rentals) of the thing until the obligation to deliver the thing arises. As the suspensive condition has not been fulfilled, the obligation to sell does not arise. Extinguishment; Assignment of Rights (2001) The sugar cane planters of Batangas entered into a long-term milling contract with the Central Azucarera de Don Pedro Inc. Ten years later, the Central assigned its rights to the said milling contract to a Taiwanese group which would take over the operations of the sugar mill. The planters filed an action to annul the said assignment on the ground that the Taiwanese group was not registered with the Board of Investments. Will the action prosper or not? Explain briefly. (5%)

(Note: The question presupposes knowledge and requires the application of the provisions of the Omnibus Investment Code, which properly belongs to Commercial law)

SUGGESTED ANSWER:

The action will prosper not on the ground invoked but on the ground that the farmers have not given their consent to the assignment. The milling contract imposes reciprocal obligations on the parties. The sugar central has the obligation to mill the sugar cane of the farmers while the latter have the obligation to deliver their sugar cane to the sugar central. As to the obligation to mill the sugar cane, the sugar central is a debtor of the farmers. In assigning its rights under the contract, the sugar central will also transfer to the Taiwanese its obligation to mill the sugar cane of the farmers. This will amount to a novation of the contract by substituting the debtor with a third party. Under Article 1293 of the Civil Code, such substitution cannot take effect without the consent of the creditor. The formers, who are creditors as far as the obligation to mill their sugar cane is

CIVIL LAW Answers to the BAR as Arranged by Topics (Year 1990-2006)

concerned, may annul such assignment for not having given their consent thereto. ALTERNATIVE ANSWER:

The assignment is valid because there is absolute freedom to transfer the credit and the creditor need not get the consent of the debtor. He only needs to notify him. Extinguishment; Cause of Action (2004) TX filed a suit for ejectment against BD for non-payment of condominium rentals amounting to P150,000. During the pendency of the case, BD offered and TX accepted the full amount due as rentals from BD, who then filed a motion to dismiss the ejectment suit on the ground that the action is already extinguished. Is BD’s contention correct? Why or why not? Reason. (5%) SUGGESTED ANSWER:

BD's contention is not correct. TX can still maintain the suit for ejectment. The acceptance by the lessor of the payment by the lessee of the rentals in arrears even during the pendency of the ejectment case does not constitute a waiver or abandonment of the ejectment case. (Spouses Clutario v. CA, 216 SCRA 341 [1992]).

Extinguishment; Compensation (2002) Stockton is a stockholder of Core Corp. He desires to sell his shares in Core Corp. In view of a court suit that Core Corp. has filed against him for damages in the amount of P 10 million, plus attorney’s fees of P 1 million, as a result of statements published by Stockton which are allegedly defamatory because it was calculated to injure and damage the corporation’s reputation and goodwill. The articles of incorporation of Core Corp. provide for a right of first refusal in favor of the corporation. Accordingly, Stockton gave written notice to the corporation of his offer to sell his shares of P 10 million. The response of Core corp. was an acceptance of the offer in the exercise of its rights of first refusal, offering for the purpose payment in form of compensation or set-off against the amount of damages it is claiming against him, exclusive of the claim for attorney’s fees. Stockton rejected the offer of the corporation, arguing that compensation between the value of the shares and the amount of damages demanded by the corporation cannot legally take effect. Is Stockton correct? Give reason for your answer. (5%) SUGGESTED ANSWERS:

Stockton is correct. There is no right of compensation between his price of P10 million and Core Corp.’s unliquidated claim for damages. In order that compensation may be proper, the two debts must be liquidated and demandable. The case for the P 10million damages being still pending in court, the corporation has as yet no claim which is due and demandable against Stockton. ANOTHER MAIN ANSWER:

The right of first refusal was not perfected as a right for the reason that there was a conditional acceptance equivalent to a counter-offer consisting in the amount of damages as being credited on the purchase price. Therefore, compensation did not result since there was no valid right of first refusal (Art. 1475 & 1319, NCC) ANOTHER MAIN ANSWER:

Even [if] assuming that there was a perfect right of first refusal, compensation did not take place because the claim is unliquidated. Extinguishment; Compensation vs. Payment (1998) Define compensation as a mode of extinguishing an obligation, and distinguish it from payment. [2%] SUGGESTED ANSWER:

COMPENSATION is a mode of extinguishing to the concurrent amount, the obligations of those persons who in their own right are reciprocally debtors and creditors of each other (Tolentino, 1991 ed., p. 365, citing 2 Castan 560 and Francia vs. IAC. 162 SCRA 753). It involves the simultaneous balancing of two obligations in order to extinguish them to the extent in which the amount of one is covered by that of the other. (De Leon, 1992 ed., p. 221, citing 8 Manresa 401). PAYMENT means not only delivery of money but also performance of an obligation (Article 1232, Civil Code). In payment, capacity to dispose of the thing paid and capacity to receive payment are required for debtor and creditor, respectively: in compensation, such capacity is not necessary, because the compensation operates by law and not by the act of the parties. In payment, the performance must be complete; while in compensation there may be partial extinguishment of an obligation (Tolentino, supra) Extinguishment; Compensation/Set-Off; Banks (1998) X, who has a savings deposit with Y Bank in the sum of P1,000,000.00 incurs a loan obligation with the said Bank in the sum of P800.000.00 which has become due. When X tries to withdraw his deposit, Y Bank allows only P200.000.00 to be withdrawn, less service charges, claiming that compensation has extinguished its obligation under the savings account to the concurrent amount of X's debt. X contends that compensation is improper when one of the debts, as here, arises from a contract of deposit. Assuming that the promissory note signed by X to evidence the loan does not provide for compensation between said loan and his savings deposit, who is correct? [3%] SUGGESTED ANSWER:

Y bank is correct. An. 1287, Civil Code, does not apply. All the requisites of Art. 1279, Civil Code are present. In the case of Gullas vs. PNB [62 Phil. 519), the Supreme Court held: "The Civil Code contains provisions regarding compensation (set off) and deposit. These portions of Philippine law provide that compensation shall take place when two persons are reciprocally creditor and debtor of each other. In this connection, it has been held that the relation existing between a depositor and a bank is that of creditor and debtor, x x x As a general rule, a bank has a right of set off of the deposits in its hands for the payment of any indebtedness to it on the part of a depositor." Hence, compensation took place between the mutual obligations of X and Y bank. Extinguishment; Condonation (2000) Arturo borrowed P500,000.00 from his father. After he had paid P300,000.00, his father died. When the administrator of his father's estate requested payment of the balance of P200,000.00. Arturo replied that the same had been

CIVIL LAW Answers to the BAR as Arranged by Topics (Year 1990-2006)

condoned by his father as evidenced by a notation at the back of his check payment for the P300,000.00 reading: "In full payment of the loan". Will this be a valid defense in an action for collection? (3%) SUGGESTED ANSWER:

It depends. If the notation "in full payment of the loan" was written by Arturo's father, there was an implied condonation of the balance that discharges the obligation. In such case, the notation is an act of the father from which condonation may be inferred. The condonation being implied, it need not comply with the formalities of a donation to be effective. The defense of full payment will, therefore, be valid. When, however, the notation was written by Arturo himself. It merely proves his intention in making that payment but in no way does it bind his father (Yam v. CA, G.R No. 104726. 11 February 1999). In such case, the notation was not the act of his father from which condonation may be inferred. There being no condonation at all the defense of full payment will not be valid. ALTERNATIVE ANSWER:

If the notation was written by Arturo's father, it amounted to an express condonation of the balance which must comply with the formalities of a donation to be valid under the 2nd paragraph of Article 1270 of the New Civil Code. Since the amount of the balance is more than 5,000 pesos, the acceptance by Arturo of the condonation must also be in writing under Article 748. There being no acceptance in writing by Arturo, the condonation is void and the obligation to pay the balance subsists. The defense of full payment is, therefore, not valid. In case the notation was not written by Arturo's father, the answer is the same as the answers above. Extinguishment; Extraordinary Inflation or Deflation (2001) On July 1, 1998, Brian leased an office space in a building for a period of five years at a rental rate of P1,000.00 a month. The contract of lease contained the proviso that "in case of inflation or devaluation of the Philippine peso, the monthly rental will automatically be increased or decreased depending on the devaluation or inflation of the peso to the dollar." Starting March 1, 2001, the lessor increased the rental to P2,000 a month, on the ground of inflation proven by the fact that the exchange rate of the Philippine peso to the dollar had increased from P25.00=$1.00 to P50.00=$1.00. Brian refused to pay the increased rate and an action for unlawful detainer was filed against him. Will the action prosper? Why? (5%) SUGGESTED ANSWER:

The unlawful detainer action will not prosper. Extraordinary inflation or deflation is defined as the sharp decrease in the purchasing power of the peso. It does not necessarily refer to the exchange rate of the peso to the dollar. Whether or not there exists an extraordinary inflation or deflation is for the courts to decide. There being no showing that the purchasing power of the peso had been reduced tremendously, there could be no inflation that would justify the increase in the amount of rental to be paid. Hence, Brian could refuse to pay the increased rate.

The action will not prosper. The existence of inflation or deflation requires an official declaration by the Bangko Sentral ng Pilipinas. ALTERNATIVE ANSWER:

The unlawful detainer action will prosper. It is a given fact in the problem, that there was inflation, which caused the exchange rate to double. Since the contract itself authorizes the increase in rental in the event of an inflation or devaluation of the Philippine peso, the doubling of the monthly rent is reasonable and is therefore a valid act under the very terms of the contract. Brian's refusal to pay is thus a ground for ejectment. Extinguishment; Loss (1994) Dino sued Ben for damages because the latter had failed to deliver the antique Marcedes Benz car Dino had purchased from Ben, which was—by agreement—due for delivery on December 31, 1993. Ben, in his answer to Dino's complaint, said Dino's claim has no basis for the suit, because as the car was being driven to be delivered to Dino on January 1, 1994, a reckless truck driver had rammed into the Mercedes Benz. The trial court dismissed Dino's complaint, saying Ben's obligation had indeed, been extinguished by force majeure. Is the trial court correct? SUGGESTED ANSWER:

a) No. Article 1262, New Civil Code provides, "An obligation which consists in the delivery of a determinate thing shall be extinguished if it should be lost or destroyed without the fault of the debtor, and before he has incurred in delay. b) The judgment of the trial court is incorrect. Loss of the thing due by fortuitous events or force majeure is a valid defense for a debtor only when the debtor has not incurred delay. Extinguishment of liability for fortuitous event requires that the debtor has not yet incurred any delay. In the present case, the debtor was in delay when the car was destroyed on January 1, 1993 since it was due for delivery on December 31, 1993. (Art. 1262 Civil Code) c) It depends whether or not Ben the seller, was already in default at the time of the accident because a demand for him to deliver on due date was not complied with by him. That fact not having been given in the problem, the trial court erred in dismissing Dino's complaint. Reason: There is default making him responsible for fortuitous events including the assumption of risk or loss. If on the other hand Ben was not in default as no demand has been sent to him prior to the accident, then we must distinguish whether the price has been paid or not. If it has been paid, the suit for damages should prosper but only to enable the buyer to recover the price paid. It should be noted that Ben, the seller, must bear the loss on the principle of res perit domino. He cannot be held answerable for damages as the loss of the car was not imputable to his fault or fraud. In any case, he can recover the value of the car from the party whose negligence caused the accident. If no price has been paid at all, the trial court acted correctly in dismissing the complaint.

ALTERNATIVE ANSWER:

Extinguishment; Loss; Impossible Service (1993) Page 86 of 119

CIVIL LAW Answers to the BAR as Arranged by Topics (Year 1990-2006)

In 1971, Able Construction, Inc. entered into a contract with Tropical Home Developers, Inc. whereby the former would build for the latter the houses within its subdivision. The cost of each house, labor and materials included, was P100,000.00. Four hundred units were to be constructed within five years. In 1973, Able found that it could no longer continue with the job due to the increase in the price of oil and its derivatives and the concomitant worldwide spiraling of prices of all commodities, including basic raw materials required for the construction of the houses. The cost of development had risen to unanticipated levels and to such a degree that the conditions and factors which formed the original basis of the contract had been totally changed. Able brought suit against Tropical Homes praying that the Court relieve it of its obligation. Is Able Construction entitled to the relief sought?

has been extinguished by the novation or extinction of the principal obligation insofar as third parties are concerned.

Extinguishment; Payment (1995) In 1983 PHILCREDIT extended loans to Rivett-Strom Machineries, Inc. (RIVETTT-STROM), consisting of US$10 Million for the cost of machineries imported and directly paid by PHTLCREDIT, and 5 Million in cash payable in installments over a period of ten (10) years on the basis of the value thereof computed at the rate of exchange of the U.S. dollar vis-à-vis the Philippine peso at the time of payment. RIVETT-STROM made payments on both loans which if based on the rate of exchange in 1983 would have fully settled the loans.

SUGGESTED ANSWER:

Yes, the Able Construction. Inc. is entitled to the relief sought under Article 1267, Civil Code. The law provides: "When the service has become so difficult as to be manifestly beyond the contemplation of the parties, the obligor may also be released therefrom, in whole or in part." Extinguishment; Novation (1994) In 1978, Bobby borrowed Pl,000,000.00 from Chito payable in two years. The loan, which was evidenced by a promissory note, was secured by a mortgage on real property. No action was filed by Chito to collect the loan or to foreclose the mortgage. But in 1991, Bobby, without receiving any amount from Chito, executed another promissory note which was worded exactly as the 1978 promissory note, except for the date thereof, which was the date of its execution. 1) Can Chito demand payment on the 1991 promissory note in 1994? 2) Can Chito foreclose the real estate mortgage if Bobby fails to make good his obligation under the 1991 promissory note?

SUGGESTED ANSWER:

1) Yes, Chito can demand payment on the 1991 promissory note in 1994. Although the 1978 promissory note for P1 million payable two years later or in 1980 became a natural obligation after the lapse of ten (10) years, such natural obligation can be a valid consideration of a novated promissory note dated in 1991 and payable two years later, or in 1993. All the elements of an implied real novation are present: a) an old valid obligation; b) a new valid obligation; c) capacity of the parties; d) animus novandi or intention to novate; and e) The old and the new obligation should be incompatible with each other on all material points (Article 1292). The two promissory notes cannot stand together, hence, the period of prescription of ten (10) years has not yet lapsed.

SUGGESTED ANSWER: 2) No. The mortgage being an accessory contract prescribed with the loan. The novation of the loan, however, did not expressly include the mortgage, hence, the mortgage is extinguished under Article 1296 of the NCC. The contract

PHILCREDIT contends that the payments on both loans should be based on the rate of exchange existing at the time of payment, which rate of exchange has been consistently increasing, and for which reason there would still be a considerable balance on each loan. Is the contention of PHILCREDIT correct? Discuss fully. SUGGESTED ANSWER:

As regards the loan consisting of dollars, the contention of PHILCREDIT is correct. It has to be paid in Philippine currency computed on the basis of the exchange rate at the TIME OF PAYMENT of each installment, as held in Kalalo v. Luz, 34 SCRA 337. As regards the P5 Million loan in Philippine pesos, PHILCREDIT is wrong. The payment thereof cannot be measured by the peso-dollar exchange rate. That will be violative of the Uniform Currency Act (RA, 529] which prohibits the payment of an obligation which, although to be paid in Philippine currency, is measured by a foreign currency. (Palanca v. CA, 238 SCRA 593). Liability; Lease; Joint Liability (2001) Four foreign medical students rented the apartment of Thelma for a period of one year. After one semester, three of them returned to their home country and the fourth transferred to a boarding house. Thelma discovered that they left unpaid telephone bills in the total amount of P80,000.00. The lease contract provided that the lessees shall pay for the telephone services in the leased premises. Thelma demanded that the fourth student pay the entire amount of the unpaid telephone bills, but the latter is willing to pay only one fourth of it. Who is correct? Why? (5%) SUGGESTED ANSWER:

The fourth student is correct. His liability is only joint, hence, pro rata. There is solidary liability only when the obligation expressly so states or when the law or nature of the obligation requires solidarity (Art. 1207, CC). The contract of lease in the problem does not, in any way, stipulate solidarity. Liability; Solidary Liability (1998) Joey, Jovy and Jojo are solidary debtors under a loan obligation of P300,000.00 which has fallen due. The creditor has, however, condoned Jojo's entire share in the debt. Since Jovy has become insolvent, the creditor makes a demand on Joey to pay the debt.

CIVIL LAW Answers to the BAR as Arranged by Topics (Year 1990-2006)

1) How much, if any, may Joey be compelled to pay? [2%] 2) To what extent, if at all, can Jojo be compelled by Joey to contribute to such payment? [3%]

promissory note as a result of the foreclosure of the chattel mortgage.

SUGGESTED ANSWER:

(c) The third defense of Y is untenable. Y is a surety of X and the extrajudicial demand against the principal debtor is not inconsistent with a judicial demand against the surety. A suretyship may co-exist with a mortgage. (d) The fourth defense of Y is untenable. Y is liable for the entire prestation since Y incurred a solidary obligation with X.

1. Joey can be compelled to pay only the remaining balance of P200.000, in view of the remission of Jojo's share by the creditor. (Art. 1219, Civil Code) 2. Jojo can be compelled by Joey to contribute P50.000 Art. 1217. par. 3, Civil Code provides. "When one of the solidary debtors cannot, because of his insolvency, reimburse his share to the debtor paying the obligation, such share shall be borne by all his co-debtors, in proportion to the debt of each." Since the insolvent debtor's share which Joey paid was P100,000, and there are only two remaining debtors - namely Joey and Jojo - these two shall share equally the burden of reimbursement. Jojo may thus be compelled by Joey to contribute P50.000.00. Liability; Solidary Obligation (1992) In June 1988, X obtained a loan from A and executed with Y as solidary co-maker a promissory note in favor of A for the sum of P200,000.00. The loan was payable at P20,000.00 with interest monthly within the first week of each month beginning July 1988 until maturity in April 1989. To secure the payment of the loan. X put up as security a chattel mortgage on his car, a Toyota Corolla sedan. Because of failure of X and Y to pay the principal amount of the loan, the car was extrajudicially foreclosed. A acquired the car at A's highest bid of P120,000.00 during the auction sale. After several fruitless letters of demand against X and Y, A sued Y alone for the recovery of P80.000.00 constituting the deficiency. Y resisted the suit raising the following defenses: a) That Y should not be liable at all because X was not sued together with Y. b) That the obligation has been paid completely by A's acquisition of the car through "dacion en pago" or payment by cession. c) That Y should not be held liable for the deficiency of P80,000.00 because he was not a co-mortgagor in the chattel mortgage of the car which contract was executed by X alone as owner and mortgagor. d) That assuming that Y is liable, he should only pay the proportionate sum of P40,000.00. Decide each defense with reasons. SUGGESTED ANSWER:

(a) This first defense of Y is untenable. Y is still liable as solidary debtor. The creditor may proceed against any one of the solidary debtors. The demand against one does not preclude further demand against the others so long as the debt is not fully paid. (b) The second defense of Y is untenable. Y is still liable. The chattel mortgage is only given as a security and not as payment for the debt in case of failure to pay. Y as a solidary co-maker is not relieved of further liability on the

(Arts. 1207, 1216. 1252 and 2047 Civil Code; Bicol Savings and Loan Associates vs. Guinhawa 188 SCRA 642)

Liability; Solidary Obligation; Mutual Guaranty (2003) A,B,C,D, and E made themselves solidarity indebted to X for the amount of P50,000.00. When X demanded payment from A, the latter refused to pay on the following grounds. a) B is only 16 years old. b) C has already been condoned by X c) D is insolvent. d) E was given by X an extension of 6 months without the consent of the other four co-debtors. State the effect of each of the above defenses put up by A on his obligation to pay X, if such defenses are found to be true. SUGGESTED ANSWERS:

(a) A may avail the minority of B as a defense, but only for B’s share of P 10,000.00. A solidary debtor may avail himself of any defense which personally belongs to a solidary co-debtor, but only as to the share of that codebtor. (b) A may avail of the condonation by X of C’s share of P 10, 000.00. A solidary debtor may, in actions filed by the creditor, avail himself of all defenses which are derived from the nature of the obligation and of those which are personal to him or pertain to his own share. With respect to those which personally belong to others, he may avail himself thereof only as regards that part of the debt for which the latter are responsible. (Article 1222, NCC). (c) A may not interpose the defense of insolvency of D as a defense. Applying the principle of mutual guaranty among solidary debtors, A guaranteed the payment of D’s share and of all the other co-debtors. Hence, A cannot avail of the defense of D’s insolvency. (d) The extension of six (6) months given by X to E may be availed of by A as a partial defense but only for the share of E, there is no novation of the obligation but only an act of liberality granted to E alone. Loss of the thing due; Force Majeure (2000) Kristina brought her diamond ring to a jewelry shop for cleaning. The jewelry shop undertook to return the ring by February 1, 1999." When the said date arrived, the jewelry shop informed Kristina that the Job was not yet finished. They asked her to return five days later. On February 6, 1999, Kristina went to the shop to claim the ring, but she was informed that the same was stolen by a thief who entered the shop the night before. Kristina filed an action

CIVIL LAW Answers to the BAR as Arranged by Topics (Year 1990-2006)

for damages against the jewelry shop which put up the defense of force majeure. Will the action prosper or not? (5%) SUGGESTED ANSWER:

The action will prosper. Since the defendant was already in default not having delivered the ring when delivery was demanded by plaintiff at due date, the defendant is liable for the loss of the thing and even when the loss was due to force majeure. Non-Payment of Amortizations; Subdivision Buyer; When justified (2005) Bernie bought on installment a residential subdivision lot from DEVLAND. After having faithfully paid the installments for 48 months, Bernie discovered that DEVLAND had failed to develop the subdivision in accordance with the approved plans and specifications within the time frame in the plan. He thus wrote a letter to DEVLAND informing it that he was stopping payment. Consequently, DEVLAND cancelled the sale and wrote Bernie, informing him that his payments are forfeited in its favor. a) Was the action of DEVLAND proper? Explain. (2%) SUGGESTED ANSWER:

No, the action of DEVLAND is not proper. Under Section 23 of Presidential Decree No. 957, otherwise known as the Subdivision and Condominium Buyer's Protection Decree, non-payment of amortizations by the buyer is justified if non-payment is due to the failure of the subdivision owner to develop the subdivision project according to the approved plans and within the limit for complying. (Eugenio v. Drilon, G.R. No. 109404, January 22, 1996)

b) Discuss the rights of Bernie under the circumstances. (2%) SUGGESTED ANSWER:

Under P.D. No. 957, a cancellation option is available to Bernie. If Bernie opts to cancel the contract, DEVLAND must reimburse Bernie the total amount paid and the amortizations interest, excluding delinquency interest, plus interest at legal rate. (Eugenio v. Drilon, G.R. No. 109404, January 22, 1996)

Bernie 50% of the total payments made. (Rillo v. Court of

Appeals, G.R. No. 125347, June 19,1997)

Period; Suspensive Period (1991) In a deed of sale of a realty, it was stipulated that the buyer would construct a commercial building on the lot while the seller would construct a private passageway bordering the lot. The building was eventually finished but the seller failed to complete the passageway as some of the squatters, who were already known to be there at the time they entered into the contract, refused to vacate the premises. In fact, prior to its execution, the seller filed ejectment cases against the squatters. The buyer now sues the seller for specific performance with damages. The defense is that the obligation to construct the passageway should be with a period which, incidentally, had not been fixed by them, hence, the need for fixing a judicial period. Will the action for specific performance of the buyer against the seller prosper? SUGGESTED ANSWER:

No. the action for specific performance filed by the buyer is premature under Art. 1197 of the Civil Code. If a period has not been fixed although contemplated by the parties, the parties themselves should fix that period, failing in which, the Court maybe asked to fix it taking into consideration the probable contemplation of the parties. Before the period is fixed, an action for specific performance is premature. ALTERNATIVE ANSWER:

It has been held in Borromeo vs. CA (47 SCRA 69), that the Supreme Court allowed the simultaneous filing of action to fix the probable contemplated period of the parties where none is fixed in the agreement if this would avoid multiplicity of suits. In addition, technicalities must be subordinated to substantial justice. ALTERNATIVE ANSWER:

The action for specific performance will not prosper. The filing of the ejectment suit by the seller was precisely in compliance with his obligations and should not, therefore, be faulted if no decision has yet been reached by the Court on the matter.

TRUST

c) Supposing DEVLAND had fully developed the subdivision but Bernie failed to pay further installments Express Trust; Prescription (1997) after 4 years due to business reverses. Discuss the rights On 01 January 1980, Redentor and Remedies entered into an agreement by virtue of which the former was to register a and obligations of the parties. (2%) SUGGESTED ANSWER: parcel of land in the name of Remedies under the explicit In this case, pursuant to Section 24 of P.D. No. 957, R.A. covenant to reconvey the land to Remigio, son of Redentor, No. 6552 otherwise known as the Realty Installment Buyer upon the son's graduation from college. In 1981, the land Protection Act, shall govern. Under Section 3 thereof, Bernie was registered in the name of Remedies. is entitled: 1) to pay without additional interest the unpaid installments due within a grace period of four (4) months or Redentor died a year later or in 1982. In March 1983, Remigio one month for every year of installment paid; 2) if the graduated from college. In February 1992, Remigio contract is cancelled, Bernie is entitled to the refund of the accidentally found a copy of the document so constituting cash surrender value equal to 50% of the total payments Remedies as the trustee of the land. In May 1994, Remigio made. filed a case against Remedies for the reconveyance of the land to him. Remedies, in her answer, averred that the action DEVLAND on the other hand has the right to cancel the already prescribed. How should the matter be decided? contract after 30 days from receipt by Bernie of notice of cancellation. DEVLAND is however obliged to refund to SUGGESTED ANSWER:

CIVIL LAW Answers to the BAR as Arranged by Topics (Year 1990-2006)

The matter should be decided in favor of Remigio (trustee) because the action has not prescribed. The case at bar involves an express trust which does not prescribe as long as they have not been repudiated by the trustee (Diaz vs. Gorricho. 103 Phil, 261).

Implied Trust (1998) Juan and his sister Juana inherited from their mother two parcels of farmland with exactly the same areas. For convenience, the Torrens certificates of title covering both lots were placed in Juan's name alone. In 1996, Juan sold to an innocent purchaser one parcel in its entirety without the knowledge and consent of Juana, and wrongfully kept for himself the entire price paid. 1. What rights of action, if any, does Juana have against and/or the buyer? |3%] 2. Since the two lots have the same area, suppose Juana flies a complaint to have herself declared sole owner of the entire remaining second lot, contending that her brother had forfeited his share thereof by wrongfully disposing of her undivided share in the first lot. Will the suit prosper? [2%] SUGGESTED ANSWER:

1. When, for convenience, the Torrens title to the two parcels of land were placed in Joan's name alone, there was created an implied trust (a resulting trust) for the benefit of Juana with Juan as trustee of one-half undivided or ideal portion of each of the two lots. Therefore, Juana can file an action for damages against Joan for having fraudulently sold one of the two parcels which he partly held in trust for Juana's benefit. Juana may claim actual or compensatory damage for the loss of her share in the land; moral damages for the mental anguish, anxiety, moral shock and wounded feelings she had suffered; exemplary damage by way of example for the common good, and attorney's fees. Juana has no cause of action against the buyer who acquired the land for value and in good faith, relying on the transfer certificate of title showing that Juan is the registered owner of the land. ANOTHER ANSWER:

1. Under Article 476 of the Civil Code, Juana can file an action for quieting of title as there is a cloud in the title to the subject real property. Second, Juana can also file an action for damages against Juan, because the settled rule is that the proper recourse of the true owner of the property who was prejudiced and fraudulently dispossessed of the same is to bring an action for damages against those who caused or employed the same. Third, since Juana had the right to her share in the property by way of inheritance, she can demand the partition of the thing owned in common, under Article 494 of the Civil Code, and ask that the title to the remaining property be declared as exclusively hers. However, since the farmland was sold to an innocent purchaser for value, then Juana has no cause of action against the buyer consistent with the established rule that the rights of an innocent purchaser for value must be respected and protected notwithstanding the fraud employed by the seller in securing his title. (Eduarte vs. CA, 253 SCRA 391) ADDITIONAL ANSWER:

1. Juana has the right of action to recover (a) her one-half share in the proceeds of the sale with legal interest thereof, and (b) such damages as she may be able to prove as having been suffered by her, which may include actual or compensatory damages as well as moral and exemplary damages due to the breach of trust and bad faith (Imperial vs. CA, 259 SCRA 65). Of course, if the buyer knew of the co-ownership over the lot he was buying, Juana can seek (c) reconvenyance of her one-half share instead but she must implead the buyer as co-defendant and allege his bad faith in purchasing the entire lot. Finally, consistent with the ruling in Imperial us. CA. Juana may seek instead (d) a declaration that she is now the sole owner of the entire remaining lot on the theory that Juan has forfeited his one-half share therein. ADDITIONAL ANSWER:

1. Juana can file an action for damages against Juan for having fraudulently sold one of the two parcels which he partly held in trust for Juana's benefit. Juana may claim actual or compensatory damage for the loss of her share in the land; moral damages for the mental anguish, anxiety, moral shock and wounded feelings she had suffered; exemplary damage by way of example for the common good, and attorney's fees. Juana has no cause of action against the buyer who acquired the land for value and in good faith, relying on the transfer certificate showing that Juan is the registered owner of the land. SUGGESTED ANSWER:

2. Juana's suit to have herself declared as sole owner of the entire remaining area will not prosper because while Juan's act in selling the other lot was wrongful. It did not have the legal effect of forfeiting his share in the remaining lot. However, Juana can file an action against Juan for partition or termination of the co-ownership with a prayer that the lot sold be adjudicated to Juan, and the remaining lot be adjudicated and reconveyed to her. ANOTHER ANSWER:

2. The suit will prosper, applying the ruling in Imperial vs. CA cited above. Both law and equity authorize such a result, said the Supreme Court. Strictly speaking, Juana's contention that her brother had forfeited his share in the second lot is incorrect. Even if the two lots have the same area, it does not follow that they have the same value. Since the sale of the first lot on the Torrens title in the name of Juan was valid, all that Juana may recover is the value of her undivided interest therein, plus damages. In addition, she can ask for partition or reconveyance of her undivided interest in the second lot, without prejudice to any agreement between them that in lieu of the payment of the value of Juana's share in the first lot and damages, the second lot be reconveyed to her. ALTERNATIVE ANSWER:

2. The suit will not prosper, since Juan's wrongful act of pocketing the entire proceeds of the sale of the first lot is not a ground for divesting him of his rights as a co-owner of the second lot. Indeed, such wrongdoing by Juan does not constitute, for the benefit of Juana, any of the modes of acquiring ownership under Art. 712, Civil Code.

CIVIL LAW Answers to the BAR as Arranged by Topics (Year 1990-2006) (Art. 1624; 1475. CC; Rodriguez v. CA, et al, G. R No. 84220, March 25. 1992 207 SCRA 553).

Trust; Implied Resulting Trust (1995) In 1960, Maureen purchased two lots in a plush subdivision registering Lot 1 in her name and Lot 2 in the name of her brother Walter with the latter's consent. The idea was to circumvent a subdivision policy against the acquisition of more than one lot by one buyer. Maureen constructed a house on Lot 1 with an extension on Lot 2 to serve as a guest house. In 1987, Walter who had suffered serious business losses demanded that Maureen remove the extension house since the lot on which the extension was built was his property. In 1992, Maureen sued for the reconveyance to her of Lot 2 asserting that a resulting trust was created when she had the lot registered in Walter's name even if she paid the purchase price. Walter opposed the suit arguing that assuming the existence of a resulting trust the action of Maureen has already prescribed since ten years have already elapsed from the registration of the title in his name. Decide. Discuss fully. SUGGESTED ANSWER:

This is a case of an implied resulting trust. If Walter claims to have acquired ownership of the land by prescription or if he anchors his defense on extinctive prescription, the ten year period must be reckoned from 1987 when he demanded that Maureen remove the extension house on Lot No. 2 because such demand amounts to an express repudiation of the trust and it was made known to Maureen. The action for reconveyance filed in 1992 is not yet barred by prescription. (Spouses Huang v. Court of Appeals, Sept. 13, 1994).

SALES Assignment of Credit vs. Subrogation (1993) Peter Co, a trader from Manila, has dealt business with Allied Commodities in Hongkong for five years. All through the years, Peter Co accumulated an indebtedness of P500,000.00 with Allied Commodities. Upon demand by its agent in Manila, Peter Co paid Allied Commodities by check the amount owed. Upon deposit in the payee's account in Manila, the check was dishonored for insufficiency of funds. For and in consideration of P1.00, Allied Commodities assigned the credit to Hadji Butu who brought suit against Peter Co in the RTC of Manila for recovery of the amount owed. Peter Co moved to dismiss the complaint against him on the ground that Hadji Butu was not a real party in interest and, therefore, without legal capacity to sue and that he had not agreed to a subrogation of creditor. Will Peter Co's defense of absence of agreement to a subrogation of creditor prosper? SUGGESTED ANSWER:

No, Co's defense will not prosper. This is not a case of subrogation, but an assignment of credit. ASSIGNMENT OF CREDIT is the process of transferring the right of the assignor to the assignee. The assignment may be done either gratuitously or onerously, in which case, the assignment has an effect similar to that of a sale (Nyco Sales Corp.v.BA Finance Corp. G.R No.71694. Aug.16, 1991 200 SCRA 637). As a result of the assignment, the plaintiff acquired all the rights of the assignor including the right to sue in his own name as the legal assignee. In assignment, the debtor's consent is not essential for the validity of the assignment

ALTERNATIVE ANSWER:

No, the defense of Peter Co will not prosper. Hadji Butu validly acquired his right by an assignment of credit under Article 1624 of the Civil Code. However, the provisions on the contract of sale (Article 1475 Civil Code) will apply, and the transaction is covered by the Statute of Frauds. (Art. 1403 par. (2) Civil Code) Conditional Sale vs. Absolute Sale (1997) Distinguish between a conditional sale, on the one hand, and an absolute sale, on the other hand. SUGGESTED ANSWER:

A CONDITIONAL SALE is one where the vendor is granted the right to unilaterally rescind the contract predicated on the fulfillment or non-fulfillment, as the case may be, of the prescribed condition. An ABSOLUTE SALE is one where the title to the property is not reserved to the vendor or if the vendor is not granted the right to rescind the contract based on the fulfillment or nonfulfillment, as the case may be, of the prescribed condition. Contract of Sale vs. Agency to Sell (1999) A granted B the exclusive right to sell his brand of Maong pants in Isabela, the price for his merchandise payable within 60 days from delivery, and promising B a commission of 20% on all sales. After the delivery of the merchandise to B but before he could sell any of them, B’s store in Isabela was completely burned without his fault, together with all of A's pants. Must B pay A for his lost pants? Why? (5%) SUGGESTED ANSWER:

The contract between A and B is a sale not an agency to sell because the price is payable by B upon 60 days from delivery even if B is unable to resell it. If B were an agent, he is not bound to pay the price if he is unable to resell it. As a buyer, ownership passed to B upon delivery and, under Art. 1504 of the Civil Code, the thing perishes for the owner. Hence, B must still pay the price. Contract of Sale; Marital Community Property; Formalities (2006) Spouses Biong and Linda wanted to sell their house. They found a prospective buyer, Ray. Linda negotiated with Ray for the sale of the property. They agreed on a fair price of P2 Million. Ray sent Linda a letter confirming his intention to buy the property. Later, another couple, Bernie and Elena, offered a similar house at a lower price of P 1.5 Million. But Ray insisted on buying the house of Biong and Linda for sentimental reasons. Ray prepared a deed of sale to be signed by the couple and a manager's check for P2 Million. After receiving the P2 Million, Biong signed the deed of sale. However, Linda was not able to sign it because she was abroad. On her return, she refused to sign the document saying she changed her mind. Linda filed suit for nullification of the deed of sale and for moral and exemplary damages against Ray. Will the suit prosper? Explain. (2.5%) ALTERNATIVE ANSWER:

CIVIL LAW Answers to the BAR as Arranged by Topics (Year 1990-2006)

No, the suit will not prosper. The contract of sale was perfected when Linda and Ray agreed on the object of the sale and the price [Art. 1475, New Civil Code]. The consent of Linda has already been given, as shown by her agreement to the price of the sale. There is therefore consent on her part as the consent need not be given in any specific form. Hence, her consent may be given by implication, especially since she was aware of, and participated in the sale of the property (Pelayo v. CA, G.R. No. 141323, June 8, 2005). Her action for moral and exemplary damages will also not prosper because the case does not fall under any of those mentioned in Art. 2219 and 2232 of the Civil Code. ALTERNATIVE ANSWER:

The suit will prosper. Sale of community property requires written consent of both spouses. The failure or refusal of Linda to affix her signature on the deed of sale, coupled with her express declaration of opposing the sale negates any valid consent on her part. The consent of Biong by himself is insufficient to effect a valid sale of community property (Art. 96, Family Code; Abalos v. Macatangay, G.R. No. 155043, September 30, 2004).

Does Ray have any cause of action against Biong and Linda? Can he also recover damages from the spouses? Explain. (2.5%) Considering that the contract has already been perfected and taken out of the operation of the statute of frauds, Ray can compel Linda and Biong to observe the form required by law in order for the property to be registered in the name of Ray which can be filed together with the action for the recovery of house [Art. 1357 New Civil Code]. In the alternative, he can recover the amount of Two million pesos (P2,000,000.00) that he paid. Otherwise, it would result in solutio indebiti or unjust enrichment. Ray can recover moral damages on the ground that the action filed by Linda is clearly an unfounded civil suit which falls under malicious prosecution {Ponce v. Legaspi, G.R. No. 79184, May 6,1992).

Contract to Sell (2001) Arturo gave Richard a receipt which states: Receipt Received from Richard as down payment for my 1995 Toyota Corolla with plate No. XYZ-1 23.............. P50.000.00 Balance payable: 12/30/01........

P50 000.00

September 15, 2001. (Sgd.) Arturo Does this receipt evidence a contract to sell? Why? (5%) SUGGESTED ANSWER:

It is a contract of sale because the seller did not reserve ownership until he was fully paid. Contract to Sell vs. Contract of Sale (1997) State the basic difference (only in their legal effects) Between a contract to sell, on the one hand, and a contract of sale, on the other. SUGGESTED ANSWER:

In a CONTRACT OF SALE, ownership is transferred to the buyer upon delivery of the object to him while in a CONTRACT TO SELL, ownership is retained by the seller until the purchase price is fully paid. In a contract to sell, delivery of the object does not confer ownership upon the buyer. In a contract of sale, there is only one contract executed between the seller and the buyer, while in a contract to sell, there are two contracts, first the contract to sell (which is a conditional or preparatory sale) and a second, the final deed of sale or the principal contract which is executed after full payment of the purchase price. Contract to Sell; Acceptance; Right of First Refusal (1991) A is the lessee of an apartment owned by Y. A allowed his married but employed daughter B, whose husband works in Kuwait, to occupy it. The relationship between Y and A soured. Since he has no reason at all to eject A, Y, in connivance with the City Engineer, secured from the latter an order for the demolition of the building. A immediately filed an action in the Regional Trial Court to annul the order and to enjoin its enforcement. Y and A were able to forge a compromise agreement under which A agreed to a twenty percent (20%) increase in the monthly rentals. They further agreed that the lease will expire two (2) years later and that in the event that Y would sell the property, either A or his daughter B shall have the right of first refusal. The Compromise Agreement was approved by the court. Six (6) months before the expiration of the lease, A died. Y sold the property to the Visorro Realty Corp. without notifying B. B then filed an action to rescind the sale in favor of the corporation and to compel Y to sell the property to her since under the Compromise Agreement, she was given the right of first refusal which, she maintains is a stipulation pour atrui under Article 1311 of the Civil Code. Is she correct? SUGGESTED ANSWER:

B is not correct. Her action cannot prosper. Article 1311 requires that the third person intended to be benefited must communicate his acceptance to the obligor before the revocation. There is no showing that B manifested her acceptance to Y at any time before the death of A and before the sale. Hence, B cannot enforce any right under the alleged stipulation pour atrui. Double Sales (2001) On June 15, 1995, Jesus sold a parcel of registered land to Jaime. On June 30, 1995, he sold the same land to Jose. Who has a better right if: a) the first sale is registered ahead of the second sale, with knowledge of the latter. Why? (3%) b) the second sale is registered ahead of the first sale, with knowledge of the latter? Why? (5%) SUGGESTED ANSWER:

(a) The first buyer has the better right if his sale was first to be registered, even though the first buyer knew of the second sale. The fact that he knew of the second sale at the time of his registration does not make him as acting in bad faith because the sale to him was ahead in time, hence, has a priority in right. What creates bad faith in the case of double sale of land is knowledge of a previous sale.

CIVIL LAW Answers to the BAR as Arranged by Topics (Year 1990-2006)

b) The first buyer is still to be preferred, where the second sale is registered ahead of the first sale but with knowledge of the latter. This is because the second buyer, who at the time he registered his sale knew that the property had already been sold to someone else, acted in bad faith. (Article 1544, C.C.)

(2) years, or until 3 June 1973. It is further stated therein that should the Vendor (Juliet) fail to exercise her right to redeem within the said period, the conveyance shall be deemed absolute and irrevocable. Romeo did not take possession of the property. He did not pay the taxes thereon.

Double Sales (2004) JV, owner of a parcel of land, sold it to PP. But the deed of sale was not registered. One year later, JV sold the parcel again to RR, who succeeded to register the deed and to obtain a transfer certificate of title over the property in his own name. Who has a better right over the parcel of land, RR or PP? Why? Explain the legal basis for your answer. (5%)

Juliet died in January I973 without having repurchased the property. Her only surviving heir, her son X, failed to repurchase the property on or before 3 June 1973. In 1975, Romeo sold the property to Y for P50,000.00. Upon learning of the sale, X filed an action for the nullification of the sale and for the recovery of the property on the ground that the so-called deed of absolute sale executed by his mother was merely an equitable mortgage, taking into account the inadequacy of the price and the failure of Romeo to take possession of the property and to pay the taxes thereon. Romeo and Y maintain that there was a valid absolute sale and that the document signed by the former on 3 June 1973 was merely a promise to sell. a) If you were the Judge, would you uphold the theory of X? b) If you decide in favor of Romeo and Y, would you

SUGGESTED ANSWER:

It depends on whether or not RR is an innocent purchaser for value. Under the Torrens System, a deed or instrument operated only as a contract between the parties and as evidence of authority to the Register of Deeds to make the registration. It is the registration of the deed or the instrument that is the operative act that conveys or affects the land. (Sec. 51, P.D. No. 1529).

uphold the validity of the promise to sell? In cases of double sale of titled land, it is a well-settled rule that the buyer who first registers the sale in good faith acquires a better right to the land. (Art. 1544, Civil Code). Persons dealing with property covered by Torrens title are not required to go beyond what appears on its face. (Orquiola v. CA 386, SCRA 301, [2002]; Domingo v. Races 401 SCRA 197, [2003]). Thus, absent any showing that RR knew

about, or ought to have known the prior sale of the land to PP or that he acted in bad faith, and being first to register the sale, RR acquired a good and a clean title to the property as against PP. Equitable Mortgage On 20 December 1970, Juliet, a widow, borrowed from (1991) Romeo P4,000.00 and, as security therefore, she executed a deed of mortgage over one of her two (2) registered lots which has a market value of P15,000.00. The document and the certificate of title of the property were delivered to Romeo. On 2 June 1971, Juliet obtained an additional sum of P3,000 from Romeo. On this date, however, Romeo caused the preparation of a deed of absolute sale of the above property, to which Juliet affixed her signature without first reading the document. The consideration indicated is P7,000.00. She thought that this document was similar to the first she signed. When she reached home, her son X, after reading the duplicate copy of the deed, informed her that what she signed was not a mortgage but a deed of absolute sale. On the following day, 3 June 1971, Juliet, accompanied by X, went back to Romeo and demanded the reformation it, Romeo prepared and signed a document wherein, as vendee in the deed of sale above mentioned, he obligated and bound himself to resell the land to Juliet or her heirs and successors for the same consideration as reflected in the deed of sale (P7,000) within a period of two

SUGGESTED ANSWER:

A. I will not uphold the theory of X for the nullification of the sale and for the recovery of the property on the ground that the so-called sale was only an equitable mortgage. An equitable mortgage may arise only if, in truth, the sale was one with the right of repurchase. The facts of the case state that the right to repurchase was granted after the absolute deed of sale was executed. Following the rule in Cruzo vs. Carriaga (174 SCRA 330), a deed of repurchase executed independently of the deed of sale where the two stipulations are found in two instruments instead of one document, the right of repurchase would amount only to one option granted by the buyer to the seller. Since the contract cannot be upheld as a contract of sale with the right to repurchase, Art. 1602 of the Civil Code on equitable mortgage will not apply. The rule could have been different if both deeds were executed on the same occasion or date, in which case, under the ruling in spouses Claravall v. CA (190 SCRA 439), the contract may still be sustained as an equitable mortgage, given the circumstances expressed in Art. 1602. The reserved right to repurchase is then deemed an original intention. B. If I were to decide in favor of Romeo and Y, I would not uphold the validity of the promise to sell, so as to enforce it by an action for specific performance. The promise to sell would only amount to a mere offer and, therefore, it is not enforceable unless it was sought to be exercised before a withdrawal or denial thereof. Even assuming the facts given at the end of the case, there would have been no separate consideration for such promise to sell. The contract would at most amount to an option which again may not be the basis for an action for specific performance. Equitable Mortgage vs. Sale (2005) On July 14, 2004, Pedro executed in favor of Juan a Deed of Absolute Sale over a parcel of land covered by TCT No. Page 93 of 119

CIVIL LAW Answers to the BAR as Arranged by Topics (Year 1990-2006)

6245. It appears in the Deed of Sale that Pedro received from Juan P120,000.00 as purchase price. However, Pedro retained the owner's duplicate of said title. Thereafter, Juan, as lessor, and Pedro, as lessee, executed a contract of lease over the property for a period of one (1) year with a monthly rental of Pl,000.00. Pedro, as lessee, was also obligated to pay the realty taxes on the property during the period of lease.

X sold a parcel of land to Y on 01 January 2002, payment and delivery to be made on 01 February 2002. It was stipulated that if payment were not to be made by Y on 01 February 2002, the sale between the parties would automatically be rescinded. Y failed to pay on 01 February 2002, but offered to pay three days later, which payment X refused to accept, claiming that their contract of sale had already been rescinded. Is X’s contention correct? Why? 5%

SUGGESTED ANSWER:

Subsequently, Pedro filed a complaint against Juan for the reformation of the Deed of Absolute Sale, alleging that the transaction covered by the deed was an equitable mortgage. In his verified answer to the complaint, Juan alleged that the property was sold to him under the Deed of Absolute Sale, and interposed counterclaims to recover possession of the property and to compel Pedro to turn over to him the owner's duplicate of title. Resolve the case with reasons. (6%) SUGGESTED ANSWER:

No, X is not correct. In the sale of immovable property, even though it may have been stipulated, as in this case, that upon failure to pay the price at the time agreed upon the rescission of the contract shall of right take place, the vendee may pay, even after the expiration of the period, as long as no demand for rescission of the contract has been made upon him either judicially or by a notarial act (Article 1592, New Civil code). Since no demand for rescission was made on Y, either judicially or by a notarial act, X cannot refuse to accept the payment offered by Y three (3) days after the expiration of the period.

The complaint of Pedro against Juan should be dismissed. The instances when a contract — regardless of its ANOTHER SUGGESTED ANSWER: nomenclature — may be presumed to be an equitable This is a contract to sell and not a contract of absolute sale, mortgage are enumerated in Article 1602 of the Civil Code: since as there has been no delivery of the land. Article 1592 of "Art. 1602. The contract shall be presumed to be an equitable the New Civil code is not applicable. Instead, Article 1595 of the New Civil Code applies. The seller has two alternative mortgage, in any of the following cases: remedies: (1) specific performance, or (2) rescission or 1 When the price of a sale with right to repurchase is resolution under Article 1191 of the New Civil code. In both unusually inadequate: remedies, damages are due because of default. 2 When the vendor remains in possession as lessee or ALTERNATIVE ANSWER: otherwise; Yes, the contract was automatically rescinded upon Y’s failure 3 When upon or after the expiration of the right to to pay on 01 February 2002. By the express terms of the repurchase another instrument extending the period of redemption contract, there is no need for X to make a demand in order or granting a new period is executed; for rescission to take place. (Article 1191, New Civil Code, Suria 4 When the purchaser retains for himself a part of the v. IAC 151 SCRA 661 [1987]; U.P. v. de los purchase price; 5 When the vendor binds himself to pay the taxes on the Angeles 35 SCRA 102 [1970]). thing sold; Maceda Law (2000) 6 In any other case where it may be fairly inferred that the Priscilla purchased a condominium unit in Makati City from real intention of the parties is that the transaction shall secure the the Citiland Corporation for a price of P10 Million, payable payment of a debt or the performance of any other obligation. P3 Million down and the balance with interest thereon at 14% per annum payable in sixty (60) equal monthly installments of P198,333.33. They executed a Deed of Conditional Sale in "In any of the foregoing cases, any money, fruits, or other which it is stipulated that should the vendee fail to pay three benefit to be received by the vendee as rent or otherwise (3) successive installments, the sale shall be deemed shall be considered as interest which shall be subject to the automatically rescinded without the necessity of judicial action usury laws." and all payments made by the vendee shall be forfeited in favor of the vendor by way of rental for the use and Article 1604 states that "the provisions of article 1602 shall occupancy of the unit and as liquidated damages. For 46 also apply to a contract purporting to be an absolute sale." months, Priscilla paid the monthly installments religiously, but on the 47th and 48th months, she failed to pay. On the 49th For Articles 1602 and 1604 to apply, two requisites must month, she tried to pay the installments due but the vendor concur: 1) the parties entered into a contract denominated as refused to receive the payments tendered by her. The a contract of sale; and 2) their intention was to secure an following month, the vendor sent her a notice that it was existing debt by way of mortgage. (Heirs of Balite v. Lim, rescinding the Deed of Conditional Sale pursuant to the G.R. No. 152168, December 10, 2004) stipulation for automatic rescission, and demanded that she vacate the premises. She replied that the contract cannot be In the given case, although Pedro retained possession of the rescinded without judicial demand or notarial act pursuant to property as lessee after the execution of the Deed of Sale, Article 1592 of the Civil Code. a) Is Article 1592 applicable? there is no showing that the intention of the parties was to (3%) b) Can the vendor rescind the contract? (2%) secure an existing debt by way of mortgage. Hence, the complaint of Pedro should be dismissed. Immovable Property; Rescission of Contract (2003)

SUGGESTED ANSWER:

Page 94 of 119

CIVIL LAW Answers to the BAR as Arranged by Topics (Year 1990-2006)

a) Article 1592 of the Civil Code does not apply to a conditional sale. In Valarao v. CA, 304 SCRA 155, the Supreme Court held that Article 1592 applies only to a contract of sale and not to a Deed of Conditional Sale where the seller has reserved title to the property until full payment of the purchase price. The law applicable is the Maceda Law. SUGGESTED ANSWER:

b) No, the vendor cannot rescind the contract under the circumstances. Under the Maceda Law, which is the law applicable, the seller on installment may not rescind the contract till after the lapse of the mandatory grace period of 30 days for every one year of installment payments, and only after 30 days from notice of cancellation or demand for rescission by a notarial act. In this case, the refusal of the seller to accept payment from the buyer on the 49th month was not justified because the buyer was entitled to 60 days grace period and the payment was tendered within that period. Moreover, the notice of rescission served by the seller on the buyer was not effective because the notice was not by a notarial act. Besides, the seller may still pay within 30 days from such notarial notice before rescission may be effected. All these requirements for a valid rescission were not complied with by the seller. Hence, the rescission is invalid. Maceda Law; Recto Law (1999) What are the so-called "Maceda" and "Recto" laws in connection with sales on installments? Give the most important features of each law. (5%) SUGGESTED ANSWER:

The MACEDA LAW (R.A. 655) is applicable to sales of immovable property on installments. The most important features are (Rillo v. CA, 247 SCRA 461): (1) After having paid installments for at least two years, the buyer is entitled to a mandatory grace period of one month for every year of installment payments made, to pay the unpaid installments without interest. If the contract is cancelled, the seller shall refund to the buyer the cash surrender value equivalent to fifty percent (50%) of the total payments made, and after five years of installments, an additional five percent (5%) every year but not to exceed ninety percent (90%) of the total payments made. (2) In case the installments paid were less than 2 years, the seller shall give the buyer a grace period of not less than 60 days. If the buyer fails to pay the installments due at the expiration of the grace period, the seller may cancel the contract after 30 days from receipt by the buyer of the notice of cancellation or demand for rescission by notarial act. The RECTO LAW (Art. 1484} refers to sale of movables payable in installments and limiting the right of seller, in case of default by the buyer, to one of three remedies: a) exact fulfillment; b) cancel the sale if two or more installments have not been paid; c) foreclose the chattel mortgage on the things sold, also in case of default of two or more installments, with no further action against the purchaser.

Option Contract (2002) Explain the nature of an option contract. (2%) SUGGESTED ANSWER:

An OPTION CONTRACT is one granting a privilege to buy or sell within an agreed time and at a determined price. It must be supported by a consideration distinct from the price. (Art. 1479 and 1482, NCC) Option Contract; Earnest Money (1993) LT applied with BPI to purchase a house and lot in Quezon City, one of its acquired assets. The amount offered was Pl,000,000.00 payable, as follows: P200,000.00 down payment, the balance of P800,000.00 payable within 90 days from June 1, 1985. BPI accepted the offer, whereupon LT drew a check for P200,000.00 in favor of BPI which the latter thereafter deposited in its account. On September 5, 1985, LT wrote BPI requesting extension until October 10, 1985 within which to pay the balance, to which BPI agreed. On October 5, 1985, due to the expected delay in the remittance of the needed amount by his financier from the United States, LT wrote BPI requesting a last extension until October 30, 1985, within which to pay the balance. BPI denied LTs request because another had offered to buy the same property for P1,500,000.00. BPI cancelled its agreement with LT and offered to return to him the amount of P200,000.00 that LT had paid to it. On October 20, 1985, upon receipt of the amount of P800,000.00 from his US financier, LT offered to pay the amount by tendering a cashier's check therefor but which BPI refused to accept. LT then filed a complaint against BPI in the RTC for specific performance and deposited in court the amount of P800,000.00. Is BPI legally correct in canceling its contract with LT? SUGGESTED ANSWER:

BPI is not correct in canceling the contract with LT. In Lina Topacio v Court of Appeals and BPI Investment (G. R No. 102606, July 3. 1993, 211 SCRA 291) the Supreme Court held

that the earnest money is part of the purchase price and is proof of the perfection of the contract. Secondly, notarial or judicial rescission under Art. 1592 and 1991 of the Civil Code is necessary (Taguba v. de Leon, 132 SCRA 722.) ALTERNATIVE ANSWER:

BPI is correct in canceling its contract with LT but BPI must do so by way of judicial rescission under Article 1191 Civil Code. The law requires a judicial action, and mere notice of rescission is insufficient if it is resisted. The law also provides that slight breach is not a ground for rescission (Song Fo & Co, vs, Hawaiian Phil Co., 47 Phils. 821), Delay in the fulfillment of the obligation (Art. 1169, Civil Code) is a ground to rescind, only if time is of the essence. Otherwise, the court may refuse the rescission if there is a just cause for the fixing of a period. Perfected Sale; Acceptance of Earnest Money (2002) Bert offers to buy Simeon’s property under the following terms and conditions: P1 million purchase price, 10% option money, the balance payable in cash upon the clearance of the property of all illegal occupants. The option money is promptly paid and Simeon clears the property of illegal occupants in no time at all. However, when Bert tenders payment of the balance and ask Simeon for the deed

CIVIL LAW Answers to the BAR as Arranged by Topics (Year 1990-2006)

for absolute sale, Simeon suddenly has a change of heart, claiming that the deal is disadvantageous to him as he has found out that the property can fetch three time the agreed purchase price. Bert seeks specific performance but Simeon contends that he has merely given Bert an option to buy and nothing more, and offers to return the option money which Bert refuses to accept. B. Will Bert’s action for specific performance prosper? Explain. (4%) C. May Simeon justify his refusal to proceed with the sale by the fact that the deal is financially disadvantageous to him? Explain. (4%) SUGGESTED ANSWER:

B. Bert’s action for specific performance will prosper because there was a binding agreement of sale, not just an option contract. The sale was perfected upon acceptance by Simeon of 10% of the agreed price. This amount is in really earnest money which, under Art. 1482, ―shall be considered as part of the price and as proof of the perfection of the contract.‖ (Topacio v. CA, 211 SCRA 291 [1992]; Villongco Realty v. Bormaheco, 65 SCRA 352 [1975]).

C. Simeon cannot justify his refusal to proceed with the sale by the fact that the deal is financially disadvantageous to him. Having made a bad bargain is not a legal ground for pulling out a biding contract of sale, in the absence of some actionable wrong by the other party (Vales v. Villa, 35 Phil 769 [1916]), and no such wrong has been committed by Bert. Redemption; Legal; Formalities (2001) Betty and Lydia were co-owners of a parcel of land. Last January 31, 2001, when she paid her real estate tax, Betty discovered that Lydia had sold her share to Emma on November 10, 2000. The following day, Betty offered to redeem her share from Emma, but the latter replied that Betty's right to redeem has already prescribed. Is Emma correct or not? Why? (5%) SUGGESTED ANSWER:

Emma, the buyer, is not correct. Betty can still enforce her right of legal redemption as a co-owner. Article 1623 of the Civil Code gives a co-owner 30 days from written notice of the sale by the vendor to exercise his right of legal redemption. In the present problem, the 30-day period for the exercise by Betty of her right of redemption had not even begun to run because no notice in writing of the sale appears to have been given to her by Lydia. Redemption; Legal; Formalities (2002) Adela and Beth are co-owners of a parcel of land. Beth sold her undivided share of the property to Xandro, who promptly notified Adela of the sale and furnished the latter a copy of the deed of absolute sale. When Xandro presented the deed for registration, the register of deeds also notified Adela of the sale, enclosing a copy of the deed with the notice. However, Adela ignored the notices. A year later, Xandro filed a petition for the partition of the property. Upon receipt of summons, Adela immediately tendered the requisite amount for the redemption. Xandro contends that Adela lost her right of redemption after the expiration of 30 days from her receipt of the notice of the sale given by him.

May Adela still exercise her right of redemption? Explain. (5%) SUGGESTED ANSWER:

Yes, Adela may still exercise her right of redemption notwithstanding the lapse of more than 30 days from notice of the sale given to her because Article 1623 of the New Civil Code requires that the notice in writing of the sale must come from the prospective vendor or vendor as the case may be. In this case, the notice of the sale was given by the vendee and the Register of Deeds. The period of 30 days never tolled. She can still avail of that right. ALTERNATIVE ANSWER:

Adela can no longer exercise her right of redemption. As co-owner, she had only 30 days from the time she received written notice of the sale which in this case took the form of a copy of the deed of sale being given to her (Conejero v. CA, 16 SCRA 775 [1966]). The law does not prescribe any particular form of written notice, nor any distinctive method for notifying the redemptioner (Etcuban v. CA, 148 SCRA 507 [1987]). So long as the redemptioner was informed in writing, he has no cause to complain (Distrito v. CA, 197 SCRA 606, 609 [1991]). In fact, in Distrito, a written notice was held unnecessary where the co-owner had actual knowledge of the sale, having acted as middleman and being present when the vendor signed the deed of sale. Right of First Refusal; Lessee; Effect (1996) Ubaldo is the owner of a building which has been leased by Remigio for the past 20 years. Ubaldo has repeatedly assured Remigio that if he should decide to sell the building, he will give Remigio the right of first refusal. On June 30, 1994, Ubaldo informed Remigio that he was willing to sell the building for P5 Million. The following day, Remigio sent a letter to Ubaldo offering to buy the building at P4.5 Million. Ubaldo did not reply. One week later, Remigio received a letter from Santos informing him that the building has been sold to him by Ubaldo for P5 Million, and that he will not renew Remigio's lease when it expires. Remigio filed an action against Ubaldo and Santos for cancellation of the sale, and to compel Ubaldo to execute a deed of absolute sale in his favor, based on his right of first refusal. a) Will the action prosper? Explain. b) If Ubaldo had given Remigio an option to purchase the building instead of a right of first refusal, will your answer be the same? Explain. SUGGESTED ANSWER:

No, the action to compel Ubaldo to execute the deed of absolute sale will not prosper. According to Ang Yu v. Court of Appeals (238 SCRA 602), the right of first refusal is not based on contract but is predicated on the provisions of human relations and, therefore, its violation is predicated on quasi-delict. Secondly, the right of first refusal implies that the offer of the person in whose favor that right was given must conform with the same terms and conditions as those given to the offeree. In this case, however, Remigio was offering only P4.5 Million instead of P5 Million. ALTERNATIVE ANSWER:

No, the action will not prosper. The lessee's right of first refusal does not go so far as to give him the power to dictate on the lessor the price at which the latter should sell

CIVIL LAW Answers to the BAR as Arranged by Topics (Year 1990-2006)

his property. Upon the facts given, the lessor had sufficiently complied with his commitment to give the lessee a right of first refusal when he offered to sell the property to the lessee for P5 Million, which was the same price he got in selling it to Santos. He certainly had the right to treat the lessee's counter-offer of a lesser amount as a rejection of his offer to sell at P5 Million. Thus, he was free to find another buyer upon receipt of such unacceptable counter-offer (Art. 1319. NCC). SUGGESTED ANSWER:

Yes, the answer will be the same. The action will not prosper because an option must be supported by a consideration separate and distinct from the purchase price. In this case there is no separate consideration. Therefore, the option may be withdrawn by Ubaldo at any time. (Art. 1324, NCC) Right of First Refusal; Lessee; Effect (1998) In a 20-year lease contract over a building, the lessee is expressly granted a right of first refusal should the lessor decide to sell both the land and building. However, the lessor sold the property to a third person who knew about the lease and in fact agreed to respect it. Consequently, the lessee brings an action against both the lessor-seller and the buyer (a) to rescind the sale and (b) to compel specific performance of his right of first refusal in the sense that the lessor should be ordered to execute a deed of absolute sale in favor of the lessee at the same price. The defendants contend that the plaintiff can neither seek rescission of the sale nor compel specific performance of a "mere" right of first refusal. Decide the case. [5%] SUGGESTED ANSWER:

The action filed by the lessee, for both rescission of the offending sale and specific performance of the right of first refusal which was violated, should prosper. The ruling in Equatorial Realty Development, Inc. vs. Mayfair Theater, Inc. (264 SCRA 483), a case with similar facts, sustains both rights

of action because the buyer in the subsequent sale knew the existence of right of first refusal, hence in bad faith. ANOTHER ANSWER:

The action to rescind the sale and to compel the right to first refusal will not prosper. (Ang Yu Asuncion vs. CA, 238 SCRA 602). The Court ruled in a unanimous en banc decision that the right of first refusal is not founded upon contract but on a quasi-delictual relationship covered by the principles of human relations and unjust enrichment (Art. 19, et seq. Civil Code). Hence the only action that will prosper according to the Supreme Court is an "action for damages in a proper forum for the purpose." Right of Repurchase (1993) On January 2, 1980, A and B entered into a contract whereby A sold to B a parcel of land for and in consideration of P10.000.00. A reserving to himself the right to repurchase the same. Because they were friends, no period was agreed upon for the repurchase of the property. 1) Until when must A exercise his right of repurchase? 2) If A fails to redeem the property within the allowable period, what would you advise B to do for his better protection?

SUGGESTED ANSWER:

1) A can exercise his right of repurchase within four (4) years from the date of the contract (Art. 1606, Civil Code). SUGGESTED ANSWER:

2} I would advise B to file an action for consolidation of title and obtain a judicial order of consolidation which must be recorded in the Registry of Property (Art. 1607. Civil Code). Transfer of Ownership; Non-Payment of the Price (1991) Pablo sold his car to Alfonso who issued a postdated check in full payment therefor. Before the maturity of the check, Alfonso sold the car to Gregorio who later sold it to Gabriel. When presented for payment, the check issued by Alfonso was dishonored by the drawee bank for the reason that he, Alfonso, had already closed his account even before he issued his check. Pablo sued to recover the car from Gabriel alleging that he (Pablo) had been unlawfully deprived of it by reason of Alfonso's deception. Will the suit prosper? SUGGESTED ANSWER:

No. The suit will not prosper because Pablo was not unlawfully deprived of the car although he was unlawfully deprived of the price. The perfection of the sale and the delivery of the car was enough to allow Alfonso to have a right of ownership over the car, which can be lawfully transferred to Gregorio. Art. 559 applies only to a person who is in possession in good faith of the property, and not to the owner thereof. Alfonso, in the problem, was the owner, and, hence, Gabriel acquired the title to the car. Non-payment of the price in a contract of sale does not render ineffective the obligation to deliver. The obligation to deliver a thing is different from the obligation to pay its price. EDCA Publishing Co. v. Santos (1990) Transfer of Ownership; Risk of Loss (1990) D sold a second-hand car to E for P150,000.00 The agreement between D and E was that half of the purchase price, or P75,000.00, shall be paid upon delivery of the car to E and the balance of P75,000.00 shall be paid in five equal monthly installments of P15,000.00 each. The car was delivered to E, and E paid the amount of P75.000.00 to D. Less than one month thereafter, the car was stolen from E's garage with no fault on E's part and was never recovered. Is E legally bound to pay the said unpaid balance of P75.000.00? Explain your answer. SUGGESTED ANSWER:

Yes, E is legally bound to pay the balance of P75,000.00. The ownership of the car sold was acquired by E from the moment it was delivered to him. Having acquired ownership, E bears the risk of the loss of the thing under the doctrine of res perit domino. [Articles 1496. 1497, Civil Code).

LEASE Extinguishment; Total Distruction; Leased Property (1993) A is the owner of a lot on which he constructed a building in the total cost of P10,000,000.00. Of that amount B

CIVIL LAW Answers to the BAR as Arranged by Topics (Year 1990-2006)

contributed P5,000,000.00 provided that the building as a whole would be leased to him (B) for a period of ten years from January 1. 1985 to December 31, 1995 at a rental of P100,000.00 a year. To such condition, A agreed. On December 20, 1990, the building was totally burned. Soon thereafter, A's workers cleared the debris and started construction of a new building. B then served notice upon A that he would occupy the building being constructed upon completion, for the unexpired portion of the lease term, explaining that he had spent partly for the construction of the building that was burned. A rejected B's demand. Did A has a right in rejecting B's demand? SUGGESTED ANSWER:

Yes. A was correct in rejecting the demand of B. As a result of the total destruction of the building by fortuitous event, the lease was extinguished. (Art. 1655, Civil Code.) Implied New Lease (1999) Under what circumstances would an implied new lease or a tacita reconduccion arise? (2%) SUGGESTED ANSWER:

An implied new lease or tacita reconduccion arises if at the end of the contract the lessee should continue enjoying the thing leased for 15 days with the acquiescence of the lessor, and unless a notice to the contrary by either parties has previously been given (Art. 1670). In short, in order that there may be tacita reconduccion there must be expiration of the contract; there must be continuation of possession for 15 days or more; and there must be no prior demand to vacate. Lease of Rural Lands (2000) In 1995, Mark leased the rice land of Narding in Nueva Ecija for an annual rental of P1,000.00 per hectare. In 1998, due to the El Nino phenomenon, the rice harvest fell to only 40% of the average harvest for the previous years. Mark asked Narding for a reduction of the rental to P500.00 per hectare for that year but the latter refused. Is Mark legally entitled to such reduction? (2%) SUGGESTED ANSWER:

No, Mark is not entitled to a reduction. Under Article 1680 of the Civil Code, the lessee of a rural land is entitled to a reduction of the rent only in case of loss of more than 1/2 of the fruits through extraordinary and unforeseen fortuitous events. While the drought brought about by the "El Nino" phenomenon may be classified as extraordinary, it is not considered as unforeseen. ALTERNATIVE ANSWER: Yes, Mark is entitled to a reduction of the rent. His loss was more than 1/2 of the fruits and the loss was due to an extraordinary and unforeseen fortuitous event. The "El Nino" phenomenon is extraordinary because it is uncommon; it does not occur with regularity. And neither could the parties have foreseen its occurrence. The event should be foreseeable by the parties so that the lessee can change the time for his planting, or refrain from planting, or take steps to avoid the loss. To be foreseeable, the time and the place of the occurrence, as well as the magnitude of the adverse effects of the fortuitous event must be capable of being predicted. Since the exact place, the exact time, and the exact magnitude of the adverse effects of the "El Nino"

phenomenon are still unpredictable despite the advances in science, the phenomenon is considered unforeseen.

Leasee & Lessor; Rights and Obligations (1990) A vacant lot several blocks from the center of the town was leased by its owner to a young businessman B for a term of fifteen (15) years renewal upon agreement of the parties. After taking possession of the lot, the lessee built thereon a building of mixed materials and a store. As the years passed, he expanded his business, earning more profits. By the tenth (10th) year of his possession, he was able to build a three (3)-story building worth at least P300,000.00. Before the end of the term of the lease, B negotiated with the landowner for its renewal, but despite their attempts to do so, they could not agree on the new conditions for the renewal. Upon the expiration of the term of the lease, the landowner asked B to vacate the premises and remove his building and other improvements. B refused unless he was reimbursed for necessary and useful expenses. B claimed that he was a possessor and builder in good faith, with right of retention. This issue is now before the court for resolution in a pending litigation. a) What are the rights of B? b) What are the rights of the landowner? SUGGESTED ANSWER:

a) B has the right to remove the building and other improvements unless the landowner decides to retain the building at the time of the termination of the lease and pay the lessee one-half of the value of the improvements at that time. The lessee may remove the building even though the principal thing may suffer damage but B should not cause any more impairment upon the property leased than is necessary. The claim of B that he was a possessor and builder in good faith with the right of retention is not tenable. B is not a builder in good faith because as lessee he does not claim ownership over the property leased. SUGGESTED ANSWER:

b) The landowner/lessor may refuse to reimburse 1/2 of the value of the improvements and require the lessee to remove the improvements. [Article 1678, Civil Code), Leasee; Death Thereof; Effects (1997) Stating briefly the thesis to support your answer to each of the following cases, will the death - a) of the lessee extinguish the lease agreement? SUGGESTED ANSWER:

No. The death of the lessee will not extinguish the lease agreement, since lease is not personal in character and the right is transmissible to the heirs. (Heirs of Dimaculangan vs. IAC, 170 SCRA 393).

Option to Buy; Expired (2001) On January 1, 1980, Nestor leased the fishpond of Mario for a period of three years at a monthly rental of P1,000.00, with an option to purchase the same during the period of the lease for the price of P500,000.00. After the expiration of the three-year period, Mario allowed Nestor to remain in the leased premises at the same rental rate. On June 15, 1983, Nestor tendered the amount of P500,000.00 to Mario and demanded that the latter execute a deed of absolute sale of the fishpond in his favor. Mario refused, on the ground that Nestor no longer had an option to buy the fishpond.

CIVIL LAW Answers to the BAR as Arranged by Topics (Year 1990-2006)

Nestor filed an action for specific performance. Will the action prosper or not? Why? (5%) SUGGESTED ANSWER:

No, the action will not prosper. The implied renewal of the lease on a month-to-month basis did not have the effect of extending the life of the option to purchase which expired at the end of the original lease period. The lessor is correct in refusing to sell on the ground that the option had expired. Sublease vs. Assignment of Lease; Rescission of Contract (2005) Under a written contract dated December 1, 1989, Victor leased his land to Joel for a period of five (5) years at a monthly rental of Pl,000.00, to be increased to Pl,200.00 and Pl,500.00 on the third and fifth year, respectively. On January 1, 1991, Joel subleased the land to Conrad for a period of two (2) years at a monthly rental of Pl,500.00. On December 31, 1992, Joel assigned the lease to his compadre, Ernie, who acted on the belief that Joel was the rightful owner and possessor of the said lot. Joel has been faithfully paying the stipulated rentals to Victor. When Victor learned on May 18, 1992 about the sublease and assignment, he sued Joel, Conrad and Ernie for rescission of the contract of lease and for damages. a) Will the action prosper? If so, against whom? Explain. (2%) SUGGESTED ANSWER:

Yes, the action of for rescission of the contract of lease and for damages will prosper. Under Article 1659 of the Civil Code, "if the lessor or the lessee should not comply with the obligations set forth in Articles 1654 and 1657, the aggrieved party may ask for rescission of the contract and indemnification for damages, or only the latter, allowing the contract to remain in force." Article 1649 of the same Code provides that "the lessee cannot assign the lease without the consent of the lessor, unless there is a stipulation to the contrary." Consent is necessary because assignment would cause novation by the substitution of one of the parties.

sublessee can invoke no right superior to that of his sublessor, the moment the sublessor is duly ousted from the premises, the sublessee has no leg to stand on. The sublessee's right, if any, is to demand reparation for damages from his sublessor, should the latter be at fault.

(Heirs ofSevilla v. Court of Appeals G.R. No. 49823, February 26, 1992).

Sublease; Delay in Payment of Rentals (1994) In January 1993, Four-Gives Corporation leased the entire twelve floors of the GQS Towers Complex, for a period of ten years at a monthly rental of P3,000,000.00. There is a provision in the contract that the monthly rentals should be paid within the first five days of the month. For the month of March, May, June, October and December 1993, the rentals were not paid on time with some rentals being delayed up to ten days. The delay was due to the heavy paper work involved in processing the checks. Four-Gives Corporation also subleased five of the twelve floors to wholly-owned subsidiaries. The lease contract expressly prohibits the assignment of the lease contract or any portion thereof. The rental value of the building has increased by 50% since its lease to Four-Gives Corporation. 1) Can the building owner eject Four-Gives Corporation on grounds of the repeated delays in the payment of the rent? 2} Can the building owner ask for the cancellation of the contract for violation of the provision against assignment? SUGGESTED ANSWERS:

1) a) The "repeated delays" in the payment of rentals would, at best, be a slight or casual breach which does not furnish a ground for ejectment especially because the delays were only due to heavy paper work. Note that there was not even a demand for payment obviously because the delay lasted for only a few days (10 days being the longest), at the end of which time payments were presumably made and were accepted. There was, therefore, no default. Note also that there was no demand made upon the lessee to vacate the premises for non-payment of the monthly rent. There is, (Bangayan v. Court of Appeals, G.R. No. 123581, August 29, therefore, no cause of action for ejectment arising from the 1997) However, the rule is different in the case of subleasing. "repeated delays". When there is no express prohibition in the Contract of Lease, b) The building owner cannot eject Four-Gives Corporation the lessee may sublet the thing leased. (Art. 1650, Civil Code) on the ground of repeated delays in the payment of rentals. The delay in the payment of the rentals is minimal and cannot In the given case, when Joel assigned the lease to Ernie, the be made the basis of an ejectment suit. The delay was due to same was done without the consent of Victor. The assignment the heavy paperwork involved in processing the checks. It is void. However, there is no indication that in the written would be otherwise if the lease contract stated that in the contract of lease between Victor and Joel, that subleasing the payment of rentals within the first five days of the month, time premises is prohibited. Hence, the sublease of Joel with is of the essence or that the lessee will be in delay if he falls to Conrad is valid. In view of the foregoing, Victor can file the pay within the agreed period without need of demand. In this case of rescission and damages only against Joel and Ernie but case he can judicially eject the tenant on the ground of lack of he cannot include Conrad. payment of the price stipulated after a demand to vacate, (Article 1673(2), New Civil Code), b) In case of rescission, discuss the rights and obligations of the parties. (2%) SUGGESTED ANSWER: c) No. Resolution of a contract will not be permitted for a Rescission of the lease necessarily requires the return of the thing slight or casual breach, but only for such substantial and to the lessor. Hence, the judgment granting rescission of the fundamental breach as would defeat the very object of the contract should also order the lessee to vacate and return the parties in making the agreement.(Zepeda v. CA, 216 SCRA leased premises to the lessor. However, since the 293]. The delay of ten (10)) days is not such a substantial

CIVIL LAW Answers to the BAR as Arranged by Topics (Year 1990-2006)

and fundamental breach to warrant the resolution of the contract of lease specially so when the delay was due to the heavy paperwork in processing the checks. SUGGESTED ANSWER:

2) a) No. Sublease is different from assignment of lease. Sublease, not being prohibited by the contract of lease is therefore allowed and cannot be invoked as a ground to cancel the lease, b) No, the lessor cannot have the lease cancelled for alleged violation of the provision against assignment. The lessee did not assign the lease, or any portion thereof, to the subsidiaries. It merely subleased some floors to its subsidiaries. Since the problem does not state that the contract of lease contains a prohibition against sublease, the sublease is lawful, the rule being that in the absence of an express prohibition a lessee may sublet the thing leased, in whole or in part, without prejudice to his/its responsibility to the lessor for the performance of the contract. Sublease; Sublessee; Liability (1999) May a lessee sublease the property leased without the consent of the lessor, and what are the respective liabilities of the lessee and sub-lessee to the lessor in case of such sublease? (3%) SUGGESTED ANSWER:

Yes, provided that there is no express prohibition against subleasing. Under the law, when in the contract of lease of things there is no express prohibition, the lessee may sublet the thing leased without prejudice to his responsibility for the performance of the contract toward the lessor. [Art, 1650) In case there is a sublease of the premises being leased, the sublessee is bound to the lessor for all the acts which refer to the use and preservation of the thing leased in the manner stipulated between the lessor and the lessee. (Art. 1651} The sublessee is subsidiarily liable to the lessor for any rent due from the lessee. However, the sublessee shall not be responsible beyond the amount of the rent due from him. (Art. 1652) As to the lessee, the latter shall still be responsible to the lessor for the rents; bring to the knowledge of the lessor every usurpation or untoward act which any third person may have committed or may be openly preparing to carry out upon the thing leased; advise the owner the need for all repairs; to return the thing leased upon the termination of the lease just as he received it, save what has been lost or impaired by the lapse of time or by ordinary wear and tear or from an inevitable cause; responsible for the deterioration or loss of the thing leased, unless he proves that it took place without his fault.

Sublease; Sublessee; Liability (2000) A leased his house to B with a condition that the leased premises shall be used for residential purposes only. B subleased the house to C who used it as a warehouse for fabrics. Upon learning this, A demanded that C stop using the house as a warehouse, but C ignored the demand, A then filed an action for ejectment against C, who raised the defense that there is no privity of contract between him and

A, and that he has not been remiss in the payment of rent. Will the action prosper? (3%) SUGGESTED ANSWER:

Yes, the action will prosper. Under Article 1651 of the Civil Code, the sublessee is bound to the lessor for all acts which refer to the use and preservation of the thing leased in the manner stipulated between the lessor and the lessee. Sublease; Validity; Assignment of Sublease (1990) A leased a parcel of land to B for a period of two years. The lease contract did not contain any express prohibition against the assignment of the leasehold or the subleasing of the leased premises. During the third year of the lease, B subleased the land to C. In turn, C, without A's consent, assigned the sublease to D. A then filed an action for the rescission of the contract of lease on the ground that B has violated the terms and conditions of the lease agreement. If you were the judge, how would you decide the case, particularly with respect to the validity of: (a) B's sublease to C? and (b) C's assignment of the sublease to D? SUGGESTED ANSWER:

(a) B's sublease to C is valid. Although the original period of two years for the lease contract has expired, the lease continued with the acquiescence of the lessor during the third year. Hence, there has been an implied renewal of the contract of lease. Under Art. 1650 of the Civil Code, the lessee may sublet the thing leased, in whole or in part, when the contract of lease does not contain any express prohibition. [Articles 1650, 1670 Civil Code). A's action for rescission should not prosper on this ground. SUGGESTED ANSWER:

(b) C's assignment of the sublease to D is not valid. Under Art. 1649, of the Civil Code, the lessee cannot assign the lease without the consent of the lessor, unless there is a stipulation to the contrary. There is no such stipulation in the contract. If the law prohibits assignment of the lease without the consent of the lessor, all the more would the assignment of a sublease be prohibited without such consent. This is a violation of the contract and is a valid ground for rescission by A.

COMMON CARRIERS Extraordinary Diligence (2000) Despite a warning from the police that an attempt to hijack a PAL plane will be made in the following week, the airline did not take extra precautions, such as frisking of passengers, for fear of being accused of violating human rights. Two days later, an armed hijacker did attempt to hijack a PAL flight to Cebu. Although he was subdued by the other passengers, he managed to fire a shot which hit and killed a female passenger. The victim's parents sued the airline for breach of contract, and the airline raised the defense of force majeure. Is the airline liable or not? (2%) SUGGESTED ANSWER:

The airline is liable. In case of death of a passenger, common carriers are presumed to have been at fault or to have acted negligently, unless they prove that they observed extraordinary diligence (Article 1756, Civil Code). The

CIVIL LAW Answers to the BAR as Arranged by Topics (Year 1990-2006)

failure of the airline to take extra precautions despite a police warning that an attempt to hijack the plane would be made, was negligence on the part of the airline. Being negligent, it is liable for the death of the passenger. The defense of force majeure is not tenable since the shooting incident would not have happened had the airline taken steps that could have prevented the hijacker from boarding the plane. ALTERNATIVE ANSWER:

Under Article 1763 of the Civil Code, the common carrier is not required to observe extraordinary diligence in preventing injury to its passengers on account of the willful acts or negligence of other passengers or of strangers. The common carrier, in that case, is required to exercise only the diligence of a good father of a family; hence, the failure of the airline to take EXTRA precautions in frisking the passengers and by leaving that matter to the security personnel of the airport, does not constitute a breach of that duty so as to make the airline liable. Besides, the use of irresistible force by the hijackers was farce majeure that could not have been prevented even by the observance of extraordinary diligence.

AGENCY

Agency (2003) Jo-Ann asked her close friend, Aissa, to buy some groceries for her in the supermarket. Was there a nominate contract entered into between Jo-Ann and Aissa? In the affirmative, what was it? Explain. 5% SUGGESTED ANSWER:

Yes, there was a nominate contract. On the assumption that Aissa accepted the request of her close friend Jo-Ann to but some groceries for her in the supermarket, what they entered into was a nominate contract of Agency. Article 1868 of the New Civil Code provides that by the contract of agency a person binds himself to render some service or to do something in representation or on behalf of another, with the consent or authority of the latter. ALTERNATIVE ANSWER:

Yes, they entered into a nominate contract of lease to service in the absence of a relation of principal and agent between them (Article 1644, New Civil Code). Agency vs. Sale (2000) A foreign manufacturer of computers and a Philippine distributor entered into a contract whereby the distributor agreed to order 1,000 units of the manufacturer's computers every month and to resell them in the Philippines at the manufacturer's suggested prices plus 10%. All unsold units at the end of the year shall be bought back by the manufacturer at the same price they were ordered. The manufacturer shall hold the distributor free and harmless from any claim for defects in the units. Is the agreement one for sale or agency? (5%) SUGGESTED ANSWER: The contract is one of agency, not sale. The notion of sale is negated by the following indicia: (1) the price is fixed by the manufacturer with the 10% mark-up constituting the commission; (2) the manufacturer reacquires the unsold units at exactly the same price; and (3) warranty for the units was borne by the manufacturer. The foregoing indicia

negate sale because they indicate that ownership over the units was never intended to transfer to the distributor.

Agency; coupled with an interest (2001) Richard sold a large parcel of land in Cebu to Leo for P100 million payable in annual installments over a period of ten years, but title will remain with Richard until the purchase price is fully paid. To enable Leo to pay the price, Richard gave him a power-of-attorney authorizing him to subdivide the land, sell the individual lots, and deliver the proceeds to Richard, to be applied to the purchase price. Five years later, Richard revoked the power of attorney and took over the sale of the subdivision lots himself. Is the revocation valid or not? Why? (5%) SUGGESTED ANSWER:

The revocation is not valid. The power of attorney given to the buyer is irrevocable because it is coupled with an interest: the agency is the means of fulfilling the obligation of the buyer to pay the price of the land (Article 1927, CC). In other words, a bilateral contract (contract to buy and sell the land) is dependent on the agency. Agency; Guarantee Commission (2004) As an agent, AL was given a guarantee commission, in addition to his regular commission, after he sold 20 units of refrigerators to a customer, HT Hotel. The customer, however, failed to pay for the units sold. AL’s principal, DRBI, demanded from AL payment for the customer’s accountability. AL objected, on the ground that his job was only to sell and not to collect payment for units bought by the customer. Is AL’s objection valid? Can DRBI collect from him or not? Reason. (5%) SUGGESTED ANSWER:

No, AL's objection is not valid and DRBI can collect from AL. Since AL accepted a guarantee commission, in addition to his regular commission, he agreed to bear the risk of collection and to pay the principal the proceeds of the sale on the same terms agreed upon with the purchaser (Article 1907, Civil Code) Agency; Real Estate Mortgage (2004) CX executed a special power of attorney authorizing DY to secure a loan from any bank and to mortgage his property covered by the owner’s certificate of title. In securing a loan from MBank, DY did not specify that he was acting for CX in the transaction with said bank. Is CX liable for the bank loan? Why or why not? Justify your answer. (5%) SUGGESTED ANSWER:

CX is liable for the bank loan because he authorized the mortgage on his property to secure the loan contracted by DY. If DY later defaults and fails to pay the loan, CX is liable to pay. However, his liability is limited to the extent of the value of the said property. ALTERNATIVE ANSWER: CX is not personally liable to the bank loan because it was contracted by DY in his personal capacity. Only the property of CX is liable. Hence, while CX has authorized the mortgage on his property to secure the loan of DY, the bank cannot sue CX to collect the loan in case DY defaults thereon. The bank can only foreclose the property of CX.

CIVIL LAW Answers to the BAR as Arranged by Topics (Year 1990-2006)

And if the proceeds of the foreclosure are not sufficient to pay the loan in full, the bank cannot run after CX for the deficiency. ALTERNATIVE ANSWER:

While as a general rule the principal is not liable for the contract entered into by his agent in case the agent acted in his own name without disclosing his principal, such rule does not apply if the contract involves a thing belonging to the principal. In such case, the principal is liable under Article 1883 of the Civil Code. The contract is deemed made on his behalf (Sy-juco v. Sy-juco 40 Phil. 634 [1920]).

All those contracts were executed by B while A was confined due to illness in the Makati Medical Center. Rule on the validity and binding effect of each of the above contracts upon A the principal. Explain your answers, SUGGESTED ANSWER:

The agency couched in general terms comprised only acts of administration (Art. 1877, Civil Code). The lease contract on the Manila parcel is not valid, not enforceable and not binding upon A. For B to lease the property to C, for more than one (1) year, A must provide B with a special power of attorney (Art. 1878. Civil Code).

ALTERNATIVE ANSWER:

CX would not be liable for the bank loan. CX's property would also not be liable on the mortgage. Since DY did not specify that he was acting for CX in the transaction with the bank, DY in effect acted in his own name. In the case of Rural Bank of Bombon v. CA, 212 SCRA, (1992), the Supreme Court, under the same facts, ruled that "in order to bind the principal by a mortgage on real property executed by an agent, it must upon its face purport to be made, signed and sealed in the name of the principal, otherwise, it will bind the agent only. It is not enough merely that the agent was in fact authorized to make the mortgage, if he, has not acted in the name of the principal. Neither is it ordinarily sufficient that in the mortgage the agent describes himself as acting by virtue of a power of attorney, if in fact the agent has acted in his own name and has set his own hand and seal to the mortgage. There is no principle of law by which a person can become liable on a real estate mortgage which she never executed in person or by attorney in fact". Appointment of Sub-Agent (1999) X appoints Y as his agent to sell his products in Cebu City. Can Y appoint a sub-agent and if he does, what are the effects of such appointment? (5%) SUGGESTED ANSWER:

Yes, the agent may appoint a substitute or sub-agent if the principal has not prohibited him from doing so, but he shall be responsible for the acts of the substitute: (1) when he was not given the power to appoint one; (2) when he was given such power, but without designating the person, and the person appointed was notoriously incompetent or insolvent. General Agency vs. Special Agency (1992) A as principal appointed B as his agent granting him general and unlimited management over A's properties, stating that A withholds no power from B and that the agent may execute such acts as he may consider appropriate. Accordingly, B leased A's parcel of land in Manila to C for four (4) years at P60,000.00 per year, payable annually in advance. B leased another parcel of land of A in Caloocan City to D without a fixed term at P3,000.00 per month payable monthly. B sold to E a third parcel of land belonging to A located in Quezon City for three (3) times the price that was listed in the inventory by A to B.

The lease of the Caloocan City property to D is valid and binding upon A. Since the lease is without a fixed term, it is understood to be from month to month, since the rental is payable monthly (Art. 1687, Civil Code). The sale of the Quezon City parcel to E is not valid and not binding upon A. B needed a special power of attorney to validly sell the land (Arts. 1877 and 1878, Civil Code). The sale of the land at a very good price does not cure the defect of the contract arising from lack of authority Powers of the Agent (1994) Prime Realty Corporation appointed Nestor the exclusive agent in the sale of lots of its newly developed subdivision. Prime Realty told Nestor that he could not collect or receive payments from the buyers. Nestor was able to sell ten lots to Jesus and to collect the down payments for said lots. He did not turn over the collections to Prime Realty. Who shall bear the loss for Nestor's defalcation, Prime Realty or Jesus? SUGGESTED ANSWER:

a) The general rule is that a person dealing with an agent must inquire into the authority of that agent. In the present case, if Jesus did not inquire into that authority, he is liable for the loss due to Nestor's defalcation unless Article 1900, Civil Code governs, in which case the developer corporation bears the loss. Art. 1900 Civil Code provides: "So far as third persons are concerned, an act is deemed to have been performed within the scope of the agent's authority, if such act is within the terms of the power of attorney, as written, even if the agent has in fact exceeded the limits of his authority according to an understanding between the principal and the agent. However, if Jesus made due inquiry and he was not informed by the principal Prime Realty of the limits of Nestor's authority. Prime Realty shall bear the loss. b) Considering that Prime Realty Corporation only "told" Nestor that he could not receive or collect payments, it appears that the limitation does not appear in his written authority or power of attorney. In this case, insofar as Jesus, who is a third person is concerned, Nestor's acts of collecting payments is deemed to have been performed within the scope of his authority {Article 1900. Civil Code). Hence, the principal is liable. However, if Jesus was aware of the limitation of Nestor's power as an agent, and Prime Realty Corporation does not

CIVIL LAW Answers to the BAR as Arranged by Topics (Year 1990-2006)

ratify the sale contract, then Jesus shall be liable (Article 1898. Civil Code).

Termination; Effect of Death of Agent (1997) Stating briefly the thesis to support your answer to each of the following cases, will the death - (c) of an agent end an agency? SUGGESTED ANSWER:

Yes. The death of an agent extinguishes the agency, by express provision of par. 3, Art 1919 of the Civil Code.

PARTNERSHIP Composition of Partnerships; Spouses; Corporations (1994) 1) Can a husband and wife form a limited partnership to engage in real estate business, with the wife being a limited partner? 2) Can two corporations organize a general partnership under the Civil Code of the Philippines? 3) Can a corporation and an individual form a general partnership? SUGGESTED ANSWER:

1) a) Yes. The Civil Code prohibits a husband and wife from constituting a universal partnership. Since a limited partnership is not a universal partnership, a husband and wife may validly form one. b) Yes. While spouses cannot enter into a universal partnership, they can enter into a limited partnership or be members thereof (CIR u. Suter, etal. 27 SCRA 152). SUGGESTED ANSWER:

2) a) No, A corporation is managed by its board of directors. If the corporation were to become a partner, co-partners would have the power to make the corporation party to transactions in an irregular manner since the partners are not agents subject to the control of the Board of Directors. But a corporation may enter into a joint venture with another corporation as long as the nature of the venture is in line with the business authorized by its charter. (Tuason & Co., Inc. v. Bolano, 95 Phil. 106).

b) As a general rule a corporation may not form a general partnership with another corporation or an individual because a corporation may not be bound by persons who are neither directors nor officers of the corporation. However, a corporation may form a general partnership with another corporation or an individual provided the following conditions are met: 1) The Articles of Incorporation of the corporation expressly allows the corporation to enter into partnerships; 2) The Articles of Partnership must provide that all partners will manage the partnership, and they shall be jointly and severally liable; and 3) In case of a foreign corporation, it must be licensed to do business in the Philippines. c) No. A corporation may not be a general partner because the principle of mutual agency in general partnership

allowing the other general partner to bind the corporation will violate the corporation law principle that only the board of directors may bind the corporation.

SUGGESTED ANSWER:

3) No, for the same reasons given in the Answer to Number 2 above. Conveyance of a Partner’s Share Dissolution (1998) Dielle, Karlo and Una are general partners in a merchandising firm. Having contributed equal amounts to the capital, they also agree on equal distribution of whatever net profit is realized per fiscal period. After two years of operation, however, Una conveys her whole interest in the partnership to Justine, without the knowledge and consent of Dielle and Karlo. 1. Is the partnership dissolved? 12%] 2. What are the rights of Justine, if any, should she desire to participate in the management of the partnership and in the distribution of a net profit of P360.000.00 which was realized after her purchase of Una's interest? [3%] SUGGESTED ANSWER:

1. No, a conveyance by a partner of his whole interest in a partnership does not of itself dissolve the partnership in the absence of an agreement. (Art. 1813. Civil Code) SUGGESTED ANSWER:

2. Justine cannot interfere or participate in the management or administration of the partnership business or affairs. She may, however, receive the net profits to which Una would have otherwise been entitled. In this case, P120.000 (Art. 1813, Civil Code) Dissolution of Partnership (1995) Pauline, Patricia and Priscilla formed a business partnership for the purpose of engaging in neon advertising for a term of five (5) years. Pauline subsequently assigned to Philip her interest in the partnership. When Patricia and Priscilla learned of the assignment, they decided to dissolve the partnership before the expiration of its term as they had an unproductive business relationship with Philip in the past. On the other hand, unaware of the move of Patricia and Priscilla but sensing their negative reaction to his acquisition of Pauline's interest, Philip simultaneously petitioned for the dissolution of the partnership. 1. Is the dissolution done by Patricia and Priscilla without the consent of Pauline or Philip valid? Explain. 2. Does Philip have any right to petition for the dissolution of the partnership before the expiration of its specified term? Explain. SUGGESTED ANSWER:

1, Under Art. 1830 (1) (c) of the NCC, the dissolution by Patricia and Priscilla is valid and did not violate the contract of partnership even though Pauline and Philip did not consent thereto. The consent of Pauline is not necessary because she had already assigned her interest to Philip. The consent of Philip is not also necessary because the assignment to him of Pauline's interest did not make him a partner, under Art, 1813 of the NCC. ALTERNATIVE ANSWER:

Interpreting Art. 1830 (1) (c) to mean that if one of the partners had assigned his interest on the partnership to

CIVIL LAW Answers to the BAR as Arranged by Topics (Year 1990-2006)

another the remaining partners may not dissolve the partnership, the dissolution by Patricia and Priscilla without the consent of Pauline or Philip is not valid. SUGGESTED ANSWER:

2. No, Philip has no right to petition for dissolution because he does not have the standing of a partner (Art. 1813 NCC). Dissolution of Partnership; Termination (1993) A, B and C formed a partnership for the purpose of contracting with the Government in the construction of one of its bridges. On June 30, 1992, after completion of the project, the bridge was turned over by the partners to the Government. On August 30, 1992, D, a supplier of materials used in the project sued A for collection of the indebtedness to him. A moved to dismiss the complaint against him on the ground that it was the ABC partnership that is liable for the debt. D replied that ABC partnership was dissolved upon completion of the project for which purpose the partnership was formed. Will you dismiss the complaint against A If you were the Judge? SUGGESTED ANSWER:

As Judge, I would not dismiss the complaint against A. because A is still liable as a general partner for his pro rata share of 1/3 (Art. 1816, C. C.J. Dissolution of a partnership caused by the termination of the particular undertaking specified in the agreement does not extinguish obligations, which must be liquidated during the "winding up" of the partnership affairs (Articles 1829 and 1830. par. 1-a, Civil Code). Effect of Death of Partner (1997) Stating briefly the thesis to support your answer to each of the following cases, will the death - of a partner terminate the partnership? SUGGESTED ANSWER:

Yes. The death of a partner will terminate the partnership, by express provision of par. 5, Art. 1830 of the Civil Code. Obligations of a Partner (1992) W, X, Y and Z organized a general partnership with W and X as industrial partners and Y and Z as capitalist partners. Y contributed P50,000.00 and Z contributed P20,000.00 to the common fund. By a unanimous vote of the partners, W and X were appointed managing partners, without any specification of their respective powers and duties. A applied for the position of Secretary and B applied for the position of Accountant of the partnership. The hiring of A was decided upon by W and X, but was opposed by Y and Z. The hiring of B was decided upon by W and Z, but was opposed by X and Y. Who of the applicants should be hired by the partnership? Explain and give your reasons. SUGGESTED ANSWER:

A should be hired as Secretary. The decision for the hiring of A prevails because it is an act of administration which can be performed by the duly appointed managing partners, W and X. B cannot be hired, because in case of a tie in the decision of the managing partners, the deadlock must be decided by the partners owning the controlling interest. In this case, the opposition of X and Y prevails because Y owns the controlling Interest (Art. 1801, Civil Code). Obligations of a Partner; Industrial Partner (2001) Joe and Rudy formed a partnership to operate a car repair shop in Quezon City. Joe provided the capital while Rudy contributed his labor and industry. On one side of their shop, Joe opened and operated a coffee shop, while on the other side, Rudy put up a car accessories store. May they engage in such separate businesses? Why? [5%] SUGGESTED ANSWER:

Joe, the capitalist partner, may engage in the restaurant business because it is not the same kind of business the partnership is engaged in. On the other hand, Rudy may not engage in any other business unless their partnership expressly permits him to do so because as an industrial partner he has to devote his full time to the business of the partnership [Art. 1789, CC).

Commodatum & Mutuum Commodatum (1993) A, upon request, loaned his passenger Jeepney to B to enable B to bring his sick wife from Paniqui. Tarlac to the Philippine General Hospital in Manila for treatment. On the way back to Paniqui, after leaving his wife at the hospital, people stopped the passenger Jeepney. B stopped for them and allowed them to ride on board, accepting payment from them just as in the case of ordinary passenger Jeepneys plying their route. As B was crossing Bamban, there was an onrush of Lahar from Mt Pinatubo, the Jeep that was loaned to him was wrecked. 1) What do you call the contract that was entered into by A and B with respect to the passenger Jeepney that was loaned by A to B to transport the latter's sick wife to Manila? 2) Is B obliged to pay A for the use of the passenger jeepney? 3) Is B liable to A for the loss of the Jeepney? SUGGESTED ANSWER:

1) The contract is called "commodatum". [Art. 1933. Civil Code). COMMODATUM is a contract by which one of the parties (bailor) delivers to another (bailee) something not consumable so that the latter may use it for a certain time and return it. 2) No, B is not obliged to pay A for the use of the passenger Jeepney because commodatum is essentially gratuitous. (Art. 1933. Civil Code] 3) Yes, because B devoted the thing to a purpose different from that for which it has been loaned (Art. 1942, par. 2, Civil Code)

CIVIL LAW Answers to the BAR as Arranged by Topics (Year 1990-2006) ALTERNATIVE ANSWER:

No, because an obligation which consists in the delivery of a determinate thing shall be extinguished if it should be lost or destroyed without the fault of the debtor, and before he has incurred in delay. (Art. 1262. Civil Code) Commodatum (2005) Before he left for Riyadh to work as a mechanic, Pedro left his Adventure van with Tito, with the understanding that the latter could use it for one year for his personal or family use while Pedro works in Riyadh. He did not tell Tito that the brakes of the van were faulty. Tito had the van tuned up and the brakes repaired. He spent a total amount of P15,000.00. After using the vehicle for two weeks, Tito discovered that it consumed too much fuel. To make up for the expenses, he leased it to Annabelle. Two months later, Pedro returned to the Philippines and asked Tito to return the van. Unfortunately, while being driven by Tito, the van was accidentally damaged by a cargo truck without his fault. a) Who shall bear the P15,000.00 spent for the repair of the van? Explain. (2%) ALTERNATIVE ANSWER:

Tito must bear the P15,000.00 expenses for the van. Generally, extraordinary expenses for the preservation of the thing loaned are paid by the bailor, he being the owner of the thing loaned. In this case however, Tito should bear the expenses because he incurred the expenses without first informing Pedro about it. Neither was the repair shown to be urgent. Under Article 1949 of the Civil Code, bailor generally bears the extraordinary expenses for the preservation of the thing and should refund the said expenses if made by the bailee; Provided, The bailee brings the same to the attention of the bailor before incurring them, except only if the repair is urgent that reply cannot be awaited. ALTERNATIVE ANSWER:

The P15,000.00 spent for the repair of the van should be borne by Pedro. Where the bailor delivers to the bailee a non-consummable thing so that the latter may use it for a certain time and return the identical thing, the contract perfected is a Contract of Commodatum. (Art. 1933, Civil Code) The bailor shall refund the extraordinary expenses during the contract for the preservation of the thing loaned provided the bailee brings the same to the knowledge of the bailor before incurring the same, except when they are so urgent that the reply to the notification cannot be awaited without danger. (Art. 1949 of the Civil Code) In the given problem, Pedro left his Adventure van with Tito so that the latter could use it for one year while he was in Riyadh. There was no mention of a consideration. Thus, the contract perfected was commodatum. The amount of P15,000.00 was spent by Tito to tune up the van and to repair its brakes. Such expenses are extra-ordinary expenses because they are necessary for the preservation of the van Thus, the same should be borne by the bailor, Pedro. b) Who shall bear the costs for the van's fuel, oil and other materials while it was with Tito? Explain. (2%) SUGGESTED ANSWER:

Tito must also pay for the ordinary expenses for the use and preservation of the thing loaned. He must pay for the gasoline, oil, greasing and spraying. He cannot ask for reimbursement because he has the obligation to return the identical thing to the bailor. Under Article 1941 of the Civil Code, the bailee is obliged to pay for the ordinary expenses for the use and preservation of the thing loaned.

c) Does Pedro have the right to retrieve the van even before the lapse of one year? Explain. (2%) ALTERNATIVE ANSWER:

No, Pedro does not have the right to retrieve the van before the lapse of one year. The parties are mutually bound by the terms of the contract. Under the Civil Code, there are only 3 instances when the bailor could validly ask for the return of the thing loaned even before the expiration of the period. These are when: (1) a precarium contract was entered (Article 1947); (2) if the bailor urgently needs the thing (Article 1946); and (3) if the bailee commits acts of ingratitude (Article 1948). Not one of the situations is present in this case. The fact that Tito had leased the thing loaned to Annabelle would not justify the demand for the return of the thing loaned before expiration of the period. Under Article 1942 of the Civil Code, leasing of the thing loaned to a third person not member of the household of the bailee, will only entitle bailor to hold bailee liable for the loss of the thing loaned. ALTERNATIVE ANSWER:

As a rule, Pedro does not have the right to retrieve the van before the lapse of one year. Article 1946 of the Code provides that "the bailor cannot demand the return of the thing loaned till after the expiration of the period stipulated, or after the accomplishment of the use for which the commodatum has been constituted. However, if in the meantime, he should have urgent need of the thing, he may demand its return or temporary use." In the given problem, Pedro allowed Tito to use the van for one year. Thus, he should be bound by the said agreement and he cannot ask for the return of the car before the expiration of the one year period. However, if Pedro has urgent need of the van, he may demand for its return or temporary use. d) Who shall bear the expenses for the accidental damage caused by the cargo truck, granting that the truck driver and truck owner are insolvent? Explain. (2%) SUGGESTED ANSWER:

Generally, extraordinary expenses arising on the occasion of the actual use of the thing loaned by the bailee, even if incurred without fault of the bailee, shall be shouldered equally by the bailor and the bailee. (Art. 1949 of the Civil Code). However, if Pedro had an urgent need for the vehicle, Tito would be in delay for failure to immediately return the same, then Tito would be held liable for the extraordinary expenses. Commodatum vs. Usufruct (1998)

CIVIL LAW Answers to the BAR as Arranged by Topics (Year 1990-2006)

Distinguish usufruct from commodatum and state whether these may be constituted over consumable goods. [2%] SUGGESTED ANSWER:

1. USUFRUCT is a right given to a person (usufructuary) to enjoy the property of another with the obligation of preserving its form and substance. (Art. 562. Civil Code) On the other hand, COMMODATUM is a contract by which one of the parties (bailor) delivers to another (bailee) something not consumable so that the latter may use it for a certain time and return it. In usufruct the usufructuary gets the right to the use and to the fruits of the same, while in commodatum, the bailee only acquires the use of the thing loaned but not its fruits. Usufruct may be constituted on the whole or a part of the fruits of the thing. (Art. 564. Civil Code). It may even be constituted over consumables like money (Alunan v. Veloso, 52 Phil. 545). On the other hand, in commodatum, consumable goods may be subject thereof only when the purpose of the contract is not the consumption of the object, as when it is merely for exhibition. (Art. 1936, Civil Code) ANOTHER ANSWER:

1. There are several points of distinction between usufruct and commodatum. Usufruct is constituted by law, by contract, by testamentary succession, or by prescription (Art. 1933. Civil Code). Usufruct creates a real right to the fruits of another's property, while commodatum creates only a purely personal right to use another's property, and requires a stipulation to enable the bailee to "make use" of the fruits (Arts. 1939& 1940, Civil Code). Usufruct maybe onerous while commodatum is always or essentially gratuitous (Arts. 1933 & 1935, Civil Code). The contract constituting usufruct is consensual, while commodatum is a real contract (perfected only by delivery of the subject matter thereof). However, both involve the enjoyment by a person of the property of another, differing only as to the extent and scope of such enjoyment [jus fruendi in one and Jus utendi in the other); both may have as subject matter either an immovable or a movable; and, both maybe constituted over consumable goods (Arts. 574 & 1936, Civil Code). A consumable thing may be the subject-matter of an abnormal usufruct but in a normal usufruct, the subject-matter may be used only for exhibition. A commodatum of a consumable thing may be only for the purpose of exhibiting, not consuming it.

Mutuum vs. Commodatum (2004) Distinguish briefly but clearly between Mutuum and commodatum. SUGGESTED ANSWER:

In MUTUUM, the object borrowed must be a consumable thing the ownership of which is transferred to the borrower who incurs the obligation to return the same consumable to the lender in an equal amount, and of the same kind and quality. In COMMODATUM, the object borrowed is usually a non-consumable thing the ownership of which is not transferred to the borrower who incurs the obligation to return the very thing to the lender.

Mutuum; Interests (2001) Samuel borrowed P300,000.00 housing loan from the bank at 18% per annum interest. However, the promissory note contained a proviso that the bank "reserves the right to increase interest within the limits allowed by law," By virtue of such proviso, over the objections of Samuel, the bank increased the interest rate periodically until it reached 48% per annum. Finally, Samuel filed an action questioning the right of the bank to increase the interest rate up to 48%. The bank raised the defense that the Central Bank of the Philippines had already suspended the Usury Law. Will the action prosper or not? Why? (5%) SUGGESTED ANSWER:

The action will prosper. While it is true that the interest ceilings set by the Usury Law are no longer in force, it has been held that PD No. 1684 and CB Circular No. 905 merely allow contracting parties to stipulate freely on any adjustment in the interest rate on a loan or forbearance of money but do not authorize a unilateral increase of the interest rate by one party without the other's consent (PNB v. CA, 238 SCRA 2O [1994]]). To say otherwise will violate the principle of mutuality of contracts under Article 1308 of the Civil Code. To be valid, therefore, any change of interest must be mutually agreed upon by the parties (Dizon v, Magsaysay, 57 SCRA 25O [1974]). In the present problem, the debtor not having given his consent to the increase in interest, the increase is void. Mutuum; Interests (2002) Carlos sues Dino for (a) collection on a promissory note for a loan, with no agreement on interest, on which Dino defaulted, and (b) damages caused by Dino on his (Carlos’) priceless Michaelangelo painting on which Dino is liable on the promissory note and awards damages to Carlos for the damaged painting, with interests for both awards. What rates of interest may the court impose with respect to both awards? Explain. (5%) SUGGESTED ANSWER:

With respect to the collection of money or promissory note, it being a forbearance of money, the legal rate of interest for having defaulted on the payment of 12% will apply. With respect to the damages to the painting, it is 6% from the time of the final demand up to the time of finality of judgment until judgment credit is fully paid. The court considers the latter as a forbearance of money. (Eastern Shipping Lines, Inc. v. CA, 234 SCRA 78 [1994]; Art 2210 and 2211, CC)

Mutuum; Interests (2004) The parties in a contract of loan of money agreed that the yearly interest rate is 12% and it can be increased if there is a law that would authorize the increase of interest rates. Suppose OB, the lender, would increase by 5% the rate of interest to be paid by TY, the borrower, without a law authorizing such increase, would OB’s action be just and valid? Why? Has TY a remedy against the imposition of the rate increase? Explain. (5%) SUGGESTED ANSWER:

OB's action is not just and valid. The debtor cannot be required to pay the increase in interest there being no law authorizing it, as stipulated in the contract. Increasing the

CIVIL LAW Answers to the BAR as Arranged by Topics (Year 1990-2006)

rate in the absence of such law violates the principle of mutuality of contracts. ALTERNATIVE ANSWER:

Even if there was a law authorizing the increase in interest rate, the stipulation is still void because there is no corresponding stipulation to decrease the interest due when the law reduces the rate of interest.

DEPOSIT Compensation; Bank Loan (1997) In order to secure a bank loan, XYZ Corporation surrendered its deposit certificate, with a maturity date of 01 September 1997 to the bank. The corporation defaulted on the due repayment of the loan, prompting the bank to encash the deposit certificate. XYZ Corporation questioned the above action taken by the bank as being a case of pactum commissorium. The bank disagrees. What is your opinion? SUGGESTED ANSWER:

We submit that there is no pactum commissorium here. Deposits of money in banks and similar institutions are governed by the provisions on simple loans (Art. 1980. Civil Code). The relationship between the depositor and a bank is one of creditor and debtor. Basically this is a matter of compensation as all the elements of compensation are present in this case (BPI vs. CA, 232 SCRA 302). ADDITIONAL ANSWER:

Where the security for the debt is also money deposited in a bank, it is not illegal for the creditor to encash the time deposit certificates to pay the debtor's overdue obligation. (Chu us. CA, et al., G.R 78519, September 26, 1989).

Deposit; Exchange (1992) X and Y staged a daring bank robbery in Manila at 10:30 AM in the morning of a regular business day, and escaped with their loot of two (2) bags, each bag containing P50,000,00. During their flight to elude the police, X and Y entered the nearby locked house of A, then working in his Quezon City office. From A's house, X and Y stole a box containing cash totaling P50,000.00 which box A had been keeping in deposit for his friend B. In their hurry, X and Y left in A's bedroom one (1) of the bags which they had taken from the bank. With X and Y now at large and nowhere to be found, the bag containing P50.000.00 is now claimed by B, by the Mayor of Manila, and by the bank. B claims that the depository. A, by force majeure had obtained the bag of money in place of the box of money deposited by B. The Mayor of Manila, on the other hand, claims that the bag of money should be deposited with the Office of the Mayor as required of the finder by the provisions of the Civil Code. The bank resists the claims of B and the Mayor of Manila.

To whom should a deliver the bag of money? Decide with reasons. SUGGESTED ANSWER:

B would have no right to claim the money. Article 1990 of the Civil Code is not applicable. The law refers to another thing received in substitution of the object deposited and is predicated upon something exchanged. The Mayor of Manila cannot invoke. Article 719 of the Civil Code which requires the finder to deposit the thing with the Mayor only when the previous possessor is unknown. In this case , a must return the bag of money to the bank as the previous possessor and known owner (Arts. 719 and 1990. Civil Code.)

SURETY Recovery of Deficiency (1997) AB sold to CD a motor vehicle for and in consideration of P120,000.00 to be paid in twelve monthly equal installments of P10,000,00, each installment being due and payable on the 15th day of each month starting January 1997. To secure the promissory note, CD (a) executed a chattel mortgage on the subject motor vehicle, and (b) furnished a surety bond issued by Philam life, CD failed to pay more than two (2) installments, AB went after the surety but he was only able to obtain three-fourths (3/4) of the total amount still due and owing from CD. AB seeks your advice on how he might, if at all, recover the deficiency. How would you counsel AB? SUGGESTED ANSWER:

Yes, he can recover the deficiency. The action of AB to go after the surety bond cannot be taken to mean a waiver of his right to demand payment for the whole debt, The amount received from the surety is only payment pro tanto, and an action may be maintained for a deficiency debt.

ANTICHRESIS Antichresis (1995) Olivia owns a vast mango plantation which she can no longer properly manage due to a lingering illness. Since she is indebted to Peter in the amount of P500.000.00 she asks Peter to manage the plantation and apply the harvest to the payment of her obligation to him, principal and interest, until her indebtedness shall have been fully paid. Peter agrees. 1) What kind of contract is entered into between Olivia and Peter? Explain. 2) What specific obligations are imposed by law on Peter as a consequence of their contract? 3) Does the law require any specific form for the validity of their contract? Explain 4) May Olivia re-acquire the plantation before her entire indebtedness shall have been fully paid? Explain. SUGGESTED ANSWER:

CIVIL LAW Answers to the BAR as Arranged by Topics (Year 1990-2006)

1. A contract of antichresis was entered into between Olivia and Peter. Under Article 2132 of the New Civil Code, by a contract of antichresis the creditor acquires the right to receive the fruits of an immovable of his debtor, with the obligation to apply them to the payment of the interest, and thereafter to the principal of his credit. SUGGESTED ANSWER:

2. Peter must pay taxes and charges upon the land and bear the necessary expenses for preservation and repair which he may deduct from the fruits. (Art, 2135, NCC)

payment of the loan. However, the loan was not paid on time. A month after 4 years, may the shares of stock pledged be deemed owned by ABC or not? Reason. (5%) SUGGESTED ANSWER:

The shares of stock cannot be deemed owned by ABC upon default of MNO. They have to be foreclosed. Under Article 2088 of the Civil Code, the creditor cannot appropriate the things given by way of pledge. And even if the parties have stipulated that ABC becomes the owner of the shares in case MNO defaults on the loan, such stipulation is void for being a pactum commissorium.

SUGGESTED ANSWER:

3. The amount of the principal and interest must be specified in writing, otherwise the antichresis will be void. (Art. 2134, NCC) SUGGESTED ANSWER:

4. No. Art. 2136 specifically provides that the debtor cannot re-acquire the enjoyment of the immovable without first having totally paid what he owes the creditor. However, it is potestative on the part of the creditor to do so in order to exempt him from his obligation under Art. 2135, NCC, The debtor cannot re-acquire the enjoyment unless Peter compels Olivia to enter again the enjoyment of the property.

PLEDGE Pledge (1994) In 1982, Steve borrowed P400.000.00 from Danny, collateralized by a pledge of shares of stock of Concepcion Corporation worth P800,000,00. In 1983, because of the economic crisis, the value of the shares pledged fell to only P100,000.00. Can Danny demand that Steve surrender the other shares worth P700,000.00? SUGGESTED ANSWER:

a) No. Bilateral contracts cannot be changed unilaterally. A pledge is only a subsidiary contract, and Steve is still indebted to Danny for the amount of P400,000.00 despite the fall in the value of the stocks pledged. b) No. Danny's right as pledgee is to sell the pledged shares at a public sale and keep the proceeds as collateral for the loan. There is no showing that the fall in the value of the pledged property was attributable to the pledger's fault or fraud. On the contrary, the economic crisis was the culprit. Had the pledgee been deceived as to the substance or quality of the pledged shares of stock, he would have had the right to claim another thing in their place or to the immediate payment of the obligation. This is not the case here.

Pledge; Mortgage; Antichresis (1996) In the province, a farmer couple borrowed money from the local merchant. To guarantee payment, they left the Torrens Title of their land with the merchant, for him to hold until they pay the loan. Is there a - a) contract of pledge, b) contract of mortgage, c) contract of antichresis, or d) none of the above? Explain.

SUGGESTED ANSWER:

None of the above. There is no pledge because only movable property may be pledged (Art. 2094. NCC). If at all, there was a pledge of the paper or document constituting the Torrens Title, as a movable by itself, but not of the land which the title represents. There is no mortgage because no deed or contract was executed in the manner required by law for a mortgage (Arts. 2085 to 2092, NCC; 2124 to 2131, NCC). There is no contract of antichresis because no right to the fruits of the property was given to the creditor (Art. 2132 NCC). A contract of simple loan was entered into with security arrangement agreed upon by the parties which is not one of those mentioned above. ALTERNATIVE ANSWER:

There is a contract of mortgage constituted over the land. There is no particular form required for the validity of a mortgage of real property. It is not covered by the statute of frauds in Art. 1403, NCC and even assuming that it is covered, the delivery of the title to the creditor has taken it out of the coverage thereof. A contract of mortgage of real property is consensual and is binding on the parties despite absence of writing. However, third parties are not bound because of the absence of a written instrument evidencing the mortgage and, therefore the absence of registration. But this does not affect the validity of the mortgage between the Pledge (2004) parties (Art. 2125, NCC), The creditor may compel the ABC loaned to MNO P40,000 for which the latter pledged 400 debtor to execute the mortgage in a public document in order shares of stock in XYZ Inc. It was agreed that if the pledgor to allow its registration (Art. 1357.NCC in relation to Art. failed to pay the loan with 10% yearly interest within four years, 1358. NCC). the pledgee is authorized to foreclose on the shares of stock. As required, MNO delivered possession of the shares to ABC with the understanding that the shares would be returned to MNO QUASI-CONTRACT upon the

CIVIL LAW Answers to the BAR as Arranged by Topics (Year 1990-2006)

Quasi-Contracts; Negotiorium Gestio (1992) In fear of reprisals from lawless elements besieging his barangay, X abandoned his fishpond, fled to Manila and left for Europe. Seeking that the fish in the fishpond were ready for harvest, Y, who is in the business of managing fishponds on a commission basis, took possession of the property, harvested the fish and sold the entire harvest to Z. Thereafter, Y borrowed money from W and used the money to buy new supplies of fish fry and to prepare the fishpond for the next crop. a) What is the Juridical relation between X and Y during X's absence? b) Upon the return of X to the barangay, what are the obligations of Y to X as regards the contract with Z? c) Upon X's return, what are the obligations of X as regards Y's contract with W? d) What legal effects will result if X expressly ratifies Y's management and what would be the obligations of X in favor of Y? Explain all your answers.

house under the principle of negotiorum gestio. He was not liable as the burning of the house is a fortuitous event. Is B liable to A for damages under the foregoing circumstances? SUGGESTED ANSWER:

No. B is not liable for damages, because he is a gestor in negotiorum gestio (Art. 2144, Civil Code) Furthermore, B is not liable to A because Article 2147 of the Civil Code is not applicable. B did not undertake risky operations which the owner was not accustomed to embark upon: a) he has not preferred his own interest to that of the owner; b) he has not failed to return the property or business after demand by the owner; and c) he has not assumed the management in bad faith. ALTERNATIVE ANSWER:

SUGGESTED ANSWER:

(a) The juridical relation is that of the quasi-contract of "negotiorum gestio". Y is the "gestor" or "officious manager" and X is the "owner" (Art. 2144, Civil Code). (b) Y must render an account of his operations and deliver to X the price he received for the sale of the harvested fish (Art, 2145, Civil Code). (c) X must pay the loan obtained by Y from W because X must answer for obligations contracted with third persons in the interest of the owner (Art. 2150, Civil Code), (d) Express ratification by X provides the effects of an express agency and X is liable to pay the commissions habitually received by the gestor as manager (Art. 2149, Civil Code). Quasi-Contracts; Negotiorium Gestio (1993) In September, 1972, upon declaration of martial rule in the Philippines. A, together with his wife and children. disappeared from his residence along A. Mabini Street. Ermita, Manila. B, his immediate neighbor, noticing that mysterious disappearance of A and his family, closed the doors and windows of his house to prevent it from being burglarized. Years passed without B hearing from A and his family, B continued taking care of A's house, even causing minor repairs to be done at his house to preserve it. In 1976, when business began to perk up in the area, an enterprising man. C, approached B and proposed that they build stores at the ground floor of the house and convert its second floor into a pension house. B agreed to Cs proposal and together they spent for the construction of stores at the ground floor and the conversion of the second floor into a pension house. While construction was going on, fire occurred at a nearby house. The houses at the entire block, including A's were burned. After the EDSA revolution in February 1986, A and his family returned from the United States where they took refuge in 1972. Upon learning of what happened to his house. A sued B for damages, B pleaded as a defense that he merely took charge of his

He would be liable under Art. 2147 (1) of the Civil Code, because he used the property for an operation which the operator is not accustomed to, and in so doing, he exposed the house to increased risk, namely the operation of a pension house on the second floor and stores on the first floor Quasi-Contracts; Negotiorium Gestio (1995) Armando owns a row of residential apartments in San Juan, Metro Manila, which he rents out to tenants. On 1 April 1991 he left for the United States without appointing any administrator to manage his apartments such that uncollected rentals accumulated for three (3) years. Amparo, a niece of Armando, concerned with the interest of her uncle, took it upon herself to administer the property. As a consequence, she incurred expenses in collecting the rents and in some instances even spent for necessary repairs to preserve the property. 1. What Juridical relation between Amparo and Armando, if any, has resulted from Amparo's unilateral act of assuming the administration of Armando's apartments? Explain. 2. What rights and obligations, if any, does Amparo have under the circumstances? Explain. SUGGESTED ANSWER:

1. Negotiorum gestio existed between Amparo and Armando, She voluntarily took charge of the agency or management of the business or property of her uncle without any power from her uncle whose property was neglected. She is called the gestor negotiorum or officious manager, (Art. 2144, NCC) 2. It is recommended by the Committee that an enumeration of any two (2) obligations and two (2) rights as enumerated in Arts. 2145 to 2152, NCC, would entitle the examinee to full credit.

Art. 2145. The officious manager shall perform his duties with all the diligence of a good father of a family, and pay the damages which through his fault or negligence may be suffered by the owner of the property or business under management.

CIVIL LAW Answers to the BAR as Arranged by Topics (Year 1990-2006)

The courts may, however, increase or moderate the indemnity according to the circumstances of each case.

Art. 2146. If the officious manager delegates to another person all or some of his duties, he shall be liable for the acts of the delegate, without prejudice to the direct obligation of the latter toward the owner of the business. The responsibility of two or more officious managers shall be solidary, unless management was assumed to save the thing or business from imminent danger. Art. 2147. The officious manager shall be liable for any fortuitous event: (1) If he undertakes risky operations which the owner was not accustomed to embark upon; (2) If he has preferred his own interest to that of the owner; (3) If he fails to return the property or business after demand by the owner, (4) If he assumed the management in bad faith. Art. 2148. Except when the management was assumed to save the property or business from imminent danger, the officious manager shall be liable for fortuitous events (1) If he is manifestly unfit to carry on the management; (2) If by his Intervention h e prevented a more competent person from taking up the management. Art. 2149. The ratification of the management by the owner of the business produces the effects of an express agency, even if the business may not have been successful. Art. 2150, Although the officious management may not have been expressly ratified, the owner of the property or business who enjoys the advantages of the same shall be liable for obligations incurred in his interest, and shall reimburse the officious manager for the necessary and useful expenses and for the damages which the latter may have suffered in the performance of his duties. The same obligation shall be incumbent upon him when the management had for its purpose the prevention of an imminent and manifest loss, although no benefit may have been derived. Art. 2151. Even though the owner did not derive any benefit and there has been no imminent and manifest danger to the property or business, the owner is liable as under the first paragraph of the preceding article, provided: (1) The officious manager has acted in good faith, and (2) The property or business is intact, ready to be returned to the owner. Art. 2152. The officious manager is personally liable for contracts which he has entered into with third persons, even though he acted in the name of the owner, and there shall be no right of action between the owner and third persons. These provisions shall not apply: (1) If the owner has expressly or tacitly ratified the management, or

(2) When the contract refers to things pertaining to the owner of the business, (NOTE: It is recommended by the Committee that an enumeration of any two (2) obligations and any two (2) rights as enumerated la Arts. 2145 to 2152, NCC would entitle the examinee to full credit.)

Quasi-Contracts; Solutio Indebiti (2004) DPO went to a store to buy a pack of cigarettes worth P225.00 only. He gave the vendor, RRA, a P500-peso bill. The vendor gave him the pack plus P375.00 change. Was there a discount, an oversight, or an error in the amount given? What would be DPO’s duty, if any, in case of an excess in the amount of change given by the vendor? How is this situational relationship between DPO and RRA denominated? Explain. (5%) SUGGESTED ANSWER:

There was error in the amount of change given by RRA. This is a case of solutio indebiti in that DPO received something that is not due him. He has the obligation to return the P100.00; otherwise, he will unjustly enrich himself at the expense of RRA. (Art. 2154, Civil Code) ALTERNATIVE ANSWER:

DPO has the duty to return to RRA the excess P100 as trustee under Article 1456 of the Civil Code which provides: If property is acquired through mistake or fraud, the person obtaining it is, by force of law, considered a trustee of an implied trust for the benefit of the person from whom the property comes. There is, in this case, an implied or constructive trust in favor of RRA.

TORTS & DAMAGES Collapse of Structures; Last Clear Chance (1990) Mr and Mrs R own a burned-out building, the firewall of which collapsed and destroyed the shop occupied by the family of Mr and Mrs S, which resulted in injuries to said couple and the death of their daughter. Mr and Mrs S had been warned by Mr & Mrs R to vacate the shop in view of its proximity to the weakened wall but the former failed to do so. Mr & Mrs S filed against Mr and Mrs R an action for recovery of damages the former suffered as a result of the collapse of the firewall. In defense, Mr and Mrs R rely on the doctrine of last clear chance alleging that Mr and Mrs S had the last clear chance to avoid the accident if only they heeded the former’s warning to vacate the shop, and therefore Mr and Mrs R’s prior negligence should be disregarded. If you were the judge, how would you decide the case? State your reasons. SUGGESTED ANSWER:

I would decide in favor of Mr & Mrs S. The proprietor of a building or structure is responsible for the damages resulting from its total or partial collapse, if it should be due to the lack of necessary repairs (Art 2190 Civil Code) As regards the defense of ―last clear chance,‖ the same is not tenable because according to the SC in one case (De Roy v CA L-80718, Jan 29, 1988, 157 S 757) the doctrine of last clear chance is not applicable to instances covered by Art 2190 of the Civil Code.

CIVIL LAW Answers to the BAR as Arranged by Topics (Year 1990-2006) Further, in Phoenix Construction, Inc. v. Intermediate Availing of that portion of Section 12 of Article II of the 1987 Constitution which reads; Appellate Court (G.R. L-65295, March 10, 1987. 148 SCRA 353)

the Supreme Court held that the role of the common law "last clear chance" doctrine in relation to Article 2179 of the Civil Code is merely to mitigate damages within the context of contributory negligence.

The State x xx shall equally protect the life of the mother and the life of the unborn from conception, "xxx" which he claims confers a civil personality on the unborn from the moment of conception.

Damages (1994) On January 5, 1992, Nonoy obtained a loan of Pl,000,000.00 from his friend Raffy. The promissory note did not stipulate any payment for Interest. The note was due on January 5, 1993 but before this date the two became political enemies. Nonoy, out of spite, deliberately defaulted in paying the note, thus forcing Raffy to sue him. 1) What actual damages can Raffy recover? 2) Can Raffy ask for moral damages from Nonoy? 3) Can Raffy ask for nominal damages? 4) Can Raffy ask for temperate damages? 5) Can Raffy ask for attorney's fees?

Boy filed a case for damages against the abortionist, praying therein that the latter be ordered to pay him: (a) P30,000.00 as indemnity for the death of the fetus, (b) P100.000.00 as moral damages for the mental anguish and anxiety he suffered, (c) P50,000.00 as exemplary damages, (d) P20,000.00 as nominal damages, and (e) P25,000.00 as attorney's fees. May actual damages be also recovered? If so, what facts should be alleged and proved?

SUGGESTED ANSWER:

SUGGESTED ANSWER:

Yes, provided that the pecuniary loss suffered should be substantiated and duly proved.

1) Raffy may recover the amount of the promissory note of P1 million, together with interest at the legal rate from the date of judicial or extrajudicial demand. In addition, however, inasmuch as the debtor is in bad faith, he is liable for all damages which may be reasonably attributed to the non-performance of the obligation. (Art. 2201(2). NCC).

Damages arising from Death of Unborn Child (2003) If a pregnant woman passenger of a bus were to suffer an abortion following a vehicular accident due to the gross negligence of the bus driver, may she and her husband claim damages from the bus company for the death of their unborn child? Explain. 5%

2) Yes, under Article 2220, NCC moral damages are recoverable in case of breach of contract where the defendant acted fraudulently or in bad faith.

No, the spouses cannot recover actual damages in the form of indemnity for the loss of life of the unborn child. This is because the unborn child is not yet considered a person and the law allows indemnity only for loss of life of person. The mother, however may recover damages for the bodily injury she suffered from the loss of the fetus which is considered part of her internal organ. The parents may also recover damages for injuries that are inflicted directly upon them, e.g., moral damages for mental anguish that attended the loss of the unborn child. Since there is gross negligence, exemplary damages can also be recovered. (Gelus v. CA, 2 SCRA 801

SUGGESTED ANSWER:

3) Nominal damages may not be recoverable in this case because Raffy may already be indemnified of his losses with the award of actual and compensatory damages. NOMINAL DAMAGES are adjudicated only in order that a right of the plaintiff, which has been violated or invaded by the defendant may be vindicated or recognized, and not for the purpose of indemnifying the plaintiff for any loss suffered by him. (Article 2231. Civil Code) 4) Raffy may ask for, but would most likely not be awarded temperate damages, for the reason that his actual damages may already be compensated upon proof thereof with the promissory note. TEMPERATE DAMAGES may be awarded only when the court finds that some pecuniary loss has been suffered but its amount cannot, from the nature of the case, be proved with certainty. (Article 2224, Civil Code) 5) Yes, under paragraph 2, Article 2208 of the Civil Code, considering that Nonoy's act or omission has compelled Raffy to litigate to protect his interests. Furthermore. attorneys' fees may be awarded by the court when it is just and equitable. (Article 2208(110) Civil Code). Damages arising from Death of Unborn Child (1991) On her third month of pregnancy, Rosemarie, married to Boy, for reasons known only to her, and without informing Boy, went to the clinic of X, a known abortionist, who for a fee, removed and expelled the fetus from her womb, Boy learned of the abortion six (6) months later.

[1961])

Death Indemnity (1994) Johnny Maton's conviction for homicide was affirmed by the Court of Appeals and in addition, although the prosecution had not appealed at all. The appellate court increased the indemnity for death from P30,000.00 to P50,000.00. On his appeal to the Supreme Court, among the other things Johnny Maton brought to the high court's attention, was the increase of indemnity imposed by the Court of Appeals despite the clear fact that the People had not appealed from the appellate court's judgment. Is Johnny Maton correct? SUGGESTED ANSWER:

a) In Abejam v. Court of Appeals, the Supreme Court said that even if the issue of damages were not raised by the appellant in the Court of Appeals but the Court of Appeals in its findings increased the damages, the Supreme Court will not disturb the findings of the Court of Appeals. b) No, the contention of the accused is not correct because upon appeal to the Appellate Court, the court acquired jurisdiction over the entire case, criminal as well as civil. Since the conviction of homicide had been appealed, there

CIVIL LAW Answers to the BAR as Arranged by Topics (Year 1990-2006)

is no finality in the amount of indemnity because the civil liability arising from the crime and the judgment on the crime has not yet become final

c) Yes. Since the civil indemnity is an award in the civil action arising from the criminal offense, the rule that a party cannot be granted affirmative relief unless he himself has appealed should apply. Therefore, it was error for the Court of Appeals to have expanded the indemnity since the judgment on the civil liability had become final. d)

No.

Courts can review matters not assigned as errors.

(Hydro Resource vs. CA . 204 SCRA 309).

Defense; Due Diligence in Selection (2003) As a result of a collision between the taxicab owned by A and another taxicab owned by B, X, a passenger of the first taxicab, was seriously injured. X later filed a criminal action against both drivers. May both taxicab owners raise the defense of due diligence in the selection and supervision of their drivers to be absolved from liability for damages to X? Reason. 5% SUGGESTED ANSWER:

It depends. If the civil action is based on a quasi-delict the taxicab owners may raise the defense of diligence of a good father of a family in the selection and supervision of the driver; if the action against them is based on culpa contractual or civil liability arising from a crime, they cannot raise the defense. Filing of Separate Civil Action; Need for Reservation (2003) As a result of a collision between the taxicab owned by A and another taxicab owned by B, X, a passenger of the first taxicab, was seriously injured. X later filed a criminal action against both drivers. Is it necessary for X to reserve his right to institute a civil action for damages against both taxicab owners before he can file a civil action for damages against them? Why SUGGESTED ANSWER:

It depends. If the separate civil action is to recover damages arising from the criminal act, reservation is necessary. If the civil action against the taxicab owners is based on culpa contractual, or on quasi-delict, there is no need for reservation. ALTERNATIVE ANSWER:

No, such reservation is not necessary. Under Section 1 of Rule 111 of the 2000 Rules on Criminal Procedure, what is ―deemed instituted‖ with the criminal action is only the action to recover civil liability arising from the crime or ex delicto. All the other civil actions under Articles 32, 33, 34 and 2176 of the New Civil Code are no longer ―deemed instituted‖, and may be filed separately and prosecuted independently even without any reservation in the criminal action (Section 3, Rule 111, Ibid). The failure to make a reservation in the criminal action is not a waiver of the right to file a separate and independent civil action based on these articles of the New Civil Code (Casupanan v. Laroya GR No. 145391, August 26, 2002).

Fortuitous Event; Mechanical Defects (2002)

A van owned by Orlando and driven by Diego, while negotiating a downhill slope of a city road, suddenly gained speed, obviously beyond the authorized limit in the area, and bumped a car in front of it, causing severed damage to the care and serious injuries to its passengers. Orlando was not in the car at the time of the incident. The car owner and the injured passengers sued Orlando and Diego for damages caused by Diego’s negligence. In their defense, Diego claims that the downhill slope caused the van to gain speed and that, as he stepped on the brakes to check the acceleration, the brakes locked, causing the van to go even faster and eventually to hit the car in front of it. Orlando and Diego contend that the sudden malfunction of the van’s brake system is a fortuitous even and that, therefore, they are exempt from any liability. Is this contention tenable? Explain. (2%)

SUGGESTED ANSWER:

No. Mechanical defects of a motor vehicle do not constitute fortuitous event, since the presence of such defects would have been readily detected by diligent maintenance check. The failure to maintain the vehicle in safe running condition constitutes negligence. Liability; Airline Company; Non-Performance of an Obligation (2004) DT and MT were prominent members of the frequent travelers’ club of FX Airlines. In Hongkong, the couple were assigned seats in Business Class for which they had bought tickets. On checking in, however, they were told they were upgraded by computer to First Class for the flight to Manila because the Business Section was overbooked. Both refused to transfer despite better seats, food, beverage and other services in First Class. They said they had guests in Business Class they should attend to. They felt humiliated, embarrassed and vexed, however, when the stewardess allegedly threatened to offload them if they did not avail of the upgrade. Thus they gave in, but during the transfer of luggage DT suffered pain in his arm and wrist. After arrival in Manila, they demanded an apology from FX’s management as well as indemnity payment. When none was forthcoming, they sued the airline for a million pesos in damages. Is the airline liable for actual and moral damages? Why or why not? Explain briefly. (5%) SUGGESTED ANSWER:

FX Airlines committed breach of contract when it upgraded DT and MT, over their objections, to First Class because they had contracted for Business Class passage. However, although there is a breach of contract, DT and MT are entitled to actual damages only for such pecuniary losses suffered by them as a result of such breach. There seems to be no showing that they incurred such pecuniary loss. There is no showing that the pain in DT's arm and wrist resulted directly from the carrier's acts complained of. Hence, they are not entitled to actual damages. Moreover, DT could have avoided the alleged injury by requesting the airline staff to do the luggage transfer as a matter of duty on their part. There is also no basis to award moral damages for such breach of contract because the facts of the problem do not show bad faith or fraud on the part of the airline. (Cathay Pacific v. Vazquez, 399 SCRA 207 [2003]). However, they

CIVIL LAW Answers to the BAR as Arranged by Topics (Year 1990-2006)

may recover moral damages if the cause of action is based on Article 21 of the Civil Code for the humiliation and embarrassment they felt when the stewardess threatened to offload them if they did not avail of the upgrade. ALTERNATIVE ANSWER:

If it can be proved that DT's pain in his arm and wrist occasioned by the transfer of luggage was caused by fault or negligence on the part of the airline's stewardess, actual damages may be recovered. The airline may be liable for moral damages pursuant to Art. 2219 (10) if the cause of action is based on Article 21 or an act contrary to morals in view of the humiliation suffered by DT and MT when they were separated from their guests and were threatened to be offloaded. Liability; Airline Company; Non-Performance of an Obligation (2005) Dr. and Mrs. Almeda are prominent citizens of the country and are frequent travelers abroad. In 1996, they booked round-trip business class tickets for the Manila-Hong Kong-Manila route of the Pinoy Airlines, where they are holders of Gold Mabalos Class Frequent Flier cards. On their return flight, Pinoy Airlines upgraded their tickets to first class without their consent and, inspite of their protestations to be allowed to remain in the business class so that they could be with their friends, they were told that the business class was already fully booked, and that they were given priority in upgrading because they are elite members/holders of Gold Mabalos Class cards. Since they were embarrassed at the discussions with the flight attendants, they were forced to take the flight at the first class section apart from their friends who were in the business class. Upon their return to Manila, they demanded a written apology from Pinoy Airlines. When it went unheeded, the couple sued Pinoy Airlines for breach of contract claiming moral and exemplary damages, as well as attorney's fees. Will the action prosper? Give reasons. (5%) ALTERNATIVE ANSWER:

Yes, the action will prosper. Article 2201 of the Civil Code entitles the person to recover damages which may be attributed to non-performance of an obligation. In Alitalia Airways v. Court of Appeals (G.R. No. 77011, July 24, 1990), when an airline issues ticket to a passenger confirmed on a particular flight, a contract of carriage arises and the passenger expects that he would fly on that day. When the airline deliberately overbooked, it took the risk of having to deprive some passengers of their seat in case all of them would show up. For the indignity and inconvenience of being refused the confirmed seat, said passenger is entitled to moral damages. In the given problem, spouses Almeda had a booked roundtrip business class ticket with Pinoy Airlines. When their tickets were upgraded to first class without their consent, Pinoy Airlines breached the contract. As ruled in Zulueta v. Pan American (G.R. No. L-28589, January 8, 1973),

in case of overbooking, airline is in bad faith. Therefore, spouses Almeda are entitled to damages. ALTERNATIVE ANSWER:

The action may or may not prosper. Moral damages include physical suffering, mental anguish, fright, serious anxiety, besmirched reputation, wounded feelings, moral shock, social humiliation, and similar injury. Although incapable of pecuniary computation, moral damages may be recovered if they are the proximate result of the defendant's wrongful act or omission. Moral damages predicated upon a breach of contract of carriage are recoverable only in instances where the carrier is guilty of fraud or bad faith or where the mishap resulted in the death of a passenger. (Cathay Pacific Airways, Ltd. v. Court of Appeals, G.R. No. 60501, March 5, 1993) Where there is no showing that the airline acted fraudulently or in bad faith, liability for damages is limited to the natural and probable consequences of the breach of the contract of carriage which the parties had foreseen or could have reasonably foreseen. In such a case the liability does not include moral and exemplary damages. In the instant case, if the involuntary upgrading of the Almedas' seat accommodation was not attended by fraud or bad faith, the award of moral damages has no leg to stand on. Thus, spouses would not also be entitled to exemplary damages. It is a requisite in the grant of exemplary damages that the act of the offender must be accompanied by bad faith or done in wanton, fraudulent or malevolent manner. (Morris v. Court of Appeals, G.R. No. 127957, February 21, 2001)

Moreover, to be entitled thereto, the claimant must first establish his right to moral, temperate, or compensatory damages. (Art. 2234, Civil Code) Since the Almedas are not entitled to any of these damages, the award for exemplary damages has no legal basis. Where the awards for moral and exemplary damages are eliminated, so must the award for attorney's fees be eliminated. (Orosa v. Court of Appeals, G.R. No. 111080, April 5, 2000; Morris v. Court of Appeals, G.R. No. 127957, February 21, 2001) The most that can be adjudged in

their favor for Pinoy Airlines' breach of contract is an award for nominal damages under Article 2221 of the Civil Code. (Cathay Pacific Airways v. Sps. Daniel & Maria Luisa Vasquez, G.R. No. 150843, March 14, 2003)

However, if spouses Almeda could prove that there was bad faith on the part of Pinoy Airlines when it breached the contract of carriage, it could be liable for moral, exemplary as well as attorney's fees. Liability; Employer; Damage caused by Employees (1997) a) When would an employer's liability for damage, caused by an employee in the performance of his assigned tasks, be primary and when would it be subsidiary in nature? b) Would the defense of due diligence in the selection and supervision of the employee be available to the employer in both instances? SUGGESTED ANSWER:: (a) The employer's liability for damage based on culpa aquiliana under Art, 2176 and 2180 of the Civil Code is primary; while that under Art. 103 of the Revised Penal Code is subsidiary.

CIVIL LAW Answers to the BAR as Arranged by Topics (Year 1990-2006)

(b) The defense of diligence in the selection and supervision of the employee under Article 2180 of the Civil Code is available only to those primarily liable thereunder, but not to those subsidiarily liable under Article 103 of the Revised Penal Code (Yumul vs. Juliano, 72 Phil. 94).

Liability; owner who was in the vehicle (1996) Marcial, who does not know how to drive, has always been driven by Ben, his driver of ten years whom he had chosen carefully and has never figured in a vehicular mishap. One day, Marcial was riding at the back seat of his Mercedes Benz being driven along EDSA by Ben. Absorbed in reading a book, Marcial did not notice that they were approaching the corner of Quezon Avenue, when the traffic light had just turned yellow. Ben suddenly stepped on the gas to cross the intersection before the traffic light could turn red. But, too late. Midway in the intersection, the traffic light changed, and a Jeepney full of passengers suddenly crossed the car's path. A collision between the two vehicles was inevitable. As a result, several jeepney passengers were seriously injured. A suit for damages based on culpa aquiliana was filed against Marcial and Ben, seeking to hold them jointly and severally liable for such injuries. May Marcial be held liable? Explain. SUGGESTED ANSWER:

Marcial may not be liable because under Art. 2184, NCC, the owner who is in the vehicle is not liable with the driver if by the exercise of due diligence he could have prevented the injury. The law does not require the owner to supervise the driver every minute that he was driving. Only when through his negligence, the owner has lost an opportunity to prevent the accident would he be liable (Caedo v. Ytt Khe Thai, 26 SCRA 410 citing Chapman v. Underwood and Manlangit v. Mauler, 250 SCRA 560). In this case, the fact that the owner

was absorbed in reading a book does not conclusively show that he lost the opportunity to prevent the accident through his negligence. ALTERNATIVE ANSWER:

Yes, Marcial should be held liable. Art. 2164. NCC makes an owner of a motor vehicle solidarily liable with the driver if, being in the vehicle at the time of the mishap, he could have prevented it by the exercise of due diligence. The traffic conditions along EDSA at any time of day or night are such as to require the observance of utmost care and total alertness in view of the large number of vehicles running at great speed. Marcial was negligent in that he rendered himself oblivious to the traffic hazards by reading a book instead of focusing his attention on the road and supervising the manner in which his car was being driven. Thus he failed to prevent his driver from attempting to beat the traffic light at the junction of Quezon Avenue and EDSA, which Marcial, without being a driver himself could have easily perceived as a reckless course of conduct. Liability; owner who was in the vehicle (1998) A Gallant driven by John and owned by Art, and a Corolla driven by its owner, Gina, collided somewhere along Adriatico Street. As a result of the accident, Gina had a concussion. Subsequently. Gina brought an action for damages against John and Art. There is no doubt that the collision is due to John's negligence. Can Art, who was in

the vehicle at the time of the accident, be held solidarily liable with his driver, John? (5%)

SUGGESTED ANSWER:

Yes. Art may be held solidary liable with John, if it was proven that the former could have prevented the misfortune with the use of due diligence. Article 2184 of the Civil Code states: "In motor mishaps, the owner is solidary liable with his driver, if the former, who was in the vehicle, could have, by the use of due diligence, prevented the misfortune, x x x" ALTERNATIVE ANSWER:

1. It depends. The Supreme Court in Chapman vs, Underwood (27 Phil 374), held: "An owner who sits in his automobile, or other vehicle, and permits his driver to continue in a violation of law by the performance of negligent acts, after he has had a reasonable opportunity to observe them and to direct that the driver cease therefrom, becomes himself responsible for such acts, x x x On the other hand, if the driver, by a sudden act of negligence, and without the owner having a reasonable opportunity to prevent the act or its continuance, injures a person or violates the criminal law, the owner of the automobile, although present therein at the time the act was committed is not responsible, either civilly or criminally, therefor. The act complained of must be continued in the presence of the owner for such a length of time that the owner, by his acquiescence, makes his driver's act his own." Liability; owner who was in the vehicle (2002) Does the presence of the owner inside the vehicle causing damage to a third party affect his liability for his driver’s negligence? Explain (2%) SUGGESTED ANSWER:

In motor vehicle mishaps, the owner is made solidarily liable with his driver if he (the owner) was in the vehicle and could have, by the use of due diligence, prevented the mishap. (Caedo v. Yu Khe Thai, 26 SCRA 410 [1968]).

Moral Damages & Atty Fees (2002) Ortillo contracts Fabricato, Inc. to supply and install tile materials in a building he is donating to his province. Ortillo pays 50% of the contract price as per agreement. It is also agreed that the balance would be payable periodically after every 10% performance until completed. After performing about 93% of the contract, for which it has been paid an additional 40% as per agreement, Fabricato, Inc. did not complete the project due to its sudden cessation of operations. Instead, Fabricato, Inc. demands payment of the last 10% of the contract despite its non-completion of the project. Ortillo refuses to pay, invoking the stipulation that payment of the last amount 10% shall be upon completion. Fabricato, Inc. brings suit for the entire 10%. Plus damages, Ortillo counters with claims for (a) moral damages for Fabricato, Inc.’s unfounded suit which has damaged his reputation as a philanthropist and respect businessman in his community, and (b) attorney’s fees. A. Does Ortillo have a legal basis for his claim for moral damages? (2%) B. How about his claim for attorney’s fees, having hired a lawyer to defend him? (3%) SUGGESTED ANSWER:

CIVIL LAW Answers to the BAR as Arranged by Topics (Year 1990-2006)

A. There is no legal basis to Ortillo’s claim for moral damages. It does not fall under the coverage of Article 2219 of the New Civil Code. B. Ortillo is entitled to attorney’s fees because Fabricato’s complaint is a case of malicious prosecution or a clearly unfounded civil action. (Art. 2208 [4] and [11], NCC).

Moral Damages; Non-Recovery Thereof (2006) Under Article 2219 of the Civil Code, moral damages may be recovered in the cases specified therein several of which are enumerated below. Choose the case wherein you cannot recover moral damages. Explain. (2.5%) a) A criminal offense resulting in physical injuries b) Quasi-delicts causing physical injuries c) Immorality or dishonesty d) Illegal search e) Malicious prosecution SUGGESTED ANSWER: Immorality and dishonesty, per se, are not among those cases enumerated in Article 2219 which can be the basis of an action for moral damages. The law specifically mentions adultery or concubinage, etc. but not any and every immoral act.

a) When a 7-year old boy injures his playmate while playing with his father's rifle. Explain. (2%) SUGGESTED ANSWER:

The parents of the 7-year old boy who caused injury to his playmate are liable under Article 219 of the Family Code, in relation to Article 2180 of the Civil Code since they exercise parental authority over the person of the boy. (Tamargo v. Court of Appeals, G.R. No. 85044, June 3, 1992; Elcano v. Hill, G.R. No. L-24803, May 26, 1977)

b) When a domestic helper, while haggling for a lower price with a fish vendor in the course of buying foodstuffs for her employer's family, slaps the fish vendor, causing her to fall and sustain injuries. Explain. (2%) SUGGESTED ANSWER:

Employer of the domestic helper who slapped a fish vendor. Under Article 2180, par. 5 of the Civil Code, "employers shall be liable for the damages caused by their employees and household helpers acting within the scope of their assigned tasks, even though the former are not engaged in any business or industry." c) A carpenter in a construction company accidentally hits the right foot of his co-worker with a hammer. Explain. (2%)

Quasi-Delict (1992) As the result of a collision between a public service passenger bus and a cargo truck owned by D, X sustained physical injuries and Y died. Both X and Y were passengers of the bus. Both drivers were at fault, and so X and Z, the only heir and legitimate child of the deceased Y, sued the owners of both vehicles. a) May the owner of the bus raise the defense of having exercised the diligence of a good father of a family? b) May D raise the same defense? c) May X claim moral damages from both defendants? d) May Z claim moral damages from both defendants? Give reasons for all your answers, SUGGESTED ANSWER:

(a) No. The owner of the bus cannot raise the defense because the carrier's liability is based on breach of contract (b) Yes. D can raise the defense because his liability is based on a quasi-delict. (c) Because X suffered physical injuries, X can claim moral damages against D, but as against the owner of the bus. X can claim moral damages only if X proves reckless negligence of the carrier amounting to fraud. (d) Z can claim moral damages against both defendants because the rules on damages arising from death due to a quasi-delict are also applicable to death of a passenger caused by breach of contract by a common carrier (Arts. 1755. 1756, 1764, 2206 and 2219. Civil Code). Quasi-Delict (2005) Under the law on quasi-delict, aside from the persons who caused injury to persons, who else are liable under the following circumstances:

SUGGESTED ANSWER:

The owner of the construction company. Article 2180, paragraph 4 states that "the owners and managers of an establishment or enterprise are likewise responsible for damages caused by their employees in the service of the branches in which the latter are employed or on the occasion of their functions." d) A 15-year old high school student stabs his classmate who is his rival for a girl while they were going out of the classroom after their last class. Explain. (2%) SUGGESTED ANSWER:

The school, teacher and administrator as they exercise special parental authority. (Art. 2180, par. 7 in relation to Art. 218 and Art. 219 of the Family Code) e) What defense, if any, is available to them? (2%) SUGGESTED ANSWER:

The defense that might be available to them is the observance of a good father of the family to prevent the damage. (Last par., Art. 2180, Civil Code) Quasi-Delict; Acts contrary to morals (1996) Rosa was leasing an apartment in the city. Because of the Rent Control Law, her landlord could not increase the rental as much as he wanted to, nor terminate her lease as long as she was paying her rent. In order to force her to leave the premises, the landlord stopped making repairs on the apartment, and caused the water and electricity services to be disconnected. The difficulty of living without electricity and running water resulted in Rosa's suffering a nervous breakdown. She sued the landlord for actual and moral damages. Will the action prosper? Explain. SUGGESTED ANSWER:

CIVIL LAW Answers to the BAR as Arranged by Topics (Year 1990-2006)

Yes, based on quasi-delict under the human relations provisions of the New Civil Code (Articles 19, 20 and 21) because the act committed by the lessor is contrary to morals. Moral damages are recoverable under Article 2219 (10) in relation to Article 21. Although the action is based on quasi-delict and not on contract, actual damages may be recovered if the lessee is able to prove the losses and expenses she suffered. ALTERNATIVE ANSWERS:

a) Yes, based on breach of contract. The lessor has the obligation to undertake repairs to make the apartment habitable and to maintain the lessee in the peaceful and adequate enjoyment of the lease for the entire duration of the contract (Article 1654. NCC). Since there was willful breach of contract by the lessor, the lessee is entitled to moral damages under Article 3220, NCC. She is also entitled to actual damages, e. g. loss of income, medical expenses, etc., which she can prove at the trial. b) Yes, based on contract and/or on tort. The lessor willfully breached his obligations under Article 1654. NCC, hence, he is liable for breach of contract. For such breach, the lessee may recover moral damages under Art. 2220 of the NCC, and actual damages that she may have suffered on account thereof. And since the conduct of the lessor was contrary to morals, he may also be held liable for quasi-delict. The lessee may recover moral damages under Article 2219 (10) in relation to Article 21, and all actual damages which she may have suffered by reason of such conduct under Articles 9, 20 and 21. c) Yes, the action should prosper for both actual and moral damages. In fact, even exemplary damages and attorney's fees can be claimed by Rosa, on the authority of Magbanua vs. IAC (137 SCRA 328), considering that, as given, the lessor's willful and illegal act of disconnecting the water and electric services resulted in Rosa's suffering a nervous breakdown. Art. 20 NCC and Art, 21, NCC authorize the award of damages for such willful and illegal conduct. Quasi-Delict; Mismanagement of Depositor’s Account (2006) Tony bought a Ford Expedition from a car dealer in Muntinlupa City. As payment, Tony issued a check drawn against his current account with Premium Bank. Since he has a good reputation, the car dealer allowed him to immediately drive home the vehicle merely on his assurance that his check is sufficiently funded. When the car dealer deposited the check, it was dishonored on the ground of "Account Closed." After an investigation, it was found that an employee of the bank misplaced Tony's account ledger. Thus, the bank erroneously assumed that his account no longer exists. Later it turned out that Tony's account has more than sufficient funds to cover the check. The dealer however, immediately filed an action for recovery of possession of the vehicle against Tony for which he was terribly humiliated and embarrassed. Does Tony have a cause of action against Premium Bank? Explain. (5%)

SUGGESTED ANSWER: Yes, Tony may file an action against Premium Bank for damages under Art. 2176. Even if there exists a contractual relationship between Tony and Premium Bank, an action

for quasi-delict may nonetheless prosper. The Supreme Court has consistently ruled that the act that breaks the contract may also be a tort. There is a fiduciary relationship between the bank and the depositor, imposing utmost diligence in managing the accounts of the depositor. The dishonor of the check adversely affected the credit standing of Tony, hence, he is entitled to damages (Singson v. BPI,

G.R. No. L-24932, June 27, 1968; American Express International, Inc. v. IAC, G.R. No. 72383, November 9, 1988; Consolidated Bank and Trust v. CA, G.R. No. L-70766 November 9,1998).

Vicarious Liability (1991) Romano was bumped by a minivan owned by the Solomon School of Practical Arts (SSPA). The minivan was driven by Peter, a student assistant whose assignment was to clean the school passageways daily one hour before and one hour after regular classes, in exchange for free tuition. Peter was able to drive the school vehicle after persuading the regular driver, Paul, to turn over the wheel to him (Peter). Romano suffered serious physical injuries. The accident happened at night when only one headlight of the vehicle was functioning and Peter only had a student driver's permit. As a consequence, Peter was convicted in the criminal case. Thereafter, Romano sued for damages against Peter and SSPA. a) Will the action for damages against Peter and SSPA prosper? b) Will your answer be the same if, Paul, the regular driver, was impleaded as party defendant for allowing Peter to drive the minivan without a regular driver's license. c) Is the exercise of due diligence in the selection and supervision of Peter and Paul a material issue to be resolved in this case? SUGGESTED ANSWER:

A. Yes. It will prosper (Art, 2180) because at the time he drove the vehicle, he was not performing his assigned tasks as provided for by Art. 2180. With respect to SSPA, it is not liable for the acts of Peter because the latter was not an employee as held by Supreme Court in Filamer Christian Institute vs. CA. (190 SCRA 485). Peter belongs to a special category of students who render service to the school in exchange for free tuition fees. B. I would maintain the same answer because the incident did not occur while the employee was in the performance of his duty as such employee. The incident occurred at night time, and in any case, there was no indication in the problem that he was performing his duties as a driver. C. In the case of Peter, if he were to be considered as employee, the exercise of due diligence in the selection and supervision of peter would not be a material issue since the conviction of Peter would result in a subsidiary liability where the defense would not be available by the employer. In the case of Paul, since the basis of subsidiary liability is the pater familias rule under Art. 2180, the defense of selection and supervision of the employee would be a valid defense. ALTERNATIVE ANSWER:

CIVIL LAW Answers to the BAR as Arranged by Topics (Year 1990-2006)

C. In the case of Peter, if he were to be considered an employee, the exercise of due diligence in the selection and supervision of Peter would not be a material issue since the conviction of Peter would result in a subsidiary liability where the defense would not be available by the employer. In the case of Paul, since he was in the performance of his work at the time the incident occurred, the school may be held subsidiarily liable not because of the conviction of Peter, but because of the negligence of Paul under Art. 2180.

Vicarious Liability (2001) After working overtime up to midnight, Alberto, an executive of an insurance company drove a company vehicle to a favorite Videoke bar where he had some drinks and sang some songs with friends to "unwind". At 2:00 a.m., he drove home, but in doing so, he bumped a tricycle, resulting in the death of its driver. May the insurance company be held liable for the negligent act of Alberto? Why? SUGGESTED ANSWER:

The insurance company is not liable because when the accident occurred, Alberto was not acting within the assigned tasks of his employment. It is true that under Art. 2180 (par. 5), employers are liable for damages caused by their employees who were acting within the scope of their assigned tasks. However, the mere fact that Alberto was using a service vehicle of the employer at the time of the injurious accident does not necessarily mean that he was operating the vehicle within the scope of his employment. In Castilex Industrial Corp. v. Vasquez Jr (321 SCRA393 [1999]). the Supreme Court held that notwithstanding the fact that the employee did some overtime work for the company, the former was, nevertheless, engaged in his own affairs or carrying out a personal purpose when he went to a restaurant at 2:00 a.m. after coming out from work. The time of the accident (also 2:00 a. m.) was outside normal working hours. ALTERNATIVE ANSWER:

The insurance company is liable if Alberto was negligent in the operation of the car and the car was assigned to him for the benefit of the insurance company, and even though he was not within the scope of his assigned tasks when the accident happened. In one case decided by the Supreme Court, where an executive of a pharmaceutical company was given the use of a company car, and after office hours, the executive made personal use of the car and met an accident, the employer was also made liable under Art. 2180 of the Civil Code for the injury caused by the negligent operation of the car by the executive, on the ground that the car which caused the injury was assigned to the executive by the employer for the prestige of the company. The insurance company was held liable even though the employee was not performing within the scope of his assigned tasks when the accident happened [Valenzuela v. CA, 253 SCRA 3O3 (1996)]. Vicarious Liability (2002) Explain the concept of vicarious liability in quasi-delicts. (1%) SUGGESTED ANSWER:

The doctrine of VICARIOUS LIABILITY is that which renders a person liable for the negligence of others for whose acts or omission the law makes him responsible on the theory that they are under his control and supervision. Vicarious Liability (2004) OJ was employed as professional driver of MM Transit bus owned by Mr. BT. In the course of his work, OJ hit a pedestrian who was seriously injured and later died in the hospital as a result of the accident. The victim’s heirs sued the driver and the owner of the bus for damages. Is there a presumption in this case that Mr. BT, the owner, had been negligent? If so, is the presumption absolute or not? Explain. (5%) SUGGESTED ANSWER:

Yes, there is a presumption of negligence on the part of the employer. However, such presumption is rebuttable. The liability of the employer shall cease when they prove that they observed the diligence of a good father of a family to prevent damage (Article 2180, Civil Code). When the employee causes damage due to his own negligence while performing his own duties, there arises the juris tantum presumption that the employer is negligent, rebuttable only by proof of observance of the diligence of a good father of a family (Metro Manila Transit v. CA, 223 SCRA 521 [1993]; Delsan Transport Lines v, C&tA Construction, 412 SCRA 524 2003).

Likewise, if the driver is charged and convicted in a criminal case for criminal negligence, BT is subsidiarily liable for the damages arising from the criminal act. Vicarious Liability (2006) Arturo sold his Pajero to Benjamin for P1 Million. Benjamin took the vehicle but did not register the sale with the Land Transportation Office. He allowed his son Carlos, a minor who did not have a driver's license, to drive the car to buy pan de sal in a bakery. On the way, Carlos driving in a reckless manner, sideswiped Dennis, then riding a bicycle. As a result, he suffered serious physical injuries. Dennis filed a criminal complaint against Carlos for reckless imprudence resulting in serious physical injuries. 1. Can Dennis file an independent civil action against Carlos and his father Benjamin for damages based on quasi-delict? Explain. (2,5%)

Yes, Dennis can file an independent civil action against Carlos and his father for damages based on quasi-delict there being an act or omission causing damage to another without contractual obligation. Under Section 1 of Rule 111 of the 2000 Rules on Criminal Procedure, what is deemed instituted with the criminal action is only the action to recover civil liability arising from the act or omission punished by law. An action based on quasi-delict is no longer deemed instituted and may be filed separately [Section 3, Rule 111, Rules of Criminal Procedure]. SUGGESTED ANSWER:

2. Assuming Dennis' action is tenable, can Benjamin raise the defense that he is not liable because the vehicle is not registered in his name? Explain. (2.5%)

CIVIL LAW Answers to the BAR as Arranged by Topics (Year 1990-2006) SUGGESTED ANSWER: No,

Benjamin cannot raise the

defense that the vehicle is not registered in his name. His liability, vicarious in character, is based on Article 2180 because he is the father of a minor who caused damage due to negligence. While the suit will prosper against the registered owner, it is the actual owner of the private vehicle who is ultimately liable (See Duavit v. CA, G.R. No. L-29759, May 18, 1989). The purpose of car registration is to reduce difficulty in identifying the party liable in case of accidents (Villanueva v. Domingo, G.R. No. 144274, September 14, 2004).

Vicarious Liability; Public Utility (2000) Silvestre leased a car from Avis-Rent-A-Car Co. at the Mactan International Airport. No sooner had he driven the car outside the airport when, due to his negligence, he bumped an FX taxi owned and driven by Victor, causing damage to the latter in the amount of P100,000.00. Victor filed an action for damages against both Silvestre and Avis, based on quasi-delict. Avis filed a motion to dismiss the complaint against it on the ground of failure to state a cause of action. Resolve the motion. (3%)

called ―oncomouse‖ in Manila? What will be your advice to him? (5%) SUGGESTED ANSWER:

(1) The reciprocity principle in private international law may be applied in our jurisdiction. Section 3 of R.A. 8293, the Intellectual Property Code, provides for reciprocity, as follows: "Any person who is a national, or who is domiciled, or has a real and effective industrial establishment in a country which is a party to any convention, treaty or agreement relating to intellectual property rights or the repression of unfair competition, to which the Philippines is also a party, or extends reciprocal rights to nationals of the Philippines by law, shall be entitled to benefits to the extent necessary to give effect to any provision of such convention, treaty or reciprocal law, in addition to the rights to which any owner of an intellectual property right is otherwise entitled by this Act. (n)" To illustrate: the Philippines may refrain from imposing a requirement of local incorporation or establishment of a local domicile for the protection of industrial property rights of foreign nationals (citizens of Canada, Switzerland, U.S.) if the countries of said foreign nationals refrain from imposing said requirement on Filipino citizens.

SUGGESTED ANSWER:

The motion to dismiss should be granted, AVIS is not the employer of Silvestre; hence, there is no right of action against AVIS under Article 2180 of the Civil Code. Not being the employer, AVIS has no duty to supervise Silvestre. Neither has AVIS the duty to observe due diligence in the selection of its customers. Besides, it was given in the problem that the cause of the accident was the negligence of Silvestre. ALTERNATIVE ANSWER:

The motion should be denied. Under the Public Service Law, the registered owner of a public utility is liable for the damages suffered by third persons through the use of such public utility. Hence, the cause of action is based in law, the Public Service Law.

INTELLECTUAL PROPERTY Intellectual Creation (2004) Dr. ALX is a scientist honored for work related to the human genome project. Among his pioneering efforts concern stem cell research for the cure of Alzheimer’s disease. Under corporate sponsorship, he helped develop a microbe that ate and digested oil spills in the sea. Now he leads a college team for cancer research in MSS State. The team has experimented on a mouse whose body cells replicate and bear cancerous tumor. Called ―oncomouse‖, it is a life-form useful for medical research and it is a novel creation. Its body cells do not naturally occur in nature but are the product of man’s intellect, industry and ingenuity. However, there is a doubt whether local property laws and ethics would allow rights of exclusive ownership on any life-form. Dr. ALX needs your advice: (1) whether the reciprocity principle in private international law could be applied in our jurisdiction; and (2) whether there are legal and ethical reasons that could frustrate his claim of exclusive ownership over the life-form

ALTERNATIVE ANSWER:

Reciprocity principle cannot be applied in our jurisdiction because the Philippines is a party to the TRIPS agreement and the WTO. The principle involved is the most-favored nation clause which is the principle of non-discrimination. The protection afforded to intellectual property protection in the Philippines also applies to other members of the WTO. Thus, it is not really reciprocity principle in private international law that applies, but the most-favored nation clause under public international law. (2) There is no legal reason why "oncomouse" cannot be protected under the law. Among those excluded from patent protection are "plant varieties or animal breeds, or essentially biological process for the production of plants and animals" (Section 22.4 Intellectual Property Code, R.A. No. 8293). The "oncomouse" in the problem is not an essentially biological process for the production of animals. It is a real invention because its body cells do not naturally occur in nature but are the product of man's ingenuity, intellect and industry. The breeding of oncomouse has novelty, inventive step and industrial application. These are the three requisites of patentability. (Sec. 29, IPC) There are no ethical reasons why Dr. ADX and his college team cannot be given exclusive ownership over their invention. The use of such genetically modified mouse, useful for cancer research, outweighs considerations for animal rights. There are no legal and ethical reasons that would frustrate Dr. ALX's claim of exclusive ownership over "oncomouse". Animals are property capable of being appropriated and owned'. In fact, one can own pet dogs or cats, or any other animal. If wild animals are capable of being owned, with more reason animals technologically enhanced or corrupted

CIVIL LAW Answers to the BAR as Arranged by Topics (Year 1990-2006)

by man's invention or industry are susceptible to exclusive ownership by the inventor. ALTERNATIVE ANSWER:

The oncomouse is a higher life form which does not fall within the definition of the term "invention". Neither may it fall within the ambit of the term "manufacture" which usually implies a non-living mechanistic product. The oncomouse is better regarded as a "discovery" which is the common patrimony of man. ALTERNATIVE ANSWER:

The "oncomouse" is a non-patentable invention. Hence, cannot be owned exclusively by its inventor. It is a method for the treatment of the human or animal body by surgery or therapy and diagnostic methods practiced on said bodies are not patentable under Sec. 22 of the IPC.

Civil Law Q&As (2007-2013)

[email protected]

[email protected]

A Compilation of the Questions and Suggested Answers In the PHILIPPINE BAR EXAMINATIONS 2007-2013 In

CIVIL LAW Compiled and Arranged By: Baratbate-Ladot, Delight Salise, Hector Christopher “Jay-Arh” Jr. M. (University of San Jose-Recoletos School of Law)

ANSWERS TO BAR EXAMINATION QUESTIONS by the UP LAW COMPLEX (2007-2013) & PHILIPPINE ASSOCIATION OF LAW SCHOOLS (2008)

“Never Let The Odds Keep You From Pursuing What You Know In Your Heart You Were Meant To Do.”-Leroy Satchel Paige

Page 1 of 180

Civil Law Q&As (2007-2013)

[email protected]

[email protected]

FOREWORD This work is a compilation of the ANSWERS TO BAR EXAMINATION QUESTIONS by the UP LAW COMPLEX , Philippine Association of Law Schools from 2007-2010 and local law students and lawyers’ forum sites from 2011-2013 and not an original creation or formulation of the author. The authors were inspired by the work of Silliman University’s College of Law and its students of producing a very good material to everyone involved in the legal field particularly the students and the reviewees for free. Hence, this work is a freeware. Everyone is free to distribute and mass produce copies of this work, however, the author accepts no liability for the content of this reviewer, or for the consequences of the usage, abuse, or any actions taken by the user on the basis of the information given. The answers (views or opinions) presented in this reviewer are solely those of the authors in the given references and do not necessarily represent those of the authors of this work. The Authors.

“Never Let The Odds Keep You From Pursuing What You Know In Your Heart You Were Meant To Do.”-Leroy Satchel Paige

Page 2 of 180

Civil Law Q&As (2007-2013)

[email protected]

[email protected]

TABLE OF CONTENTS (Titles are based on Silliman’s Compilation [Arranged by Topic])

Persons Capacity: Juridical Capacity (2008)...............................................................................12 Capacity; Juridical Capacity of Donee; Requisites for Acceptance (2012).......................12 Capacity: Legal Capacity; Lex Rei Sitae (2007)...............................................................13 Correction of Entries; Clerical Error Act (2008).............................................................14 Nationality Principle (2009)...........................................................................................14 Nationality Principle; Change of Name not Covered (2009).............................................15

Conflict of Laws Processual Presumption (2009)......................................................................................16 Jurisdiction; Courts may Assume Jurisdiction over Conflict of Laws Cases (2010).........17

Adoption Adoption; Termination; Death of Adopter (2009)............................................................17 Adoption; Illegitimate Child (2010)................................................................................18 Adoption; Illegitimate Child; Use of Mother’s Surname as Middle Name (2012)..............19 Consent; Consent of the Adopter’s Heirs (2008).............................................................19 Qualifications of Adopter (2010)....................................................................................20

“Never Let The Odds Keep You From Pursuing What You Know In Your Heart You Were Meant To Do.”-Leroy Satchel Paige

Page 3 of 180

Civil Law Q&As (2007-2013)

[email protected]

[email protected]

Family Code Marriage; Annulment; Grounds (2009)...........................................................................20 Marriage; Annulment; Grounds (2007)...........................................................................21 Marriage; Annulment; Parties (2012)..............................................................................22 Marriage; Annulment; Support Pendente Lite (2010)......................................................22 Marriage; Divorce Decrees; Filipino Spouse Becoming Alien (2009)................................23 Marriage; Divorce Decrees; Foreign Spouse Divorces Filipino Spouse (2012)..................24 Marriage; Divorce Decrees; Foreign Spouse Divorces Filipino Spouse (2010)..................25 Marriage; Legal Separation; Prescription (2012).............................................................25 Marriage; Legal Separation; Prescription (2007).............................................................26 Marriage; Psychological Incapacity (2013).....................................................................26 Marriage; Psychological Incapacity (2012).....................................................................28 Marriage; Requisites (2008)...........................................................................................28 Marriage; Subsequent Marriage (2008)...........................................................................29 Marriage; Void Marriages; By Reason of Public Policy (2008)..........................................30 Marriage; Void Marriages; By Reason of Public Policy (2007)..........................................30 Marriage; Void Marriages; Property Relations (2009)......................................................30 Marriage; Void Marriages; Status of Children (2009).......................................................31 Parental Authority; Illegitimate Minor Child (2009).......................................................32 Parental Authority; In Vitro Fertilization (2010)............................................................32 “Never Let The Odds Keep You From Pursuing What You Know In Your Heart You Were Meant To Do.”-Leroy Satchel Paige

Page 4 of 180

Civil Law Q&As (2007-2013)

[email protected]

[email protected]

Paternity & Filiation; Child Born Under a Void Marriage (2010)....................................33 Paternity & Filiation; Impugning Legitimacy (2010).......................................................34 Paternity & Filiation; In Vitro Fertilization; Surrogate Mother’s Remedy to Regain Custody (2010)...................34 Paternity & Filiation; Legitimacy; Presumption (2008)..................................................35 Paternity & Filiation; Legitimation of a Child from a Previous Valid Marriage (2008).....36 Paternity & Filiation; Legitimation of a Dead Child (2009).............................................37 Paternity & Filiation; Support: Ascendants & Descendants; Collateral Blood Relatives (2008)...................................................37 Paternity & Filiation; Use of Surname; Illegitimate Child (2009)....................................38 Paternity & Filiation; Who May Impugn Legitimacy (2009)............................................39 Property Relations; Adulterous Relationship (2009).......................................................39 Property Relations; Accession (2012).............................................................................40 Property Relations; Ante-Nuptial Debt (2007)................................................................40 Property Relations; Unions Without Marriage (2012)......................................................41 Property Relations; Unions Without Marriage (2012)......................................................42 Property Relations; Void Marriages (2010).....................................................................43 Property Relations; Void Marriages (2010).....................................................................45

“Never Let The Odds Keep You From Pursuing What You Know In Your Heart You Were Meant To Do.”-Leroy Satchel Paige

Page 5 of 180

Civil Law Q&As (2007-2013)

[email protected]

[email protected]

Succession Disposition; Mortis Causa vs. Intervivos; Corpse (2009).................................................46 Heirs; Fideicommissary Substitution (2008)..................................................................46 Heirs; Intestate Succession; Legitime; Computation (2010)...........................................47 Heirs; Representation; Iron-Curtain Rule (2012)............................................................49 Heirs; Reserva Troncal (2009)........................................................................................49 Intestate Succession (2008)...........................................................................................50 Intestate Succession (2008)...........................................................................................51 Intestate Succession; Rights of Representation: Illegitimate, Adopted Child; Iron Curtain Rule (2007)....................51 Legitimes; Compulsory Heirs (2012)..............................................................................53 Legitime; Compulsory Heirs (2008)................................................................................53 Preterition; Disinheritance (2008).................................................................................54 Succession; Proof of Death between persons called to succeed each other (2008)..........55 Succession; Rule on Survivorship (2009).......................................................................56 Wills; Holographic Wills; Insertions & Cancellations (2012)............................................57 Wills; Holographic Wills; Probate (2009).........................................................................57 Wills; Joint Wills (2008).................................................................................................59 Wills; Joint Wills; Probate (2012)...................................................................................59

“Never Let The Odds Keep You From Pursuing What You Know In Your Heart You Were Meant To Do.”-Leroy Satchel Paige

Page 6 of 180

Civil Law Q&As (2007-2013)

[email protected]

[email protected]

Wills; Prohibition to Partition of a Co-Owned Property (2010)........................................60 Wills; Notarial Wills; Blind Testator; Requisites (2008)...................................................61 Wills; Testamentary Disposition; Period to Prohibit Partition (2008)..............................61 Wills; Witnesses to a Will, Presence required; Thumbmark as Signature (2007)..............62

Donation Donations; Formalities; In Writing (2007)......................................................................63 Donations; Illegal & Impossible Conditions (2007).........................................................64 Donation; Inter Vivos (2013)..........................................................................................64

Property Accretion; Alluvium (2008)............................................................................................65 Accretion; Rights of the Riparian Owner (2009).............................................................67 Builder; Good Faith; Requisites (2013)...........................................................................68 Easement; Prescription; Acquisitive Prescription (2009)................................................70 Easement; Right of Way (2013)......................................................................................70 Easement; Right of Way (2010)......................................................................................72 Hidden Treasure (2008).................................................................................................73 Mortgage; Public or Private Instrument (2013)...............................................................74 Occupation vs. Possession (2007)..................................................................................76 Ownership; Co-Ownership (2009)...................................................................................76 Ownership; Co-Ownership (2008)...................................................................................77

“Never Let The Odds Keep You From Pursuing What You Know In Your Heart You Were Meant To Do.”-Leroy Satchel Paige

Page 7 of 180

Civil Law Q&As (2007-2013)

[email protected]

[email protected]

Property; Movable or Immovable (2007).........................................................................78

Land Titles and Deeds Acquisition of Lands; Sale of Real Property to an Alien (2009).......................................79 Non-Registrable Properties (2007)..................................................................................80 Prescription; Acquisitive Prescription (2008).................................................................81 Prescription; Judicially Foreclosed Real Property Mortgage (2012)................................82 Purchaser in Good Faith; Mortgaged Property (2008).....................................................83 Registration; Governing Law (2007)...............................................................................84 Registration; Party Who First took Possession (2013)....................................................85 Registration; Requisites; Proof (2013)............................................................................86 Remedies; Fraud; Rights of Innocent Purchaser (2009)..................................................89

Contracts Contract to Sell vs. Conditional Contract of Sale (2012)................................................90 Rescission of Contract; Fortuitous Event (2008)............................................................90 Stipulation; Arbitration Clause (2009)............................................................................91

Obligations Extinguishment; Compensation (2009)..........................................................................91 Extinguishment; Compensation (2008)..........................................................................92 Extinguishment; Novation (2008)..................................................................................93 Extinguishment; Payment of Check (2013)....................................................................94 “Never Let The Odds Keep You From Pursuing What You Know In Your Heart You Were Meant To Do.”-Leroy Satchel Paige

Page 8 of 180

Civil Law Q&As (2007-2013)

[email protected]

[email protected]

Extinguishment; Payment of Check; Legal Tender (2008)..............................................95 Liability; Solidary Liability (2008)..................................................................................96 Obligations; Without Agreement (2007).........................................................................97

Trust Trust De Son Tort (2007)...............................................................................................98

Sales Condominium Act; Partition of a Condominium (2009)..................................................99 Mortgage; Equitable Mortgage (2012).............................................................................99 Option Contract; Liquor & “Pulutan” as Consideration (2013)......................................100 Right of First Refusal; Lessee; Effect (2008).................................................................101

Lease Builder; Good Faith; Useful Improvements (2013)........................................................103 Lease; Caveat Emptor (2009).......................................................................................104

Agency Agency; Sale of a Real Property through an Agent (2010).............................................104

Partnership Liability; Liability of a Partner (2010)..........................................................................105 Oral Partnership (2009)...............................................................................................106 Share; Demand during the Existence of Partnership (2012)..........................................107

“Never Let The Odds Keep You From Pursuing What You Know In Your Heart You Were Meant To Do.”-Leroy Satchel Paige

Page 9 of 180

Civil Law Q&As (2007-2013)

[email protected]

[email protected]

Commodatum & Mutuum Mutuum; Interest; Solutio Indebiti (2012)....................................................................107

Guaranty Guaranty (2009)...........................................................................................................108

Surety Surety (2010)...............................................................................................................108

Pledge Pledge; Pactum Commissorium (2009).........................................................................109

Torts and Damages Damages (2012)...........................................................................................................109 Damages; Moral & Exemplary (2009)............................................................................110 Damages; Public Officers acting in the Performance of their Duties (2012)..................111 Death Indemnity (2009)...............................................................................................112 Doctrine of Discovered Peril (Last Clear Chance) (2007)...............................................112 Liability; Owner of a Pet; Fortuitous Event (2010).......................................................113 Liability; Special Parental Authority (2010).................................................................115 Quasi-Delict; Claims; Requisites (2013)........................................................................115 Quasi Tort (2010).........................................................................................................116

“Never Let The Odds Keep You From Pursuing What You Know In Your Heart You Were Meant To Do.”-Leroy Satchel Paige

Page 10 of 180

Civil Law Q&As (2007-2013)

[email protected]

[email protected]

MULTIPLE CHOICE QUESTIONS 2013 Civil Law Exam MCQ (October 13, 2013)...….………………………………………..........118 2012 Civil Law Exam MCQ (October 14, 2012).….……………………………………...............130 2011 Civil Law Exam MCQ (November 13, 2011).………………………………….……............149 2010 Civil Law Exam MCQ (September 12, 2010).………………………………….……...........176 2007 Civil Law Exam MCQ (September 09, 2007).………………………………….……...........179

“Never Let The Odds Keep You From Pursuing What You Know In Your Heart You Were Meant To Do.”-Leroy Satchel Paige

Page 11 of 180

Civil Law Q&As (2007-2013)

[email protected]

Persons

[email protected]

mother's womb. However, if the fetus had an intra-uterine life of less than seven months, it is not deemed born if it

Capacity: Juridical Capacity (2008)

dies within twenty-four (24) hours after No. II. At age 18, Marian found out that she

its complete delivery from the maternal

was pregnant. She insured her own life and

womb. The act of naming the unborn

named

sole

child as sole beneficiary in the insurance

beneficiary. When she was already due to

is favorable to the conceived child and

give birth, she and her boyfriend Pietro, the

therefore the fetus acquires presumptive

father

were

or provisional personality. However, said

kidnapped in a resort in Bataan where they

presumptive personality only becomes

were vacationing. The military gave chase

conclusive if the child is born alive. The

and after one week, they were found in an

child need not survive for twenty-four

abandoned hut in Cavite. Marian and Pietro

(24) hours as required under Art. 41 of

were hacked with bolos. Marian and the

the Code because "Marian was already

baby delivered were both found dead, with

due to give birth," indicating that the

the baby's umbilical cord already cut. Pietro

child was more than seven months old.

her

unborn

of

her

child

unboarn

as

her

child,

survived. (A). Can Marian's baby be the beneficiary of the insurance taken on the life of the mother? (2%)

Capacity; Juridical Capacity of Donee; Requisites for Acceptance (2012) No.I. b) Ricky donated P 1 Million to the

SUGGESTED ANSWER:

unborn child of his pregnant girlfriend,

Yes, the baby can be the beneficiary of

which she accepted. After six (6) months of

the life insurance of Marian. Art. 40 NCC

pregnancy, the fetus was born and baptized

provides

as Angela. However, Angela died 20 hours

that

personality;

but

"birth the

determines

conceived

child

after birth. Ricky sought to recover the P 1

shall be considered born for all purposes

Million.

that are favorable to it, provided that it

Explain. (5%)

be

born

later

with

the

conditions

specified in Art. 41. Article 41 states

Is

Ricky

entitled

to

recover?

SUGGESTED ANSWER:

that "for civil purposes, the fetus shall

Yes, Ricky is entitled to recover the

be considered born if it is alive at the

P1,000,000.00. The NCC considers a

time it is completely delivered from the “Never Let The Odds Keep You From Pursuing What You Know In Your Heart You Were Meant To Do.”-Leroy Satchel Paige

Page 12 of 180

Civil Law Q&As (2007-2013)

[email protected]

a

[email protected]

fetus a person for purposes favorable to

acquire

house

in

Australia

because

it provided it is born later in accordance

Australian Laws allow aliens to acquire

with the provision of the NCC. While the

property from the age of 16.

donation is favorable to the fetus, the donation did not take effect because the fetus was not born in accordance with the NCC.

SUGGESTED ANSWER: TRUE. Since Australian Law allows alien to acquire property from the age of 16,

To be considered born, the fetus that

Roberta may validly own a house in

had an intrauterine life of less than

Australia, following the principle of lex

seven (7) months should live for 24

rei sitae enshrined in Art. 16, NCC,

hours from its complete delivery from

which states "Real property as well as

the mother’s womb. Since Angela had an

personal property is subject to the law of

intrauterine life of less than seven (7)

the

months but did not live for 24 hours, she

Moreover,

was not considered born and, therefore,

capacity of

did not become a person. Not being a

contract in Australia is governed by

person, she has no juridical capacity to

Philippine

be a donee, hence, the donation to her

ownership

did not take effect. The donation not

until the contract is annulled.

being effective, the amount donated may be recovered. To retain it will be unjust enrichment.

country

where

even

over

is

assuming

Roberta Law,

it

that

legal

in entering the

she the

situated."

will

property

acquire bought

ALTERNATIVE ANSWER: FALSE. Laws relating to family rights and duties, or to the status, condition or legal capacity of persons are binding

Capacity: Legal Capacity; Lex Rei Sitae

upon the citizens of the Philippines,

(2007)

even though living abroad (Art. 15, NCC). The age of majority under Philippine law

No.VII. Write "TRUE" if the statement is

is 18 years (R.A. No. 6809); hence,

true or "FALSE" if the statement is false. If

Roberta, being only 17 years old, has no

the statement is FALSE, state the reason.

legal capacity to acquire and own land.

(2% each). (1). Roberta, a Filipino, 17 years of age, without the knowledge of his parents, can

“Never Let The Odds Keep You From Pursuing What You Know In Your Heart You Were Meant To Do.”-Leroy Satchel Paige

Page 13 of 180

Civil Law Q&As (2007-2013)

[email protected]

[email protected]

Correction of Entries; Clerical Error Act

the Rules of Court because said changes

(2008)

are substantive corrections.

No. IV. Gianna was born to Andy and

(B).

Aimee, who at the time Gianna's birth were

administrative proceedings be brought for

not married to each other. While Andy was

the

single at the time, Aimee was still in the

corrections? (2%)

Instead

of

purpose

a of

judicial

action,

making

the

can above

process of securing a judicial declaration of nullity on her marriage to her ex-husband. Gianna's birth certificate, which was signed by both Andy and Aimee, registered the status

of

Gianna

as

"legitimate",

her

surname carrying that of Andy's and that her parents were married to each other.

SUGGESTED ANSWER: No. An administrative proceeding cannot be brought for the purpose of making the above corrections. R.A. 9048, otherwise known as the Clerical Error Act, which authorizes the city or municipal civil

(A). Can a judicial action for correction of

registrar or the consul general to correct

entries in Gianna's birth certificate be

a clerical or typographical error in an

successfully maintained to:

entry and/or change the first name or nickname in the civil register without

a). Change her status from "legitimate" to

need of a judicial order. Errors that

"illegitimate" (1%);

involve the change of nationality, age, status, surname or sex of petitioner are

and

not included from the coverage of the

b). Change her surname from that of Andy's to Aimee's maiden surname? (1%) SUGGESTED ANSWER:

entries in Gianna's birth certificate can be successfully maintained to change (a) status

from

"legitimate"

to

"illegitimate," and (b) her surname from that

of

Andy's

to

174689, 22 Oct., 2007).

Nationality Principle (2009)

Yes, a judicial action for correction of

her

said Act (Silverio v. Republic, G.R. No.

Aimee's

maiden

surname in accordance with Rule 108 of

No.XII. Emmanuel and Margarita, American citizens and employees of the U.S. State Department, got married in the African state of Kenya where sterility is a ground for annulment of marriage. Thereafter, the spouses were assigned to the U.S. Embassy in Manila. On the first year of the spouses’

“Never Let The Odds Keep You From Pursuing What You Know In Your Heart You Were Meant To Do.”-Leroy Satchel Paige

Page 14 of 180

Civil Law Q&As (2007-2013)

[email protected]

[email protected]

tour of duty in the Philippines, Margarita filed an annulment case against Emmanuel

ALTERNATIVE ANSWER:

before a Philippine court on the ground of

The

her husband’s sterility at the time of the

action for the annulment of marriage

celebration of the marriage.

solemnized elsewhere but only when the

forum

has

jurisdiction

over

an

party bringing the actions is domiciled (A). Will the suit prosper? Explain your

in the forum. In this case, none of the

answer. (3%)

parties to the marriage is domiciled in the

SUGGESTED ANSWER: No, the suits will not prosper. As applied to foreign nationals with the respect to family relations and status of persons, the nationality principle set forth in Article 15 of the Civil Code will govern the

relations

Margarita.

of

Since

Emmanuel

they

are

Philippines.

They

are

here

as

officials of the US Embassy whose stay in the country is merely temporary, lasting only during their fixed tour of duty. Hence, the Philippine courts have no jurisdiction over the action.

and

American

citizens, the governing law as to the ground for annulment is not Kenyan Law

Nationality Principle; Change of Name not Covered (2009)

which Magarita invokes in support of

No.XX. (A). If Ligaya, a Filipino citizen

sterility as such ground; but should be

residing in the United States, files a petition

U.S. Law, which is the national Law of

for change of name before the District Court

both

of New York, what law shall apply? Explain.

Emmanuel

and

Margarita

as

recognized under Philippine Law. Hence,

(2%)

the Philippine court will not give due course to the case based on Kenyan Law.

SUGGESTED ANSWER:

The nationality principle as expressed in

New York law shall apply. The petition of

the application of national law of foreign

change of name file din New York does

nationals

by

Philippine

courts

is

not concern the legal capacity or status

precedents

(Pilapil

v.

of the petitioner. Moreover, it does nto

653[1989],

affect the registry of any other country

Garcia v. Recio, 366 SCRA 437 [2001],

including the country of birth of the

Llorente v. Court of Appeals 345 SCRA

petitioner.

92 [2000], and Bayot v. Court of Appeals

rendered in that petition will have effect

570 SCRA 472 [2008]).

only in New York. The New York court

established Ibay-Somera,

by

174

SCRA

Whatever

judgment

is

“Never Let The Odds Keep You From Pursuing What You Know In Your Heart You Were Meant To Do.”-Leroy Satchel Paige

Page 15 of 180

Civil Law Q&As (2007-2013)

[email protected]

[email protected]

cannot, for instance, order the Civil

petitioner and his transactions in the

Registrar in the Philippines to change its

Philippines. The Philippine court can

records. The judgment of the New York

never

court allowing a change in the name of

custodian in the US of the records of the

the petitioner will be limited to the

petitioner. Moreover, change of name

records of the petitioner in New York

has nothing to do with the legal capacity

and the use of her new name in all

or status of the alien. Since Philippine

transactions in New York. Since the

records and transactions are the only

records and processes in New York are

ones affected, the Philippine court may

the only ones affected, the New York

effect the change only in accordance

court

with the laws governing those records

will

apply

New

YorK

law

in

acquire

jurisdiction

over

the

resolving the petition.

and transactions that law cannot be but

ALTERNATIVE ANSWER:

Philippine law.

Philippine law shall apply (Art 15, NCC).

ALTERNATIVE ANSWER:

Status, conditions, family rights and

U.S. law shall apply as it is his national

duties are governed by Philippine laws as

law. This is pursuant to the application

to

of

Filipinos

even

though

sojourning

lex

patriae

or

the

nationality

abroad.

principle, by which his legal status is

ALTENATIVE ANSWER:

governed by national law, the matter of

If Ligaya, a Filipino, files a petition for

change of name being included in the

change of name with the District Court

legal status. The Supreme Court has

of New YoRk, the laws of New York will

reiterate in several cases, that the lex

govern since change of name is not one

patriae as provided in Article 15 of the

of those covered by the principles of

Civil

nationality.

nationals

Code

is

in

applicable

to

determining

foreign

their

legal

status (supra). (B). If Henry, an American citizen residing in the Philippines, files a petition for change

Conflict of Laws

of name before a Philippine court, what law shall apply? Explain. (2%)

Processual Presumption (2009)

SUGGESTED ANSWER: Philippine law will apply. The petition for change of name in the Philippines will

affect

only

the

records

of

the

No.I. TRUE or FALSE. Answer TRUE if the statement

is

true,

or

FALSE

if

the

statement is false. Explain your answer in not more than two (2) sentences.

“Never Let The Odds Keep You From Pursuing What You Know In Your Heart You Were Meant To Do.”-Leroy Satchel Paige

Page 16 of 180

Civil Law Q&As (2007-2013)

(A).

The

[email protected]

doctrine

of

[email protected]

"processual

(1) Public Order. To maintain peace and

presumption" allows the court of the forum

order, disputes that disturb the peace of

to presume that the foreign law applicable

the forum should be settled by the court

to the case is the same as the local or

of the forum even though the application

domestic law. (1%)

of the foreign law is necessary for the purpose.

SUGGESTED ANSWER: TRUE. If the foreign law necessary to the

(2) Humanitarian Principle. An aggrieved

resolve an issue is not proven as a fact,

party should not be left without remedy

the court of the forum may presume that

in a forum even though the application

the foreign law is the same as the law of

of the foreign law by the courts of the

the forum.

forum is unavoidable in order to extend relief.

Jurisdiction;

Courts

may

Adoption

Assume

Jurisdiction over Conflict of Laws Cases (2010)

Adoption; Termination; Death of Adopter (2009)

No.III. Define, Enumerate or Explain. (2% each)

No.XIII. Rafael, a wealthy bachelor, filed a petition for the adoption of Dolly, a one-

(C) Give at least two reasons why a court

year old foundling who had a severe heart

may assume jurisdiction over a conflict of

ailment.

laws case.

adoption proceedings, Rafael died of natural

During

the

pendency

of

the

causes. The Office of the Solicitor General files a motion to dismiss the petition on the

SUGGESTED ANSWER: (1) Statute theory. There is a domestic law

authorizing

the

local

court

to

assume jurisdiction. (2)

Comity

assumes

theory.

ground that the case can no longer proceed because of the petitioner’s death. (A). Should the case be dismissed? Explain.

The

jurisdiction

local

based

principle of comity or courtesy. ALTERNATIVE ANSWER:

on

court the

(2%) SUGGESTED ANSWER: It

depends

on

the

stage

of

the

proceedings when Rafael died. If he died after all the requirements under the law

“Never Let The Odds Keep You From Pursuing What You Know In Your Heart You Were Meant To Do.”-Leroy Satchel Paige

Page 17 of 180

Civil Law Q&As (2007-2013)

[email protected]

have been complied with and the case is

[email protected]

Adoption; Illegitimate Child (2010)

already submitted for resolution, the court may grant the petition and issue a

No.VIII. Spouses Rex and Lea bore two

decree of adoption despite the death of

children now aged 14 and 8. During the

the

8552).

subsistence of their marriage, Rex begot a

Otherwise, the death of the petitioner

child by another woman. He is now 10

shall have the effect terminating the

years of age.

adopter

(Section

13,

RA

proceedings.

On Lea’s discovery of Rex’s fathering a child

(B). Will your answer be the same if it was

by another woman, she filed a petition for

Dolly who died during the pendency of the

legal separation which was granted.

adoption proceedings? Explain. (2%)

Rex now wants to adopt his illegitimate child.

SUGGESTED ANSWER: No, if it was Dolly who died, the case should

be

terminates

dismissed. the

Her

death

proceedings (Art. 13,

Domestic Adoption Law).

(A) Whose consent is needed for Rex’s adoption of his illegitimate child? (2.5%) SUGGESTED ANSWER:

ALTERNATIVE ANSWER:

The

It depends. If all the requirements under

legitimate child, of the 10- year -old

the law have already been complied with

illegitimate child and of the biological

and the case is already submitted for

mother of the illegitimate child are

resolution, the death of the adoptee

needed for the adoption (Section 7 and

should not abate the proceedings. The

9, RA 8552). The consent of Lea is no

court

longer

should

issue

the

decree

of

adoption if will be for the best interest of

consent

required

of

the

14-year-old

because

there

was

already a final decree of legal separation.

the adoptee. While RA8552 provides only for the case where it is the petitioner

(B) If there was no legal separation, can Rex

who dies before the decree is issued, it is

still adopt his illegitimate child? Explain.

with more compelling reason that the

(2.5%)

decree should be allowed in case it is the adoptee who dies because adoption is primarily for his benefit.

SUGGESTED ANSWER: Yes, he can still adopt his illegitimate child but with the consent of his spouse,

“Never Let The Odds Keep You From Pursuing What You Know In Your Heart You Were Meant To Do.”-Leroy Satchel Paige

Page 18 of 180

Civil Law Q&As (2007-2013)

[email protected]

[email protected]

of his 14-year-old legitimate child, of the

Adoption of Stephanie Nathy Astorga

illegitimate child, and of the biological

Garcia, G.R. No. 148311, March 31,

mother of the illegitimate child (Section

2005, the Supreme Court ruled that the

7 and 9, RA 8552).

adopted child may use the surname of the natural mother as his middle name because there is no prohibition in the

Adoption; Mother’s

Illegitimate Surname

as

Child;

Use

Middle

of

Name

(2012)

minor

illegitimate

child

Stephanie,

alleging that Stephanie’s mother is Gemma Astorga Garcia; that Stephanie has been using

for the benefit of the adopted child who shall

preserve

his

lineage

on

his

mother’s side and reinforce his right to

No.IV.b) Honorato filed a petition to adopt his

law against it. Moreover, it will also be

her

mother’s

middle

name

and

inherit from his mother and her family. Lastly, it will make the adopted child conform with the time-honored Filipino tradition

of

carrying

the

mother’s

surname as the person’s middle name.

surname; and that he is now a widower and qualified to be her adopting parent. He prayed that Stephanie’s middle name be

Consent; Consent of the Adopter’s Heirs

changed from "Astorga" to "Garcia," which

(2008)

is her mother’s surname and that her surname "Garcia" be changed to "Catindig,"

No.V. Despite several relationships with

which is his surname. This the trial court

different

denied. Was the trial court correct in

unmarried.

denying Hororato’s request for Stephanie’s

Brenda produced a daughter, Amy, now 30

use of her mother’s surname as her middle

years old. His second, with Carla, produced

name? Explain. (5%)

two sons: Jon and Ryan. His third, with

women, His

Andrew

first

remained

relationship

with

Donna, bore him no children although SUGGESTED ANSWER: No, the trial court was not correct. There is no law prohibiting an illegitimate child adopted by his natural father to use as middle name his mother’s surname. The law is silent as to what middle name an adoptee may use. In case of In re:

Elena has a daughter Jane, from a previous relationship. His last, with Fe, produced no biological

children

but

they

informally

adopted without court proceedings, Sandy's now 13 years old, whom they consider as their own. Sandy was orphaned as a baby and was entrusted to them by the midwife who attended to Sandy's birth. All the

“Never Let The Odds Keep You From Pursuing What You Know In Your Heart You Were Meant To Do.”-Leroy Satchel Paige

Page 19 of 180

Civil Law Q&As (2007-2013)

[email protected]

children, including Amy, now live with

[email protected]

Qualifications of Adopter (2010)

andrew in his house. No.IX. Eighteen-year old Filipina Patrice (A). Is there any legal obstacle to the legal

had a daughter out of wedlock whom she

adoption of Amy by Andrew? To the legal

named

adoption of Sandy by Andrew and Elena?

American citizen John who brought her to

(2%)

live with him in the United States of

Laurie.

America.

John

At

at

26,

Patrice

once

married

signified

his

SUGGESTED ANSWER:

willingness to adopt Laurie.

Yes, there is a legal obstacle to the legal

Can John file the petition for adoption? If

adoption of Amy by Andrew. Under Sec.

yes, what are the requirements? If no, why?

9(d) of RA 8552, the New Domestic

(5%)

Adoption

Act

consent

of

of

1998, the

the

written

illegitimate

sons/daughters, ten (10) years of age or over, of the adopter, if living with said adopter and the latter's spouse, if any, is necessary

to

the

adoption.

All

the

children of Andrew are living with him. Andrew needs to get the written consent of Jon, Ryan, Vina and Wilma, who are all ten (10) years old or more. Sandy's consent

to

Amy's

adoption

is

SUGGESTED ANSWER: No, John cannot file the petition to adopt

alone.

Philippine

law

requires

husband and wife to adopt jointly except on certain situations enumerated in the law. The case of John does not fall in any of the exceptions (R.A. 8552).

Family Code

not

necessary because she was not legally

Marriage; Annulment; Grounds (2009)

adopted by Andrew. Jane's consent is likewise not necessary because she is

No.XII. Emmanuel and Margarita, American

not a child of Andrew. Sandy, an orphan

citizens and employees of the U.S. State

since birth, is eligible for adoption under

Department, got married in the African

Sec. 8(f) of RA 8552, provided that

state of Kenya where sterility is a ground

Andrew obtains the written consent of

for annulment of marriage. Thereafter, the

the other children mentioned above,

spouses were assigned to the U.S. Embassy

including Amy and Elena obtains the

in Manila. On the first year of the spouses’

written consent of Jane, if she is over

tour of duty in the Philippines, Margarita

ten years old (Sec. 9(d), RA 8552).

filed an annulment case against Emmanuel before a Philippine court on the ground of

“Never Let The Odds Keep You From Pursuing What You Know In Your Heart You Were Meant To Do.”-Leroy Satchel Paige

Page 20 of 180

Civil Law Q&As (2007-2013)

[email protected]

[email protected]

her husband’s sterility at the time of the

shall be determined by applying Kenyan

celebration of the marriage.

law and not Philippine law.

(B). Assume Emmanuel and Margarita are

However, while Kenyan law governs the

both

in

formal validity of the marriage, the legal

up

capacity of the Filipino parties to the

residence in the Philippines. Can their

marriage is governed not by Kenyan law

marriage be annulled on the ground of

but by Philippine law (Article 15, NCC).

Emmanuel’s sterility? Explain. (3%)

Sterility of a party as a ground for the

Filipinos.

Kenya,

they

After

come

their

back

wedding

and

take

annulment of the marriage is not a

SUGGESTED ANSWER: No, the marriage cannot be annulled under the Philippine law. Sterility is not a ground for annulment of marriage under Article 45 of the Family Code.

must

apply

Phillippine

law

in

determining the status of the marriage legal capacity of the Filipino parties.

No, the marriage cannot be annulled in the Philippines. Philippine

capacity. Hence, the Philippine court

on the ground of absence or defect in the

ALTERNATIVE ANSWER:

The

matter of form but a matter of legal

Since

sterility

does

not

constitute

absence or defect in the legal capacity of the parties under Philippine law, there is

court

shall

have

jurisdiction over the action to annul the marriage not only because the parties

no

ground

to

avoid

or

annul

the

marriage. Hence, the Philippine court has to deny the petition.

are residents of the Philippines but because they are Filipino citizens. The Philippine court, however, shall apply the law of the place where the marriage

Marriage; Annulment; Grounds (2007)

was celebrated in determining its formal

No. VII. Write "TRUE" if the statement is

validity (Article 26, FC; Article 17, NCC).

true or "FALSE" if the statement is false. If the statement is FALSE, state the reason.

Since the marriage was celebrated in

(2% each).

Kenya in accordance with Kenyan law, the formal validity of such marriage is

(4). The day after John and Marsha got

governed by Kenyan law and any issue as

married,

to the formal validity of that marriage

impotent. Marsha continued to live with

John

told

her

that

he

was

“Never Let The Odds Keep You From Pursuing What You Know In Your Heart You Were Meant To Do.”-Leroy Satchel Paige

Page 21 of 180

Civil Law Q&As (2007-2013)

[email protected]

[email protected]

John for 2 years. Marsha is now estopped

No, I do not agree. There are others who

from filing an annulment case against

may file a petition for declaration of

John.

nullity such as the other spouse in bigamous marriages.

SUGGESTED ANSWER: FALSE. Marsha is not estopped from filing an annulment case against John

Marriage; Annulment; Support Pendente

on the ground of his impotence, because

Lite (2010)

she learned of his impotence after the celebration of the marriage and not

No.V. G filed on July 8, 2000 a petition for

before.

to

declaration of nullity of her marriage to B.

consummate is a valid ground for the

During the pendency of the case, the couple

annulment of marriage if such incapacity

entered into a compromise agreement to

was existing at the time of the marriage,

dissolve

continues and appears to be incurable.

property. B ceded his right to their house

The marriage may be annulled on this

and lot and all his shares in two business

ground

firms to G and their two children, aged 18

Physical

within

incapacity

five

years

from

its

their

absolute

community

of

and 19.

celebration.

B also opened a bank account in the amount of P3 million in the name of the two children to answer for their educational

Marriage; Annulment; Parties (2012)

expenses until they finish their college No.IX.b) A petition for declaration of nullity

degrees.

of a void marriage can only be filed by either the husband or the wife? Do you

For her part, G undertook to shoulder the

agree? Explain your answer. (5%)

day-to-day living expenses and upkeep of the

I

agree.

Under

The

Court

approved

the

spouses’ agreement on September 8, 2000.

SUGGESTED ANSWER: Yes,

children.

the

rules

(A) Suppose the business firms suffered

promulgated by the Supreme Court, a

reverses, rendering G unable to support

direct action for declaration of nullity

herself and the children. Can G still ask for

may only be filed by any of the spouses.

support pendente lite from B? Explain. (3%)

ALTERNATIVE SUGGESTED ANSWER: “Never Let The Odds Keep You From Pursuing What You Know In Your Heart You Were Meant To Do.”-Leroy Satchel Paige

Page 22 of 180

Civil Law Q&As (2007-2013)

[email protected]

Marriage;

SUGGESTED ANSWER: If B acquiesces and does not file the

[email protected]

Divorce

Decrees;

Filipino

Spouse Becoming Alien (2009)

action to impugn the legitimacy of the

No.IV. Harry married Wilma, a very wealthy

child within the prescriptive period for

woman. Barely five (5) years into the

doing so in Article 170 of the Family

marriage, Wilma fell in love with Joseph.

Code, G's daughter by another man shall

Thus, Wilma went to a small country in

be

Europe, became a naturalized citizen of

conclusively

presumed

as

the

legitimate daughter of B by G.

that country, divorced Harry, and married

(B) Suppose in late 2004 the two children had squandered the P3 million fund for their education before they could obtain

Joseph. Joseph

A

year

thereafter,

returned

Wilma

and

and

established

permanent residence in the Philippines.

their college degrees, can they ask for more

(A). Is the divorce obtained by Wilma from

support from B? Explain. (3%)

Harry

Yes, the two children can still ask for support for schooling or training for trade

or

vocation,

even beyond the age of majority until they shall have finished or completed their education (Article 194, Paragraph 2, Family Code; Javier v. Lucero, 94 Phil.

634

{1954}].Their

having

squandered the money given to them for their education will not deprive them of their right to complete an education, or to

extinguish

the

obligation

of

the

parents to ensure the future of their children.

in

the

Philippines?

Explain your answer. (3%)

SUGGESTED ANSWER:

some professions,

recognized

SUGGESTED ANSRWER : As to Wilma, the divorced obtained by her

is

recognized

Philippines

as

because

valid

she

is

in

the

now

a

foreigner. Philippine personal laws do not

apply

to

a

foreigner.

However,

recognition of the divorce as regards Harry will depend on the applicability to his case of the second paragraph of Article 26 of the Family Code. If it is applicable, divorce is recognized as to him and, therefore, he can remarry. However, if it is not applicable, divorce is

not

recognized

as

to

him

and,

consequently, he cannot remarry. ALTERNATIVE ANSWER:

“Never Let The Odds Keep You From Pursuing What You Know In Your Heart You Were Meant To Do.”-Leroy Satchel Paige

Page 23 of 180

Civil Law Q&As (2007-2013)

[email protected]

Yes , the divorce obtained by Wilma is

[email protected]

SUGGESTED ANSWER :

recognized as valid in the Philippines. At the time she got the divorce, she was

Yes, he can validly marry Elizabeth,

already a foreign national having been

applying the doctrine laid down by the

naturalized as a citizen of that “small

Supreme Court in Republic v. Obrecido

country in Europe.” Based on precedents

(427 SCRA 114 [2005]). Under the second

established by the Supreme Court (

paragraph of Article 26 of the Family

Bayot v. CA, 570 SCRA 472 [2008]),

Code, for the Filipino spouse to have

divorce

is

capacity to remarry, the law expressly

recognized in the Philippines if validly

requires the spouse who obtained the

obtained in accordance with his or her

divorce to be a foreigner at the time of

national law .

the marriage. Applying this requirement

obtained

by

a

foreigner

to the case of Harry it would seem that (B). If Harry hires you as his lawyer, what

he is not given the capacity to remarry.

legal recourse would you advise him to

This is because Wilma was a Filipino at

take? Why? (2%)

the time of her marriage to Harry.

SUGGESTED ANSWER:

In Republic v. Obrecido, however, the Supreme Court ruled that a Filipino

I will advice Harry to:

spouse is given the capacity to remarry

(1) Dissolve and liquidate his property relations with Wilma ; and

the recognition and enforcement of the judgment

of

divorced

the divorce was a Filipino at the time of the marriage, if the latter was already a

(2) If he will remarry, file a petition for foreign

even though the spouse who obtained

(Rule

39,Rules of Court ). (C). Harry tells you that he has fallen in love with another woman, Elizabeth, and wants to marry her because, after all,

foreigner when the divorce was already obtained abroad. According to the court, to rule otherwise will violate the equal protection clause of the Constitution.

Marriage;

Divorce

Decrees;

Foreign

Spouse Divorces Filipino Spouse (2012)

Wilma is already married to Joseph. Can

(b) Cipriano and Lady Miros married each

Harry legally marry Elizabeth? Explain.

other. Lady Miros then left for the US and

(2%)

there, she obtained American citizenship.

“Never Let The Odds Keep You From Pursuing What You Know In Your Heart You Were Meant To Do.”-Leroy Satchel Paige

Page 24 of 180

Civil Law Q&As (2007-2013)

Cipriano

later

[email protected]

learned

all

about

[email protected]

this

proving only that the foreign spouse has

including the fact that Lady Miros has

obtained a divorce against her or him

divorced him in America and that she had

abroad. (1%)

remarried there. He then filed a petition for authority to remarry, invoking Par. 2, Art. 26

of

the

Family

Code.

Is

Cipriano

capacitated to re-marry by virtue of the divorce decree obtained by his Filipino spouse who was later naturalized as an American citizen? Explain. (5%)

SUGGESTED ANSWER : FALSE, In Garcia v. Recio , 366 SCRA 437 (2001) , the SC held Filipino

spouse

to

that for a

have

capacity

to

contract a subsequent marriage, it must also be proven that the foreign divorced obtained abroad by the foreigner spouse

SUGGESTED ANSWER:

give such foreigner spouse capacity to

Yes, he is capacitated to remarry. While the second paragraph of Art 26 of the Family Code is applicable only to a

remarry. ALTERNATIVE ANSWER:

Filipino who married a foreigner at the

TRUE, Art 26 (2) (FC), clearly provides

time of marriage, the Supreme Court

that the decree of divorce obtained

ruled in the case of Republic v. Orbecido,

abroad

G.R. No. 154380, 5 Oct, 2005, that the

sufficient

said

spouse to remarry.

provision

equally

applies

to

a

by

the to

foreigner

capacitate

spouse

the

is

Filipino

Filipino who married another Filipino at the time of the marriage, but who was already a foreigner when the divorce was Marriage; Legal Separation; Prescription

obtained.

(2012) No.IV.a) Marriage;

Divorce

Decrees;

Foreign

Spouse Divorces Filipino Spouse (2010) No.I. True or False. (A). Under Article 26 of the Family Code, when a foreign spouse divorces his/her Filipino spouse, the latter may re-marry by

After

they

got

married,

Nikki

discovered that Christian was having an affair with another woman.

But

Nikki

decided to give it a try and lived with him for two (2) years. After two (2) years, Nikki filed an action for legal separation on the ground of Christian’s sexual infidelity. Will the action prosper? Explain. (5%)

“Never Let The Odds Keep You From Pursuing What You Know In Your Heart You Were Meant To Do.”-Leroy Satchel Paige

Page 25 of 180

Civil Law Q&As (2007-2013)

[email protected]

[email protected]

the sexual infidelity committed in 2003,

SUGGESTED ANSWER: Although the action for legal separation has not yet prescribed, the prescriptive period being 5 years, if Obecido’s affair with another woman was ended when

the prescriptive period runs from 2003 and

so

on.

The

action

for

legal

separation for the last act of sexual infidelity in 2005 will prescribe in 2010.

Nikki decided to live with him again, Nikki’s

action

account

of

will

not

condonation.

prosper

on

However,

if

such affair is still continuing, Nikki’s

Marriage;

Psychological

Incapacity

(2013)

action would prosper because the action will surely be within five (5) years from

No.I. You are a Family Court judge and

the commission of the latest act of

before you is a Petition for the Declaration

sexual infidelity. Every act of sexual

of Nullity of Marriage (under Article 36 of

liaison is a ground for legal separation.

the Family Code)filed by Maria against Neil. Maria claims that Neil is psychologically incapacitated to comply with the essential

Marriage; Legal Separation; Prescription (2007)

obligations of marriage because Neil is a drunkard, a womanizer, a gambler, and a mama's boy- traits that she never knew or

No.VII. Write "TRUE" if the statement is true or "FALSE" if the statement is false. If the statement is FALSE, state the reason. (2% each).

saw when Neil was courting her. Although summoned, Neil did not answer Maria's petition and never appeared in court. To support her petition, Maria presented

(2). If a man commits several acts of sexual infidelity, particularly in 2002, 2003, 2004, 2005, the prescriptive period to file for legal separation runs from 2002.

three witnesses- herself, Dr. Elsie Chan, and Ambrosia. Dr. Chan testified on the psychological

report

on

Neil

that

she

prepared. Since Neil never acknowledged n9r

responded

to

her

invitation

for

interviews, her report is solely based on her

SUGGESTED ANSWER:

interviews with Maria and the spouses' FALSE. The five-year prescriptive period

minor children. Dr. Chan concluded that

for filing legal separation runs from the

Neil

occurrence

infidelity

Personality Disorder, an ailment that she

committed in 2002 runs from 2002, for

found to be already present since Neil's

of

sexual

is

suffering

from

Narcissistic

“Never Let The Odds Keep You From Pursuing What You Know In Your Heart You Were Meant To Do.”-Leroy Satchel Paige

Page 26 of 180

Civil Law Q&As (2007-2013)

[email protected]

[email protected]

early adulthood and one that is grave and

mere conclusions. Being a drunkard, a

incurable. Maria testified on the specific

womanizer, a gambler and a mama’s boy,

instances when she found Neil drunk, with

merely shows Neil’s failure to perform

another

the

his marital obligations. In a number of

family's resources in a casino. Ambrosia,

cases, the Supreme Court did not find

the

the existence of psychological incapacity

woman,

spouses'

or

current

squandering household

help,

corroborated Maria's testimony.

in cases where the respondent showed habitual

drunkenness

(Republic

v.

On the basis of the evidence presented, will

Melgar, G.R. No. 139676, 2006), blatant

you grant the petition? (8%)

display of infidelity and irresponsibility (Dedel v. CA, 2004) or being hooked to

SUGGESTED ANSWER:

gambling and drugs (Republic v. TanyagSan Jose, G.R. No. 168328, 2007).

No. The petition should be denied. The psychological incapacity under Art. 36

of

the

Family

Code

must

be

characterized by (a) gravity, (b) juridical

ALTERNATIVE ANSWER: Yes. The petition should be granted.

antecedence, and (c) incurability. It is

The personal medical or psychological

not enough to prove that the parties

examination

failed to meet their responsibilities and

requirement

duties as married persons; it is essential

psychological

that they must be shown to be incapable

totality of the evidence presented which

of doing so, due to some physiological

shall

(not physical) illness (Republic v. CA and

psychological

Molina, G.R. No. 108763, Feb 13, 1997).

Marcos, G.R. No. 136490, Oct 19, 2000).

In this case, the pieces of evidence presented are not sufficient to conclude that

indeed

Neil

psychological Personality before

the

is

suffering

incapacity

Disorder] marriage,

from

[Narcissistic

existing

already

incurable

Dr.

of

respondent for

Chan’s

Maria’s

therefore,

the

incapacity report

and

not

declaration

incapacity.

determine

is It

of

is

the

existence

of

(Marcos

v.

corroborated

Ambrosia’s

sufficiently

a

by

testimonies, prove

Neil’s

psychological incapacity to assume his marital obligations.

and

serious enough to prevent Neil from performing

his

essential

marital

obligations. Dr. Chan’s report contains

“Never Let The Odds Keep You From Pursuing What You Know In Your Heart You Were Meant To Do.”-Leroy Satchel Paige

Page 27 of 180

Civil Law Q&As (2007-2013)

Marriage;

[email protected]

Psychological

Incapacity

(2012)

[email protected]

the said report is the only evidence of respondent’s psychological incapacity.

No.II.b) The petitioner filed a petition for declaration of nullity of marriage based

Marriage; Requisites (2008)

allegedly on the psychological incapacity of the respondent, but the psychologist was

No. III. Roderick and Faye were high school

not

the

sweethearts. When Roderick was 18 and

respondent and the psychological report

Faye, 16 years old, they started to live

was

of

together as husband and wife without the

be

benefit of marriage. When Faye reached 18

able

to

based

petitioner.

personally

only

on

Should

examine

the

the

narration

annulment

years of age, her parents forcibly took her

granted? Explain. (5%)

back and arranged for her marriage to Brad. Although Faye lived with Brad after

SUGGESTED ANSWER: The annulment cannot be guaranteed solely on the basis of the psychological report. For the report to prove the psychological respondent,

it

incapacity

of

the

is

that

the

required

psychologist should personally examine the respondent and the psychological report

should

be

based

on

the

psychologist’s independent assessment of the facts as to whether or not the respondent

is

psychologically

incapacitated. Since,

the

the

marriage,

Roderick

continued

to

regularly visit Faye while Brad was away at work. During their marriage, Faye gave birth to a baby girl, Laica. When Faye was 25 years old, Brad discovered her continued liason with Roderick and in one of their heated arguments, Faye shot Brad to death. She lost no time in marrying her true love Roderick,

without

a

marriage

license,

claiming that they have been continuously cohabiting for more than 5 years. (A). Was the marriage of Roderick and Faye

psychologist

did

not

personally examine the respondent, and

valid? (2%) SUGGESTED ANSWER:

his report is based solely on the story of the petitioner who has an interest in the

No. The marriage of Roderick and Faye is

outcome of the petition, the marriage

not valid. Art. 4, FC provides that the

cannot be annulled on the ground of

absence of any of the essential or formal

respondent’s psychological incapacity if

requisites renders the marriage void ab initio.

However,

no

license

shall

be

“Never Let The Odds Keep You From Pursuing What You Know In Your Heart You Were Meant To Do.”-Leroy Satchel Paige

Page 28 of 180

Civil Law Q&As (2007-2013)

[email protected]

[email protected]

necessary for the marriage of a man and

reqiured to submit the required certificate

a woman who have lived together as

of capacity to marry from the German

husband and wife for at least 5 years and

Embassy in Manila, Adolf stated in the

without any legal impediment to marry

application for marriage license that he was

each other. In Republic v. Dayot, G.R.

a Filipino citizen. With the marriage license

No. 175581, 28 March 2008, reiterating

stating that Adolf was a Filipino, the couple

the doctrine in Niñal v. Bayadog, G.R.

got married in a ceremony officiated by the

No. 133778, 14 March 2000, this five-

Parish Priest of Calamba, Laguna in a

year

beach in Nasugbu, Batangas, as the local

period

exclusivity

is

and

characterized continuity.

by

In

the

parish

priest

refused

to

solemnize

present case, the marriage of Roderick

marriages except in his church. Is the

and Faye cannot be considered as a

marriage valid? Explain fully. (5%)

marriage

of

exceptional

character,

because there were 2 legal impediments during their cohabitation: minority on the part of Faye, during the first two years of cohabitation; and, lack of legal capacity, since Faye married Brad at the age of 18. The absence of a marriage license made the marriage of Faye and Roderick void ab initio.

SUGGESTED ANSWER: No. The marriage is not valid. Art. 41 FC allows the present spouse to contract a subsequent

marriage

during

the

subsistence of his previous marriage provided that: (a) his prior spouse in the first marriage had been absent for four consecutive years; (b) that the spouse present has a well-founded belief that the absent spouse was already dead, and

Marriage; Subsequent Marriage (2008)

(C) present spouse instituted a summary No. I. Ana Rivera had a husband, a Filipino

proceeding for the declaration of the

citizen

the

presumptive death of absent spouse.

passengers on board a commercial jet plane

Otherwise, the second marriage shall be

which crashed in the Atlantic Ocean ten

null and void. In the instant case, the

(10) years earlier and had never been heard

husband

of ever since. Believing that her husband

passengers on board a commercial jet

had died, Ana married Adolf Cruz Staedtler,

plane which crashed in the Atlantic

a divorced German national born of a

Ocean.

German

father

mother

husband was not recovered to confirm

residing

in

being

his death. Thus, following Art. 41, Ana

like

her,

who

and

a

Stuttgart.

was

among

Filipino To

avoid

of

The

Ana

body

was

of

among

the

the

deceased

“Never Let The Odds Keep You From Pursuing What You Know In Your Heart You Were Meant To Do.”-Leroy Satchel Paige

Page 29 of 180

Civil Law Q&As (2007-2013)

should

have

first

[email protected]

secured

a

judicial

[email protected]

while Jane is a child of Elena from a

declaration of his presumptive death

previous

before she married Adolf. The absence of

marriage is not one of the prohibited

the

marriages enumerated under Art. 38 of

said

judicial

declaration

incapacitated Ana from contracting her

relationship.

Thus,

their

the FC.

second marriage, making it void ab initio. Marriage; Void Marriages; By Reason of Marriage; Void Marriages; By Reason of

Public Policy (2007)

Public Policy (2008)

No. VII. Write "TRUE" if the statement is true or "FALSE" if the statement is false. If

No.V. Despite several relationships with

the statement is FALSE, state the reason.

different

(2% each).

unmarried.

women, His

Andrew

first

remained

relationship

with

Brenda produced a daughter, Amy, now 30

(5). Amor gave birth to Thelma when she

years old. His second, with Carla, produced

was 15 years old. Thereafter, Amor met

two sons: Jon and Ryan. His third, with

David and they got married when she was

Donna, bore him no children although

20 years old. David had a son, Julian, with

Elena has a daughter Jane, from a previous

his ex-girlfriend Sandra. Julian and Thelma

relationship. His last, with Fe, produced no

can get married.

biological

children

but

they

informally

adopted without court proceedings, Sandy's now 13 years old, whom they consider as their own. Sandy was orphaned as a baby and was entrusted to them by the midwife who attended to Sandy's birth. All the children, including Amy, now live with andrew in his house. (D). Can Jon and Jane legally marry? (1%)

SUGGESTED ANSWER: TRUE.

Julian

and

Thelma

can

get

married. Marriage between stepbrothers and

stepsisters

are

not

among

the

marriages prohibited under the Family Code.

Marriage;

Void

Marriages;

Property

SUGGESTED ANSWER:

Relations (2009)

Yes. Jon and Jane can marry each other;

No. III. In December 2000, Michael and

Jon is an illegitimate child of Andrew

Anna, after obtaining a valid marriage

“Never Let The Odds Keep You From Pursuing What You Know In Your Heart You Were Meant To Do.”-Leroy Satchel Paige

Page 30 of 180

Civil Law Q&As (2007-2013)

[email protected]

[email protected]

license, went to the Office of the Mayor of

impediment for them to validity marry

Urbano,

each other.

Bulacan, to get married. The

Mayor was not there, but the Mayor’s secretary asked Michael and Anna and their witnesses to fill up and sign the

Marriage;

required

Children (2009)

marriage

contract

forms.

The

Void

Marriages;

Status

of

secretary then told them to wait, and went out to look for the Mayor who was attending

No. III. In December 2000, Michael and

a wedding in a neighboring municipality.

Anna, after obtaining a valid marriage license, went to the Office of the Mayor of

When the secretary caught up with the

Urbano,

Mayor

she

Mayor was not there, but the Mayor’s

showed him the marriage contract forms

secretary asked Michael and Anna and

and told him that the couple and their

their witnesses to fill up and sign the

witnesses were waiting in his office. The

required

Mayor forthwith signed all the copies of the

secretary then told them to wait, and went

marriage

out to look for the Mayor who was attending

at

the

wedding

contract,

gave

reception,

them

to

the

secretary who returned to the Mayor’s

Bulacan, to get married. The

marriage

contract

forms.

The

a wedding in a neighboring municipality.

office. She then gave copies of the marriage contract to the parties, and told Michael

When the secretary caught up with the

and Anna that they were already married.

Mayor

Thereafter, the couple lived together as

showed him the marriage contract forms

husband and wife, and had three sons.

and told him that the couple and their

at

the

wedding

reception,

she

witnesses were waiting in his office. The (C). What property regime governs the

Mayor forthwith signed all the copies of the

properties acquired by the couple? Explain.

marriage

(2%)

secretary who returned to the Mayor’s

contract,

gave

them

to

the

office. She then gave copies of the marriage SUGGESTED ANSWER:

contract to the parties, and told Michael and Anna that they were already married.

The marriage being void, the property

Thereafter, the couple lived together as

relationship that governed their union is

husband and wife, and had three sons.

special co-ownership under Article 147 of the Family Code. This is on the

(A). Is the marriage of Michael and Anna

assumption

valid,

that

there

was

no

voidable,

or

void?

Explain

your

answer. (3%) “Never Let The Odds Keep You From Pursuing What You Know In Your Heart You Were Meant To Do.”-Leroy Satchel Paige

Page 31 of 180

Civil Law Q&As (2007-2013)

[email protected]

SUGGESTED ANSWER :

[email protected]

(C). When Rona reaches seven (7) years old, she tells Rodolfo that she prefers to live

The marriage is void because the formal

with

requisite

financially than Nanette. If Rodolfo files an

of

marriage

ceremony

was

absent ( Art.3, F.C. 209, Family Code).

him,

because

he

is

better

off

action for the custody of Rona, alleging that he is Rona’s choice as custodial parent, will

ALTERNATIVE ANSWER: The

marriage

is

void

the court grant Rodolfo’s petition? Why or because

an

why not? (2%)

essential requisite was absent: consent of

the

parties

freely

given

in

the

SUGGESTED ANSWER:

presence of the solemnizing officer (Art

No, because Rodolfo has no parental

.2, FC).

authority over Rona. He who has the parental

authority

has

the

right

to

(B). What is the status of the three children

custody. Under the Family Code, the

of Michael and Anna? Explain your answer.

mother alone has parental authority over

(2%)

the illegitimate child. This is true even if illegitimate father recognized the child

SUGGESTED ANSWER:

and even though he is giving support for

The children are illegitimate, having

the child. To acquire custody over Rona,

been born outside a valid marriage.

Rodolfo should first deprive Nanette of parental authority if there is ground under the law, and in a proper court

Parental Authority; Illegitimate Minor

proceedings. In the same action, the

Child (2009)

court may award custody of Rona to

No.XIV. Rodolfo, married to Sharon, had an

Rodolfo if it is for her best interest.

illicit affair with his secretary, Nanette, a 19-year old girl, and begot a baby girl, Rona. Nanette sued Rodolfo for damages: actual, for hospital and other medical expenses

in

delivering

No.VI. Gigolo entered into an agreement

caesarean section; moral, claiming that

with Majorette for her to carry in her womb

Rodolfo

her,

his baby via in vitro fertilization. Gigolo

representing that he was single when, in

undertook to underwrite Majorette’s pre-

fact, he was not; and exemplary, to teach a

natal expenses as well as those attendant

to

child

(2010)

by

promised

the

Parental Authority; In Vitro Fertilization

marry

lesson to like-minded Lotharios. “Never Let The Odds Keep You From Pursuing What You Know In Your Heart You Were Meant To Do.”-Leroy Satchel Paige

Page 32 of 180

Civil Law Q&As (2007-2013)

[email protected]

[email protected]

to her delivery. Gigolo would thereafter pay

in Parañaque. After four (4) years or in

Majorette P2 million and, in return, she

2001,

would give custody of the baby to him.

college degree as a fulltime student, she and

After Majorette gives birth and delivers the

G B

having

completed

contracted

her

marriage

4-year

without

a

license.

baby to Gigolo following her receipt of P2 million, she engages your services as her

The marriage of B and G was, two years

lawyer to regain custody of the baby.

later, declared null and void due to the absence of a marriage license.

(C) Who of the two can exercise parental authority over the child? Explain. (2.5%)

(B). Is Venus legitimate, illegitimate, or legitimated? Explain briefly. (3%)

SUGGESTED ANSWER: Majorette,

the

mother,

can

exercise

SUGGESTED ANSWER:

parental authority. Since the child was

Venus is illegitimate. She was conceived

born

is

and born outside a valid marriage. Thus,

illegitimate and the mother has the

she is considered illegitimate (Art 165,

exclusive parental authority and custody

Family

over the child.

legitimated by the subsequent marriage

out

of

wedlock,

the

child

ALTERNATIVE ANSWER:

Code).

While

Venus

was

of her parents, such legitimation was rendered

ineffective

when

the

said

Gigolo can exercise parental authority

marriage was later on declared null and

over the child. Majorette has no blood

void

relation to the child. She is just a

license.

“carrier” of the child.

due

to

absence

of

a

marriage

Under Article 178 of the Family Code, “legitimation subsequent

shall valid

take

place

marriage

by

a

between

Paternity & Filiation; Child Born Under a

parents. The annulment of a voidable

Void Marriage (2010)

marriage

shall

legitimation.”

not

The

affect

inclusion

the

of

the

No.X. In 1997, B and G started living

underscored

together without the benefit of marriage.

necessarily implies that the Article's

The relationship produced one offspring,

application

Venus. The couple acquired a residential lot

marriages.

portion is It

in

limited follows

the to

that

Article voidable

when

the

“Never Let The Odds Keep You From Pursuing What You Know In Your Heart You Were Meant To Do.”-Leroy Satchel Paige

Page 33 of 180

Civil Law Q&As (2007-2013)

[email protected]

[email protected]

subsequent marriage is null or void, the

action to impugn, B can pray for the

legitimation must also be null and void.

correction of the status of the said

In the present problem, the marriage

daughter in her record of birth.

between B and G was not voidable but void. Hence, Venus has remained an

(B). If B acquiesces to the use of his

illegitimate child.

surname by G’s daughter by another man, what is/are the consequence/s? Explain. (5%)

Paternity

&

Filiation;

Impugning

Legitimacy (2010)

SUGGESTED ANSWER: If B acquiesces and does not file the

two

action to impugn the legitimacy of the

serious

child within the prescriptive period for

spouses

doing so in Article 170 of the Family

continued to live under one roof. B begot a

Code, G's daughter by another man shall

son by another woman. G also begot a

be

daughter by another man.

legitimate daughter of B by G.

No.IV.

Spouses

offsprings.

B

Albeit

personality

and they

differences,

G

begot

had the

conclusively

presumed

as

the

(A). If G gives the surname of B to her daughter by another man, what can B do to protect their legitimate children's interests? Explain. (5%)

Paternity

&

Fertilization;

Filiation;

In

Surrogate

Vitro

Mother’s

Remedy to Regain Custody (2010)

SUGGESTED ANSWER:

No.VI. Gigolo entered into an agreement

B can impugn the status of G's daughter

with Majorette for her to carry in her womb

by

his baby via in vitro fertilization. Gigolo

another

daughter

on

man the

as

his

ground

legitimate that

for

undertook to underwrite Majorette’s pre-

biological reason he could not have been

natal expenses as well as those attendant

the father of the child, a fact that may

to her delivery. Gigolo would thereafter pay

be proven by the DNA test. Having been

Majorette P2 million and, in return, she

born during the marriage between B and

would give custody of the baby to him.

G, G's daughter by another man is presumed as the child of B under Article

After Majorette gives birth and delivers the

164 of the Family Code. In the same

baby to Gigolo following her receipt of P2

“Never Let The Odds Keep You From Pursuing What You Know In Your Heart You Were Meant To Do.”-Leroy Satchel Paige

Page 34 of 180

Civil Law Q&As (2007-2013)

[email protected]

[email protected]

million, she engages your services as her

No, he cannot. Both he and Majorette are

lawyer to regain custody of the baby.

guilty of violating the provision of the Anti-Child Abuse Law (RA7610) on child

(A) What legal action can you file on behalf

trafficking. Being in pari delicto, the

of Majorette? Explain. (2.5%)

partners shall be left where they are and Gigolo cannot demand the return of

SUGGESTED ANSWER:

what he paid.

As her lawyer, I can file a petition for habeas corpus on behalf Majorette to

ALTERNATIVE ANSWER:

recover custody of her child. Since she is

Yes. The agreement between Gigolo and

the mother of the child that was born

Majorette is a valid agreement.

out

of

wedlock,

she

has

exclusive

parental authority and custody over the

(D) Is the child entitled to support and

child. Gigolo, therefore, has no right to

inheritance from Gigolo? Explain. (2.5%)

have custody of the child and his refusal to give up custody will constitute illegal

SUGGESTED ANSWER:

detention for which habeas corpus is the

If Gigolo voluntarily recognized the child

proper remedy.

as his illegitimate child in accordance

ALTERNATIVE ANSWER:

with Article 175 in relation to Article 172 of the Family Code, the child is

The action to regain custody will not

entitled to support and inheritance from

prosper. In the first place Majorette

Gigolo.

cannot regain custody of the baby. As surrogate mother she merely carries the

ALTERNATIVE ANSWER:

child in her womb for its development.

Yes, because Gigolo is the natural and

The child is the child of the natural

biological parent of the baby.

parents- Gigolo and his partner. The agreement between Gigolo and Majorette is a valid agreement. Paternity

&

Filiation;

Legitimacy;

(B) Can Gigolo demand from Majorette the

Presumption (2008)

return of the P2 million if he returns the

No. III. Roderick and Faye were high school

baby? Explain. (2.5%)

sweethearts. When Roderick was 18 and Faye, 16 years old, they started to live

SUGGESTED ANSWER:

together as husband and wife without the

“Never Let The Odds Keep You From Pursuing What You Know In Your Heart You Were Meant To Do.”-Leroy Satchel Paige

Page 35 of 180

Civil Law Q&As (2007-2013)

[email protected]

[email protected]

benefit of marriage. When Faye reached 18

March 2002, the Supreme Court ruled

years of age, her parents forcibly took her

that impugning the legitimacy of the

back and arranged for her marriage to

child is a strictly personal right of

Brad. Although Faye lived with Brad after

husband, except: (a) when the husband

the

to

died before the expiration of the period

regularly visit Faye while Brad was away at

fixed for bringing the action; (b) if he

work. During their marriage, Faye gave

should

birth to a baby girl, Laica. When Faye was

complaint,

25 years old, Brad discovered her continued

therefrom, or (c) if the child was born

liason with Roderick and in one of their

after the death of the husband. Laica's

heated arguments, Faye shot Brad to death.

case does not fall under any of the

She lost no time in marrying her true love

exceptions.

marriage,

Roderick,

Roderick

without

a

continued

marriage

die

after

the

without

filing

having

of

the

desisted

license,

claiming that they have been continuosly

(D).

cohabiting for more than 5 years.

marriage of her biological parents? (1%)

(B). What is the filiation status of Laica?

SUGGESTED ANSWER:

(2%)

Can

Laica

be

legitimated

by

the

No. Laica cannot be legitimated by the marriage

SUGGESTED ANSWER:

of

her

biological

parents

because only children conceived and Laica

is

legitimate

children

born outside of wedlock of parents who

conceived or born during the marriage of

at the time of the conception of the

the

former were not disqualified by any

parents

are

because presumed

to

be

legitimate (Art. 164, FC).

impediment to marry each other may be legitimated (Art. 177, FC).

(C).Can Laica bring an action to impugn her own status on the ground that based on DNA results, Roderick is her biological father? (2%)

Paternity & Filiation; Legitimation of a Child from a Previous Valid Marriage

SUGGESTED ANSWER:

(2008)

No. Laica cannot bring an action to impugn her own status. In Liyao Jr. v. Tanhoti-Liyao,

G.R.

No.

138961,

07

No. IV. Gianna was born to Andy and Aimee, who at the time Gianna's birth were not married to each other. While Andy was single at the time, Aimee was still in the

“Never Let The Odds Keep You From Pursuing What You Know In Your Heart You Were Meant To Do.”-Leroy Satchel Paige

Page 36 of 180

Civil Law Q&As (2007-2013)

[email protected]

[email protected]

process of securing a judicial declaration of

statement is false. Explain your answer in

nullity on her marriage to her ex-husband.

not more than two (2) sentences.

Gianna's birth certificate, which was signed by both Andy and Aimee, registered the status

of

Gianna

as

"legitimate",

her

surname carrying that of Andy's and that her parents were married to each other.

(E). A dead child can be legitimated. (1%) SUGGESTED ANSWER: TRUE To be legitimated, the law does

(C). Assuming that Aimee is successful in declaring her former marriage void, and Andy and Aimee subsequently married each other, would Gianna be legitimated? (1%)

not require a child to be alive at the same time of the marriage of his / her parents ( Article 177, FC ). Furthermore, Art. 181 of the Family Code which states that “[Th]e llegitimation of children who died before the celebration of marriage

SUGGESTED ANSWER:

will benefit their descendants,” does not Gianna cannot be legitimated by the

preclude

instances

where

such

subsequent marriage of Andy and Aimee.

legitimation will benefit no one but the

Art. 177 of the FC provides that "only

child's ascendants ,or other relatives .

children conceived and born outside of wedlock of parents who, at the time of the conception of the former, were not

Paternity

disqualified by any impediment to marry

Ascendants & Descendants; Collateral

each other may be legitimated." In the

Blood Relatives (2008)

present case, a legal impediment was existing at the time of the conception of Gianna. Her mother, Aimee, was still alive in the process of securing judicial declaration of nullity on her marriage to her ex-husband.

&

Filiation;

Support:

No.V. Despite several relationships with different unmarried.

women, His

Andrew

first

remained

relationship

with

Brenda produced a daughter, Amy, now 30 years old. His second, with Carla, produced two sons: Jon and Ryan. His third, with Donna, bore him no children although

Paternity & Filiation; Legitimation of a

Elena has a daughter Jane, from a previous

Dead Child (2009)

relationship. His last, with Fe, produced no biological

children

but

they

informally

No. I. TRUE or FALSE. Answer TRUE if the

adopted without court proceedings, Sandy's

statement

now 13 years old, whom they consider as

is

true,

or

FALSE

if

the

“Never Let The Odds Keep You From Pursuing What You Know In Your Heart You Were Meant To Do.”-Leroy Satchel Paige

Page 37 of 180

Civil Law Q&As (2007-2013)

[email protected]

[email protected]

their own. Sandy was orphaned as a baby

Paternity & Filiation; Use of Surname;

and was entrusted to them by the midwife

Illegitimate Child (2009)

who attended to Sandy's birth. All the children, including Amy, now live with

No.XIV. Rodolfo, married to Sharon, had an

andrew in his house.

illicit affair with his secretary, Nanette, a 19-year old girl, and begot a baby girl,

(B). In his old age, can Andrew be legally

Rona. Nanette sued Rodolfo for damages:

entitled to claim support from Amy, Jon,

actual, for hospital and other medical

Ryan, Vina, Wilma, and Sandy assuming

expenses

that all of them have the means to support

caesarean section; moral, claiming that

him? (1%)

Rodolfo

in

delivering

promised

to

the

child

marry

by her,

representing that he was single when, in SUGGESTED ANSWER: Andrew, in his old age, cannot be legally

fact, he was not; and exemplary, to teach a lesson to like-minded Lotharios.

entitled to claim support because Art.

(B). Suppose Rodolfo later on acknowledges

195, par 2 of the FC limits the giving of

Rona and gives her regular support, can he

support to "legitimate ascendants and

compel her to use his surname? Why or

descendants."

why not? (2%)

(C). Can Amy, Jon, Ryan, Vina, Wilma, and

SUGGESTED ANSWER:

Sandy legally claim support from each

No. he has no right to compel Rona to

other? (2%)

use his surname. The law does not give

SUGGESTED ANSWER: Amy, Jon, Ryan, Vina, Wilma and Sandy cannot legally claim support from each other because Art. 195, par 5 limits the giving of support to "legitimate brothers and sisters, whether full or half blood."

him the right simply because he gave her support (RA 9255). Under the Family Code, an illegitimate child was required to use only the surname of the mother. Under RA 9255, otherwise known as the Revilla law, however, the illegitimate child is given the option to use the surname of the illegitimate father when the latter has recognized the former in accordance with law. Since the choice belongs to the illegitimate

child,

Rodolfo

cannot

“Never Let The Odds Keep You From Pursuing What You Know In Your Heart You Were Meant To Do.”-Leroy Satchel Paige

Page 38 of 180

Civil Law Q&As (2007-2013)

[email protected]

[email protected]

compel Rona, if already of age, to use

They are not related at all to Edilberto.

the surname against her will. If Rona is

They were born during the marriage of

still a minor, to use the surname of

Conrado

Rodolfo

of

considered legitimate children of the

Rona's mother who has sole parental

said spouses. This status is conferred on

authority over her.

them at birth by law.

will

require

the

consent

and

Clarita,

hence,

are

Under Philippine law, a person cannot Paternity & Filiation; Who May Impugn

have more than one natural filiation.

Legitimacy (2009)

The legitimate filiation of a person can be changed only if the legitimate father

No.V.

Four

children,

namely:

Alberto,

will successfully impugn such status.

Baldomero, Caridad, and Dioscoro, were born to the spouses Conrado and Clarita de

In the problem, therefore, the filiation of

la Costa. The children’s birth certificates

Alberto and Baldomero as legitimate

were duly signed by Conrado, showing

children of Condrado cannot be changed

them to be the couple’s legitimate children.

by their recognition by Edilberto as his

Later, one Edilberto de la Cruz executed a notarial document acknowledging Alberto and Baldomero as his illegitimate children >with

Clarita.

Edilberto

died

leaving

substantial properties. In the settlement of his

estate,

Alberto

intervened

claiming

deceased’s

illegitimate

and shares

Baldomero as

the

children.

The

legitimate family of Edilberto opposed the claim.

illegitimate children. Before they can be conferred illegitimate first

the

status

children,

impugn

their

of

Edilberto’s

Condrado

must

legitimacy.

Since

Condrado has not initiated any action to impugn their legitimacy, they continue to be the legitimate of Condrado. They cannot be the illegitimate children of Edilberto at the same time. Not being the illegitimate children of Edilberto, they have no right to inherit from him.

Are Alberto and Baldomero entitled to share in the estate of Edilberto? Explain. (4%)

Property

Alberto

and

Baldomero

Adulterous

Relationship (2009)

SUGGESTED ANSWER: No,

Relations;

are

not

entitled to share in Edilberto’s estate.

No. XI. TRUE or FALSE. Answer TRUE if the statement is true, or FALSE if the

“Never Let The Odds Keep You From Pursuing What You Know In Your Heart You Were Meant To Do.”-Leroy Satchel Paige

Page 39 of 180

Civil Law Q&As (2007-2013)

[email protected]

statement is false. Explain your answer in not more than two (2) sentences.

[email protected]

SUGGESTED ANSWER: It depends. If the value of the building is

(B). If there is no marriage settlement, the

more than the value of the land, the

salary of a "spouse" in an adulterous

building

marriage

becomes conjugal property under Art.

belongs

to

the

conjugal

is

conjugal

and

the

land

120 of the Family Code. This is a case of

partnership of gains. (1%)

reverse accession, where the building is SUGGESTED ANSWER:

considered as the principal and the land,

False. In adulterous relationship, the

the accessory. If, on the other hand, the

salary of a married partner belongs to

value of the land is more than the value

the absolute community, or conjugal

of the building, then the ordinary rule of

partnership, of such married partner

accession applies where the land is the

with his or her lawful spouse. Under

principal

and

Articles 148 of the Family Code, the

accessory.

In

property

remains paraphernal property and the

partner

relations and

governed

between

his/her

by

married

paramour

ordinary

is

co-ownership

where the partners become co-owners only

when

they

acquisition paramour

of is

contributed the

deemed

to

property. to

have

the The not

contributed in the earning of the salary of the married partner.

the such

building, case,

the

the land

building becomes paraphernal propery. Note: The rule on reverse accession is applicable only to the regime of conjugal partnership of gains in both the Family Code and the New Civil Code. The foregoing answer assumes that CPG is the regime of the property relations of the spouses.

Property Relations; Ante-Nuptial Debt Property Relations; Accession (2012) No.III.(a) Maria, wife of Pedro, withdrew P 5 Million from their conjugal funds. With this money, she constructed a building on a lot which she inherited from her father. Is the

(2007) No. VII. Write "TRUE" if the statement is true or "FALSE" if the statement is false. If the statement is FALSE, state the reason. (2% each).

building conjugal or paraphernal? Reasons. (5%)

(3). An individual, While single, purchases a house and lot in 1990 and borrows money

“Never Let The Odds Keep You From Pursuing What You Know In Your Heart You Were Meant To Do.”-Leroy Satchel Paige

Page 40 of 180

Civil Law Q&As (2007-2013)

in

1992

to

repair

[email protected]

it.

In

1995,

such

individual gets married while the debt is

Property

[email protected]

Relations;

Unions

Without

Marriage (2012)

still being paid. After the marriage, the debt is still the responsibility of such individual.

No.V. a) Spouses Primo and Monina Lim, childless, were entrusted with the custody

SUGGESTED ANSWER:

of two (2) minor children, the parents of whom were unknown. Eager of having

FALSE.

The

absolute

Community

of

children of their own, the spouses made it

property is liable for the ante-nuptial

appear

debts of either spouse in so far as the

parents by naming them Michelle P. Lim

same redounded to the benefit of the

and

family (Art. 94 par.7, FC).

Monina married Angel Olario after Primo’s

The

responsibility

debt

is

of

the

already

the

community

property, because the property already constitutes

absolute

Michael

they

were

Jude

the

Lim.

children’s

Subsequently,

death.

ALTERNATIVE ANSWER: FALSE.

that

community

property under Art. 91 of FC which took effect in 1988 while the house and lot here involved was purchased in 1990.

She decided to adopt the children by availing the amnesty given under R.A. 8552 to those individuals who simulated the birth of a child. She filed separate petitions for the adoption of Michelle, then 25 years old and Michael, 18. Both Michelle and Michael gave consent to the adoption.

There is no indication that the spouse

The trial court dismissed the petition and

who bought the property had legitimate

ruled that Monina should have filed the

descendants by a former marriage, which

petition jointly with her new husband.

would exclude the house and lot from

Monina, in a Motion for Reconsideration

the community property, Art. 92 par 3,

argues that mere consent of her husband

FC). If the spouses established a conjugal

would suffice and that joint adoption is not

partnership, the property belongs to the

needed,

individual spouse if full ownership was

emancipated.

for

the

adoptees

are

already

vested before marriage (Art. 118, FC). Is the trial court correct in dismissing the petitions for adoption? Explain. (5%) SUGGESTED ANSWER:

“Never Let The Odds Keep You From Pursuing What You Know In Your Heart You Were Meant To Do.”-Leroy Satchel Paige

Page 41 of 180

Civil Law Q&As (2007-2013)

[email protected]

[email protected]

Yes, the trial court was correct. At the

Borromeo discovered that titles to the three

time the petitions for adoptions were

(3) lots have been transfereed in the name

filed, petitioner had already remarried.

of Descallar. Who is the rightful owner of

Under the law, husband and wife shall

the properties? Explain. (5%)

adopt

jointly,

except

in

the

cases

enumerated in the law. The adoption cases of Michelle and James do not fall in any of the exceptions provided in the law where a spouse is permitted to adopt alone.

Hence,

Monina

should

adopt

jointly with her husband Angel (Adoption of Michelle P. Lim, G.R. Nos. 168992-93, May 21, 2009).

SUGGESTED ANSWER: It depends. On the assumption that the Family Code is the applicable law, the ownership of the properties depends on whether or not, Jambrich and Descallar are capacitated to marry each other during their cohabitation, and whether or not both have contributed funds for the acquisition of the properties.

Property

Relations;

Unions

Without

Marriage (2012)

If both of them are capacitated to marry each other, Art 147- co-ownership will apply to their property relations and the

No.V. b) Jambrich, an Austrian, fell in-love

properties in question are owned by

and lived together with Descallar

and

them in equal shares even though all the

bought their houses and lots at Agro-Macro

funds used in acquiring the properties

Subdivision.

came only from the salaries or wages, or

In

the

Contracts

to

Sell,

Jambrich and Descallar were referred to as

the

the buyers. When the Deed of Absolute Sale

business or profession. In such case,

was presented for registration before the

while Jambrich is disqualified to own

Register of Deeds, it was refused because

any

Jambrich was an alien and could not

subsequent transfer of all his interest

acquire

public

therein to Borromeo, a Filipino, was

Descallar

valid as it removed the disqualification.

separated, Jambrich purchased an engine

In such case, the properties are owned

and some accessories for his boat from

by Borromeo and Descallar in equal

Borromeo. To pay for his debt, he sold his

shares.

domain.

alienable After

lands

Jambrich

of

the

and

rights and interests in the Agro-Macro properties to Borromeo.

income

part

of

of

Jambrich

the

from

properties,

his

his

If, on the other hand, Jambrich and Descallar were not capacitated to marry

“Never Let The Odds Keep You From Pursuing What You Know In Your Heart You Were Meant To Do.”-Leroy Satchel Paige

Page 42 of 180

Civil Law Q&As (2007-2013)

each

other

Art.

[email protected]

148-co-ownership

governs their property relations. Under

marriage,

[email protected]

the

couple

possessed

the

following properties:

this regime, Jambrich and Descallar are co-owners of the properties but only if

a house and lot acquired by B on

both

August 3, 1988, one third (1/3) of

of

them

contributed

in

their

acquisition. If all the funds used in

the

acquiring

downpayment) of which he paid; one

the

properties

in

question

purchase

price

came from Jambrich, the entire property

third (1/3)

is his even though he is disqualified from

February 14, 1990 out of a cash gift

owning it. His subsequent transfer to

given to her by her parents on her

Borromeo,

it

graduation on April 6, 1989; and the

removed the disqualification. In such

balance was paid out of the spouses’

case, all the properties are owned by

joint income; and

Borromeo. Descallar

however,

If,

on

is

the

valid

as

was

(representing

paid

by

G

on

other

hand

an apartment unit donated to B by

to

their

an uncle on June 19, 1987.

contributed

acquisition, the properties are co-owned by Descallar and Borromeo in proportion to the respective contributions of the Descallar and Jambrich.

(A) Who owns the foregoing properties? Explain. (5%) SUGGESTED ANSWER:

Note: The facts of the problem are not exactly the same as in the case of Borromeo v. Descallar, G.R. NO. 159310, Feb 24, 2009, hence, the difference in the resulting answer.

Since the marriage was declared void ab initio in 2001, no Absolute Community or

Conjugal

Partnership

established between property

relation

is

was

ever

B and G. Their governed

by

a

“special co-ownership” under Article 147 Property

Relations;

Void

Marriages

(2010) No.VII. G and B were married on July 3, 1989. On March 4, 2001, the marriage, which bore no offspring, was declared void ab initio under Article 36 of the Family Code. At the time of the dissolution of the

of the Family Code because they were capacitated to marry each other. Under

that

Article

147,

wages

and

salaries of the “former spouses” earned during their cohabitation shall be owned by them in equal shares while properties acquired thru their work for industry shall be owned by them in proportion to

“Never Let The Odds Keep You From Pursuing What You Know In Your Heart You Were Meant To Do.”-Leroy Satchel Paige

Page 43 of 180

Civil Law Q&As (2007-2013)

[email protected]

[email protected]

their respective contributions. Care and

G. She is an undivided co-owner to the

maintenance of the family is recognized

extent

as

the

acquisition when she paid 1/3 of the

absence of proof as to the value of their

purchase price using the gift from her

respective

parents. Although the gift was acquired

a

valuable

contribution.

contributions,

In

they

shall

for

her

contribution

in

its

by G during her cohabitation with B, it is

share equally. If ownership of the house and lot was acquired by B on August 3, 1988 at the time he bought it on installment before he got married, he shall remain owner of the house and lot but he must reimburse G for all the amounts she advanced to pay the purchase price and for one-half share in the last payment from their joint income. In such case, the house and lot were not acquired during their cohabitation, hence, are not co-owned by B and G.

her exclusive property. It did not consist of wage or salary or fruit of her work or industry. (3) 1/3 of the house is co-owned by B and G because the payment came from their co-owned funds, i.e., their joint income during their cohabitation which is shared by them equally in the absence of any proof to the contrary. After

summing

up

their

prospective

shares, B and G are undivided co-owners

But if the ownership of the house and lot was acquired during the cohabitation, the house and lot will be owned as follows:

of the house and lot in equal shares. As

to

the

apartment,

it

is

owned

exclusive by B because he acquired it before their cohabitation. Even if he

(1) 1/3 of the house and lot is owned by

acquired it during their cohabitation, it

B. He is an undivided co-owner to that

will

extent

its

because it did not come from his wage or

acquisition in the form of the down

salary, or from his work or industry. It

payment he made before the celebration

was

of the marriage. The money he used to

uncle.

for

his

contributions

in

still

be

acquired

his

exclusive

gratuitously

property

from

his

pay the down payment was not earned during the cohabitation, hence, it is his exclusive property. (2) 1/3 of the house and lot is owned by

(B) If G and B had married on July 3, 1987 and their marriage was dissolved in 2007, who owns the properties? Explain. (5%) SUGGESTED ANSWER:

“Never Let The Odds Keep You From Pursuing What You Know In Your Heart You Were Meant To Do.”-Leroy Satchel Paige

Page 44 of 180

Civil Law Q&As (2007-2013)

[email protected]

[email protected]

The answer is the same as in letter A.

in Parañaque. After four (4) years or in

Since the parties to the marriage which

2001,

was later declared void ab initio were

college degree as a fulltime student, she

capacitated to marry each other, the

and

applicable law under the New Civil Code

license.

was

Article

144.This

Article

G B

having contracted

completed

her

marriage

4-year

without

a

is

substantially the same as Article 147 of

The marriage of B and G was, two years

the Family Code.

later, declared null and void due to the absence of a marriage license.

Hence, the determination of ownership will remain the same as in question A.

(A). If you were the judge who declared the

And

nullity of the marriage, to whom would you

even

assuming

that

the

two

provisions are not the same, Article 147

award the lot? Explain briefly. (3%)

of the Family Code is still the law that will govern the property relations of B and G because under Article 256, the Family

Code

has

retroactive

effect

insofar as it does not prejudice or impair vested or acquired rights under the new Civil Code or other laws. Applying Article 147 retroactively to the case of G and B will not impair any vested right. Until the declaration of nullity of the marriage under the Family Code, B and G have not as yet acquired any vested right over the properties

acquired

during

their

cohabitation.

SUGGESTED ANSWER: Since the marriage was null and void, no Absolute

Community

or

Conjugal

Partnership was established between B and G. Their properties are governed by the “special co-ownership” provision of Article 147 of the Family Code because both B and G were capacitated to marry each other. The said Article provides that when a man and a woman who are capacitated to marry each other,

live

exclusively with each other as husband and wife without the benefit of marriage, or under a void marriage: (1) their wages

Property

Relations;

Void

Marriages

(2010) No.X. In 1997, B and G started living together without the benefit of marriage. The relationship produced one offspring, Venus. The couple acquired a residential lot

and salaries shall be owned by them in equal shares; and (2) property acquired by both of them through their work or industry shall be governed by the rules on co-ownership. In co-ownership, the parties are co-owners if they contributed something of value in the acquisition of

“Never Let The Odds Keep You From Pursuing What You Know In Your Heart You Were Meant To Do.”-Leroy Satchel Paige

Page 45 of 180

Civil Law Q&As (2007-2013)

the

property.

proportion

[email protected]

Their

to

contributions.

share

their

In

an

is

family

is

not

Succession

in

respective

ordinary

co-

ownership the care and maintenance of the

[email protected]

recognized

as

Disposition; Mortis Causa vs. Intervivos; Corpse (2009)

a

valuable contribution for the acquisition

No. XI. TRUE or FALSE. Answer TRUE if

of a property. In the Article 147 “special

the statement is true, or FALSE if the

co-ownership”

statement is false. Explain your answer in

however,

care

and

maintenance is recognized as a valuable contribution contributor

which to

half

will of

entitle the

the

property

(E). A person can dispose of his corpse through an act intervivos. (1%)

acquired. Having

not more than two (2) sentences.

been

cohabitation,

acquired the

during

residential

their lot

is

SUGGESTED ANSWER: False.

A persons cannot dispose of his

presumed acquired through their joint

corpse through an act inter vivos, i.e.,

work and industr

under Article 147,

an act to take effect during his lifetime.

hence, B and G are co-owners of the said

Before his death there is no corpse to

property in equal shares.

dispose. But he is allowed to do so

Article 147 also provides that when a party to the void marriage was in bad

through an act mortis causa, i.e., an act to take effect upon his death.

faith, he forfeits his share in the coownership

in

favor

of

the

common

children or descendants, the default of

Heirs;

children or descendants, the forfeited

(2008)

Fideicommissary

Substitution

share shall belong to the innocent party. In the foregoing problem, there is no

No. XIII. Raymond, single, named his sister

showing that one party was in bad faith.

Ruffa in his will as a devisee of a parcel of

Hence, both shall be presumed in good

land which he owned. The will imposed

faith and no forfeiture shall take place.

upon Ruffa the obligation of preseving the land and transferring it, upon her death, to her illegitimate daughter Scarlet who was then only one year old. Raymond later died, leaving behind his widowed mother, Ruffa and Scarlet.

“Never Let The Odds Keep You From Pursuing What You Know In Your Heart You Were Meant To Do.”-Leroy Satchel Paige

Page 46 of 180

Civil Law Q&As (2007-2013)

[email protected]

[email protected]

(A). Is the condition imposed upon Ruffa, to

Ruffa (Art. 992, Civil Code). Moreover,

preserve the property and to transmit it

Scarlet is not a compulsory heir of

upon her death to Scarlet, valid? (1%)

Raymond, hence she can inherit only by testamentary

SUGGESTED ANSWER:

succession.

Since

Raymond executed a will in the case at

Yes, the condition imposed upon Ruffa

bar, Scarlet may inherit from Raymond.

to preserve the property and to transmit it upon her death to Scarlet is valid because

it

is

tantamount

to

fideicommissary substitution under Art.

Heirs; Intestate Succession; Legitime; Computation (2010)

863 of the Civil Code. No.XI. The spouses Peter and Paula had (B).

If

Scarlet

predeceases

Ruffa,

who

inherits the property? (2%)

three (3) children. Paula later obtained a judgment of nullity of marriage. Their absolute community of property having

SUGGESTED ANSWER: Ruffa

will

inherit

been dissolved, they delivered P1 million to the

property

as

Scarlet's heir. Scarlet acquires a right to the

succession

3

children

as

their

presumptive legitimes.

she

children by his second wife Marie. Peter

should predecease Ruffa (Art. 866, Civil

and Marie, having successfully engaged in

Code).

business, acquired real properties. Peter

even

time

their

Peter later re-married and had two (2)

death,

the

of

of

Raymond's

from

each

though

(C). If Ruffa predeceases Raymond, can

later died intestate.

Scarlet inherit the property directly from

(A). Who are Peter’s legal heirs and how will

Raymond? (2%)

his estate be divided among them? (5%)

SUGGESTED ANSWER:

SUGGESTED ANSWER:

If

Ruffa

Raymond's

predeceases widowed

mother

Raymond, will

be

entitled to the inheritance. Scarlet, an illegitimate child, cannot inherit the property by intestate succession from Raymond who is a legitimate relative of

The legal heirs of Peter are his children by the first and second marriages and his surviving second wife. Their shares in the estate of Peter will depend, however, on the cause of the

“Never Let The Odds Keep You From Pursuing What You Know In Your Heart You Were Meant To Do.”-Leroy Satchel Paige

Page 47 of 180

Civil Law Q&As (2007-2013)

[email protected]

nullity of the first marriage. If the

(B)

nullity

psychological capacity:

of

the

first

marriage

was

If

the

[email protected]

psychological incapacity of one or both

2

spouses, the three children of that void

children

ground

in equal shares. If the judgment of nullity was for other causes, the three

each

Surviving

shall

children

such

that

an

not

of

second

¼ of the estate

second spouse 3

distributed

is

marriage

children are illegitimate and the estate be

nullity

legitimate ¼ of the estate for

marriage are legitimate and all of the legal heirs shall share the estate of Peter

of

illegitimate 1/12

of

estate

for

each of first marriage

illegitimate child of the first marriage shall receive half of the share of a legitimate child of the second marriage, and the second wife will inherit a share equal to that of a legitimate child. In no case may the two legitimate children of the second marriage receive a share less than one-half of the estate which is their legitime.

When

the

estate

is

not

sufficient to pay all the legitimes of the compulsory heirs, the legitime of the

Note: The legitime of an illegitimate child is supposed to be ½ the legitime of a legitimate child or 1/8 of the estate. But the estate will not be sufficient to pay

the

said

legitime

of

the

3

illegitimate children, because only ¼ of the

estate

is

left

after

paying

the

legitime of the surviving spouse which is preferred.

spouse is preferred and the illegitimate

Hence, the remaining ¼ of the estate

children suffer the reduction.

shall be divided among the 3 illegitimate children.

Computation: (A)

If

the

ground

of

nullity

is

psychological incapacity: 3

children

marriage

by

first 1/6 of the estate for each

2 children by second 1/6 of the estate marriage

for each

Surviving

second 1/6 of the estate

spouse

(B). What is the effect of the receipt by Peter’s 3 children by his first marriage of their presumptive legitimes on their right to inherit following Peter’s death? (5%) SUGGESTED ANSWER: In the distribution of Peter’s estate, ½ of the

presumptive

received

by

the

3

children of the first marriage shall be collated to Peter’s estate and shall be

“Never Let The Odds Keep You From Pursuing What You Know In Your Heart You Were Meant To Do.”-Leroy Satchel Paige

Page 48 of 180

Civil Law Q&As (2007-2013)

imputed

Art 992 of the NCC, an illegitimate child

respective inheritance from Peter. Only

has no right to inherit ab intestato from

half

the legitimate children and relatives of

the

an

advance

presumptive

of

[email protected]

their

of

as

[email protected]

legitime

is

collated to the estate of Peter because

his

father

or

the other half shall be collated to the

disqualified

estate of his first wife.

because Arnel is an illegitimate child of

to

mother. inherit

Arnel from

is

Ricky

Franco and Ricky is a legitimate relative of Franco. Heirs; Representation; Iron-Curtain Rule (2012) Heirs; Reserva Troncal (2009) No.VIII.a) Ricky and Arlene are married. They begot Franco during their marriage.

No. I. TRUE or FALSE. Answer TRUE if the

Franco had an illicit relationship with

statement

Audrey and out of which, they begot Arnel.

statement is false. Explain your answer in

Frnaco

not more than two (2) sentences.

predeceased

Ricky,

Arlene

and

is

true,

or

FALSE

if

the

Arnel. Before Ricky died, he executed a will which when submitted to probate was

(B).In reservatroncal, all reservatarios (reser

opposed by Arnel on the ground that he

vees) inherit as a class and in equal shares

should be given the share of his father,

regardless of their proximity in degree to

Franco. Is the opposition of Arnel correct?

the prepositus. (1%)

Why? (5%)

SUGGESTED ANSWER: FALSE. Not all the relatives within the

SUGGESTED ANSWER:

third degree will inherit as reservatario , No, his opposition is not correct. Arnel

and not all those who are entitled to

cannot

inherit will inherit in the equal shares .

inherit

from

Ricky

in

the

representation of his father Franco. In

The

representation, the representative must

succession will determine who among

not only be a legal heir of the person he

the relatives will inherit as reservatarios

is representing, he must also be a legal

and what shares they will tak, i.e., the

heir of the decedent he seeks to inherit

direct line excludes the collateral, the

from.

descending

While Arnel is a legal heir of Franco, he is not a legal heir of Ricky because under

applicable

direct

laws

line

of

intestate

excludes

the

ascending ,the nearer excludes the more remote, the nephews and nieces exclude

“Never Let The Odds Keep You From Pursuing What You Know In Your Heart You Were Meant To Do.”-Leroy Satchel Paige

Page 49 of 180

Civil Law Q&As (2007-2013)

[email protected]

[email protected]

the uncles and the aunts, and half blood

(1). The wife of Ramon will, therefore,

relatives

receive one half (½) of the estate or the

inherit

half

the

share

of

full-blooded relatives.

amount of P5,000,000.00. (2). The three (3) full-blood brothers, will, therefore, receive P1,000,000.00 each. (3).

Intestate Succession (2008)

leaving a net estate of P10,000,000.00. Determine how much each heir will receive from the estate:

full-blood brothers, two half-brothers, and one nephew (the son of a deceased fullblood brother)? Explain. (3%)

by

receive

right

of

representation. (4). The two (2) half-brothers will receive P500,000.00 each.

Ramon shall be inherited by his wife and his full and half blood siblings or their respective representatives. In intestacy, if the wife concurs with no one but the siblings of the husband, all of them are of

the

deceased

husband. The wife will receive half of the intestate estate, while the siblings or respective

of a

deceased full-blood brother)? Explain. (3%) SUGGESTED ANSWER: estate or P5,000,000.00. The other half

Having died intestate, the estate of

heirs

sister, and three nephews (sons

The wife will receive one half (1/2) of the

SUGGESTED ANSWER:

their

will

(B). If Ramon is survived by his wife, a half-

(A). If Ramon is survived by his wife, three

intestate

nephew

P1,000,000.00

No. VII. Ramon Mayaman died intestate,

the

The

representatives,

will

inherit the other half to be divided among them equally. If some siblings are of the full-blood and the other of the half blood, a half blood sibling will receive half the share of a full-blood sibling.

shall be inherited by (1) the full-blood brother,

represented

by

his

three

children, and (2) the half-sister. They will divide the other half between them such that the share of the half-sister is just half the share of the full-blood brother.

The share

of

the full-blood

brother shall in turn be inherited by the three nephews in equal shares by right of presentation. Therefore, the three (3) nephews will receive P1,111,111.10 each the halfsister

will

receive

the

sum

of

P1,666,666.60.

“Never Let The Odds Keep You From Pursuing What You Know In Your Heart You Were Meant To Do.”-Leroy Satchel Paige

Page 50 of 180

Civil Law Q&As (2007-2013)

[email protected]

Intestate Succession (2008)

[email protected]

(D). How should the house and lot, and the cash be distributed? (1%)

No.X.

Arthur

executed

a

will

which

contained only: (i) a provision disinheriting

SUGGESTED ANSWER:

his daughter Bernica for running off with a married man, and (ii) a provision disposing

Since the probate of the will cannot be

of his share in the family house and lot in

allowed,

favor of his other children Connie and Dora.

succession apply. Under Art. 996 of the

He did not make any provisions in favor of

Civil Code, if a widow or widower and

his wife Erica, because as the will stated,

legitimate children or descendants are

she would anyway get ½ of the house and

left, the surviving spouse has the same

lot as her conjugal share. The will was very

share as of the children. Thus, ownership

brief and straightforward and both the

over the house and lot will be created

above provisions were contained in page 1,

among

which Arthur and his instrumental witness,

Bernice, Connie and Dora. Similarly, the

signed at the bottom. Page 2 contained the

amount of P 1 million will be equally

attestation clause and the signatures, at

divided among them.

the

wife

rules

Erica

and

on

her

intestate

children

the bottom thereof, of the 3 instrumental witnesses which included Lambert, the driver of Arthur; Yoly, the family cook, and Attorney Zorba, the lawyer who prepared the will. There was a 3rd page, but this only contained the notarial acknowledgement.

Intestate

Succession;

Representation:

Rights

Illegitimate,

of

Adopted

Child; Iron Curtain Rule (2007)

The attestation clause stated the will was

No. X. For purpose of this question, assume

signed on the same occasion by Arthur and

all formalities and procedural requirements

his instrumental witnesses who all signed

have been complied with.

in the presence of each other, and the notary public who notarized the will. There

In 1970, Ramon and Dessa got married.

are no marginal signatures or pagination

Prior to their marriage, Ramon had a child,

appearing on any of the 3 pages. Upon his

Anna. In 1971 and 1972, Ramon and Dessa

death, it was discovered that apart from the

legally

house and lot, he had a P 1 million account

respectively. In 1973, Dessa died while

deposited with ABC bank.

giving birth to Larry Anna had a child, Lia.

adopted

Cherry

and

Michelle

Anna never married. Cherry, on the other hand, legally adopted Shelly. Larry had

“Never Let The Odds Keep You From Pursuing What You Know In Your Heart You Were Meant To Do.”-Leroy Satchel Paige

Page 51 of 180

Civil Law Q&As (2007-2013)

[email protected]

[email protected]

twins, Hans and Gretel, with his girlfriend,

also of the person from whom the person

Fiona. In 2005, Anna, Larry and Cherry

being

died in a car accident. In 2007, Ramon

inherit. While Shelly is a legal heir of

died. Who may inherit from Ramon and

Cherry, Shelly is not a legal heir of

who may not? Give your reason briefly.

Ramon.

Adoption

(10%)

personal

legal

represented

was

supposed

created

relation

a

only

to

purely between

Cherry and Shelly. SUGGESTED ANSWER: (2). Hans and Gretel are barred from The following may inherit from Ramon: (1). Michelle, as an adopted child of Ramon, will inherit as a legitimate child of Ramon. As an adopted child, Michelle

inheriting from Ramon under Art. 992, NCC. Being illegitimate children, they cannot inherit ab intestao from Ramon. ALTERNATIVE ANSWER:

has all the rights of a legitimate child (Sec 18, Domestic Adoption Law).

The problem expressly mentioned the dates of the adoption of Cherry and

(2). Lia will inherit in representation of

Michelle as 1971 and 1972. During that

Anna. Although Lia is an illegitimate

time, adoption was governed by the New

child, she is not barred by Articles 992,

Civil Code. Under the New Civil Code,

because

an

husband and wife were allowed to adopt

illegitimate herself. She will represent

separately or not jointly with the other

Anna as regards Anna's legitime under

spouse. And since the problem does not

Art. 902, NCC and as regards Anna's

specifically and categorically state, it is

intestate share under Art. 990, NCC.

possible to construe the use of the word

her

mother

Anna

is

The following may not inherit from Ramon:

cannot represent Cherry. This is because adoption creates a personal legal relation between

adopted.

The

the

adopter

law

on

and

the

representation

requires the representative to be a legal heir of the person he is representing and

in

the

problem

as

indicative of the situation that Cherry was

(1). Shelly, being an adopted child, she

only

"respectively" adopted

by

Ramon

alone

and

Michelle was adopted by Dessa alone. In such

case

of

separate

adoption

the

alternative answer to the problem will be as follows: Only Lia will inherit from Ramon in representation of Ramon's illegitimate daughter Anna. Although Lia is an illegitimate child, she is not barred from inheriting from Ramon because her

“Never Let The Odds Keep You From Pursuing What You Know In Your Heart You Were Meant To Do.”-Leroy Satchel Paige

Page 52 of 180

Civil Law Q&As (2007-2013)

[email protected]

mother is herself illegitimate. Shelly cannot

inherit

in

representation

of

Cherry because Shelly is just an adopted child of Cherry. In representation, the representative must not only be a legal heir of the person he is representing but also of the decedent from whom the represented

person

is

supposed

to

inherit. In the case of Shelly, while she is a legal heir of Cherry by virtue of

[email protected]

SUGGESTED ANSWER: A testator may dispose of by will the free portion of his estate. Since the legitime of JCP is 1/8 of the estate, SGO is ¼ of the estate and that of HBR and RVC is ½ of the hereditary estate under Art 889 of the NCC, the remaining 1/8 of the estate is the free portion which the testator may dispose of by will.

adoption, she is not a legal heir of Ramon.

Adoption

creates

a

personal

legal relation only between the adopting

Legitime; Compulsory Heirs (2008)

parent and the adopted child (Teotico v. Del Val, 13 SCRA 406, 1965. Michelle

No.

cannot inherit from Ramon, because she

worker, was coming home to the Philippines

was adopted not by Ramon but by Dessa.

after working for so many years in the

In the eyes of the law, she is not related

Middle East. He had saved P100.000 in his

to Ramon at all. Hence, she is not a legal

saving account in Manila which intended to

heir of Ramon. Hans and Gretel are not

use to start a business in his home

entitled to inherit from Ramon, because

country. On his flight home, Ernesto had a

they are barred by Art. 992 NCC. Being

fatal heart attack. He left behind his

illegitimate

they

widowed mother, his common-law wife and

legitimate

their twins sons. He left no will, no debts,

relatives of their father Larry. Ramon is

no other relatives and no other properties

a legitimate relative of Larry who is the

except the money in his saving account.

legitimate father.

Who are the heirs entitled to inherint from

cannot

children

inherit

from

of

Larry,

the

him

XII.

Ernesto,

and

how

an

overseas

much

should

Filipino

each

receive?(3%) Legitimes; Compulsory Heirs (2012)

SUGGESTED ANSWER:

No.VIII.b) How can RJP distribute his estate

The mother and twin sons are entitled to

by will, if his heirs are JCP, his wife; HBR

inherit from Ernesto. Art. 991 of the

and RVC, his parents; and an illegitimate

Civil Code, provides that if legitimate

child, SGO? “Never Let The Odds Keep You From Pursuing What You Know In Your Heart You Were Meant To Do.”-Leroy Satchel Paige

Page 53 of 180

Civil Law Q&As (2007-2013)

[email protected]

[email protected]

ascendants are left, the twin sons shall

The attestation clause stated the will was

divide the inheritance with them taking

signed on the same occasion by Arthur and

one-half of the estate. Thus, the widowed

his instrumental witnesses who all signed

mother gets P50,000.00 while the twin

in the presence of each other, and the

sons shall receive P25,000.00 each. The

notary public who notarized the will. There

common-law wife cannot inherit from

are no marginal signatures or pagination

him because when the law speaks "widow

appearing on any of the 3 pages. Upon his

or widower" as a compulsory heir, the

death, it was discovered that apart from the

law refers to a legitimate spouse (Art.

house and lot, he had a P 1 million account

887, par 3, Civil Code).

deposited with ABC bank. (A). Was Erica preterited? (1%)

Preterition; Disinheritance (2008) No.X.

Arthur

executed

a

will

SUGGESTED ANSWER: which

Erica cannot be preterited. Art. 854 of

contained only: (i) a provision disinheriting

the

Civil

Code

provides

that

only

his daughter Bernica for running off with a

compulsory heirs in the direct line can

married man, and (ii) a provision disposing

be preterited.

of his share in the family house and lot in favor of his other children Connie and Dora.

(B). What other defects of the will, if any,

He did not make any provisions in favor of

can cause denial of probate? (2%)

his wife Erica, because as the will stated, she would anyway get ½ of the house and lot as her conjugal share. The will was very brief and straightforward and both the above provisions were contained in page 1, which Arthur and his instrumental witness, signed at the bottom. Page 2 contained the attestation clause and the signatures, at the bottom thereof, of the 3 instrumental witnesses which included Lambert, the driver of Arthur; Yoly, the family cook, and Attorney Zorba, the lawyer who prepared the will. There was a 3rd page, but this only contained the notarial acknowledgement.

SUGGESTED ANSWER: The other defects of the will that can cause its denial are as follows: (a) Atty. Zorba, the one who prepared the will was one of the three witnesses, violating the three-witnesses rule;

(b) no marginal

signature at the last page; (c ) the attestation did not state the number of pages upon which the will is written; and,

(d)

no

pagination

appearing

correlatively in letters on the upper part of the three pages (Azuela v. C.A., G.R.

“Never Let The Odds Keep You From Pursuing What You Know In Your Heart You Were Meant To Do.”-Leroy Satchel Paige

Page 54 of 180

Civil Law Q&As (2007-2013)

[email protected]

[email protected]

No. 122880, 12 Apr 2006 and cited cases

(B). Between Marian and the baby, who is

therein, Art 805 and 806, Civil Code).

presumed to have died ahead? (1%)

(C). Was the disinheritance valid? (1%)

SUGGESTED ANSWER:

SUGGESTED ANSWER:

Marian is presumed to have died ahead of the baby. Art. 43 applies to persons

Yes, the disinheritance was valid. Art.

who are called to succeed each other.

919, par 7, Civil Code provides that

The proof of death must be established

"when a child or descendant leads a

by positive or circumstantial evidence

dishonorable

like

derived from facts. It can never be

running off with a married man, there is

established from mere inference. In the

sufficient cause for disinheritance."

present case, it is very clear that only

or

disgraceful

life,

Marian and Pietro were hacked with bolos. There was no showing that the Succession;

Proof

of

Death

between

persons called to succeed each other (2008) No. II. At age 18, Marian found out that she was pregnant. She insured her own life and named

her

unborn

child

as

her

sole

beneficiary. When she was already due to give birth, she and her boyfriend Pietro, the father

of

her

unboarn

child,

were

kidnapped in a resort in Bataan where they were vacationing. The military gave chase and after one week, they were found in an abandoned hut in Cavite. Marian and Pietro were hacked with bolos. Marian and the baby delivered were both found dead, with the baby's umbilical cord already cut. Pietro survived.

baby was also hacked to death. The baby's death could have been due to lack of nutrition. ALTERNATIVE ANSWER: The baby is presumed to have died ahead of Marian. Under Par. 5, rule 131, Sec. 5 (KK) of the Rules of Court, if one is under 15 or above 60 and the age of the other is in between 15 and 60, the latter is presumed to have survived. In the instant case, Marian was already 18 when

she

found

out

that

she

was

pregnant. She could be of the same age or maybe 19 years of age when she gave birth. (C). Will Pietro, as surviving biological father of the baby, be entitled to claim the

“Never Let The Odds Keep You From Pursuing What You Know In Your Heart You Were Meant To Do.”-Leroy Satchel Paige

Page 55 of 180

Civil Law Q&As (2007-2013)

[email protected]

[email protected]

proceeds of the life insurance on the life of Marian? (2%)

Marilyn is not entitled to a share in the estate of Dr. Lopez.

SUGGESTED ANSWER:

succession,

Pietro, as the biological father of the baby, shall be entitled to claim the proceeds of life insurance of the Marian because he is a compulsory

heir of his

child.

Succession; Rule on Survivorship (2009)

Dr.

For purpose of

Lopez

and

his

son

Roberto are presumed to have died at the same time, there being no evidence to prove otherwise, and there shall be no transmission of rights from one to the other (Article 43, NCC). Hence, Roberto, inherited nothing from his father that Marilyn

would

Roberto

.The

in

turn

children

inherit of

from

Roberto,

however, will succeed their grandfather, Dr. Lopez ,in representation of their

No. II. Dr. Lopez, a 70-year old widower,

father Roberto and together Roberto will

and his son Roberto both died in a fire that

receive 1/3 of the estate of Dr. Lopez

gutted their home while they were sleeping

since their father Roberto was one of the

in their air-conditioned rooms. Roberto’s

three children of Dr. Lopez . Marilyn

wife, Marilyn, and their two children were

cannot represent her husband Roberto

spared because they were in the province at

because the right is not given by the law

the time. Dr. Lopez left an estate worth

to a surviving spouse.

P20M and a life insurance policy in the amount of P1M with his three children ---

As to the proceeds of the insurance on

one of whom is Roberto --- as beneficiaries.

the life of Dr. Lopez:

Marilyn is now claiming for herself and her

Since succession is

children her husband’s share in the estate

regards

left by Dr. Lopez, and her husband’s share

provisions of the Rules of Court (Rule

in the proceeds of Dr. Lopez’s life insurance

131, Sec. 3 , [jj] [5] ) on survivorship

policy. Rule on the validity of Marilyn’s

shall apply. Under the Rules, Dr. Lopez,

claims with reasons. (4%)

who was 70 years old, is presumed to

SUGGESTED ANSWER : As to the Estate of Dr. Lopez:

the

not involved as

insurance

contract,

the

have died ahead of Roberto who is presumably between the ages 15 and 60. Having survived the insured, Roberto's right as a beneficiary became vested

“Never Let The Odds Keep You From Pursuing What You Know In Your Heart You Were Meant To Do.”-Leroy Satchel Paige

Page 56 of 180

Civil Law Q&As (2007-2013)

[email protected]

[email protected]

upon the death of Dr. Lopez. When

should be given effect must be denied.

Roberto died after Dr. Lopez, his right to

The said cancellation has revoked the

receive the insurance became part of his

entire will as nothing remains of the will

hereditary estate, which in turn was

after the name of Rosa was cancelled.

inherited in equal shares by his legal

Such cancellation is valid revocation of

heirs, namely, his spouse and children.

the

Therefore, Roberto's children and his

authentication by the full signature of

spouse

the testator to be effective.

are

one-third

entitled

share

in

to

Roberto's

the

insurance

proceeds.

will

and

does

not

require

However, if the cancellation of Rosa’s name was not done by the testator himself, such cancellation shall not be effective and the will in its original tenor

Wills; Holographic Wills; Insertions &

shall remain valid. The effectively of the

Cancellations (2012)

holographic will cannot be left to the mercy of unscrupulous third parties.

No.VII.a) Natividad’s holographic will, which had only one (1) substantial provision, as

The writing of Gregorio’s name as sole

first written, named Rosa as her sole heir.

heir was ineffective, even though written

However, when Gregorio presented it for

by the testator himself, because such is

probate, it already contained an alteration,

an

naming Gregorio, instead of Rosa, as sole

authentication by the full signature of

heir,

by

the testator to be valid and effective. Not

Natividad’s signature. Rosa opposes the

having an authenticated, the designation

probate

of Gregorio as an heir was ineffective,

but

without

alleging

authentication.

authentication

such She

lack

claims

of

proper

that

the

unaltered form of the will should be given

alteration

that

requires

(Kalaw v. Relova, G.R. No. L-40207, Sept 28, 1984).

effect. Whose claim should be granted? Explain. (5%) SUGGESTED ANSWER:

Wills; Holographic Wills; Probate (2009)

It depends. If the cancellation of Rosa’s

No.VI. On December 1, 2000, Dr. Juanito

name in the will was done by the

Fuentes

testator himself, Rosa’s claimed that the

wherein he gave nothing to his recognized

holographic will in its original tenor

illegitimate son, Jay. Dr. Fuentes left for the

executed

a

holographic

will,

“Never Let The Odds Keep You From Pursuing What You Know In Your Heart You Were Meant To Do.”-Leroy Satchel Paige

Page 57 of 180

Civil Law Q&As (2007-2013)

[email protected]

[email protected]

United States, passed the New York medical

court shall apply the New Civil Code in

licensure examinations, resided therein,

determining the formal validity of the

and became a naturalized American citizen.

holographic will. The subsequent change

He died in New York in 2007. The laws of

in the citizenship of Dr. Fuentes did not

New York do not recognize holographic wills

affect the law governing the validity of

or compulsory heirs.

his will. Under the new Civil Code, which was the law used by Dr. Fuentes, the law

(A). Can the holographic will of Dr. Fuentes

enforced at the time of execution of the

be admitted to probate in the Philippines?

will shall govern the formal validity of

Why or why not? (3%)

the will (Art. 795, NCC).

SUGGESTED ANSWER:

(B). Assuming that the will is probated in

Yes, the holographic will of Dr. Fuentes

the Philippines, can Jay validly insist that

may be admitted to probate in the

he be given his legitime? Why or why not?

Philippines because there is no public

(3%)

policy violated by such probate. The only issue at probate is the due execution of

SUGGESTED ANSWER:

the

formal

No, Jay cannot insist because under New

validity of the will. As regards formal

York law he is not a compulsory heir

validity, the only issue the court will

entitled to a legitime.

will

which

includes

the

resolve at probate is whether or not the will was executed in accordance with the

The

form prescribed by the law observed by

determines who his heirs are, the order

the testator in the execution of his will.

that

For

the

successional rights are, and whether or

may

not a testamentary disposition in his will

observe the law of the place where the

is valid (Art 16, NCC). Since, Dr. Fuentes

will was executed (Art 17, NCC), or the

was a US citizen, the laws of the New

formalities of the law of the place where

York determines who his heirs are. And

he

since

purposes

Philippines,

resides,

of

an

or

probate

alien

in

testator

according

to

the

national they

the

law

succeed,

New

of

the

how

York

testator

much

law

does

their

not

formalities of the law of his own country,

recognize the concept of compulsory

or in accordance with the Philippine

heirs, Jay is not a compulsory heir of Dr.

Civil Code (Art. 816, NCC). Since Dr.

Fuentes entitled to a legitime.

Fuentes executed his will in accordance with the Philippine law, the Philippine

“Never Let The Odds Keep You From Pursuing What You Know In Your Heart You Were Meant To Do.”-Leroy Satchel Paige

Page 58 of 180

Civil Law Q&As (2007-2013)

[email protected]

[email protected]

Wills; Joint Wills (2008)

SUGGESTED ANSWER:

No. XI. John and Paula, British citizens at

No. The testamentary dispositions are

birth, acquired Philippine citizenship by

not valid because (a) omission of Mary, a

naturalization after their marriage. During

legitimate

their

preterition

marriage

the

couple

acquired

child,

is

which

tantamount

shall

annul

to the

substanial landholdings in London and in

institution of Peter and Paul as heirs

Makati. Paula bore John three children,

(Art.

Peter, Paul and Mary. In one of their trips

disposition that Peter and Paul could not

to London, the couple executed a joint will

dispose of nor divide the London estate

appointing each other as their heirs and

for more than 20 years is void (Art. 870,

providing that upon the death of the

Civil Code).

854,

Civil

Code);

and,

(b)

the

survivor between them the entire estate would go to Peter and Paul only but the two could not dispose of nor divide the London estate as long as they live. John and Paul died

tragically

in

the

London

Subway

terrorist attack in 2005. Peter and Paul filed a petition for probate of their parent's will before a Makati Regional Trial Court. (A). Should the will be admitted to probate? (2%)

Wills; Joint Wills; Probate (2012) No.VII.b) John Sagun and Maria Carla Camua, British citizens at birth, acquired Philippine

citizenship

by

naturalization

after their marriage. During their marriage, the

couple

acquired

substantial

landholdings in London and in Makati. Maria

begot

three

(3)

children,

Jorge,

Luisito, and Joshur. In one of their trips to

SUGGESTED ANSWER:

London, the couple executed a joint will No. The will cannot be admitted to

appointing each other as their heirs and

probate because a joint will is expressly

providing that upon the death of the

prohibited under Art. 818 of the Civil

survivor between them, the entire estate

Code. This provision applies John and

would go to Jorge and Luisito only but the

Paula became Filipino citizens after their

two (2) could not dispose of nor divide the

marriage.

London estate as long as they live. John and Maria died tragically in the London

(B).

Are

valid? (2%)

the

testamentary

dispositions

subway terrorist attack in 2005. Jorge and Luisito filed a petition for probate of their parents’ will before a Makati Regional Trial

“Never Let The Odds Keep You From Pursuing What You Know In Your Heart You Were Meant To Do.”-Leroy Satchel Paige

Page 59 of 180

Civil Law Q&As (2007-2013)

[email protected]

Court. Joshur vehemently objected because he was preterited.

[email protected]

SUGGESTED ANSWER: Assuming the will of John and Maria was

(1) Should the will be admitted to probate?

valid, the testamentary prohibition on the

Explain. (2%)

division of the London estate shall be valid but only for 20 years. Under Arts 1083 and 494 of the NCC, a testamentary disposition

SUGGESTED ANSWER: No, the will should not be admitted to probate. Since the couples are both Filipino citizens, Art 818 and 819 of the

of the testator cannot forbid the partition of all or part of the estate for a period longer than twenty (20) years.

NCC shall apply. Said articles prohibits the execution of joint wills and make them void, even though authorized of

Wills; Prohibition to Partition of a Co-

the country where they were executed.

Owned Property (2010)

(2) Are the testamentary dispositions valid?

No.I. True or False.

Explain. (2%) (B) X, a widower, died leaving a will stating that the house and lot where he lived

SUGGESTED ANSWER: Since the joint will is void, all the testamentary disposition written therein are also void. However, if the will is valid, the institutions of the heirs shall be

annulled

because

Joshur

was

cannot be partitioned for as long as the youngest of his four children desires to stay there. As coheirs and co-owners, the other three may demand partition anytime. (1%) SUGGESTED ANSWER:

preterited. He was preterited because he will receive nothing from the will, will

FALSE, The other three co – heirs may

receive nothing in testacy, and the facts

not anytime demand the partition of the

do not show that he received anything as

house and lot since it was expressly

an advance on his inheritance. He was

provided by the decedent in his will that

totally excluded from the inheritance of

the same cannot be partitioned while his

his parents.

youngest child desires to stay there. Article 1083 of the New Civil Code allows

(3) Is the testamentary prohibition against

a decedent to prohibit, by will, the

the division of the London estate valid?

partition of a property and his estate for

Explain. (1%)

a period not longer than 20 years no

“Never Let The Odds Keep You From Pursuing What You Know In Your Heart You Were Meant To Do.”-Leroy Satchel Paige

Page 60 of 180

Civil Law Q&As (2007-2013)

[email protected]

matter what his reason maybe. Hence,

[email protected]

(B). Act as a witness to a will? (1%)

the three co-heir cannot demand its partition at anytime but only after 20 years from the death of their father. Even if the deceased parent did not leave

a

will,

if

the

house

and

lot

constituted their family home, Article 159 of the Family Code prohibits its partition for a period of ten (10) years, or for

as

long

as

there

is

a

minor

beneficiary living in the family home.

SUGGESTED ANSWER: Stevie cannot be a witness to a will. Art. 820 of the Civil Code provides that "any person of sound mind and of the age of eighteen years or more, and not blind, deaf or dumb, and able to read and write, may be a witness to the execution of a will. (C). In either of the above instances, must the will be read to him? (1%)

Wills;

Notarial

Wills;

Blind

Testator;

Requisites (2008) No. XIV. Stevie was born blind. He went to school for the blind, and learned to read in Baille

Language.

He

Speaks

English

fluently. Can he:

SUGGESTED ANSWER: If Stevie makes a will, the will must be read to him twice, once by one of the subscribing witnesses, and again, by the notary public before whom the will is acknowledged (Art. 808, Civil Code).

(A). Make a will? (1%) SUGGESTED ANSWER: Assuming that he is of legal age (Art. 797, Civil Code) and of sound mind at the time of execution of the will (Art. 798, Civil Code), Stevie, a blind person, can make a notarial will, subject to compliance with the "two-reading rule" (Art. 808, Civil Code) and the provisions of Arts. 804, 805 and 806 of the Civil Code.

Wills; Testamentary Disposition; Period to Prohibit Partition (2008) No. XI. John and Paula, British citizens at birth, acquired Philippine citizenship by naturalization after their marriage. During their

marriage

the

couple

acquired

substanial landholdings in London and in Makati. Paula bore John three children, Peter, Paul and Mary. In one of their trips to London, the couple executed a joint will appointing each other as their heirs and

“Never Let The Odds Keep You From Pursuing What You Know In Your Heart You Were Meant To Do.”-Leroy Satchel Paige

Page 61 of 180

Civil Law Q&As (2007-2013)

[email protected]

[email protected]

providing that upon the death of the

that she can sign her full name later. While

survivor between them the entire estate

the

would go to Peter and Paul only but the two

experienced a stomach ache and kept going

could not dispose of nor divide the London

to the restroom for long periods of time.

estate as long as they live. John and Paul

Hannah, while waiting for her turn to sign

died

Subway

the will, was reading the 7th Harry Potter

terrorist attack in 2005. Peter and Paul

book on the couch, beside the table on

filed a petition for probate of their parent's

which everyone was signing. Benjamin,

will before a Makati Regional Trial Court.

aside from witnessing the will, also offered

tragically

in

the

London

will

was

being

signed,

Roberta

to notarize it. A week after, Clara was run (C). Is the testamentary prohibition against

over by a drunk driver while crossing the

the division of the London estate valid? (2%)

street in Greenbelt.

SUGGESTED ANSWER:

May the will of Clara be admitted to

No. the testamentary prohibition against the division of the London estate is void (Art.

870,

Civil

Code).

A

probate? Give your reasons briefly. (10%) SUGGESTED ANSWER:

testator,

however, may prohibit partition for a

Probate

should

be

denied.

The

period which shall not exceed twenty

requirement that the testator and at

(20) years (Art. 870 in relation to Art.

least three (3) witnesses must sign all in

494, par 3, Civil Code).

the "presence" of one another was not complied with. Benjamin who notarized the will is disqualified as a witness,

Wills; Witnesses to required;

a

Thumbmark

Will, Presence as

Signature

(2007)

the three witnesses (Cruz v. Villasor, 54 SCRA 31, 1973). The testatrix and the other witnesses signed the will not in

No.VI. Clara, thinking of her mortality, drafted a will and asked Roberta, Hannah, Luisa

hence he cannot be counted as one of

and

Benjamin

to

be

witnesses.

During the day of signing of her will, Clara fell down the stairs and broke her arms. Coming from the hospital, Clara insisted on signing her will by thumb mark and said

the presence of Roberta because she was in the restroom for extended periods of time. Inside the restroom, Roberta could not have possibly seen the testatrix and the other witnesses sign the will by merely casting her eyes in the proper direction (Jaboneta v. Gustilo, 5 Phil 541, 1906; Nera v. Rimando, 18 Phil

“Never Let The Odds Keep You From Pursuing What You Know In Your Heart You Were Meant To Do.”-Leroy Satchel Paige

Page 62 of 180

Civil Law Q&As (2007-2013)

451,

1914).

[email protected]

Therefore,

the

[email protected]

testatrix

Because the Picasso painting reminded

signed the will in the presence of only

Angie of him, Brad in his will bequeathed

two witnesses, and only two witnesses

the painting to Angie. Brad died in 1995.

signed the will in the presence of the

Saddened by Brad's death, Jennifer asked

testatrix and of one another.

for the Picasso painting as a remembrance of him. Angie refused and claimed that

It is to be noted, however, that the

Brad, in his will, bequeathed the painting to

thumb mark intended by the testator to

her. Is Angie correct? Why or why not?

be his signature in executing his last will

(10%)

and

testament

is

valid

(Payad

v.

Tolentino, 62 Phil 848, 1936; Matias v.

SUGGESTED ANSWER:

Salud, L-104 Phil 1046, 23 June, 1958). The problem, however, states that Clara

NO. Angie is not correct. The Picasso

"said that she can sign her full name

painting is not given or donated by

later;" Hence, she did not consider her

Jennifer to Brad. She merely "placed it

thumb mark as her "complete" signature,

in his bedroom." Hence, she is still the

and intended further action on her part.

owner of the painting. Not being the

The testatrix and the other witness

owner of the Picasso painting, Brad

signed

of

cannot validly bequeath the same to

Hannah, because she was aware of her

Angie (Art. 930, NCC). Even assuming

function and role as witness and was in a

that the painting was impliedly given or

position to see the testatrix and the

donated

other witnesses sign by merely casting

donation is nevertheless void for not

her eyes in the proper direction.

being in writing. The Picasso painting

the

will

in

the

presence

Donation Donations; Formalities; In Writing (2007)

by

Jennifer

to

Brad,

the

must be worth more than 5,000 pesos. Under Art. 748, NCC, the donation and acceptance of a movable worth more than 5,000 pesos must be in writing,

No. VIII. In 1986, Jennifer and Brad were

otherwise the donation is void. The

madly in love. In 1989, because a certain

donation being void, Jennifer remained

Picasso painting reminded Brad of her,

the owner of the Picasso painting and

Jennifer acquired it and placed it in his

Brad could not have validly disposed of

bedroom. In 1990, Brad and Jennifer broke

said painting in favor of Angie in his will.

up. While Brad was mending his broken heart, he met Angie and fell in love.

ALTERNATIVE ANSWER:

“Never Let The Odds Keep You From Pursuing What You Know In Your Heart You Were Meant To Do.”-Leroy Satchel Paige

Page 63 of 180

Civil Law Q&As (2007-2013)

[email protected]

[email protected]

YES. Angie is correct. Even assuming

illegal and impossible donations imposed

that there was void donation because the

in an onerous donation shall annul the

same was not in writing, Brad was in

donation (Art. 1183, NCC). This is so,

uninterrupted possession of the Picasso

because onerous donations are governed

painting from 1989 to 1995, lasting for

by the law on contracts (Art. 733, NCC).

six (6) years prior to his death. Brad has already

acquired

ownership

painting

through

prescription.

Under

ownership

of

of

the

acquisitive

Art.

movables

1132,

NCC,

prescribes

through continuous possession for four (4) years in good faith and for eight (8) years without need of other conditions. A void donation may be the basis of

Donation; Inter Vivos (2013) No.V. Josefa executed a deed of donation covering a one-hectare rice land in favor of her

daughter,

Jennifer.

The

deed

specifically provides that:

possession in the concept of owner and

"For and in consideration of her love

of just title for purposes of acquisitive

and service Jennifer has shown and

prescription.

given

to

me,

I

hereby

freely,

voluntarily and irrevocably donate to her my one-hectare rice land covered Donations;

Illegal

&

by TCT No. 11550, located in San

Impossible

Fernando, Pampanga. This donation

Conditions (2007) No.I. Distinguish the following concepts:

shall take effect upon my death." The deed also contained Jennifer's signed

(B). Illegal and impossible conditions in a

acceptance,

and

an

attached notarized

simple donation v. illegal and impossible

declaration by Josefa and Jennifer that the

conditions in an onerous donation. (5%)

land will remain in Josefa's possession and cannot be alienated, encumbered, sold or

SUGGESTED ANSWER:

disposed of while Josefa is still alive.

Illegal and impossible conditions in a

Advise Jennifer on whether the deed is a

simple donation are considered as not

donation inter vivos or mortis causa and

written. Such conditions, shall therefore,

explain the reasons supporting your advice.

be disregarded but the donation remains

(8%)

valid (Art. 727, NCC). On the other hand,

“Never Let The Odds Keep You From Pursuing What You Know In Your Heart You Were Meant To Do.”-Leroy Satchel Paige

Page 64 of 180

Civil Law Q&As (2007-2013)

[email protected]

SUGGESTED ANSWER:

[email protected]

same should be harmonized with its express irrevocability (Austria-Magat v.

The donation is a donation inter vivos.

CA, G.R. No. 106755, Feb 1, 2002).

When

ALTERNATIVE ANSWER:

the

donor

intends

that

the

donation shall take effect during the lifetime

of

the

donor,

though

the

The donation is donation mortis causa.

property shall not be delivered till after the

donor’s

death,

this

shall

be

a

The

deed

clearly

states

that

the

donation inter vivos (Art. 729, Civil

donation shall take effect upon the

Code).

death of the donor, Josefa. The donor, moreover, retained ownership of the

The

Civil

Code

prefers

inter

vivos

subject property as it was declared that

transmissions. Moreover, mortis causa

the

property

cannot

be

alienated,

donations should follow the formalities

encumbered, sold or disposed of while

of a will (Art. 728, Civil Code). Here there

the donor is still alive.

is no showing that such formalities were followed. Thus, it is favorable to Jennifer

As the donation is in the nature of a

that the deed is a donation inter vivos.

mortis causa disposition, the formalities of a will should have been complied with

Furthermore, what is most significant in

under

determining the type of donation is the

otherwise, the donation is void and

absence of stipulation that the donor

would produce no effect (The National

could

the

Treasure of the Philippines v. Vda. de

contrary, the deeds expressly declare

Meimban, G.R. No. L-61023, Aug 22,

them

1984).

revoke to

be

the

donation;

“irrevocable,”

a

on

quality

Art.

728

of

the

Civil

Code,

absolutely incompatible with the idea of conveyances

mortis

causa

where

Property

revocability is the essence of the act, to the

extent

that

a

testator

cannot

Accretion; Alluvium (2008)

lawfully waive or restrict his right of revocation. The provisions of the deed of

No. IX. The properties of Jessica and Jenny,

donation which state that the same will

who are neighbors, lie along the banks of

only take effect upon the death of the

the Marikina River. At certain times of the

donor and that there is a prohibition to

year, the river would swell and as the water

alienate, encumber, dispose, or sell the

recedes, soil, rocks and other materials are

“Never Let The Odds Keep You From Pursuing What You Know In Your Heart You Were Meant To Do.”-Leroy Satchel Paige

Page 65 of 180

Civil Law Q&As (2007-2013)

deposited properties.

on

[email protected]

Jessica's

This

swelling, receding and depositing soil and

man, it is man-made accretion and a

other materials being deposited on the

part of the public domain (Tiongco v.

neighbors' properties have gone on for

Director of Lands, 16 C.A. Rep 211, cited

many years. Knowing his pattern, Jessica

in Nazareno v. C.A., G.R. No. 98045, 26

constructed a concrete barrier about 2

June 1996). Thus, Jessica cannot legally

meters

claim ownership of the additional 2

property

the

land but is also the consequences of the direct and deliberate intervention of

her

of

Jenny's river

from

pattern

and

[email protected]

line

and

extending towards the river, so that when

meters

the water recedes, soil and other materials

because

are trapped within this barrier. After several

barrier about 2 meters from her property

years, the area between Jessica's property

causing

line to the concrete barrier was completely

materials when the water recedes. In

filled

increasing

other words, the increase in her property

Jessica's property by 2 meters. Jenny's

was not caused by nature but was man-

property, where no barrier was constructed,

made.

with

soil,

effectively

of

land

she

along

her

constructed

deposits

of

a

soil

property concrete

and

other

also increased by one meter along the side (B). If Jessica's and Jenny's properties are

of the river.

registered,

will

the

benefit

of

such

(A). Can Jessica and Jenny legally claim

registration extend to the increased area of

ownership over the additional 2 meters and

their properties? (2%)

one meter, respectively, of land deposited along their properties?(2%)

SUGGESTED ANSWER:

SUGGESTED ANSWER:

If the properties of Jessica and Jenny are

registered,

the

the

land

increased

deposited along her property. Art. 457 of

Accretion

the Civil Code provides that "to the

become registered land because there is

owners of lands adjoining the banks of

a specific technical description of the lot

river belong the accretion which they

in its Torrens title. There must be a

gradually receive from the effects of the

separate application for registration of

current of the water." Where the land is

the alluvial deposits under the Torrens

not formed solely by the natural effect of

System (Grande v. CA, G.R. No. L-17652,

the water current of the river bordering

30 June, 1962).

meter

of

area does

of

extend

their not

to

such

registration

one

not

of

Only Jenny can claim ownership over additional

does

benefit

the

properties.

automatically

“Never Let The Odds Keep You From Pursuing What You Know In Your Heart You Were Meant To Do.”-Leroy Satchel Paige

Page 66 of 180

Civil Law Q&As (2007-2013)

[email protected]

[email protected]

(C). Assume the two properties are on a cliff

Ulpiano built three huts on this additional

adjoining the shore of Laguna Lake. Jessica

area,

and Jenny had a hotel built on the

children live. On this same area, Ulpiano

properties. They had the erath and rocks

and his family planted peanuts, monggo

excavated from the properties dumped on

beans

the adjoining shore, giving rise to a new

regularly paid taxes on the land, as shown

patch of dry land. Can they validly lay claim

by tax declarations, for over thirty years.

where

he

and

and

his

vegetables.

two

married

Ulpiano

also

to the patch of land? (2%) When Marciano learned of the increase in SUGGESTED ANSWER:

the size of the land, he ordered Ulpiano to demolish the huts, and demanded that he

No. Jessica and Jenny cannot validly lay

be paid his share in the proceeds of the

claim to the patch of land because in

harvest. Marciano claims that under the

order to acquire land by accretion, there

Civil Code, the alluvium belongs to him as a

should

actual

registered riparian owner to whose land the

continuity of the accretion to the land of

accretion attaches, and that his right is

the riparian owner caused by natural ebb

enforceable against the whole world.

be

a

natural

and

and flow of the current of the river (Delgado v. Samonte, CA-G.R. No. 34979-

(A). Is Marciano correct? Explain. (3%)

R, 10 Aug 1966). SUGGESTED ANSWER: Marciano’s contention is correct. Since that accretion was deposited on his land Accretion; Rights of the Riparian Owner

by the action of the waters of the river

(2009)

and he did not construct any structure

No.XVI. Marciano is the owner of a parcel of land through which a river runs out into the sea. The land had been brought under the Torrens System, and is cultivated by Ulpiano and his family as farmworkers therein. Over the years, the river has brought silt and sediment from its sources up in the mountains and forests so that gradually the land owned by Marciano increased

in

area

by

three

hectares.

to increase the deposition of soil and silt, Marciano automatically owns the accretion. His real right of ownership is enforceable

against

the

whole

world

including Ulpiano and his two married children. Although Marciano’s land is registered, the three (3) hectares land deposited

through accretion was not

automatically unregistered

registered. land,

it

is

As

an

subject

to

acquisitive prescription by third persons.

“Never Let The Odds Keep You From Pursuing What You Know In Your Heart You Were Meant To Do.”-Leroy Satchel Paige

Page 67 of 180

Civil Law Q&As (2007-2013)

[email protected]

[email protected]

production, gathering and preservation Although Ulpiano and his children live in

of the fruits (Art 443, NCC).

the three (3) hectare unregistered land owned

by

Marciano,

they

are

farm

He may also ask for reimbursement of

workers; therefore, they are possessors

the taxes he has paid, as these are

not in the concept of owners but in the

charges on the land owned by Marciano.

concept of mere holders. Even if they

This obligation is based on a quasi-

possess the land for more than 30 years,

contract (Art 2175, NCC).

they cannot become the owners thereof through

extraordinary

acquisitive

prescription, because the law requires possession in the concept of the owner. Payment of taxes and tax declaration are not enough to make their possession one in the concept of owner. They must repudiate the possession in the concept of holder by executing unequivocal acts of repudiation amounting to ouster of Marciano, known to Marciano and must be

proven

evidence.

by Only

clear

and

then

convincing would

his

possession become adverse.

against Marciano? Explain. (3%)

faith, because he knew he does not own the land, he will lose the three huts he faith

meter land (Lot A) in Paranaque. The land now has a fair market value of Pl,200,000. CRC likewise sold to the spouses Rodriguez, a 700-square meter land (Lot B) which is adjacent to Lot A. Lot B has a present fair market value of P1,500,000.

on Lot B, relying on their presentation of the CRC sales agent that it is the property of their house did the spouses Dela Cruz

Although Ulpiano is a possessor in bad

bad

sold to the spouses Del a Cruz a500-square

they purchased. Only upon the completion

SUGGESTED ANSWER:

in

No.VIII. Ciriaco Realty Corporation (CRC)

The spouses Dela Cruz constructed a house

(B). What rights, if any, does Ulpiano have

built

Builder; Good Faith; Requisites (2013)

and

make

an

accounting of the fruits he has gathered, he has the right to deduct from the value of the fruits the expenses for

discovered that they had built on Lot B owned by the spouses Rodriguez, not on Lot A that they purchased. They spent P 1 000,000 for the house. As their lawyer, advise the spouses Dela Cruz on their rights and obligations under the given circumstances, and the recourses

“Never Let The Odds Keep You From Pursuing What You Know In Your Heart You Were Meant To Do.”-Leroy Satchel Paige

Page 68 of 180

Civil Law Q&As (2007-2013)

[email protected]

[email protected]

and options open to them to protect their

However, the builder cannot be obliged

interests. (8%)

to

buy

the

considerable SUGGESTED ANSWER:

land more

if

its

than

value

that

of

is the

building.. In such case, he shall pay

Based on the fact as stated, the spouses Dela Cruz as builders and the spouses Rodriguez as land owners, are both in good faith. The spouses Dela Cruz are

reasonable rent of the owner of the land does

not

choose

to

appropriate

the

building or trees after proper indemnity (Art 448, Civil Code).

builder in good faith because before

The house constructed by the spouses

constructing the house they exercised

Dela Cruz is considered as a useful

due diligence by asking the Agent of CRC

expense, since it increased the value of

the location of the lot A, and they relied

the lot. As such, should the spouses

on the information given by the agent

Rodriguez decides to appropriate the

who is presumed to know the identity of

house,

the lot purchased by the Dela Cruz

entitled to the right of retention pending

spouses (Pleasantville v. CA, 253 SCRA

reimbursement of the expenses they

10, 1996). On the other hand, there is no

incurred or the increase in value which

showing that the land owners, spouse

the thing may have acquired by reason

Rodriguez acted in bad faith. The facts

of

do not show that the building was done

Code). Thus, the spouses Dela Cruz may

with

without

demand P1,000,000.00 as payment of

opposition on their part (Art 453, Civil

the expenses in building the house or

Code). The good faith is always presumed

increase in value of the land because of

(Art. 527, Civil Code).

the house as a useful improvement, as

their

knowledge

and

The owner of the land on which anything has been built, sown, or planted in good faith shall have the right: (1) to appropriate as his own the works

the

the

spouses

improvement

Dela

(Art

Cruz

546,

are

Civil

may be determined by the court form the evidence presented during the trial (Depra v. Dumlao, 136 SCRA 475, 1985; Technogas Phils v. CA, 268 SCRA 5, 1997).

after payment of the indemnity provided for in Art 546 and 548, or (2) to oblige the one who built to pay the price of the land.

“Never Let The Odds Keep You From Pursuing What You Know In Your Heart You Were Meant To Do.”-Leroy Satchel Paige

Page 69 of 180

Civil Law Q&As (2007-2013)

Easement;

Prescription;

[email protected]

Acquisitive

Prescription (2009)

[email protected]

In 2006, Brand0 fenced off his property, thereby blocking Andres' access to the national highway. Andres demanded that

No. XI. TRUE or FALSE. Answer TRUE if

part of the fence be removed to maintain

the statement is true, or FALSE if the

his

statement is false. Explain your answer in

(pathway A), but Brando refused, claiming

not more than two (2) sentences.

that there was another available pathway

(C). Acquisitive prescription of a negative easement runs from the time the owner of the dominant estate forbids, in a notarized document, the owner of the servient estate

old

access

route

to

the

highway

(pathway B) for ingress and egress to the highway. Andres countered that pathway B has defects, is circuitous, and is extremely inconvenient to use.

from executing an act which would be

To settle their dispute, Andres and Brando

lawful without the easement. (1%)

hired Damian, a geodetic and civil engineer, to survey and examine the two pathways

SUGGESTED ANSWER: True. In negative easements, acquisitive prescription runs from the moment the owner of the dominant estate forbade, by an

instrument

acknowledged

before

notary public, the owner of the servient estate from executing an act which would be lawful without the easement (Art. 621, NCC).

and

the

determine prejudicial

surrounding the way

areas,

shortest through

and the

and the

to

least

servient

estates. After the survey, the engineer concluded that pathway B is the longer route and will need improvements and repairs, but will not significantly affect the use of Brando's property. On the other hand, pathway A that had long been in place, is the shorter route but would significantly affect the use of Brando's

Easement; Right of Way (2013)

property.

No.VII.In 2005, Andres built a residential

In light of the engineer's findings and the

house on a lot whose only access to the

circumstances of the case, resolve the

national highway was a pathway crossing

parties' right of way dispute. (6%)

Brando's property. Andres and others have been using this pathway (pathway A) since

SUGGESTED ANSWER:

1980.

“Never Let The Odds Keep You From Pursuing What You Know In Your Heart You Were Meant To Do.”-Leroy Satchel Paige

Page 70 of 180

Civil Law Q&As (2007-2013)

[email protected]

[email protected]

Andres is not entitled to the easement of

(Pathway B). Second, the right of way

right of way for Pathway A. Pathway B

obtained (Pathway A) is not the least

must be used.

prejudicial

to

Brando’s

property,

as

evidence by the reports of the geodetic The owner of a dominant estate may

and civil engineer.

validly obtain a compulsory right of way only

after

he

has

established

the

existence of four requisites, to wit:

When

there

adequate

is

outlet

already from

an

the

existing dominant

estate to the public highway, even if the (1) The (dominant) estate is surrounded

said outlet, for one reason or another, be

by other immovables and is without

inconvenient,

adequate outlet to a public highway;

another servitude is entirely unjustified

(2)

After

payment

of

the

proper

indemnity;

proprietor’s own acts; and

need

to

open

up

(Costabella Corporation v. CA, G.R. No. 80511, Jan 25, 1991). The rule that the easement

(3) The isolation was not due to the

the

of

right

of

way

shall

be

established at the point least prejudicial to the servient estate is controlling (Quimen v. Quimen and CA, G.R. No.

(4) The right of way claimed is at a point least prejudicial to the servient estate, and insofar as consistent with this rule, where the distance from the dominant estate to the public highway maybe the shortest (Art 650, civil Code).

112331, May 29, 1996). (Note: It is not clear from the problem if there exists an easement in favor of the lot belonging to Andres and if Brando’s lot is burdened as a servient estate by a right of way as a servient estate. If there is such an

has

easement burdening Brando’s lot, was it

consistently ruled that in case both

created as legal easement or as a voluntary

criteria cannot be complied with, the

easement. If the used pathway was only a

right of way shall be established at the

tolerance, then Brando may close it. Andres

point least prejudicial to the servient

must ask for the constitution of a legal

estate.

easement through Brando’s lot by proving

However,

the

Supreme

Court

the four requisites required by Art 649 and The first and fourth requisites are not

65, Civil Code).

complied with. First, there is another available outlet to the national highway

“Never Let The Odds Keep You From Pursuing What You Know In Your Heart You Were Meant To Do.”-Leroy Satchel Paige

Page 71 of 180

Civil Law Q&As (2007-2013)

[email protected]

Easement; Right of Way (2010)

[email protected]

easement or servitude, even if the deed of sale is silent on the matter.

No.XIII. Franz was the owner of Lot E which was surrounded by four (4) lots one of

(3) The vendee of the property in which a

which – Lot C – he also owned. He promised

servitude

Ava that if she bought Lot E, he would give

close or put obstructions thereon to

her a right of way in Lot C.

prevent the dominant estate from using

Convinced, Ava bought Lot E and, as promised, Franz gave her a right of way in Lot C. Ava cultivated Lot E and used the right of way granted by Franz. Ava later found gainful employment abroad.

or

easement

exists

cannot

it. (4) Ava’s working abroad for more than ten (10) years should not be construed as non-user, because it cannot be implied from the fact that she or those she left behind to cultivate the lot no longer use the right of way.

On her return after more than 10 years, the right of way was no longer available to her

Note:

Since

a

right

of

way

is

a

because Franz had in the meantime sold

discontinuous easement, the period of

Lot C to Julia who had it fenced.

ten years of non-user, shall be computed from the day it ceased to be used under

(A). Does Ava have a right to demand from Julia the activation of her right of way? Explain. (2.5%) SUGGESTED ANSWER:

Act 6341 (2) CC. (5)

Renunciation

easement

must

or be

waiver specific,

of

an

clear,

express and made in a public instrument in accordance of Art 1358 of the New

Yes. Ava has the right to demand from

Civil Code.

Julia the activation of the right of way,

ALTERNATIVE ANSWER:

for the following reasons: Yes. Ava has the right to demand from (1) The easement of the right of way is a

Julia the activation of the right of way.

real right which attaches to, and is

A voluntary easement of right of way,

inseperable from, the estate to which it

like

belongs.

extinguished only by mutual agreement

any

other

contract,

could

be

or by renunciation of the owner of the (2) The sale of the property includes the

dominant estate. Also, like any other

“Never Let The Odds Keep You From Pursuing What You Know In Your Heart You Were Meant To Do.”-Leroy Satchel Paige

Page 72 of 180

Civil Law Q&As (2007-2013)

contract,

an

[email protected]

easement

is

[email protected]

generally

her in lot C if Ava purchase lot E. The

effective between parties, their heirs and

promise was not reduced to writing (Obra

assignees, except in case where the

v. Baldria, 529 SCRA 621 [2007]). Hence,

rights and obligations arising from the

it was not or could not have been

contract are not transmissible by their

registered as to warn buyers of lot C

nature, or by stipulations or by provision

about the existence of the easement on

of law (Unisource Commercial v. Chung,

the property. Not having been annotated

593 SCRA 530 [2009]).

on the TCT to lot C, the buyer acquired lot C free from such right of way granted

(B). Assuming Ava opts to demand a right of

to Ava.

way from any of the owners of Lots A, B, and D, can she do that? Explain. (2.5%) SUGGESTED ANSWER:

Hidden Treasure (2008)

Yes. Ava has the option to demand a

No. VIII. Adam, a building contractor, was

right of way on any of the remaining lots

engaged by Blas to construct a house on a

of Franz more so after Franz sold lot C to

lot which he (Blas) owns. While digging on

Julia. The essential elements of a legal

the lot in order to lay down the foudation of

right of way under Art 649 and 650 of

the house, Adam hit a very hard object. It

the New Civil Code are complied with.

turned out to be the vault of the old Banco de las Islas Filipinas. Using a detonation

ALTERNATIVE ANSWER:

device, Adam was able to open the vault

Yes. Ava has the option to demand a

containing old notes and coins which were

right of way from the other lots. The law

in circulation during the Spanish era. While

provides that whenever a piece of land

the notes and coins are no longer legal

acquired by sale, exchange or partition is

tender, they were valued at P100 million

surrounded

the

because of their historical value and the

vendor, exchanger, or co-owner, he shall

coins silver nickel content. The following

be obliged to grant a right of way

filed legal claims over the notes and coins:

by

other

estates

of

without indemnity (Art 652, NCC). (i). Adam, as finder; ALTERNATIVE ANSWER: (ii). Blas, as owner of the property where No. There was merely a promise to Ava

they were found;

that a right of way shall be granted to

“Never Let The Odds Keep You From Pursuing What You Know In Your Heart You Were Meant To Do.”-Leroy Satchel Paige

Page 73 of 180

Civil Law Q&As (2007-2013)

[email protected]

[email protected]

(iii). Bank of the Philippine Islands, as

present case, Adam, as finder, and Blas,

successor-in-interest of the owner of the

as owner of the land, are entitled to

vault; and

share

50-50

in

the

treasure.

The

government can only claim if it can (iv). The Philippine Government because of

establish that the notes and coins are of

their historical value.

interest to science or the arts, then it must pay just price of the things found,

(A). Who owns the notes and coins? (4%)

to be divided equally between Adam and Blas (Art. 438, Civil Code).

SUGGESTED ANSWER: The notes and coins are no longer owned by the Bank of the Philippine Islands, which has either lost or abandoned the vault and its contents, and it has not taken any effort to search, locate or recover the vault. In any case, since the vault is in actual possession of Adam,

(B). Assuming that either or both Adam and Blas are adjudged as owners, will the notes and coins be deemed part of their absolute community or conjugal partnership of gains with their respective spouses? (2%) SUGGESTED ANSWER:

BPI may attempt, in a judicial action to

If either or both Adam and Blas are

recover, to rebut the presumption of

adjudged as owners, the notes and coins

ownership in favor of Adam and Blas

shall be deemed part of their absolute

(Art. 433, Civil Code). Hidden treasure is

community or conjugal partnership of

any hidden and unknown deposit of

gains with their respective spouses (Art.

money,

117, par 4, FC).

jewelry,

or

other

precious

objects, the lawful ownership of which does not appear. Given the age and importance of the items found, it would be safe to consider the vault, notes and

Mortgage; Public or Private Instrument

coins

(2013)

abandoned

by

BPI

and

its

predecessor (Art. 439, Civil Code). It belongs to the owner of the land on which it is found. When the discovery is made on the property of another, or of the State and by chance, one-half of it shall belong to the finder who is not a trespasser (Art. 438, Civil Code). In the

No.VI. Lito obtained a loan of P1,000,000 from Ferdie, payable within one year. To secure payment, Lito executed a chattel mortgage on a Toyota Avanza and a real estate mortgage on a 200-square meter piece of property.

“Never Let The Odds Keep You From Pursuing What You Know In Your Heart You Were Meant To Do.”-Leroy Satchel Paige

Page 74 of 180

Civil Law Q&As (2007-2013)

[email protected]

[email protected]

(A) Would it be legally significant - from the

exceeds Five Hundred pesos (P500.00)

point of view of validity and enforceability -

must appear in writing, even in private

if the loan and the mortgages were in public

one. However, the requirement is not for

or private instruments? (6%)

validity of the contract, but only for its greater efficacy.

SUGGESTED ANSWER: With regard to the chattel mortgage, Art. From the point of view of validity and

1508,

enforceability,

legal

requires an affidavit of good faith stating

significance if the mortgage was in a

that the chattel mortgage is supposed to

public or private instrument. As for the

stand as security of the loan; thus, for

loan,

significance

the validity of the chattel mortgage, it

except of interest were charged on the

must be in a public document and

loan, in which case, the charging of

recorded

interest must be in writing.

Register in the Register of Deeds. A real

there

there

is

no

would

legal

be

A contract of loan is a real contract and is perfected upon delivery of the object of the obligation (Art 1934, Civil Code). Thus, a contract of loan is valid and enforceable even if it is neither in a private nor in a public document. As a rule, contracts shall be obligatory in whatever form they

may

have been

entered into provided all the essential requisites for their validity are present. With regards to its enforceability, a contact of loan is not among those enumerated under Art. 1403 (2) of the Civil Code, which are covered by the Statute of Frauds. It is important to note that under Art. 1358 of the Civil Code, all the other contracts where the amount involved

the

Chattel

in

the

Mortgage

Chattel

Law,

Mortgage

estate mortgage, under the provisions of Art. 2125 of the Civil Code, requires that in order that a mortgage may be validly constituted the document in which it appears be recorded. If the instrument is not

recorded,

the

mortgage

is

nevertheless valid and binding between the parties. Hence, for validity of both chattel and real estate mortgages, they must appear in a public instrument. But the

purpose

of

enforceability,

it

is

submitted that the form of the contract, whether in a public or private document, would

be

immaterial

(Mobil

Oil

v.

Diocaresa, 29 SCRA 656, 1969). Also, under Art 1358, acts and contracts which have for their object the creation or

transmission

of

real

rights

over

immovable property must be in a public

“Never Let The Odds Keep You From Pursuing What You Know In Your Heart You Were Meant To Do.”-Leroy Satchel Paige

Page 75 of 180

Civil Law Q&As (2007-2013)

[email protected]

[email protected]

document for greater efficacy and a real

right. Possession may be the real right of

estate mortgage is a real right over

possession or jus possessiones or it can

immovable property.

be merely the right to possess or jus possedendi, which are among the basic rights of ownership. If the real right of possession is possession in the concept

Occupation vs. Possession (2007)

of

owner,

limitations,

No.I. Distinguish the following concepts:

but it

subject may

to

ripen

certain into

full

ownership of the thing or property right through

(A). Occupation v. possession. (5%)

acquisitive

prescription

depending on whether it is a case of SUGGESTED ANSWER:

ordinary or extraordinary prescription and whether the property is movable or

Occupation

is

an

original

mode

of

immovable.

acquiring ownership (Art. 712, NCC). Things appropriable by nature which are without an owner, such as animals that

Ownership; Co-Ownership (2009)

are the object of hunting and fishing, hidden

treasure

and

abandoned

No. XI. TRUE or FALSE. Answer TRUE if

movables, are acquired by occupation

the statement is true, or FALSE if the

(Art. 713, NCC). However, ownership of a

statement is false. Explain your answer in

piece of land cannot be acquired by

not more than two (2) sentences.

occupation (Art. 714, NCC). (D). The renunciation by a co-owner of his ALTERNATIVE ANSWER:

undivided share in the co-owned property in lieu of the performance of his obligation

Occupation

is

a

mode

of

acquiring

to contribute to taxes and expenses for the

dominion by the seizure of corporeal

preservation

things which have no owner, with the

constitutes dacion en pago. (1%)

of

the

property

intention of acquiring the ownership thereof.

It

is

an

original

mode

of

SUGGESTED ANSWER:

acquiring ownership upon seizure of a

True, Under the Civil Code, a co-owner

res nullius by the occupant who has the

may renounce his share in the co-owned

intention to become the owner thereof.

property in lieu of paying for his share in

Possession, on the other hand, is the

the

holding of the thing or an enjoyment of a

preservation of the co-owned property.

taxes

and

expenses

for

the

“Never Let The Odds Keep You From Pursuing What You Know In Your Heart You Were Meant To Do.”-Leroy Satchel Paige

Page 76 of 180

Civil Law Q&As (2007-2013)

In

effect,

there

is

[email protected]

dacion

en

pago

[email protected]

SUGGESTED ANSWER:

because the co-owner is discharging his monetary obligation by paying it with

Yes, Cathy can lawfully ask for the

his non-monetary interest in the co-

demolition

owned property. The fact that he is

there are two or more heirs, the whole

giving up his entire interest simply

estate

means that he is accepting the value of

partition, owned in common by such

his interest as equivalent to his share in

heirs, subject to the payment of debts of

the taxes and expenses of preservation.

the deceased (Art. 1078, Civil Code),

of

of

Bobby's

the

house.

decedent,

is,

Where before

Under the rules on co-ownership, "none of Ownership; Co-Ownership (2008)

the

co-owners

shall,

without

the

consent of the others make alterations in the thing owned in common, even

No. VI. Alex died without a will, leaving only

though benefits for all would results

an undeveloped and untitled lot in Tagiug

therefrom." In Cruz v. Catapang, G.R. No.

City. He is survived by his wife and 4

164110, 12 Feb., 2008, the Court held

children. His wife told the children that she

that "alterations include any act of strict

is waiving her share in the property, and

dominion

allowed Bobby, the eldest son who was

construction of a house." In the present

about to get married, to construct his

case, of Alex is the real owner of the

house on ¼ of the lot, without however

undeveloped and untitled lot in Taguig,

obtaining the consent of his siblings. After

co-ownership is created among his wife

settlement of Alex's estate and partition

and four children over said property

among the heirs, it was discovered that

upon his death. Since the construction

Bobby's house was constructed on the

of the house by Bobby was done without

portion allocated to his sister, Cathy asked

obtaining the consent of his siblings, the

Bobby to demolish his house and vacate

alteration effected is illegal. Bobby is

the portion alloted to her. In leiu of

considered to be in bad faith and as a

demolition, Bobby offered to purchase from

sanction for his conduct, he can be

Cathy the lot portion on which his house

compelled

was constructed. At that time, the house

remove

constructed was valued at P350.000.

expense.

(A). Can Cathy lawfully ask for demolition of

(B). Can Bobby legally insist on purchasing

Bobby's house? (3%)

the land? (2%)

or

by

the

ownership

Cathy

to

structure

such

demolish at

his

as

or own

“Never Let The Odds Keep You From Pursuing What You Know In Your Heart You Were Meant To Do.”-Leroy Satchel Paige

Page 77 of 180

Civil Law Q&As (2007-2013)

[email protected]

SUGGESTED ANSWER:

[email protected]

nature and object to remain at a fixed place on a river, lake or coast." Since the

No.

Bobby

cannot

legally

insist

on

floating

platform

is

a

petroleum

purchasing the land. Being in bad faith,

operation

he has no option to pay for the price of

remain permanently where it is situated,

the lot (Art. 450, Civil Code).

even if it is tethered to a ship which is

facility,

it

is

intended

to

anchored to the seabed. ALTERNATIVE ANSWER: Property; Movable or Immovable (2007) The platform is a movable property No.II. Manila Petroleum Co. owned and

because it is attached to a movable

operated a petroleum operation facility off

property,

the coast of Manila. The facility was located

merely anchored to the seabed. The fact

on a floating platform made of wood and

that the vessel is merely anchored to the

metal,

sea

upon

which

was

permanently

bed

i.e.

only

the

vessel

shows

which

that

it

is

was

not

attached the heavy equipment for the

intended to remain at a fixed place;

petroleum operations and living quarters of

hence, it remains a movable property. If

the crew. The floating platform likewise

the intention was to make the platform

contained a garden area, where trees,

stay permanent where it was moored, it

plants

The

would not have been simply tethered to

platform was tethered to a ship, the MV

a vessel but itself anchored to the

101, which was anchored to the seabed.

seabed.

Please briefly give the reason for your

(B). Are the equipment and living quarters

answers. (10%)

movable or immovable property?

(A).Is the platform movable or immovable

SUGGESTED ANSWER:

and

flowers

were

planted.

property? The thing and living quarters of the crew SUGGESTED ANSWER:

are immovable property under Art. 415

The platform is an immovable property under Art. 415 (9) NCC, which provides that

"docks

and

structures

which,

though floating, are intended by their

(3)

NCC, classifies as an immovable

"everything attached to an immovable in a fixed manner, in such a way that it cannot be separated therefrom without breaking the material or deterioration of

“Never Let The Odds Keep You From Pursuing What You Know In Your Heart You Were Meant To Do.”-Leroy Satchel Paige

Page 78 of 180

Civil Law Q&As (2007-2013)

[email protected]

[email protected]

the object." Both the equipment and the

The trees, plants and flowers planted in

living quarters are permanently attached

the garden area of the platform are

to

an

immovable property under Art. 415 (2)

immovable. The equipment can also be

NCC which classifies as an immovable

classified

property

the

platform as

an

which

is

immovable

also

property

"trees,

plants

and

growing

under Art. 415 (5) NCC because such

fruits, while they are attached to the

equipment are "machinery, receptacles,

land or form an integral part of an

instruments or implements intended by

immovable,

the

facility.

owner

of

the

tenement

for

an

the

petroleum

operation

industry or works which may be carried on in a building or on a piece of land and which tend directly to meet the needs of the industry or works." It is logically assumed that the petroleum industry may be carried on in a building or on a piece

of

land

and

the

platform

is

analogous to a building.

ALTERNATIVE ANSWER: The trees, plants and flowers planted in the garden area of the platform are movable property because they are not permanently attached t the land and do not

form

immovable.

an

integral

The

part

of

an

is

not

an

platform

immovable property for the same reason

ALTERNATIVE ANSWER:

already given in the Alternative Answer The equipment and living quarters of the

to Item (a) above.

crew are movable properties since they are attached to a platform which is also

Land Titles and Deeds

a movable property, because it is simply attached to a

vessel is likewise a

movable property since it was merely

Acquisition

of

Lands;

Sale

of

Real

Property to an Alien (2009)

anchored on the seabed only shows that it is not intended to remain at a fixed

No.XIX. In 1972, Luciano de la Cruz sold to

place;

Chua Chung Chun, a Chinese citizen, a

hence,

it

remains

a

movable

property.

parcel of land in Binondo. Chua died in 1990, leaving behind his wife and three

(C). Are the trees, plants and flowers

children,

immovable or movable property?

naturalized Filipino citizen. Six years after Chua’s

SUGGESTED ANSWER:

one death,

of

whom,

the

heirs

Julian,

is

executed

a an

extrajudicial settlement of estate, and the parcel of land was allocated to Julian. In

“Never Let The Odds Keep You From Pursuing What You Know In Your Heart You Were Meant To Do.”-Leroy Satchel Paige

Page 79 of 180

Civil Law Q&As (2007-2013)

[email protected]

2007, Luciano filed suit to recover the land

[email protected]

Non-Registrable Properties (2007)

he sold to Chua, alleging that the sale was void

because

it

contravened

the

Constitution which prohibits the sale of private lands to aliens. Julian moved to dismiss

the

suit

on

grounds

of pari

delicto, laches and acquisitive prescription. Decide the case with reasons. (4%)

No.IV.

What

properties

are

not

registrable? (5%) Supply this information. SUGGESTED ANSWER: The

SUGGESTED ANSWER:

(B).

following

properties

are

not

registrable:

The case must be dismissed. Julian, who is a naturialized Filipino citizen and to

(1.) Properties of the Public dominion;

whom the property was allocated in a n

(2.) Properties for public use or public

extra-judicial partition of the estate, is

service;

now the owner of the property. The defect in ownership of the property of

(3.)

Inalienable

Julian’s alien father has already been

domain;

lands

of

the

public

cured by its transfer to Julian. It has been validated by the transfer of the

(4.) Military installations, civil and quasi-

property to a Filipino citizen. Hence,

public lands; and

there

is

no

more

violation

of

the

Constitution because the subject real property is now owned by a Filipino citizen (Halili v. CA, 287 SCRA 465, [1998]). Further, after the lapse of 35

(5.) All lands not classified as alienable and disposable. ALTERNATIVE ANSWER:

year, laches has set in and the motion to

(1).

dismiss may be granted, for the failure of

intended

Luciano to question the ownership of

canals, rivers, torrents, ports and bridges

Chua before its transfer of ownership to

constructed by the State, banks, shores,

Julian.

roadsteads, and the like, are incapable of

Properties

private

for

of

public

public

use,

appropriation,

dominium like

much

roads,

less

registration (Art. 420 NCC). This includes public markets, public plazas, municipal streets

and

public

buildings

“Never Let The Odds Keep You From Pursuing What You Know In Your Heart You Were Meant To Do.”-Leroy Satchel Paige

Page 80 of 180

Civil Law Q&As (2007-2013)

[email protected]

[email protected]

(Municipality of Antipolo v. Zapanta, 133

(7.) Lands reclaimed by the government

SCRA 820, 1986; Martinez v. CA, 56

from the sea, lakes, or other bodies of

SCRA 647, 1974; Navera v. Quicho, 5

water are disposed or acquisible only by

SCRA 454, 1962).

lease

and

not

otherwise,

under

the

Public Land Act. (2.) Lands proclaimed or classified as forest, timberlands, mineral lands and national parks. Under Sec 2, Art XII, Constitution of the Philippines, these

Prescription;

Acquisitive

Prescription

lands are inalienable.

(2008)

(3.) Lands that are reserved by law or

No. VII. Anthony bought a piece of untitled

Presidential proclamation for military,

agricultural land from Bert. Bert, in turn,

civic or quasi-public purpose, Under Sec

acquired the property by forging carlo's

88, Chapter XII of the Public Land Act,

signature in a deed of sale over the

such lands shall be inalienable and shall

property. Carlo had been in possession of

not be subject to occupation, entry, sale,

the property for 8 years, declared it for tax

lease or other disposition.

purposes, and religiously paid all taxes due on the property. Anthony is not aware of

(4.) In general, all lands of the public

the defect in Bert's title, but has been in

domain that has not been classified as

actual physical possession of the property

alienable and disposable under the Public

from the time he bought it from Bert, who

Land Act.

had

never

been

in

possession

of

the

property for one year. (5.) Lands that form part of the seabed, riverbed or lakebed. These lands are not

(A). Can Anthony acquire ownership of the

susceptible to private appropriation.

property by acquisitive prescription? How many more years does he have possess it to

(6.) Foreshore lands is that strip of land

acquire ownership? (2%)

that lies between the high and low water marks

and

alternately

wet

and

dry

SUGGESTED ANSWER:

according to the flow of the tide belong to the public domain, and can only be

Yes, Anthony can acquire ownership of

acquired by lease if not needed by the

the

government for public or quasi-public

prescription.

purposes.

Anthony is a buyer/possessor in good

property

through In

the

acquisitive

present

case,

“Never Let The Odds Keep You From Pursuing What You Know In Your Heart You Were Meant To Do.”-Leroy Satchel Paige

Page 81 of 180

Civil Law Q&As (2007-2013)

[email protected]

[email protected]

faith because he was not aware of the

shall have a right to a part of the

defect in Bert's title (Art. 526, Civil

expenses of cultivation, and to a part of

Code). As such, Anthony can acquire

the net harvest of the standing crops,

ownership and other real rights over

both in proportion to the time of the

immovable

possession (Art 545, Civil Code).

property

through

open,

continuous possession of 10 years (Art. 1134, Civil Code). Anthony needs nine (9) more years of possession, in addition to his one (1) year of possession in good faith. (B).If Carlo is able to legally recover his property,

can

he

require

Anthony

to

account for all the fruits he has harvested from the property while in possession? (2%)

If Carlo is able to legally recover his property, he cannot require Anthony to account for all the fruits harvested from the property. Anthony is entitled to the fruits harvested in good faith before his possession was legally interrupted (Art. 544, Civil Code).

property when Carlo recovers possession, can Carlo appropriate them? (2%)

issuance of a writ of possession over a foreclosed real property prescribe in five (5) years? (5%)

Yes, it prescribes in five (5) years. If the real property mortgaged is judicially foreclosed,

the

action

for

judicial

foreclosure should be filed within a period of ten (10) years. The request for issuance of a writ of possession should be filed upon motion of the winning

appropriate

judgment of foreclosure. The writ of possession is an order commanding the sheriff to place a person named therein in possession of real property (BPI v. Icot. G.R. No. 168081, Oct 12, 2009).

SUGGESTED ANSWER: can

No.IX.a) Does the right to request for the

bidder within five (5) years after the

(C).If there are standing crops on the

Carlos

Property Mortgage (2012)

SUGGESTED ANSWER:

SUGGESTED ANSWER:

Yes,

Prescription; Judicially Foreclosed Real

only

a

portion of the standing crops on the property once he recovers possession. Anthony being a possessor in good faith,

“Never Let The Odds Keep You From Pursuing What You Know In Your Heart You Were Meant To Do.”-Leroy Satchel Paige

Page 82 of 180

Civil Law Q&As (2007-2013)

Purchaser

in

[email protected]

Good

Faith;

Mortgaged

[email protected]

of Deeds under Act. 3344 and obtained a

Property (2008)

tax declaration in its name.

No. XIX. Juliet offered to sell her house and

(A). Was Dehlma a purchaser in good faith?

lot, together with all the furniture and

(2%)

appliances

therein

to

Dehlma.

Before

agreeing to purchase the property, Dehlma went to the Register of Deeds to verify Juliet's title. She discovered that while the property was registered in Juliet's name under

the

amended

Land

by

the

Registration Property

Act,

as

Registration

Decree, it property, Dehlma told Juliet to redeem the property from Elaine, and gave her an advance payment to be used for purposes of realesing the mortgage on the property. When the mortgage was released, Juliet executed a Deed of Absolute Sale over the property which was duly registered with the Registry of Deeds, and a new TCT was issued in Dehlma's name. Dehlma immediately took possession over the house and

lot

and

the

movables

therein.

Thereafter, Dehlma went to theAssessor's Office to get a new tax declaration under

SUGGESTED ANSWER: Yes, Dehlma is a purchaser in good faith. In

the

present

case,

before

Dehlma

bought the property, she went to the Register of Deeds to verify Juliet's title. When she discovered that the property was mortgaged to Elaine, she gave an advance payment so that Juliet could release the mortgage. It was only after the mortgage was released and free from the claims of other persons that Dehlma bought the property. Thus, Dehlma is a purchaser in good faith (Mathay v. CA, G.R. No. 115788, 17 Sept, 1998). (B). Who as between Dehlma and XYZ Bank has a better right to the house and lot? (2%) SUGGESTED ANSWER:

her name. She was surprised to find out that the property was already declared for

Between Dehlma and XYZ Bank, Dehlma

tax purposes in the name of XYZ Bank

has a better right to the house and lot.

which had foreclosed the mortgage on the

After the release of the mortgage, the

property before it was sold to her. XYZ

Deed of Absolute Sale was registered and

Bank

the

a new title was issued in Dehlma's name.

foreclosure sale of the property. At that

Act 3344 is applicable exclusively to

time, the property was still unregistered but

instruments resulting from agreement of

XYZ Bank registered the Sheriff's Deed of

parties thereto and does not apply to

Conveyance in the day book of the Register

deeds

was

also

the

purchaser

in

of

a

sheriff

conveying

to

a

“Never Let The Odds Keep You From Pursuing What You Know In Your Heart You Were Meant To Do.”-Leroy Satchel Paige

Page 83 of 180

Civil Law Q&As (2007-2013)

[email protected]

[email protected]

purchaser unregistered lands sold to him

land registration and acquisition of title to

under execution (Williams v. Suñer, 49

land.

Phil. ,534).

following items:

(C). Who owns the movables inside the

(A). What is the governing law? (5%)

The

manual

should

include

the

house? (2%) SUGGESTED ANSWER: SUGGESTED ANSWER: The Dehlma

owns

is

the

Land

Registration Act as amended by Property

when she acquired the house and lot

Registration Decree (Act 496 as amended

from

by PD 1529).

all

movables

law

because

Juliet,

the

governing

the

furniture

and

appliances therein were included in the sale. As owner of the real property,

[Note: It is respectfully recommended

Dehlma also owns the movables found

that full credit be given to examinees

therein (Art. 542, Civil Code).

who did not give the exact title or number of the law but merely stated a description of the law.]

Registration; Governing Law (2007)

ALTERNATIVE ANSWER:

No.IV.

Property

In general, the governing law relating to

development

registration and acquisition of title to

company engaged in developing and selling

land is Act 496 of 1902 as amended by

subdivisions,

PD 1529, otherwise known as Property

Bedrock

Development

Land

Corp.

is

& a

condominium

units

and

industrial estates. In order to replenish its

Registration Decree of June 11, 1978.

inventories, it embarked on an aggressive land banking program. It employed "scouts"

(1.)

who roam all over the Philippines to look for

registration

and conduct investigations on prospective

Torrens System by voluntary ordinary

sites

judicial proceedings.

for

acquisition

and

development,

whether developed, semi-developed or raw land. The management of Bedrock asks you as the company counsel to prepare a manual

containing

a

summary

of

the

Chapter of

III-I land

governs title

original

under

the

(2.) Chapter II-II governs compulsory registration of lands through cadastral proceedings.

pertinent laws and regulations relating to

“Never Let The Odds Keep You From Pursuing What You Know In Your Heart You Were Meant To Do.”-Leroy Satchel Paige

Page 84 of 180

Civil Law Q&As (2007-2013)

[email protected]

[email protected]

(3.) Section 103 governs registration of

(c) Replacement of lost or destroyed

homestead, sales, free patent under CA

owner's duplicate certificate of title.

No. 141, as amended, otherwise known (9.)

as the Public Land Act.

R.A.

No.

reconstitution (4.) Section 104 governs registration of certificates

of

land

26 of

governs lost

or

judicial destroyed

originals of the certificate of title.

transfers,

emancipation patents and Certificates of

(10.)

Land Ownership Award (CLOA) under

administrative reconstitution of lost or

Comprehensive Land Reform Law.

destroyed original certificates of title.

(5.) Chapter V governs the registration of

(11.)

land dealings on registered land like

registration

conveyances,

unregistered private lands.

transfers,

mortgages,

leases, powers of attorney, trusts and similar contracts inter vivos.

R.A.

No.

Section of

6732

113

governs

governs

instruments

the

affecting

(12.) Section 117 governs "consultas," where the Register of Deeds refuses to

(6.) Chapter V-II governs the registration

register a deed or when he is in doubt as

of involuntary dealings on registered

to what action to take on an instrument

land like attachments, adverse claims,

presented for registration.

enforcement of liens on registered land, notices of lis pendens. (7.) Chapter VI governs the registration of judgments, orders and partitions, condemnation in eminent domain proceedings, judicial and extra-judicial settlement of estates. (8.) Sections 107, 108 and 109 govern petitions

and

registration

actions

after

original

like:

(a).Compulsory

surrender of withheld owner's duplicate certificate of title; (b)

Amendment

certificate of title;

Registration;

Party

Who

First

took

Possession (2013) No.IX.Rica petitioned for the annulment of her

ten-year

old

marriage

to

Richard.

Richard hired Atty. Cruz to represent him in the proceedings. In payment for Atty. Cruz's acceptance and legal fees, Richard conveyed to Atty. Cruz a parcel of land in Taguig that he recently purchased with his

and

alteration

of

lotto winnings. The transfer documents were

duly

signed

and

Atty.

Cruz

“Never Let The Odds Keep You From Pursuing What You Know In Your Heart You Were Meant To Do.”-Leroy Satchel Paige

Page 85 of 180

Civil Law Q&As (2007-2013)

[email protected]

[email protected]

immediately took possession by fencing off

the

liquidation

of

the

absolute

the property's entire perimeter.

community or conjugal partnership of the spouses as the case may be (Art. 50

Desperately needing money to pay for his

in relation to Art 43 of the Family Code).

mounting legal fees and his other needs

Richard purchased the land with his

and despite the transfer to Atty. Cruz,

lotto winnings during the pendency of

Richard offered the same parcel of land for

the suit for annulment and on the

sale to the spouses Garcia. After inspection

assumption

of the land, the spouses considered it a

governed by

good investment and purchased it from

community

Richard. Immediately after the sale, the

winnings from gambling or betting will

spouses

form part thereof. Also, since the land is

Garcia

commenced

the

that

the

parties

the regime or

conjugal

of

are

absolute

partnership,

construction of a three-story building over

part

the land, but they were prevented from

conjugal

doing this by Atty. Cruz who claimed he

Rica, it may not be sold or alienated

has a better right in light of the prior

without the consent of the latter and

conveyance in his favor.

any disposition or encumbrance of the

of

property

Is Atty. Cruz's claim correct? (8%)

the

absolute

partnership

of

the

community of

Richard

community

or

or and

the

conjugal property without the consent of the other spouse is void (Art 96 and Art

SUGGESTED ANSWER:

124, Family Code).

No. Atty. Cruz is not correct. At first glance, it may appear that Atty. Cruz is the one who has the better right because he first took possession of the property.

Registration; Requisites; Proof (2013)

However, a lawyer is prohibited under

No.X. Manuel was born on 12 March 1940

Art

in a 1 000-square meter property where he

1491

of

the

Civil

Code

from

acquiring the property and rights which

grew

may be the object of any litigation in

cultivate the land. Michael has lived on the

which they may take part by virtue of

property since the land was opened for

their profession. While the suit is for

settlement

annulment of marriage and it may be

Commonwealth government in 1935, but

urged that the land itself is not the

for some reason never secured any title to

object of the litigation, the annulment of

the property other than a tax declaration in

marriage, if granted, will carry with it

his name. He has held the property through

up

helping

at

his

about

father,

the

time

Michael,

of

the

“Never Let The Odds Keep You From Pursuing What You Know In Your Heart You Were Meant To Do.”-Leroy Satchel Paige

Page 86 of 180

Civil Law Q&As (2007-2013)

[email protected]

[email protected]

the years in the concept of an owner and

ownership

since

his stay was uncontested by others. He has

earlier. However, it is only necessary

also conscientiously and continuously paid

that the land is already declared A & D

the realty taxes on the land.

land “at the time for application for registration

is

June

filed”

12,

1945,

(Malabanan

or

v.

Michael died in 2000 and Manuel - as

Republic, G.R. No. 180067, June 30,

Michael’s only son and heir -now wants to

2009).

secure and register title to the land in his own name. He consults you for legal advice

Manuel could also invoke Sec 14 (2) of

as he wants to perfect his title to the land

the

and secure its registration in his name.

registration through ordinary acquisitive

same

Decree,

which

allows

prescription for thirty years, provided, (A) What are the laws that you need to

however, that the land is “patrimonial”

consider in advising Manuel on how he can

in character, i.e. already declared by the

perfect his title and register the land in his

government (a) as A & D, and (b) no

name? Explain the relevance of these laws

longer needed for public use or public

to your projected course of action. (4%)

service (Malabanan, supra).

SUGGESTED ANSWER:

Manuel could also file an application for

(Note:

With

all

due

respect,

it

is

recommended that the examiner accept and give full credit to any of the answers given in each of the following paragraphs.) I

would

advice

Manuel

to

file

an

of Pres. Decree No. 1529, or the Property Registration Decree (PRD), specifically Sec14 (1) which requires (a) that the land applied for forms part of the alienable and disposable (A & D) portion of the public domain, and (b) that the applicant been

notorious thereof

in

open,

possession under

bona

continuous and fide

incomplete

and

occupation claim

of

of title’

imperfect through

or

“judicial

legalization” under Sec. 48 (b) of CA no. 141, or the Public Land Act (PLA). But, as

application for registration under Sec 14

has

“confirmation

held

in

substantial

Malabanan, difference

there between

is

no this

provision and Sec 14 (1) of the PRD. Both refer to agricultural lands already classified as alienable and disposable at the time the application is filed, and require possession and occupation since June 12, 1945. The only difference is that under the PRD, there already exists a title which is to be confirmed, whereas under the PLA, the presumption is that land is still public land (Republic v.

“Never Let The Odds Keep You From Pursuing What You Know In Your Heart You Were Meant To Do.”-Leroy Satchel Paige

Page 87 of 180

Civil Law Q&As (2007-2013)

[email protected]

[email protected]

Aquino, G.R. No. L-33983, January 27,

Manuel has a the burden to overcome

1983).

the presumption of State ownership by “well-nigh

incontrovertible”

evidence

Manuel may also invoke “vested rights’

(Ong

acquired under Rep. Act. No. 1942, dated

March 12, 2008). Accordingly, he must

June 2, 1957, which amended Sec. 48 (b)

show that ht eland is already classified

of the PLA by providing for a prescriptive

as A & D “at the time the application for

period

registration is filed” and that he has

of

thirty

years

for

judicial

v.

Republic,

G.R.

No.

175746,

confirmation of imperfect title. It must

been

only be demonstrated that possession

thereof” in the manner required by law

and occupation commenced on January

since June 12, 1945, or earlier.

in

“possession

and

occupation

24, 1947 and the 30-year period was completed prior to the effectivity of PD

Manuel may tack his possession to that

No. 1073 on January 25, 1977. PD No.

of his predecessor-in-interest (Michael)

1073

and

by the testimony of disinterested and

1945

knowledgeable eyewitnesses. Overt acts

(Republic v. Espinosa, G.R. No. 171514,

of possession may consist in introducing

July 18, 2012).

valuable improvements like fencing the

now

occupation

requires since

possession June

12,

land, constructing a residential house Another alternative is for Manuel to

thereon,

secure

planting fruit bearing trees, declaring

title

through

administrative

cultivating

the

land

and

proceedings under the homestead or free

the

patent provisions of the PLA. The title

paying realty taxes, all of which are

issued has the same efficacy and validity

corroborative proof of possession.

as

a

title

issued

through

land

for

taxation

purposes

and

judicial

proceedings, but with the limitations

To identify the land, he must submit the

that the land cannot be sold or disposed

tracing cloth plan or a duly-certified

of within five years from the issuance of

blueprint

patent

(Director of Lands v. Reyes, G.R. No. L-

(Sec.

118,

CA

No.

141,

as

amended).

or

whiteprint

copy

thereof

27594, November 28, 1975; Director of Lands v. CA and Iglesia ni Cristo, G.R.

(B) What do you have to prove to secure Manuel's

objectives

and

documentation are necessary? (4%)

No. L-56613, March 14, 1988).

what To show the classification of the land as A

SUGGESTED ANSWER:

&

D,

the

application

must

be

accompanied by (1) a CENRO or PENRO

“Never Let The Odds Keep You From Pursuing What You Know In Your Heart You Were Meant To Do.”-Leroy Satchel Paige

Page 88 of 180

Civil Law Q&As (2007-2013)

[email protected]

[email protected]

certification; and (2) a certified true copy

of

approved

the by

original the

classification

DENR

Secretary

I will institute the following actions against Atty. Tan:

(Republic v. Bantigue, G.R.No. 162322, March

14,

2012).

A

presidential

or

legislative act may also be considered.

(a). A civil action for damage for the fraudulent transfer of the title in his name and to recover the value of the property;

Remedies; Fraud; Rights of Innocent Purchaser (2009) No.IX. Before migrating to Canada in 1992, the spouses Teodoro and Anita entrusted all their legal papers and documents to their nephew, Atty. Tan. Taking advantage of the situation, Atty. Tan forged a deed of sale, making it appear that he had bought the couple’s property in Quezon City. In 2000, he succeeded in obtaining a TCT over the property in his name. Subsequently, Atty. Tan sold the same property to Luis, who built an auto repair shop on the property. In 2004, Luis registered the deed of conveyance, and title over the property was transferred in his name.

(b).

An

action

against

the

National

Treasurer for compensation from the State Assurance Fund which is set aside by law to pay those who lose their land suffer damages as a consequence of the operation of the Torrens system; (c). A criminal action for forgery or falsification of public document; (d).

A

complaint

with

the

Supreme

Court/Integrated Bar of the Philippines to

disbar

or

suspend

him

or

other

disciplinary action for violation or the Code of Professional Ethics. Any action against Luis will not prosper

In 2006, the spouses Teodoro and Anita

because he is an innocent purchaser for

came to the Philippines for a visit and

value. The Title to the land he bought

discovered what had happened to their

was already in the name of the person

property. They immediately hire you as

who sold the property to him, and there

lawyer. What action or actions will you

is nothing on the title which will make

institute in order to vindicate their rights?

him suspect about the fraud committed

Explain fully. (4%)

by Atty. Tan.

SUGGESTED ANSWER:

“Never Let The Odds Keep You From Pursuing What You Know In Your Heart You Were Meant To Do.”-Leroy Satchel Paige

Page 89 of 180

Civil Law Q&As (2007-2013)

[email protected]

Contracts

[email protected]

Rescission of Contract; Fortuitous Event (2008)

Contract to Sell vs. Conditional Contract of Sale (2012)

No.XVIII. AB Corp. entered into a contract with XY Corp. whereby the former agreed to

No.X.a) A contract to sell is the same as a

construct

the

research

and

laboratory

conditional contract of sale. Do you agree?

facilities of the latter. Under the terms of

Explain your answer. (5%)

the contract, AB Corp. agreed to complete the facility in 18 months, at the total

SUGGESTED ANSWER:

contract price of P10 million. XY Corp. paid

No. A contract to sell is a species of conditional sale. The contract to sell does not sell a thing or property; it sells the right to buy property. A conditional sale is a sale subject to the happening or performance of a condition, such as payment of the full purchase price, or the performance of other prestation to give, to do or not to do. Compliance with the condition automatically gives the right to the vendee to demand the delivery of the object of the sale. In a contract

to

sell,

however,

the

compliance with the condition does not automatically sell the property to the vendee. It merely gives the vendee the

50% of the total contract price, the balance to be paid upon completion of the work. The work stated immediately, but AB Corp. later experienced work slippage because of labor unrest

in

his

company.

AB

Corp.'s

employees claimed that they are not being paid on time; hence, the work slowdown. As of the 17th month, work was only 45% completed. AB Corp. asked for extension of time, claiming that its labor problems is a case of fortuitous event, but this was denied by XY Corp. When it became certain that the contruction could not be finished on time, XY Corp. sent written notice cancelling the contract, and requiring AB Corp. to immediately vacate the premises.

right to compel the vendor to execute

(A). Can the labor unrest be considered a

the deed of absolute sale.

fortuitous event? (1%) SUGGESTED ANSWER: No.

The

labor

unrest

cannot

be

considered a fortuitous event under Art. 1174 of the Civil Code. A fortuitous event should occur independent of the “Never Let The Odds Keep You From Pursuing What You Know In Your Heart You Were Meant To Do.”-Leroy Satchel Paige

Page 90 of 180

Civil Law Q&As (2007-2013)

will

of

the

debtor

[email protected]

or

without

his

participation or aggravation (Paras, Civil

[email protected]

statement is false. Explain your answer in not more than two (2) sentences.

Code Annotated, vol. IV, 2000 ed., p 159). As mentioned in the facts, labor

(A). A clause in an arbitration contract

unrest of the employees was caused by

granting one of the parties the power to

AB Corp.'s failure to pay its employees

choose more arbitrators than the other

on time.

renders the arbitration contract void. (1%)

(B).

Can

XY

Corp.

unilaterrally

and

SUGGESTED ANSWER: True. The Civil Code provides that “Any

immediately cancel the contract? (2%)

clause giving one of the parties power to SUGGESTED ANSWER:

choose more arbitrators than the other is void and of no effect” (Art 2045, NCC).

No, XY Corp. cannot unilaterally and

Obligations

immediately cancel the contract. In the absence of any stipulation for automatic rescission, rescission must be judicial (Art. 1191, Civil Code). (C).

Must

AB

Corp.

Extinguishment; Compensation (2009) No.XV. Sarah had a deposit in a savings

return

the

50%

downpayment? (2%) SUGGESTED ANSWER:

account with Filipino Universal Bank in the amount

of

five

million

pesos

(P5,000,000.00). To buy a new car, she obtained a loan from the same bank in the amount of P1,200,000.00, payable in twelve

AB Corp. need not return the 50% down

monthly installments. Sarah issued in favor

payment because 45% of the work was

of the bank post-dated checks, each in the

already completed, otherwise, XY Corp.

amount of P100,000.00, to cover the twelve

would be unjustly enriching itself at the

monthly installment payments. On the

expense of AB Corp.

third,

fourth

and

fifth

months,

the

corresponding checks bounced. The Stipulation; Arbitration Clause (2009)

bank

then

declared

the

whole

obligation due, and proceeded to deduct the amount

of

one

million

pesos

No. XI. TRUE or FALSE. Answer TRUE if

(P1,000,000.00) from Sarah’s deposit after

the statement is true, or FALSE if the

notice to her that this is a form of

“Never Let The Odds Keep You From Pursuing What You Know In Your Heart You Were Meant To Do.”-Leroy Satchel Paige

Page 91 of 180

Civil Law Q&As (2007-2013)

[email protected]

compensation allowed by law. Is the bank

[email protected]

Extinguishment; Compensation (2008)

correct? Explain. (4%) No. XV. Eduardo was granted a loan by SUGGESTED ANSWER:

XYZ Bank for the purpose of improving a

No, the bank is not correct. While the

building

Bank is correct about the applicability of

Eduardo, executed the promissory note

compensation, it was not correct as to

("PN") in favor of the bank, with his friend

the amount compensated.

Recardo as co-signatory. In the PN, they both

which

XYZ

acknowledged

leased

that

from

him.

they

are

A bank deposit is a contract of loan,

"individually and collectively" liable and

where the depositor is the creditor and

waived the need for prior demand. To

the bank the debtor. Since Sarah is also

secure the PN, Recardo executed a real

the debtor of the bank with respect to

estate mortgage on his own property. When

the loan, both are mutually principal

Eduardo defaulted on the PN, XYZ stopped

debtors and creditors of each other. Both

payment of rentals on the building on the

obligation

and

ground that legal compensation had set in.

liquidated but only up to the extent of

Since there was still a balance due on the

P300,000.00 (covering the unpaid third,

PN

fourth and fifth monthly installments).

foreclosed the real estate mortgage over

The entire one million was not yet due

Recardo's property. Recardo opposed the

because the loan has no acceleration

foreclosure on the ground that he is only a

clause in case of default. And since there

co-signatory; that no demand was made

is

upon him for payment, and assuming he is

no

are

due,

retention

commenced

or

the

rentals,

XYZ

the

half the balance of the loan. Further,

debtor, then all the requisites of legal

Recardo said that when the bank invoked

compensation are present but only up to

compensation between the reantals and the

the amount of P300,000.00. The bank,

amount of the loan, it amounted to a new

therefore, may deduct P300,000.00 from

contract or novation, and had the effect of

Sarah’s

extinguishing the security since he did not

compensation.

in

due

deposit

person

applying

liable, his liability should not go beyond

bank

third

controversy

after

and

communicated

by

demandable

time

by

to

way

of

give his consent (as owner of the property under the real estate mortgage) thereto. (A). Can XYZ Bank validly assert legal compensation? (2%)

“Never Let The Odds Keep You From Pursuing What You Know In Your Heart You Were Meant To Do.”-Leroy Satchel Paige

Page 92 of 180

Civil Law Q&As (2007-2013)

[email protected]

SUGGESTED ANSWER:

[email protected]

ground that legal compensation had set in. Since there was still a balance due on the

Yes, XYZ Bank can validly assert legal

PN

compensation. In the present case, all of

foreclosed the real estate mortgage over

the elements of legal compensation are

Recardo's property. Recardo opposed the

present: (1) XYZ Bank is the creditor of

foreclosure on the ground that he is only a

Eduardo while Eduardo is the lessor of

co-signatory; that no demand was made

XYZ Bank; (2) both debts consist in a

upon him for payment, and assuming he is

sum of money, or if the things due are

liable, his liability should not go beyond

consumable, they be of the same kind,

half the balance of the loan. Further,

and also of the same quality if the latter

Recardo said that when the bank invoked

has been stated; (3) the two debts be

compensation between the reantals and the

due;

and

amount of the loan, it amounted to a new

demandable, and (5) over neither of them

contract or novation, and had the effect of

there be any retention or controversy,

extinguishing the security since he did not

commenced

give his consent (as owner of the property

(4)

they

by

be

liquidated

third

persons

and

communicated in due time to the debtor

after

applying

the

rentals,

XYZ

under the real estate mortgage) thereto.

(Art. 1279, Civil Code). (C). Does Recardo have basis under the Civil Code for claiming that the original contract was novated? (2%)

Extinguishment; Novation (2008) No. XV. Eduardo was granted a loan by

SUGGESTED ANSWER:

XYZ Bank for the purpose of improving a

No. Recardo has no basis for claiming

building

him.

novation of the original contract when

Eduardo, executed the promissory note

the bank invoked compensation because

("PN") in favor of the bank, with his friend

there was simply partial compensation

Recardo as co-signatory. In the PN, they

(Art. 1290, Civil Code) and this would

both

not bar the bank from recovering the

which

XYZ

acknowledged

leased

that

from

they

are

"individually and collectively" liable and

remaining balance of the obligation.

waived the need for prior demand. To secure the PN, Recardo executed a real

ALTERNATIVE ANSWER:

estate mortgage on his own property. When Eduardo defaulted on the PN, XYZ stopped

No. In order that an obligation may be

payment of rentals on the building on the

extinguished by another, it is imperative

“Never Let The Odds Keep You From Pursuing What You Know In Your Heart You Were Meant To Do.”-Leroy Satchel Paige

Page 93 of 180

Civil Law Q&As (2007-2013)

[email protected]

[email protected]

that it be so declared in unequivocal

or creditors (Philippine Airlines v. CA

terms,

and

or

that

the

old

and

new

Amelia

Tan,

G.R.

No.

L-49188,

obligations be on every point compatible

1990). Mere delivery of checks does not

with

discharge

each

other.

Novation

is

never

presumed (Art. 1292, Civil Code).

the

obligation

under

a

judgment. A check shall produce the effect of payment only when they have been cashed or where through the fault

Extinguishment;

Payment

of

Check

(2013) No.VI. Lito obtained a loan of P1,000,000 from Ferdie, payable within one year. To secure payment, Lito executed a chattel mortgage on a Toyota Avanza and a real estate mortgage on a 200-square meter piece of property. (B) Lito's failure to pay led to the extrajudicial foreclosure of the mortgaged real property. Within a year from foreclosure, Lito tendered a manager's check to Ferdie to redeem the property. Ferdie refused to accept payment on the ground that he wanted payment in cash: the check does not qualify as legal tender and does not include the interest payment. Is Ferdie's refusal justified? (4%) SUGGESTED ANSWER:

of the creditor they have been impaired (Art 1249, Civil Code). However, it is not necessary that the right of redemption be exercised by delivery of legal tender. A check may be used

for

the

exercise

of

right

of

redemption, the same being a right and not an obligation. The tender of a check is sufficient to compel redemption but is not in itself a payment that relieves the redemptioner from his liability to pay the redemption price (Biana v. Gimenez, G.R. No. 132768, Sept 9, 2005, citing Fortunado v. CA). Redemption within the period allowed by law is not a matter of intent but a question of payment or valid tender of full redemption prices within the said period. Whether redemption is being made under Art. 3135 or under the General Banking Law, the mortgagor or

A check, whether a manager’s check or

his

assignee is required to tender

an ordinary check is not legal tender,

payment to make said redemption valid

and an offer of a check in payment of a

(Heirs of Quisumbing v. PNB and SLDC,

debt is not a valid tender of payment

G.R. No. 178242, Jan 20, 2009).

and may be refused receipt by the oblige

“Never Let The Odds Keep You From Pursuing What You Know In Your Heart You Were Meant To Do.”-Leroy Satchel Paige

Page 94 of 180

Civil Law Q&As (2007-2013)

[email protected]

[email protected]

Moreover, Ferdie’s refusal was justified

there has been extraordinary deflation since

on the ground that the amount tendered

1998, and therefore, Felipe should pay him

does not include interest. In order to

the value of the debt at the time it was

effect the redemption of the foreclosed

incurred. Felipe refused to pay him again,

property, the payment to the purchaser

claiming that Gustavo is estopped from

must include the following sums: (a) the

raising the issue of legal tender, having

bid price; (b) the interest on the bid

accepted the check in March, and that it

price, computed at one per centum (1%)

was Gustavo's negligence in not depositing

per month; and (c) the assessments and

the check immediately that caused the

taxes, if any, paid by the purchaser with

check to become stale.

the same rate of interest (Sec 28, 1997 Rules of Civil Procedure). Unless there is

(A). Can Gustavo now raised the issue that

an express stipulation to that effect, the

the cashier's check is not legal tender? (2%)

creditor cannot be compelled to receive partial payment of the prestation (Art. 1248, Civil Code).

SUGGESTED ANSWER: No. Gustavo previously accepted a check as payment. It was his fault why the check became stale. He is now estopped

Extinguishment;

Payment

of

Check;

check is not legal tender.

Legal Tender (2008) No.

XVII.

Felipe

borrowed

$100

from

Gustavo in 1998, when the Phil P - US$ exchange rate was P56 - US$1. On March 1, 2008, Felipe tendered to Gustavo a cashier's check in the amount of P4,135 in payment of his US$ 100 debt, based on the Phil P - US$ exchange rat at that time. Gustavo accepted the check, but forgot to deposit it until Sept. 12, 2008. His bank refused to accepted the check because it had become stale. Gustavo now wants Felipe to pay him in cash the amount of P5,600.

Claiming

from raising the issue that a cashier's

that

the

previous

payment was not in legal tender, and that

(B).

Can

Felipe

validly

refuse

to

pay

Gustavo again? (2%) SUGGESTED ANSWER: Yes, Felipe can refuse to pay Gustavo, who allowed the check to become stale. Although a check is not legal tender (Belisario v. Natividad. 60 Phil 156), there

are

instances

when

a

check

produces the effects of payment, for example: (a) when the creditor is in estoppel or he had previously promised he would accept a check (Paras, Civil

“Never Let The Odds Keep You From Pursuing What You Know In Your Heart You Were Meant To Do.”-Leroy Satchel Paige

Page 95 of 180

Civil Law Q&As (2007-2013)

[email protected]

[email protected]

Code Annotated, Vol IV, 2000 ed., p.

secure the PN, Recardo executed a real

394); (b) when the check has lost its

estate mortgage on his own property. When

value because of the fault of the creditor

Eduardo defaulted on the PN, XYZ stopped

(Art. 1249, 2nd par.),as when he was

payment of rentals on the building on the

unreasonably delayed in presenting the

ground that legal compensation had set in.

check for payment (PNB v. Seeto, G.R.

Since there was still a balance due on the

No, L-4388, 13 August 1952).

PN

after

applying

the

rentals,

XYZ

foreclosed the real estate mortgage over (C). Can Felipe compel Gustavo to receive

Recardo's property. Recardo opposed the

US$100 instead? (1%)

foreclosure on the ground that he is only a co-signatory; that no demand was made

SUGGESTED ANSWER:

upon him for payment, and assuming he is

Felipe cannot compel Gustavo to receive US$100 because under RA 529, payment of loans should be at Philippine currency at the rate of exchange prevailing at the time of the stipulated date of payment. Felipe could only compel Gustavo to receive US$ 100 if they stipulated that obligation be paid in foreign currency (R.A. 4100).

liable, his liability should not go beyond half the balance of the loan. Further, Recardo said that when the bank invoked compensation between the reantals and the amount of the loan, it amounted to a new contract or novation, and had the effect of extinguishing the security since he did not give his consent (as owner of the property under the real estate mortgage) thereto. (B). Can Recardo's property be foreclosed to pay the full balance of the loan? (2%)

Liability; Solidary Liability (2008)

SUGGESTED ANSWER:

No. XV. Eduardo was granted a loan by XYZ Bank for the purpose of improving a building

which

XYZ

leased

from

him.

Eduardo, executed the promissory note ("PN") in favor of the bank, with his friend Recardo as co-signatory. In the PN, they both

acknowledged

that

they

are

"individually and collectively" liable and waived the need for prior demand. To

Yes,

Recardo's

property

can

be

foreclosed to pay the full balance of the loan because when he signed as cosignatory in the promissory note, he acknowledged he is solidarily liable with Eduardo.

In

solidary

obligations,

a

creditor has the right to demand full payment of the obligation from any of

“Never Let The Odds Keep You From Pursuing What You Know In Your Heart You Were Meant To Do.”-Leroy Satchel Paige

Page 96 of 180

Civil Law Q&As (2007-2013)

[email protected]

[email protected]

the solidary debtors (Art. 1207, Civil

latter, is obliged to continue the same

Code).

until the termination of the affair and its

situations

giving

five rise

to

examples

of

this

of

type

to

require

concerned

to

substitute

the

person

him,

if

the

2144, NCC).

No.V. What are obligations without an Give

or

owner is in a position to do so (Art.

Obligations; Without Agreement (2007)

agreement"?

incidents,

obligations? (10%)

Second

example,

indebiti

may

also

a

case give

of

solutio

rise

to

an

obligation without an agreement. This refers to the obligation to return which arises when something is received when

SUGGESTED ANSWER:

there is no right to demand it, and it was

"Obligations without an agreement" are

unduly delivered through mistake (Art.

obligations

2154, NCC).

that

do

not

arise

from

contract such as those arising from: 1. delicts;

2.

quasi-delicts;

3.

solutio

indebiti; 4. negotiorum gestio; and 5. all other obligations arising from law.

Third example, is when without the knowledge of the person obliged to give support, it is given by a stranger, the latter shall have a right to claim the same from the former, unless it appears

ALTERNATIVE ANSWER:

that he gave it out of piety and without "Obligations without an agreement" refer

intention of being repaid (Art. 2164,

to the juridical relation of quasi-contract

NCC).

which

arise

from

certain

lawful,

voluntary and unilateral acts to the end

Fourth

example,

is

when

through

that no one shall be unjustly enriched or

accident or other causes a person is

benefited at the expense of another. (Art.

injured or becomes seriously ill, and he

2142, NCC)

is treated or helped while he is not in a condition to give consent to a contract,

First Example of an obligation without

he shall be liable to pay for the services

an agreement is a case of negotiorum

of the physician or other person aiding

gestio,

whereby

him,

takes

charge

one of

who the

voluntarily agency

or

management of the business or property

unless

the

service

has

been

rendered out of pure generosity (Art. 2167, NCC).

of another without any power from the “Never Let The Odds Keep You From Pursuing What You Know In Your Heart You Were Meant To Do.”-Leroy Satchel Paige

Page 97 of 180

Civil Law Q&As (2007-2013)

[email protected]

not,

[email protected]

Fifth instance of an obligation without

ought

in

equity

and

good

an agreement is when the person obliged

conscience, to hold (Heirs of Lorenzo

to support an orphan or an insane or

Yap v. CA, 371 Phil 523, 1991). The

other indigent person unjustly refuses to

following are examples of constructive

give support to the latter, any third

trust: 1. Art. 1456 NCC which provides:

person may furnish support to the needy

"If property is acquired through mistake

individual, with right of reimbursement

or fraud, the person obtaining it is, by

from the person obliged to give support.

force of law considered a trustee of an

The provisions of this article apply when

implied trust for the benefit of the

the father or mother of a child under

person for whom the property comes." 2.

eighteen years of age unjustly refuses to

Art 1451 NCC which provides: "When

support him (Art. 2166, NCC).

land passes by succession through any person and he causes the legal title to be

Trust

put in the name of another, a trust is established by implication of law for the

Trust De Son Tort (2007)

benefit of the true owner." 3. Art 1454 NCC which provides: "If an absolute

No.III. Explain the following concepts and

conveyance of property is made in order

doctrines and give an example of each:

to

(A).

concept

of

trust de

son

tort (constructive trust) (5%)

secure

the

performance

of

an

obligation of the grantor toward the grantee, a trust by virtue of law is established. If the fulfillment of the obligation is offered by the grantor when

SUGGESTED ANSWER:

it becomes due, he may demand the

A constructive trust is a trust NOT

reconveyance of the property to him." 4.

created by any word or phrase, either

Art 1455 NCC which provides: "When any

expressly or impliedly, evincing a direct

trustee, guardian or any person holding a

intention to create a trust, but is one

fiduciary relationship uses trust funds

that

the

for the purchase of property and causes

demands of justice. It does not come

conveyance to be made to him or to

about by agreement or intention but

third person, a trust is established by

mainly operation of law and construed as

operation of law in favor of the person to

a trust against one who, by fraud, duress

whom the funds belong."

arises

in

order

to

satisfy

or abuse of confidence, obtains or holds the legal right to property which he “Never Let The Odds Keep You From Pursuing What You Know In Your Heart You Were Meant To Do.”-Leroy Satchel Paige

Page 98 of 180

Civil Law Q&As (2007-2013)

[email protected]

Sales

advanced

for

[email protected]

her

employees (biyaheros).

She required them to surrender TCT of Condominium

Act;

Partition

of

a

Condominium (2009) No.XVIII.

The

Ifugao

Arms

is

a

strong earthquake occurred which left huge cracks in the outer walls of the building. As a result, a number of condominium units were rendered unfit for use. May Edwin, owner of one of the condominium units affected, legally sue for partition by sale of the whole project? Explain. (4%)

Yes, Edwin may legally sue for partition of

the

whole

condominium

project under the following conditions: (a) the damage or destruction caused by the earthquake has rendered one-half (1/2)

or

more

of

the

untenantable,

and

condominium

owners

(b)

units

therein

that holding

the an

aggregate of more than thirty percent (30%) interests of the common areas are opposed

to

the

and

to

execute

the

Domeng Bandong was not required to post any security but when Eulalia discovered that

he

incurred

shortage

in

cattle

procurement operation, he was required to execute a Deed of Sale over a parcel of land in favor of Eulalia. She sold the property to her

grandneice

Jocelyn

who

thereafter

instituted an action for ejectment against the Spouses Bandong. To assert their right, Spouses Bandong filed

SUGGESTED ANSWER: sale

properties

corresponding Deeds of Sale in her favor.

condominium project in Baguio City. A

by

their

restoration

of

the

an action for annulment of sale against Eulalia and Jocelyn alleging that there was no

sale

intended

but

only

equitable

mortgage for the purpose of securing the shortage

incurred

by

Domeng

in

the

amount of P 70, 000.00 while employed as "biyahero" by Eulalia. Was the Deed of Sale between Domeng and Eulalia a contract of sale or an equitable mortgage? Explain. (5%) SUGGESTED ANSWER:

condominium project (Sec 8 [b], Republic Act No. 472 “Condominium Act”).

The contract between Domeng Bandong and Eulalia was an equitable mortgage rather than a contract of sale. The

Mortgage; Equitable Mortgage (2012)

purported deed of sale was actually intended to merely secure the payment

No.VI. (b) Eulalia was engaged in the

of the shortage incurred by Domeng in

business of buying and selling large cattle.

the

In order to secure the financial capital, she

operations. Under Art 1602, Civil Code,

conduct

of

the

cattle-buying

“Never Let The Odds Keep You From Pursuing What You Know In Your Heart You Were Meant To Do.”-Leroy Satchel Paige

Page 99 of 180

Civil Law Q&As (2007-2013)

[email protected]

[email protected]

the contract shall be presumed to be an

offering P800,000 in ready cash for the

equitable mortgage when it may be fairly

land. When Roberto confirmed that he

inferred that the real intention of the

could pay in cash as soon as Sergio could

parties is simply to secure the payment

get

of a debt or the performance of any

decided to withdraw his offer to Marcelo,

other obligation. The present transaction

hoping to just explain matters to his friend.

was clearly intended to just secure the

Marcelo,

shortage incurred by Eulalia because

withdrawal was communicated to him,

Bandung remained in possession of the

taking the position that they have a firm

property inspite of the execution of the

and binding agreement that Sergio cannot

sale.

simply walk away from because he has an

the

documentation

however,

ready,

objected

Sergio

when

the

option to buy that is duly supported by a duly accepted valuable consideration. Option Contract; Liquor & “Pulutan” as Consideration (2013)

against Sergio? (5%)

No.III.Sergio is the registered owner of a 500-square meter land. His friend, Marcelo, who

has

long

been

(A) Does Marcelo have a cause of action

interested

in

the

property, succeeded in persuading Sergio to

SUGGESTED ANSWER: Yes. Marcelo has a cause of action against Sergio.

sell it to him. On June 2, 2012, they agreed on the purchase price of P600,000 and that

Under Art. 1324, when the offerer has

Sergio would give Marcelo up to June30,

allowed the offeree a certain period to

2012 within which to raise the amount.

accept, the offer may be withdrawn at

Marcelo, in a light tone usual between

any

them, said that they should seal their

communicating such withdrawal, except

agreement through a case of Jack Daniels

when

Black and P5,000 "pulutan" money which

consideration,

he immediately handed to Sergio and which

promised.

time the

before option as

acceptance is

founded

something

by upon

paid

or

the latter accepted. The friends then sat down and drank the first bottle from the

An accepted unilateral promise to buy or

case of bourbon.

sell a determinate thing for a certain

is

binding

upon

him

price if

the

On June 15, 2013, Sergio learned of

promise is supported by a consideration

another

distinct

buyer,

Roberto,

who

was

from

the

price

(Art.

1479).

“Never Let The Odds Keep You From Pursuing What You Know In Your Heart You Were Meant To Do.”-Leroy Satchel Paige

Page 100 of 180

Civil Law Q&As (2007-2013)

[email protected]

[email protected]

Consideration in an option contract may

The

Statute

be anything of value,, unlike in sale

agreement for the sale of real property

where it must be the price certain in

or

money or its equivalent (San Miguel

agreement is unenforceable by action,

Properties Inc. v. Spouses Huang, G.R.

unless

No. 137290, July 31, 2000).

memorandum, thereof, be in writing,

of

of

an the

Frauds

interest same,

covers

therein.

or

some

an Such

note

or

(Art. 1403 (e), Civil Code). Here, Marcelo Here, the case of Jack Daniels Black and

and

the P5,000.00 “pulutan” money was a

Option

consideration to “seal their agreement,”

unilateral promise to buy or sell, which

an agreement that Marcelo is given until

need not be in writing to be enforceable

June 30, 2012 to buy the parcel of land.

(Sanchez v. Rigos, G.R. No. L-25494,

There is also no showing that such

June 14, 1972, citing Atkins, Kroll and

consideration will be considered part of

Co.

the

Southwestern Sugar & Molasses Co. v.

purchase

unilateral

price.

withdrawal

Thus, of

Sergio’s

the

offer

Sergio

merely

Contract,

Inc.

v.

entered

which

Cua

into

refers

Hian

an

to

Tek

a

and

Atlantic Gulf & Pacific Co.).

violated the Option Contract between him and Marcelo.

ALTERNATIVE ANSWER:

(B) Can Sergio claim that whatever they

No. Sergio’s claim has no legal basis.

might have agreed upon cannot be enforced because any agreement relating to the sale

The contract of sale has already been

of real property must be supported by

partially executed which takes it outside

evidence in writing and they never reduced

the ambit of the Statute of Frauds is

their agreement to writing? (3%)

applicable only to executory contracts, not to contracts that are totally or

SUGGESTED ANSWER:

partially performed (Carbonnel v. Poncio, G.R. No. L-11231, May 12, 1958).

No. Sergio’s claim has no legal basis. The contract at issue in the present case is the option contract, not the contract

Right of First Refusal; Lessee; Effect

of sale for the real property. Therefore,

(2008)

Art. 1403 does not apply. No.XVI. Dux leased his house to Iris for a period of 2 years, at the rate of P25,000.00

“Never Let The Odds Keep You From Pursuing What You Know In Your Heart You Were Meant To Do.”-Leroy Satchel Paige

Page 101 of 180

Civil Law Q&As (2007-2013)

[email protected]

[email protected]

monthly, payable annually in advance. The

refusal. This makes the mother a buyer

contract stipulated that it may be renewed

in bad faith, hence giving more ground

for another 2-year period upon mutual

for

agreement of the parties. The contract also

(Equatorial

granted Iris the right of first refusal to

Theater, G.R. No. 106063, 21 Nov. 1996).

rescission

of

Realty,

the et

sale al.

v.

to

her

Mayfair

purchase the property at any time during the lease, if Dux decides to sell the property at the same price that the property is offered for sale to a third party. Twentythree months after execution of the lease contract, Dux sold breach of her right of first refusal. Dux said there was no breach because the property was sold to his mother who is not a third party. Iris filed an action to rescind the sale and to compel Dux to sell the property to her at the same price. Alternatively, she asked the court to

ALTERNATIVE ANSWER: No, Iris cannot seek rescission of the sale of the property to Dux’s mother because the sale is not one of those rescissible contracts under Art. 1381 of the Civil Code. (B). Will the alternative prayer for extension of the lease prosper? (2%) SUGGESTED ANSWER:

extend the lease for another 2 years on the No. The contract stipulated that it may

same terms.

be renewed for another 2-year period (A). Can Iris seek rescission of the sale of

upon mutual agreement of the parties.

the property to Dux's mother? (3%)

Contracts

are

binding

between

the

parties; validity or compliance cannot be

SUGGESTED ANSWER:

left to the will of one of the parties (Art.

Yes, because the right of first refusal is included in the contract signed by the parties. Only if the lessee failed to

1308, Civil Code). ALTERNATIVE ANSWER:

exercise the right of first refusal could

It depends. The alternative prayer for

the

subject

the extension of the lease may prosper if

property to others, under no less than

(a) there is a stipulation in the contract

the

conditions

of sale; (b) Dux's mother is aware of the

previously offered to the lessee. Granting

existing contract of lease; or (c) the lease

that the mother is not a third party, this

is recorded in the Registry of Property

would make her privy to the agreement

(Art. 1676, Civil Code).

lessor same

lawfully terms

sell and

the

of Dux and Iris, aware of the right of first “Never Let The Odds Keep You From Pursuing What You Know In Your Heart You Were Meant To Do.”-Leroy Satchel Paige

Page 102 of 180

Civil Law Q&As (2007-2013)

[email protected]

Lease

[email protected]

reimbursed the value of the improvements he introduced. (4%)

Builder;

Good

Faith;

Useful

Improvements (2013) No.IV.Anselmo is the registered owner of a land and a house that his friend Boboy occupied for a nominal rental and on the condition that Boboy would vacate the property

on

demand.

With

Anselmo's

knowledge, Boboy introduced renovations consisting of an additional bedroom, a covered veranda, and a concrete block fence, at his own expense.

SUGGESTED ANSWER: Boboy’s claim that he is a builder in good faith has no basis. A builder in good faith

is

someone

who

occupies

the

property in concept of an owner. The provisions

on

builder-planter-sower

under the Civil Code cover cases in which the builder, planter and sower believe themselves to be owners of the land, or at least, to have a claim of title thereto.

Subsequently, Anselmo needed the property as his residence and thus asked Boboy to vacate and turn it over to him. Boboy, despite an extension, failed to vacate the property, forcing Anselmo to send him a written demand to vacate.

As Boboy is a lessee of the property, even if he was paying nominal rental, Art. 1678, Civil Code, is applicable. Under

this

makes,

provision,

in

good

if

the

lessee

faith,

useful

improvements which are suitable to the

In his own written reply, Boboy signified

use for which the lease is intended,

that he was ready to leave but Anselmo

without altering the form or substance of

must first reimburse him the value of the

the property leased, the lessor upon the

improvements

termination of the lease, shall pay the

he

introduced

on

the

property as he is a builder in good faith.

lessee

Anselmo

Boboy

improvements at that time. Should the

cannot ask for reimbursement as he is a

lessor refuse to reimburse said amount,

mere lessee. Boboy responded by removing

the

the improvements and leaving the building

improvements,

even

in its original state.

principal

may

refused,

insisting

that

one-half

lessee thing

of

may

the

value

of

remove

the

though

the

suffer

damage

thereby. (IVa) Resolve Boboy's claim that as a builder

in

good

faith,

he

should

be

“Never Let The Odds Keep You From Pursuing What You Know In Your Heart You Were Meant To Do.”-Leroy Satchel Paige

Page 103 of 180

Civil Law Q&As (2007-2013)

[email protected]

[email protected]

(IVb) Can Boboy be held liable for damages

lease contracts between Jude and his

for

tenants? Explain your answer. (3%)

removing

the

improvements

over

Anselmo's objection? (4%) SUGGESTED ANSWER: SUGGESTED ANSWER:

Yes, Ildefonso must respect the lease contracts between Jude and his tenants.

No. Boboy cannot be held liable for

While it is true that the said lease

damages.

contracts

The

lessor,

Anselmo,

refused

to

reimburse one-half of the value of the improvements, so the lessee, Boboy, may remove principal thereby.

the

same,

thing If

in

improvements

even

may

suffer

removing Boboy

though

the

damage

the

useful

caused

more

impairment in the property leased than is necessary he will be liable for damages (Art. 1678, Civil Code).

were

not

registered

and

annotated on the title to the property, Ildefonso

is

still

not

an

innocent

purchaser for value. He ought to know the existence of the lease because the building was already occupied by the tenants

at

the

time

he

bought

it.

Applying the principle of caveat emptor, he should have checked and known the status of the occupants of their right to occupy the building before buying it.

Agency Lease; Caveat Emptor (2009)

Agency; Sale of a Real Property through

No.VIII. Jude owned a building which he had leased to several tenants. Without informing

his

tenants,

Jude

sold

the

building to Ildefonso. Thereafter, the latter notified all the tenants that he is the new owner of the building. Ildefonso ordered the tenants to vacate the premises within thirty

an Agent (2010) No.XVI. X was the owner of an unregistered parcel of land in Cabanatuan City. As she was abroad, she advised her sister Y via overseas call to sell the land and sign a contract of sale on her behalf.

(30) days from notice because he had other

Y thus sold the land to B1 on March 31,

plans for the building. The tenants refused

2001 and executed a deed of absolute sale

to vacate, insisting that they will only do so

on behalf of X. B1 fully paid the purchase

when the term of their lease shall have

price.

expired. Is Ildefonso bound to respect the

“Never Let The Odds Keep You From Pursuing What You Know In Your Heart You Were Meant To Do.”-Leroy Satchel Paige

Page 104 of 180

Civil Law Q&As (2007-2013)

[email protected]

[email protected]

B2, unaware of the sale of the land to B1,

double sales of an immovable property,

signified to Y his interest to buy it but

the

asked Y for her authority from X. Without

person who is in good faith was first in

informing X that she had sold the land to

possession and in the absence thereof to

B1, Y sought X for a written authority to

the person who presents the oldest title,

sell.

provide there is good faith.

X e-mailed Y an authority to sell the land. Y

In a case, the Supreme Court has held

thereafter sold the land on May 1, 2001 to

that in a sale of real estate the execution

B2 on monthly installment basis for two

of

years, the first installment to be paid at the

tantamount to delivery of the possession

end of May 2001.

of the property sold. The ownership of

ownership

a

shall

notarial

pertain

document

to

of

the

sale

is

the land therefore pertains to the first Who between B1 and B2 has a better right

buyer. It may also be mentioned that

over the land? Explain. (5%)

under Art 3344 no instruments or deed establishing,

SUGGESTED ANSWER: B-2 has a better title. This is not a case of double sale. Since the first sale was void. The law provides that when a sale of a piece of land or any interest therein is through an agent, the authority of the

acknowledging,

Hence, the sale to B1 was void. ALTERNATIVE ANSWER:

or

registered under Act 496 shall be valid except as between the parties. Thus, the Deed of Sale of B-2 has no binding effect on B-1.

Partnership

sale shall be void (Art 1874, NCC). The written authority from the owner X.

modifying,

extinguishing right to real property not

latter shall be in writing; otherwise, the property was sold by Y to B1 wihtout any

transmitting,

Liability; Liability of a Partner (2010) No.XV.

A,

partnership

B,

and

to

C

operate

entered a

into

a

restaurant

business. When the restaurant had gone

Under the facts, B-1 has a better right to

past break-even stage and started to garner

the land. Given the fact that the Deed of

considerable profits, C died. A and B

Sale in favor of B-1 and B-2 are not

continued the business without dissolving

inscribed in the Registry of Deeds, the

the partnership. They in fact opened a

case is governed by Art 1544 of the New

branch

Civil Code which provides that in case of

obligations in the process. Creditors started

of

the

restaurant,

incurring

“Never Let The Odds Keep You From Pursuing What You Know In Your Heart You Were Meant To Do.”-Leroy Satchel Paige

Page 105 of 180

Civil Law Q&As (2007-2013)

demanding

for

[email protected]

the

payment

of

their

obligations.

[email protected]

individual properties shall be subject first to the payment of his separate debts (Art 1835. NCC).

(A). Who are liable for the settlement of the partnership’s obligations? Explain? (3%) SUGGESTED ANSWER:

Oral Partnership (2009)

The two remaining partners, A and B, are

No.I. TRUE or FALSE. Answer TRUE if the

liable. When any partner dies and the

statement

business

statement is false. Explain your answer in

is

continued

without

any

settlement of accounts as between him

is

true,

or

FALSE

if

the

not more than two (2) sentences.

or his estate, the surviving partners are held liable for continuing the business

(C). An oral partnership is valid. (1%)

despite the death of C (Art 1841, 1785, SUGGESTED ANSWER:

par 2, and Art 1833 of NCC).

TRUE.

Partnership

is

a

consensual

(B).What are the creditors’ recourse/s?

contract, hence, it is valid even though

Explain. (3%)

not in writing.

SUGGESTED ANSWER:

ALTERNATIVE ANSWER:

Creditors actions,

can for

file

the

instance,

an

appropriate action

for

collection of sum of money against the “partnership at will” and if there are no sufficient funds, the creditors may go after the private properties of A and B (Art 816, NCC). Creditors may also sue the estate of C. The estate is not excused

from

the

liabilities

of

the

partnership even if C is dead already but only up to the time that he remained a partner (Art 1829, 1835, par 2, NCC; Testate Estate of Mota v. Serra, 47 Phil 464 [1925]). However, the liability of C’s

TRUE. An oral is a consensual of the partnership is valid even though not in writing.

However,

If

it

involves

contribution of an immovable property or a real right, an oral contract of partnership is void. In such a case, the contract of partnership to be valid, must be in a public instrument ( Art. 1771 ,NCC

),

and

the

inventory

of

said

property signed by the parties must be attached to said public instrument (Art. 1773, NCC). ALTERNATIVE ANSWER:

“Never Let The Odds Keep You From Pursuing What You Know In Your Heart You Were Meant To Do.”-Leroy Satchel Paige

Page 106 of 180

Civil Law Q&As (2007-2013)

TRUE.

[email protected]

consensual

Yes, he is not entitled to the return of

contract, hence, it is valid even though

his contribution to the capital of the

not in writing. The oral contract of

partnership, but only to the net profits

partnership is also valid even if an

from the partnership business during the

immovable

is

life of the partnership period. If he is a

contributed thereto. While the law, in

limited partner, however, he may ask for

such a case, requires the partnership to

the

be in a public document, the law does

provided in Art 1856 and 1857, Civil

not expressly declare the contract void if

Code.

not

Partnership

is

property

executed

in

a

[email protected]

or real

the

right

required

form

inferred that the said requirement is prohibitory NCC),

the

or

mandatory

said

oral

(Article

contract

of

his

contributions

as

Commodatum & Mutuum

(Article 1409 (7 ,NCC ). And there being nothing in the law from which it can be

return

Mutuum;

Interest;

Solutio

Indebiti

(2012)

5, of

No.VI.a)

Siga-an

granted

a

loan

to

partnership must also be valid. The

Villanueva in the amount of P 540, 000.00.

interested party may simply require the

Such agreement was not reduced to writing.

contract

public

Siga-an demanded interest which was paid

document in order to comply with the

by Villanueva in cash and checks. The total

required form (Article 1357, NCC). The

amount

purpose of the law in requiring a public

to P 1, 200, 000.00. Upon advice of her

document is simply to notify the public

lawyer, Villanueva demanded for the return

about the contribution.

of the excess amount of P 660, 000.00

to

be

made

into

a

Villanueva

paid

accumulated

which was ignored by Siga-an. Share; Demand during the Existence of

(1)

Partnership (2012)

Explain. (3%)

No.X.b) A partner cannot demand the

SUGGESTED ANSWER:

Is

the

payment

of

interest

valid?

return of his share (contribution) during the existence of a partnership. Do you agree?

No, Art. 1956, Civil Code, provides that

Explain your answer. (5%)

“no interest shall be due unless it has been expressly stipulated in writing.”

SUGGESTED ANSWER: (2) Is solution indebiti applicable? Explain. (2%) “Never Let The Odds Keep You From Pursuing What You Know In Your Heart You Were Meant To Do.”-Leroy Satchel Paige

Page 107 of 180

Civil Law Q&As (2007-2013)

[email protected]

enforceable ( Article 1403 [2] b, NCC).The

SUGGESTED ANSWER:

validity Yes,

Solutio

because

Indebiti

Villanueva

P600,000.00

[email protected]

is

applicable

Overpaid

representing

of

the

be

Surety

interest

therefore, demand its return.

should

distinguished from its enforceability .

by

payment which is not due. He can,

contract

Surety (2010) No.III. Define, Enumerate or Explain. (2%

Guaranty

each)

Guaranty (2009)

(A).

What

is

the

difference

between

"guaranty" and "suretyship"? No.I. TRUE or FALSE. Answer TRUE if the statement

is

true,

or

FALSE

if

the

statement is false. Explain your answer in not more than two (2) sentences. (D). An oral promise of guaranty is valid and binding. (1%)

SUGGESTED ANSWER: Guaranty and Suretyship distinguished (1)The

obligation

in

guaranty

is

secondary; whereas, in suretyship, it is primary.

SUGGESTED ANSWER :

(2) In guranty, the undertaking is to pay if

the

principal

debtor

cannot

pay;

FALSE. An oral contract of guaranty,

whereas, in suretyship, the undertaking

being a special promise to answer for the

is to pay if the principal debtor does not

debt of

pay .

another,

is

unenforceable

unless

in (3) In guranty, the guarantor is entitled

writing (Article 1403 [2] b, NCC ).

to the benefit of excussion; whereas, in suretyship the surety

ALTERNATIVE ANSWER:

is not entitled.

TRUE. An oral promise of guaranty is

(4) Liability in guaranty depends upon

valid and binding. While the contract is

an independent agreement to pay the

valid,

unenforceable

obligations of the principal if he fails to

because it is not writing . Being a special

do so; whereas, in suretyship, the surety

promise

assumes liability as a regular party.

however answer

,it for

is

the

debt,

or

miscarriage of another, the Statute of Frauds requires it to be in writing to be “Never Let The Odds Keep You From Pursuing What You Know In Your Heart You Were Meant To Do.”-Leroy Satchel Paige

Page 108 of 180

Civil Law Q&As (2007-2013)

[email protected]

[email protected]

(5)The Guarantor insures the solvency

(B). Will your answer to [a] be the same if

of the principal debtor; whereas, the

the contract stipulates that upon failure of

surety insures the debt.

Rosario to redeem the ring on due date,

(6)In

a

guaranty,

subsidiarlty suretyship,

liable; the

the

guarantor

whereas,

surety

in

binds

is a

himself

solidarity with the principal debtor (Art 2047, Civil Code).

herself

as

full

payment

of

the

loan?

Reasons. (3%)

No, my answer will be different. While the contract of pledge is valid, the stipulation authorizing the pledgee to

Pledge; Pactum Commissorium (2009) Rosario

appropriate the entire proceeds thereof for

SUGGESTED ANSWER:

Pledge

No.XVII.

Jennifer may immediately sell the ring and

obtained

a

loan

immediately sell the thing pledged is of

P100,000.00 from Jennifer, and pledged her diamond ring. The contract signed by the parties stipulated that if Rosario is unable to redeem the ring on due date, she will execute a document in favor of Jennifer providing that the ring shall automatically

void under Art 2088 of the New Civil Code, which provides that “the creditor cannot appropriate the things given by way of pledge or mortgage, or dispose of them xxx.” Jennifer cannot immediately sell by herself the thing pledged. It must be foreclosed by selling it at a public auction

be considered full payment of the loan.

in

accordance

with

the

procedure under Art 2112 of the New Civil Code.

(A). Is the contract valid? Explain. (3%)

Torts and Damages

SUGGESTED ANSWER: The contract is valid because Rosario

Damages (2012)

has to execute a document in favor of Jennifer to transfer the ownership of the

No.I. a) Roberto was in Nikko Hotel when he

pledged ring to the latter. The contract

bumped into a friend who was then on her

does

way to a wedding reception being held in

not

commissorium

amount because

to it

pactum does

not

said hotel. Roberto alleged that he was then

provide for the automatic appropriation

invited by his friend to join her at the

by the pledgee of the thing pledged in

wedding reception and carried the basket

case of default by the pledgor.

full of fruits which she was bringing to the affair.

At

the

reception,

the

wedding

“Never Let The Odds Keep You From Pursuing What You Know In Your Heart You Were Meant To Do.”-Leroy Satchel Paige

Page 109 of 180

Civil Law Q&As (2007-2013)

[email protected]

[email protected]

coordinator of the hotel noticed him and

prosper. Otherwise, Roberto’s action will

asked him, allegedly in a loud voice, to

not prosper.

leave as he was not in the guest list. He retorted that he had been invited to the affair by his friend, who however denied doing

so.

Deeply

embarrassed

by

the

The hotel is liable for the wrongful acts of its employees. COMMENT:

incident, Roberto then sued the hotel for damages under Articles 19 and 21 of the Civil Code. Will Roberto’s action prosper?

The facts of the problem are almost similar to the facts of Nikko Hotel Manila

Explain. (5%)

Garden

v.

Reyes,

G.R.

No.

154259, Feb 28, 2005. In the said case, however, there is a categorical finding

SUGGESTED ANSWER: No. Roberto’s action will not prosper. From the facts given in the problem, the wedding coordinator did not abuse her right when she asked him to leave the wedding reception because he was not in the guest list. Hotel Nikko could not be held liable for damages as its liable

that the hotel employee did not, exposed the complainant to the ridicule, shame or

embarrassment;

hence,

did

not

commit any abuse of right. The present problem makes no statement of that finding. In the contrary, the problem states that it is a mere allegation.

spring from the liability of its employee (Nikko Hotel Manila Garden v. Reyes, G.R. No. 154259, Feb 28, 2005).

Damages; Moral & Exemplary (2009)

ALTERNATIVE ANSWER:

No.XIV. Rodolfo, married to Sharon, had an

It depends. While the hotel has the right to exclude an uninvited guest from the wedding reception, that does not give the

hotel

the

license

to

humiliate

Roberto. If the wedding coordinator of the hotel acted wrongfully e.g. with the abuse of right, unfairly, or in a matter that exposed Roberto to unnecessary ridicule

or

shame,

his

action

will

illicit affair with his secretary, Nanette, a 19-year old girl, and begot a baby girl, Rona. Nanette sued Rodolfo for damages: actual, for hospital and other medical expenses

in

delivering

the

child

by

caesarean section; moral, claiming that Rodolfo

promised

to

marry

her,

representing that he was single when, in fact, he was not; and exemplary, to teach a lesson to like-minded Lotharios.

“Never Let The Odds Keep You From Pursuing What You Know In Your Heart You Were Meant To Do.”-Leroy Satchel Paige

Page 110 of 180

Civil Law Q&As (2007-2013)

[email protected]

[email protected]

(A). If you were the judge, would you award

Vinzons-Chato filed a Motion to Dismiss

all the claims of Nanette? Explain. (3%)

arguing that she cannot be held liable for damages for acts she performed while in

SUGGESTED ANSWER:

the

If Rodolfo's marriage could not have

Commissioner. Is she correct? Explain. (5%)

discharge

of

her

duties

as

BIR

been possibly known to Nanette or there is no gross negligence on the part of Nanette, Rodolfo could be held liable for moral damages.

Yes. As a general rule, a public officer is not liable for acts performed in the

If there is gross negligence in a suit for quasi-delict,

SUGGESTED ANSWER:

exemplary

could

be

awarded.

discharge of his duties. The exceptions are when he acted with malice, bad faith, or gross negligence in the performance of his duty, or when his act is in violation of a Constitutional guaranteed right and liberties of a person under

Damages; Public Officers acting in the Performance of their Duties (2012)

Art32 of the NCC. The public officer is not automatically

No.II.a) Liwayway Vinzons-Chato was then

considered to have violated the rights or

the

liberties of a person simply because the

Commissioner

of

Internal

Revenue

while Fortune Tobacco Corporation is an

rule

entity

declared

engaged

in

the

manufacture

of

the

public invalid

officer by

issued

was

court.

The

the

different brands of cigarettes, among which

complainant must still allege and prove

are

the particular injury or prejudice he has

"Champion,"

"Hope,"

and

"More"

cigarettes.

suffered

from

the

violation

of

his

constitutional right by the issuance of Fortune filed a complaint against VinzonsChato to recover damages for the alleged violation of its constitutional rights arising from Vinzons-Chato’s issuance of Revenue Memorandum Circular No. 37-934 (which re-classified Fortune cigarettes as locally manufactured with foreign brands and thereby imposed higher taxes), which the Supreme Court later declared invalid.

the invalidated rule. The problem does not state any fact from which any malice, bad faith or gross negligence on the part of VinzonsChato may be inferred, or the particular injury or prejudice the complainant may have suffered as a result of the violation of his constitutional right. Hence, she

“Never Let The Odds Keep You From Pursuing What You Know In Your Heart You Were Meant To Do.”-Leroy Satchel Paige

Page 111 of 180

Civil Law Q&As (2007-2013)

cannot

be

held

[email protected]

liable.

The

facts

presented are similar to facts of the case

[email protected]

the driver and the defense of diligence is not available.

of Vinzons-Chato v. Fortune, G.R. No. (B).Would your answer be the same if

141309, Dec 23, 2008.

Rommel was in the car at the time of the accident? Explain. (2%) Death Indemnity (2009)

SUGGESTED ANSWER: Yes, my answer would be the same.

No. X. Rommel’s private car, while being

Rommel, who was in the car, shall be

driven by the regular family driver, Amado,

liable for damages if he could have

hits a pedestrian causing the latter’s death.

prevented the misfortune by the use of

Rommel is not in the car when the incident

due diligence in supervising his driver

happened.

but failed to exercise it (Art. 2184, NCC).

(A). Is Rommel liable for damages to the heirs of the deceased? Explain. (2%)

Yes, my answer will be the same except

Yes, Rommel may be held liable for damages if he fails to prove that he exercised the diligence of a good father of a family (Art. 2180, par 5, NCC) in and

supervising

his

family

driver. The owner is presumed liable unless

he

proves

his driver. ALTERNATIVE ANSWER:

SUGGESTED ANSWER:

selecting

In such case, his liability is solidary with

the

defense

of

diligence. If the driver was performing his assigned task when the accident happened, Rommel shall be solidarily

that in such case the liability of the owner is not presumed. When the owner is inside the vehicle, he becomes liable only when it is shown that he could have prevented the misfortune by the use of due diligence (Art. 2184, NCC). For the owner to be held liable, the burden of proving that he could have prevented the misfortune rests on the shoulder of the victim.

liable with the driver. In

case

the

driver

is

convicted

of

reckless imprudence and cannot pay the

Doctrine of Discovered Peril (Last Clear Chance) (2007)

civil liability, Rommel is subsidiarily liable for the damage awarded against

No.III. Explain the following concepts and doctrines and give an example of each:

“Never Let The Odds Keep You From Pursuing What You Know In Your Heart You Were Meant To Do.”-Leroy Satchel Paige

Page 112 of 180

Civil Law Q&As (2007-2013)

[email protected]

[email protected]

(B). doctrine of discovered peril (last clear

typhoon knocked down the fence of the

chance) (5%)

pond and the iguana crawled out of the gate of Primo’s residence. N, a neighbor

SUGGESTED ANSWER:

who was passing by, started throwing

The doctrine of last clear chance states that where the plaintiff was guilty of prior or antecedent negligence, but the defendant,

who

opportunity

to

had avoid

the the

ultimate impending

harm failed to do so, it is the defendant

stones at the iguana, drawing the iguana to move toward him. N panicked and ran but tripped on something and suffered a broken leg. Is anyone liable for N’s injuries? Explain. (4%)

who is liable for all the consequences of the accident notwithstanding the prior negligence of the plaintiff. An example is where a person was riding a pony on a bridge and improperly pulled the pony to the wrong side when he saw a car coming. The driver of the car did not stop or change direction, and nearly hit the horse, and, the frightened animal jumped to its death. The driver of the car is guilty of negligence because he had a fair opportunity to avoid the

SUGGESTED ANSWER: No one is liable. The possessor of an animal or whoever may make use of the same is responsible for the damage it may cause, although it may escape or be lost. This responsibility shall cease only in case the damage should come from force majeure or from the fault of the person who has suffered damage (Art 2183, NCC).

accident and failed to avail himself of that opportunity. He is liable under the doctrine of last clear chance (Picart v. Smith, 37 Phil. 809, 1918).

Liability;

Special

Parental

Authority

(2010) No.XII. On May 5, 1989, 16-year old

Liability; Owner of a Pet; Fortuitous

Rozanno, who was issued a student permit,

Event (2010)

drove to school a car, a gift from his parents. On even date, as his class was

No.XIV. Primo owns a pet iguana which he

scheduled to go on a field trip, his teacher

keeps in a man-made pond enclosed by a

requested him to accommodate in his car,

fence situated in his residential lot. A

as he did, four (4) of his classmates

“Never Let The Odds Keep You From Pursuing What You Know In Your Heart You Were Meant To Do.”-Leroy Satchel Paige

Page 113 of 180

Civil Law Q&As (2007-2013)

[email protected]

[email protected]

because the van rented by the school was

(B). How about the damage to the jeepney?

too crowded. On the way to a museum

Explain. (2%)

which the students were scheduled to visit, Rozanno made a wrong maneuver, causing a collision with a jeepney. One of his classmates died. He and the three (3) others were badly injured.

SUGGESTED ANSWER: With respect to the damages caused to the jeepney, only Rozanno should be held liable because his negligence or

(A). Who is liable for the death of Rozanno’s classmate and the injuries suffered by Rozanno and his 3 other classmates? Explain. (2%)

tortuous act was the sole, proximate and immediate cause thereof. (C). Under the same facts, except the date of occurrence of the incident, this time in mid-1994, what would be your answer?

SUGGESTED ANSWER:

Explain. (2%)

At the time the incident occurred in May 1989, Rozanno was still a minor. Being a minor, Art 218 of the Family Code applies. Pursuant to Art 218, the school, its administrators and teachers shall be liable for the acts of minor Rozanno because of the special parental authority and responsibility that they exercise over him. The authority applies to all authorized activities, whether inside or

SUGGESTED ANSWER: Since Rozanno was 16 years old in 1989, if the incident happened sometime in the middle of 1994, Rozanno have been 21 years old at the time. Hence, he was already of legal age. The law reducing the age of majority to 18 years took effect in December 1989.

the school,

Being of legal age, articles 218, 219, and

entity or institution. The field trip on

221 of the Family Code are no longer

which occasion Rozanno drove the car,

applicable. In such case, only Rozanno

was an authorized activity, and , thus,

will be personally responsible for all the

covered by the provision. Furthermore,

consequences

the parents of Rozanno are subsidiarily

school or his parents were themselves

liable pursuant to Art 219 (FC), and

also

principally liable under Art 221 (FC), if

contributed to the happening of the

they are negligent.

incident. In that event, the school or his

outside the premises

of

negligent

of

his and

act such

unless

his

negligence

parents are not liable under Art 218, 218 or 221 of the Family Code, but will be “Never Let The Odds Keep You From Pursuing What You Know In Your Heart You Were Meant To Do.”-Leroy Satchel Paige

Page 114 of 180

Civil Law Q&As (2007-2013)

[email protected]

[email protected]

liable under general provision on the

court to justify the damages that your client

Civil Code on quasi-delict.

claims? (8%) SUGGESTED ANSWER:

Quasi-Delict; Claims; Requisites (2013) No.II. A collision occurred at an intersection involving a bicycle and a taxicab. Both the bicycle rider (a businessman then doing his morning

exercise)

and

the

taxi

driver

claimed that the other was at fault. Based on the police report, the bicycle crossed the intersection first but the taxicab, crossing at a fast clip from the bicycle's left, could

I will the base the claim of my client on quasi-delict under Art 2176 of the Civil Code of the Philippines. The requisites for a claim under quasi-delict to prosper are as follows: (1) Act or omission, there being fault or negligence; (2) Damage or injury; and

not brake in time and hit the bicycle's rear wheel, toppling it and throwing the bicycle

(3)

rider into the sidewalk 5 meters away.

damage and the act or omission.

The bicycle rider suffered a fractured right

The case clearly involves quasi-delict

knee, sustained when he fell on his right

where

side on the concrete side walk. He was

suffered

hospitalized

negligence

and

was

subsequently

operated on, rendering him immobile for 3

Causal

my

connection

client,

injury of

as the

the a

between

bicycle result

the

rider, of

the

over-speeding

taxi

driver, without fault on my client’s part.

weeks and requiring physical rehabilitation for another 3 months. In his complaint for

To prove actual damages aside from the

damages, the rider prayed for the award

testimony of client, I will present his

ofP1,000,000

hospital and medical bills. Receipts paid

moral

actual

damages,P200,000

damages, P200,000

damages, P1

00,000

exemplary

nominal

damages

on

the

rehabilitation

will

also

be

presented. [The sentence in red should be replaced with the following sentence

and P50,000 attorney's fees.

because he is a businessman and not an Assuming the police report to be correct

employee. Furthermore, I will present

and as the lawyer for the bicycle rider, what

income tax returns, contracts and other

evidence

testimonial)

documents to prove unrealized profits as

and legal arguments will you present in

a result of this temporary injury.] I will

(documentary

and

“Never Let The Odds Keep You From Pursuing What You Know In Your Heart You Were Meant To Do.”-Leroy Satchel Paige

Page 115 of 180

Civil Law Q&As (2007-2013)

[email protected]

[email protected]

also call the attending physician to

fraudulent,

reckless,

testify as to the extent of the injuries

malevolent manner. While the amount of

suffered by my client, and to corroborate

exemplary damages may not be proved,

the contents of the medical documents.

the

plaintiff

entitled

to

must moral

oppressive,

show or

that

he

or

is

compensatory

Based on Art. 2202, in quasi-delicts, the

damages.

defendant shall be liable for all damages

present the police report showing the

which

probable

circumstance under which the accident

consequences of the act or omission

took place, taking into account the

complained of. It is not necessary that

actions of the parties. I will ask the

the damages have been foreseen or could

officials who responded to the accident

have been foreseen by the defendant.

to testify as to the conduct of the

Unlike

are

the

actual

natural

damages,

and

no

proof

of

pecuniary loss is necessary in order that moral, nominal, temperate liquidated or

In

support

of

this,

I

will

parties at the time of the accident in order to determine whether defendant was guilty of gross negligence.

exemplary damages may be adjudicated.

Finally, attorney’s fees may be recovered

The assessment is left to the discretion

when exemplary damages are awarded

of the Court (Art. 2216, Civil Code).

(Art 2208, Civil Code).

There

must

be

proof

pecuniary

estimation, however. Moral damages can be recovered by my

Quasi Tort (2010)

client under Articles 2219 and 2200. Moral damages may be recovered in case

No.III. Define, Enumerate or Explain. (2%

of

each)

a

quasi-delict

causing

physical

injuries. Additionally, it must be proved that such damages were the proximate result of the act complained of. Medical certificates will be presented, along with the testimony from my client and other eyewitness accounts, in order to support the award for moral damages.

defendant

acted

in

liable under quasi torts and what are the defenses available to them? Note: It is recommended that the examiner exercise leniency and liberality in grading the answers given to this question. The term

Exemplary damages may be granted if the

(B). Define quasi tort. Who are the persons

wanton,

quasi-tort is not part of legal developments in civil law. In Philippine legal tradition, quasi-

“Never Let The Odds Keep You From Pursuing What You Know In Your Heart You Were Meant To Do.”-Leroy Satchel Paige

Page 116 of 180

Civil Law Q&As (2007-2013)

[email protected]

[email protected]

delict has been treated as the closest civil

who can be held liable and their defenses

law equivalent of the common law tort. In

would also apply.

fact,

in

a

decisions,

number the

two

of

Supreme

terms

have

Court been

Those liable for quasi-delict include:

considered synonymous. In reality, however,

(1)

the common law tort is much broader in

causing damage to another through fault

scope than the civil law quasi-delict. In

or negligence ( Article 2176 NCC ); and

recent developments in common law, the

(2)

concept of “quasi-torts” can be considered as

Article 2180 (NCC ).

Those

tortfeasor

Persons

or

vicariously

the

person

liable

under

the closest common law equivalent of the civil law concept of quasi-delict. This is because it is argued that the growing

The defenses available include:

recognition of quasi-torts as a source of

(a) That the defendant was not negligent

obligation is hinged on the acceptance at

or that he exercised due diligence (

common law of the civil law principles of

Article 2176 NCC );

quasi-delict. (b) SUGGESTED ANSWER:

That

negligent

Quasi -tort is a legal concept upholding the doctrine that some legal duty exists

although his

the

defendant

negligence

is

not

is the

proximate cause of the injury ( Article 2179 NCC );

that cannot be classified strictly as a

(c) That the plaintiff's own negligence

personal duty (thus resulting in a tort),

was the immediate and proximate cause

nor as a contractual duty but rather

of his injury ( Article 2179 NCC );

some other kind of duty recognizable by the law. ” Tort “ or ” Quasi-tort” is an

(d ) That the person vicariously liable

Law

has observed all the diligence of a good

concept, while “Delict” or “Quasi-Delict“

father of a family to prevent damage (

is

Article 2180 NCC ); and

Anglo a

American Civil

Law

or

Common

concept

(Wikipedia

encyclopedia).

(e)

That

the

cause

of

action

prescribed after the lapse

ALTERNATIVE ANSWER:

has

s (Article

2179 NCC ). Quasi

-tort

is

considered

as

the

equivalent of quasi-delict. Hence the

The

fact

that

the

plaintiff

had

rules of the latter pertaining to persons

committed contributory negligence is a partial defense (Art 2179, NCC).

“Never Let The Odds Keep You From Pursuing What You Know In Your Heart You Were Meant To Do.”-Leroy Satchel Paige

Page 117 of 180

Civil Law Q&As (2007-2013)

[email protected]

MULTIPLE CHOICE

per

Taxation

Law

and

not

by

right

of

representation (Art 975, Civil Code)

QUESTIONS 2013

capita,

[email protected]

Exam

MCQ (October 13, 2013)

I. (2) How much is Dante's share in the net estate? (1%) (A) P150,000. (B) P200,000.

I. Armand died intestate. His full-blood

(C) P300,000.

brothers, Bobby and Conrad, and half-

(D) P400,000.

blood brothers, Danny, Edward and Floro,

(E) None of the above.

all predeceased him. The following are the surviving relatives:

SUGGESTED ANSWER:

1. Benny and Bonnie, legitimate children of

E. None of the above.

Bobby;

There is no showing that Danny is an

2. Cesar, legitimate child of Conrad;

illegitimate

3. Dante, illegitimate child of Danny;

Armand. In the absence of proof to the

4. Ernie, adopted child of Edward; and

contrary, the law presumes that the

5. Felix, grandson of Floro. The

net

value

of

Armand's

half-blood

brother

of

relationship is legitimate. Thus, Dante, estate

is

Pl,200,000.

an illegitimate child of Danny, is barred from inheriting from Armand pursuant to

the

“iron

curtain

rule”

which

I. (1) How much do Benny and Bonnie

disqualifies an illegitimate child from

stand to inherit by right of representation?

inheriting

(1%)

legitimate children and relatives of his

(A) P200,000

father or mother, and vice versa (Art

(B) P300,000

992, Civil Code).

ab

intestao

from

the

(C) P400,000 (D) P150,000

I. (3) How much is Ernie's share in the net

(E) None of the above.

estate . (1%) (A) P 0.

SUGGESTED ANSWER:

(B) P400,000.

(E) None of the above.

(C) P150,000.

If all the brothers/sisters are disqualified

(D) P200,000.

to inherit, the nephews/nieces inherit

(E) None of the above. SUGGESTED ANSWER:

“Never Let The Odds Keep You From Pursuing What You Know In Your Heart You Were Meant To Do.”-Leroy Satchel Paige

Page 118 of 180

Civil Law Q&As (2007-2013)

[email protected]

(A) 0 or (E) None of the above. The

legal

relationship

[email protected]

Should the share of insolvent debtor C be

created

by

divided

only

between

the

two

other

adoption is strictly between the adopter

remaining debtors, A and B? (1%)

and the adopted. It does not extend to

(A) Yes. Remission of D's share carries with

the relatives of either party (Sayson v.

it total extinguishment of his obligation to

CA, G.R. Nos. 89224-25, Jan 23, 1992).

the benefit of the solidary debtors.

(Note: “E. None of the above’” is another

(B) Yes. The Civil Code recognizes remission

answer because Ernie has no share at

as a mode of extinguishing an obligation.

all in the net estate).

This clearly applies to D. (C) No. The rule is that gratuitous acts

I. (4) How much is Felix's share in the net

should be restrictively construed, allowing

estate? (1%)

only the least transmission of rights.

(A) P400,000.

(D) No, as the release of the share of one

(B) P150,000.

debtor would then increase the burden of

(C) P300,000.

the other debtors without their consent.

(D) P0. (E) None of the above.

SUGGESTED ANSWER: (D). No, as the release of the share of one

SUGGESTED ANSWER:

debtor would then increase the burden of

(D). 0. Or (E) None of the above.

the other debtors without their consent.

In the collateral line, representation is

When one of the solidary debtors cannot,

granted only to children of brother or

because of his insolvency, reimburse his

sisters,

share

Felix

is

a

grandson

of

a

to

the

debtor

paying

the

predeceased brother.

obligation, such share shall be borne by

(Note: “E. None of the above: is another

all his co-debtors, in proportion to the

answer because Felix has no share at

debt of each (Art 1217, Civil Code).

all in the net estate)

Additionally, D was released only from his share of P10,000.00 not from the

II. A, B, C and D are the solidary debtors of

solidary tie that binds him to A, B and C.

X for P40,000. X released D from the payment of his share of PI 0,000. When the

III. Amador obtained a loan of P300,000

obligation became due and demandable, C

from Basilio payable on March25, 2012. As

turned out to be insolvent.

security for the payment of his loan, Amador constituted a mortgage on his residential house and lot in Basilio's favor.

“Never Let The Odds Keep You From Pursuing What You Know In Your Heart You Were Meant To Do.”-Leroy Satchel Paige

Page 119 of 180

Civil Law Q&As (2007-2013)

[email protected]

[email protected]

Cacho, a good friend of Amador, guaranteed

Basilio

(the

creditor)

must

first

be

and obligated himself to pay Basilio, in case

exhausted”)

Amador fails to pay his loan at maturity.

III. (2) If Amador sells his residential house and lot to Diego, can Basilio foreclose the

III. (1) If Amador fails to pay Basilio his loan

real estate mortgage? (1%)

on March 25, 2012, can Basilio compel Cacho to pay? (1%)

(A) Yes, Basilio can foreclose the real estate mortgage because real estate mortgage

(A) No, Basilio cannot compel Cacho to pay

creates a real right that attaches to the

because as guarantor, Cacho can invoke

property.

the principle of excussion, i.e., all the

(B) Yes, Basilio can foreclose the real

assets of Basilio must first be exhausted.

estate mortgage. It is binding upon Diego

(B) No, Basilio cannot compel Cacho to

as the mortgage is embodied in a public

pay because Basilio has not exhausted

instrument.

the available remedies against Amador.

(C) No, Basilio cannot foreclose the real

(C) Yes, Basilio can compel Cacho to pay

estate

because the nature of Cacho's undertaking

ownership on the buyer, Diego, who

indicates

must therefore consent.

that

he

has

bound

himself

mortgage.

The

sale

confers

solidarily with Amador.

(D) No, Basilio cannot foreclose the real

(D) Yes, Basilio can compel Cacho who

estate mortgage. To deprive the new owner

bound himself to unconditionally pay in

of ownership and possession is unjust and

case Amador fails to pay; thus the benefit of

inequitable.

excussion will not apply. SUGGESTED ANSWER: SUGGESTED ANSWER:

(B) Yes, Basilio can foreclose the real

(B) No, Basilio cannot compel Cacho to

estate mortgage. It is binding upon Diego

pay because Basilio has not exhausted

as the mortgage is embodied in a public

the available remedies against Amador.

instrument.

The guarantor cannot be compelled to

Since

pay the creditor unless the latter has

instrument, there is constructive notice

exhausted all the property of the debtor

to

and has resorted to all the legal remedies

mortgaged property.

the

Diego,

mortgage who

is

is

the

in

a

buyer

public if

the

against the debtor (Art. 2058, Civil Code) (Note: “A” is not the correct answer

ALTERNATIVE ANSWER:

because it states that “all the assets of

“Never Let The Odds Keep You From Pursuing What You Know In Your Heart You Were Meant To Do.”-Leroy Satchel Paige

Page 120 of 180

Civil Law Q&As (2007-2013)

[email protected]

[email protected]

(C) No, Basilio cannot foreclose the real

(D) No, Jose's refusal is not justified. The

estate

confers

expenses he incurred are useful for the

ownership on the buyer, Diego, who

preservation of the thing loaned. It is

must therefore consent.

Jose's obligation to shoulder these useful

The mortgage is not registered, thus,

expenses.

mortgage.

The

sale

cannot be binding against third persons (Art. 2125, Civil Code)

SUGGESTED ANSWER: (D) No, Jose's refusal is not justified. The

IV. Cruz lent Jose his car until Jose

expenses he incurred are useful for the

finished his Bar exams. Soon after Cruz

preservation of the thing loaned. It is

delivered

Jose's obligation to shoulder these useful

the

car,

Jose

brought

it

to

Mitsubishi Cubao for maintenance check

expenses.

up and incurred costs of P8,000. Seeing the

In commodatum, the bailee is obliged to

car's peeling and faded paint, Jose also had

pay for the ordinary expenses for the use

the car repainted for P10,000. Answer the

and preservation of the thing loaned (Art

two

1941, Civil Code).

questions

below

based

on

these

common facts.

The

bailee,

Jose,

has

no

right

of

retention on the ground that the bailor IV. (1) After the bar exams, Cruz asked for

owes him something, even if it may be

the return of his car. Jose said he would

by reason of expenses. He can only

return it as soon as Cruz has reimbursed

retain it if he suffers damages by reason

him for the car maintenance and repainting

of a flaw or defect in the thing loaned of

costs of P 18,000.

which the bailor knows (Art 1951, Civil

Is Jose's refusal justified? (1%)

Code).

(A) No, Jose's refusal is not justified. In this kind of contract, Jose is obliged to pay for

IV. (2) During the bar exam month, Jose

all

lent the car to his girlfriend, Jolie, who

the

expenses

incurred

for

the

preservation of the thing loaned.

parked the car at the Mall of Asia's open

(B) Yes, Jose's refusal is justified. He is

parking lot, with the ignition key inside the

obliged to pay for all the ordinary and

car. Car thieves broke into and took the

extraordinary

car.

expenses,

but

subject

to

reimbursement from Cruz. (C) Yes, Jose's refusal is justified. The

Is Jose liable to Cruz for the loss of the car

principle of unjust enrichment warrants the

due to Jolie's negligence? (1%)

reimbursement of Jose's expenses.

“Never Let The Odds Keep You From Pursuing What You Know In Your Heart You Were Meant To Do.”-Leroy Satchel Paige

Page 121 of 180

Civil Law Q&As (2007-2013)

[email protected]

[email protected]

(A) No, Jose is not liable to Cruz as the loss

Securities

and

was not due to his fault or negligence.

designated L and 0 as managing partners; L

(B) No, Jose is not liable to Cruz. In the

was liable only to the extent of his capital

absence of any prohibition, Jose could lend

contribution; and P was not liable for

the car to Jolie. Since the loss was due to

losses.

force majeure, neither Jose nor Jolie is

In 2006, the partnership earned a net profit

liable.

of P800,000. In the same year, P engaged in

(C) Yes, Jose is liable to Cruz. Since Jose

a different business with the consent of all

lent the car to Jolie without Cruz's

the

consent, Jose must bear the consequent

partnership

loss of the car.

of P500,000. In 2008,the partners dissolved

(D) Yes, Jose is liable to Cruz. The contract

the partnership. The proceeds of the sale of

between them is personal in nature. Jose

partnership

can neither lend nor lease the car to a third

settle its obligation. After liquidation, the

person.

partnership

partners.

Exchange

However, incurred

assets had

were an

Commission,

in

2007,

a

net

loss

insufficient

unpaid

the

to

liability

ofP300,000. SUGGESTED ANSWER: (C) Yes, Jose is liable to Cruz. Since Jose

V. (l) Assuming that the just and equitable

lent the car to Jolie without Cruz's

share of the industrial partner, P, in the

consent, Jose must bear the consequent

profit in 2006 amounted to P1 00,000, how

loss of the car.

much is the share of 0, a limited partner, in

The bailee is liable for the loss of the

the P800,000 net profit? (1%)

thing, even if it should be through a

(A) P160,000.

fortuitous event if he lends or leases the

(B) P175,000.

thing to a third person, who is not a

(C) P280,000.

member of his household (Art 1942, Civil

(D) P200,000.

Code).

(E) None of the above.

V. In 2005, L, M, N, 0 and P formed a

SUGGESTED ANSWER:

partnership. L, M and N were capitalist

(C) P280,000.

partners who contributed P500,000 each,

First, deduct the share of P from the

while 0, a limited partner, contributed P1

profits.

,000,000. P joined as an industrial partner,

P700,000. Next, get the share of O by

contributing only his services. The Articles

following the proportion that the shares

of

of L, M, N, O is 1:1:1:2, respectively.

Partnership,

registered

with

the

P800,000

less

P100,000

is

“Never Let The Odds Keep You From Pursuing What You Know In Your Heart You Were Meant To Do.”-Leroy Satchel Paige

Page 122 of 180

Civil Law Q&As (2007-2013)

[email protected]

[email protected]

(B) No. P is not liable because there is a V. (2) In 2007, how much is the share of 0,

valid

a

losses. Since the other partners allowed

limited

partner,

in

the

net

loss

stipulation

exempting

him

from

of P500,000? (1%)

him to engage in an outside business

(A) P 0.

activity, the stipulation absolving P from

(B) P1 00,000.

liability is valid. For 0, it is basic that a

(C) P125,000.

limited partner is liable only up to the

(D) P200,000.

extent of his capital contribution.

(E) None of the above.

(C) Yes. The stipulations exempting P and L from losses are not binding upon the

SUGGESTED ANSWER:

creditors. 0 is likewise liable because the

(D) P200,000

partnership was not formed in accordance

A limited partner shall not become liable

with

a s a general partner unless, in addition

partnership.

to the exercise of his rights and powers

(D) No. The Civil Code allows the partners

as a limited partner, he takes part in the

to stipulate that a partner shall not be

control of the business (Art 1948, Civil

liable for losses. The registration of the

Code). In the absence of stipulation as to

Articles of Partnership embodying such

profits and losses, the share of each

stipulations serves as constructive notice to

partner

the partnership creditors.(E) None of the

in

the

losses

shall

be

the

requirements

of

a

limited

proportionate to what he may have

above is completely accurate.

contributed (Art 1797).

(E) None of the above is completely accurate.

V. (3) Can the partnership creditors hold L, 0 and Pliable after all the assets of the

SUGGESTED ANSWER:

partnership are exhausted? (1%)

(E) None of the above is completely

(A) Yes. The stipulation exempting P from

accurate.

losses is valid only among the partners. L is

VI. Gary is a tobacco trader and also a

liable because the agreement limiting his

lending investor. He sold tobacco leaves to

liability to his capital contribution is not

Homer

valid insofar as the creditors are concerned.

although the period for delivery was not

Having taken part in the management of

guaranteed. Despite Gary's efforts to deliver

the partnership, 0 is liable as capitalist

on

partner.

government red tape hindered his efforts

for

time,

delivery

within

transportation

a

month,

problems

and

and he could only deliver after 30 days.

“Never Let The Odds Keep You From Pursuing What You Know In Your Heart You Were Meant To Do.”-Leroy Satchel Paige

Page 123 of 180

Civil Law Q&As (2007-2013)

[email protected]

[email protected]

Homer refused to accept the late delivery

(D) No. Homer was not justified in refusing

and to pay on the ground that the agreed

to accept the tobacco leaves. There was no

term had not been complied with.

term in the contract but a mixed condition.

As

lending

investor,

Gary

granted

a

The fulfillment of the condition did not

Pl,000,000 loan to Isaac to be paid within

depend purely on Gary's will but on other

two years from execution of the contract. As

factors, e.g., the shipping company and the

security for the loan, Isaac promised to

government. Homer should comply with his

deliver to Gary his Toyota Innova within

obligation.

seven (7) days, but Isaac failed to do so. Gary

was

thus

compelled

to

demand

SUGGESTED ANSWER:

payment for the loan before the end of the

(B)

No.

Homer

was

not

justified

in

agreed two-year term.

refusing to accept the tobacco leaves. He consented to the terms and conditions

VI. (l) Was Homer justified in refusing to

of

accept the tobacco leaves? (1%)

Obligations arising from contract have

(A) Yes. Homer was justified in refusing to

the force of law between the contracting

accept the tobacco leaves. The delivery was

parties.

to be made within a month. Gary's promise

It is clear under the facts that the period

of delivery on a "best effort" basis made the

of delivery of the tobacco leaves was not

delivery uncertain. The term, therefore, was

guaranteed.

ambiguous.

factors which may prevent him from

(B)

No.

abide

anticipated

by

it.

other

True enough, transportation problems

consented to the terms and conditions

and government red

of

slight delay was, thus, excusable.

must

justified

Gary

must

refusing to accept the tobacco leaves. He and

not

and

making the delivery within a month.

sale

was

sale

in

the

Homer

the

abide

by

it.

tape

did. Such

Obligations arising from contract have

Obligations arising from contract have

the force of law between the contracting

the force of law between the contracting

parties.

parties and should be complied with in

(C) Yes. Homer was justified in his refusal

good faith (Art. 1160, Civil Code)

to

accept

the

delivery.

The

contract

contemplates an obligation with a term.

VI. (2) Can Gary compel Isaac to pay his

Since the delivery was made after 30 days,

loan even before the end of the two-year

contrary to the terms agreed upon, Gary

period? (1%)

could not insist that Homer accept the tobacco leaves.

“Never Let The Odds Keep You From Pursuing What You Know In Your Heart You Were Meant To Do.”-Leroy Satchel Paige

Page 124 of 180

Civil Law Q&As (2007-2013)

(A)

Yes,

Gary

[email protected]

to

Under Art 1198 (2) of the Civil Code, the

Non-

debtor shall lose every right to make use

compliance with the promised guaranty

of the period when he does not furnish

or

to

immediately security

can pay

compel the

renders

Isaac

[email protected]

loan.

the

obligation

immediately demandable. Isaac lost his

the

creditor

the

guaranties

or

securities which he has promised.

right to make use of the period. (B)

Yes,

to

VII. Lito was a commercial pilot who flew for

immediately pay the loan. The delivery of

Pacific-Micronesian Air. In 1998, he was

the Toyota Innova is a condition for the

the co-pilot of the airline's Flight MA916

loan. Isaac's failure to deliver the car

that mysteriously disappeared two hours

violated the condition upon which the loan

after

was granted. It is but fair for Gary to

presumably over the Pacific Ocean. No trace

demand immediate payment.

of the plane and its 105 passengers and

(C)

compel Isaac to

crew was ever found despite diligent search;

immediately pay the loan. The delivery of

Lito himself was never heard of again. Lito

the car as security for the loan is an

left behind his wife, Lita, and their two

accessory contract; the principal contract is

children.

No,

Gary

Gary

can

compel

cannot

Isaac

take-off

from

Agana,

Guam,

still the P 1,000,000 loan. Thus, Isaac can still make use of the period.

In 2008, Lita met and married Jaime. They

(D)

now have a child of their own.

No,

Gary cannot

compel Isaac to

immediately pay the loan. Equity dictates

While on a tour with her former high school

that Gary should have granted a reasonable

classmates in a remote province of China in

extension of time for Isaac to deliver his

2010, Lita was surprised to see Lito or

Toyota Innova. It would be unfair and

somebody who looked exactly like him, but

burdensome

pay

she was sure it was Lito because of the

the

extreme surprise that registered in his face

for

the P1,000,000

Isaac

simply

to

because

promised security was not delivered.

when he also saw her. Shocked, she immediately fled to her hotel and post haste

SUGGESTED ANSWER: (A)

Yes,

Gary

immediately

can pay

returned to the country the next day. Lita compel

the

Isaac

loan.

to

Non-

now comes to you for legal advice. She asks you the following questions:

compliance with the promised guaranty or

security

renders

the

obligation

immediately demandable. Isaac lost his

VII. (l) If Lito is alive, what is the status of his marriage to Lita? (1%)

right to make use of the period.

“Never Let The Odds Keep You From Pursuing What You Know In Your Heart You Were Meant To Do.”-Leroy Satchel Paige

Page 125 of 180

Civil Law Q&As (2007-2013)

[email protected]

[email protected]

(A) The marriage subsists because the

(B) The marriage is valid. After an absence

marital bond has not been terminated by

of more than 10 years, Lito is already

death.

presumed dead for all purposes.

(B) The marriage was terminated when Lita

(C)

married Jaime.

absence, however lengthy, is insufficient to

(C) The marriage subsists because Lita's

authorize Lita to contract a subsequent

marriage to Jaime is void.

marriage.

(D) The marriage is terminated because Lito

(D) The marriage is void. If Lito is indeed

is presumed dead after his plane has been

alive, his marriage to Lita

missing for more than 4 years.

dissolved and they can resume their marital

(E) The marriage can be formally declared

relations at any time.

The marriage

is

void.

Lito's mere

was never

terminated if Lito would not resurface. SUGGESTED ANSWER: SUGGESTED ANSWER:

Any answer is correct.

(C) The marriage subsists because Lita's

Under Art 390 of the Civil Code, after an

marriage to Jaime is void.

absence

For

the

purpose

years,

it

being

unknown whether or not the absentee

subsequent marriage under Art 41 of the

still lives, he shall be presumed dead doe

Family Code, the spouse present must

all

institute

succession.

summary

contracting

seven

the

a

of

of

proceeding

as

purposes,

except

This

for

provision

those was

of not

provided in the Family Code for the

repealed by the present Family Code.

declaration of presumptive death of the

Applying this to the problem, (A) may be

absentee, without prejudice to the effect

correct. (B) may also be correct. (C) and

of

(D) may also be correct under Art 41 of

the

reappearance

of

the

absent

spouse.

the Family Code. VIII.Which

VII. (2) If Lito is alive, what is the status of

of

the

following

actions

or

defenses are meritorious: (1%)

Lita's marriage to Jaime? (1%) (A) An action for recovery of down (A) The marriage is valid because Lita's

payment paid under a rescinded oral sale

marriage to Lito was terminated upon Lito's

of real property.

disappearance for more than seven years.

(B) A defense in an action for ejectment that the lessor verbally promised to extend or renew the lease.

“Never Let The Odds Keep You From Pursuing What You Know In Your Heart You Were Meant To Do.”-Leroy Satchel Paige

Page 126 of 180

Civil Law Q&As (2007-2013)

[email protected]

[email protected]

(C) An action for payment of sum of money filed against one who orally promised to

ALTERNATIVE ANSWER:

answer another's debt in case the latter

(E) None of the above.

defaults.

(a) The recovery of the down payments

(D) A defense in an action for damages that

should be made in the same action for

the debtor has sufficient, but unliquidated

rescission. Otherwise, it would be a

assets to satisfy the credit acquired when it

ground for dismissal under Rule 2, Sec 4

becomes due.

of Rules of Court.

(E) None of the above.

(b) Lease of a real property is covered by the Statute of Frauds. Furthermore, it

SUGGESTED ANSWER:

also consists of interest in real property.

(A) An action for recovery of down

Hence, it must be in writing. (Art 1403,

payment paid under a rescinded oral sale

Civil Code)

of real property.

(c) A contract of guaranty is a promise to

An oral sale of real property is an

answer for the debt of another and

unenforceable

the

hence, it is also covered by the Statute

Statute of Frauds. Since, in the problem,

of Frauds. It must be in writing before it

the vendee paid down payment, it takes

can be enforced in a court action. (Art

it out of the ambit of Statute of Frauds.

1403, Civil Code)

The rescission here must be in the sense

(d)

of resolution of the reciprocal obligation

unliquidated assets does not excuse him

arising from the contract of sale. If

from paying his debt.

rescinded (resolved) by the vendee on

(e) In the technical meaning of rescission

account of the vendors’ failure to deliver

under Art 1191 of the Civil Code will be

the thing sold, the parties will go back to

adhered to, then there is no absolutely

their status prior to the contract. If the

correct answer. Hence, letter E is also a

vendor

possible answer.

refuses

contract

to

return

under

the

down

The

fact

that

a

debtor

has

payment, then the vendee can file an action to recover the down payment.

IX. Betty entrusted to her agent, Aida,

If, on the other hand, the vendor and the

several pieces of jewelry to be sold on

vendee mutually agree to rescind i.e.

commission with the express obligation to

cancel the contract, the vendee likewise

turn over to Betty the proceeds of the sale,

can file an action for the recovery of the

or to return the jewelries if not sold in a

down payment on the basis of solution

month's

indebiti.

jewelries,

time.

Instead

of

selling

the

Aida pawned them with the

“Never Let The Odds Keep You From Pursuing What You Know In Your Heart You Were Meant To Do.”-Leroy Satchel Paige

Page 127 of 180

Civil Law Q&As (2007-2013)

Tambunting

[email protected]

Pawnshop,

and

used

[email protected]

the

(C) I will rule in favor of Tambunting. Its

money for herself. Aida failed to redeem the

good faith takes precedence over the right

pawned jewelries and after a month, Betty

of Betty to recover the jewelries.

discovered what Aida had done. Betty

(D) I will rule in favor of Tambunting. Good

brought criminal charges which resulted in

faith is always presumed. Tambunting's

Aida's conviction for estafa.

lawful acquisition in the ordinary course of business coupled with good faith gives it

Betty thereafter filed an action against

legal right over the jewelries.

Tambunting Pawnshop for the recovery of the

jewelries.

Tambunting

raised

the

SUGGESTED ANSWER:

defense of ownership, additionally arguing

(A) I will rule in favor of Betty. My ruling

that it is duly licensed to engage in the

is based on the Civil Code provision that

pawnshop and lending business, and that it

one who has lost any movable or has

accepted the mortgage of the jewelry in

been unlawfully deprived thereof may

good faith and in the regular course of its

recover it from the person in possession

business.

of the same. Tam bunting's claim of good faith is inconsequential.

If you were the judge, how will you decide the case? (1%)

Although possession of movable property acquired in good faith is equivalent to a

(A) I will rule in favor of Betty. My ruling

title, nevertheless, one who has lost any

is based on the Civil Code provision that

movable or has been unlawfully deprived

one who has lost any movable or has

thereof may recover it from the person

been unlawfully deprived thereof may

in possession of the same. Betty has

recover it from the person in possession

been deprived unlawfully of her jewelries

of the same. Tam bunting's claim of good

by the estafa committed by Aida. The

faith is inconsequential.

pledge of the said jewelries by Aida to

(B)

I

will

rule

Betty.

Tambunting pawnshop is void because

Tambunting's claim of good faith pales into

the pledgor is not the owner (Art 2085

insignificance

unlawful

(2), Civil Code). Tambunting’s claim of

deprivation of the jewelries. However, equity

good faith is inconsequential, because,

dictates

be

aside from good faith, Tambunting must

reimbursed for the pawn value of the

prove also that it acquired the jewelries

jewelries.

at a public sale in order to be able to

that

in

in light

favor of

the

Tambunting

of

must

retain the jewelries until reimbursed by

“Never Let The Odds Keep You From Pursuing What You Know In Your Heart You Were Meant To Do.”-Leroy Satchel Paige

Page 128 of 180

Civil Law Q&As (2007-2013)

[email protected]

[email protected]

Betty the amount of loan including

If you were the judge, would you grant

interest (Art 559, Civil Code).

Arlene's motion? (1%)

The only exception the law allows is

(A) Yes, I will grant the motion because the

when there is acquisition in good faith of

lease contract between Arlene and Janet

the possessor at a public sale, in which

was not in writing, hence, Janet may not

case, the owner cannot obtain its return

enforce any right arising from the same

without reimbursing the price (Dizon v.

contract.

Suntay, 47 SCRA 160, Sept 29, 1972).

(B) No, I will not grant the motion because to allow Arlene to retain the advance

X. Arlene owns a row of apartment houses

payments

would

amount

to

unjust

in Kamuning, Quezon City. She agreed to

enrichment.

lease Apartment No. 1 to Janet for a period

(C) Yes, I will grant the motion because the

of 18 months at the rate of P10,000 per

action for recovery is premature; Janet

month. The lease was not covered by any

should first secure a judicial rescission of

contract. Janet promptly gave Arlene two

the contract of lease.

(2) months deposit and 18 checks covering

(D) No. I will not grant the motion

the rental payment for 18 months. This

because the cause of action does not

show of good faith prompted Arlene to

seek to enforce any right under the

promise Janet that should Arlene decide to

contract of lease.

sell the property, she would give Janet the right of first refusal.

SUGGESTED ANSWER:

X. (1) Not long after Janet moved in, she

(D) No. I will not grant the motion

received news that her application for a

because the cause of action does not

Master

seek to enforce any right under the

of

Laws

scholarship

at

King's

College in London had been approved.

contract of lease.

Since her acceptance of the scholarship

Janet is not asking for the continued use

entailed a transfer of residence, Janet

of the leased premises. Moreover, the

asked Arlene to return the advance rental

contract

payments

Statute of Frauds as the same has

she

made.

Arlene

refused,

prompting Janet to file an action to recover

is

aside

the

ambit

of

the

already been partially performed.

the payments. Arlene filed a motion to dismiss, claiming that the lease on which

X. (2)Assume that Janet decided not to

the action is based, is unenforceable.

accept

the

scholarship

and

continued

leasing Apartment No. 1. Midway through

“Never Let The Odds Keep You From Pursuing What You Know In Your Heart You Were Meant To Do.”-Leroy Satchel Paige

Page 129 of 180

Civil Law Q&As (2007-2013)

[email protected]

[email protected]

the lease period, Arlene decided to sell

Frauds under Art 1403 (2)(e) of the Civil

Apartment No. 1 to Jun in breach of her

Code. It must be in writing in order to be

promise to Janet to grant her the right of

enforceable.

first refusal. Thus, Janet filed an action seeking the recognition of her right of first refusal, the payment of damages for the violation of this right, and the rescission of the sale between Arlene and Jun.

2012

Taxation

Law

Exam

MCQ (October 14, 2012) 1. Which of the following is NOT included in

Is Janet's action meritorious? (1%)

the attributes of juridical capacity? a) Juridical capacity is inherent in

(A) Yes, under the Civil Code, a promise to buy

and

sell

a

determinate

thing

every natural person, and therefore

is

it is not acquired.

reciprocally demandable.

b) Juridical capacity is lost only

(B) No, the promise to buy and sell a

through death.

determinate thing was not supported by a

c) Juridical capacity is the fitness to

consideration.

be the subject of legal relations.

(C) Yes, Janet's right of first refusal was

d) Juridical capacity cannot exist

clearly violated when the property was not

without capacity to act.

offered for sale to her before it was sold to Jun. (D) No, a right of first refusal involves an interest over real property that must be embodied in a written contract to be enforceable. (E) None of the above. SUGGESTED ANSWER: (D) No, a right of first refusal involves an interest over real property that must be embodied in a written contract to be enforceable. The right of first refusal involves a transfer of interest in the real property.

2.

Which

of

the

following

is

NOT

a

restriction on one’s capacity to act? a) Minority b) Marriage c) Deaf-mute d) Civil Interdiction SUGGESTED ANSWER: This question should be disregarded. (NOTE: There is no correct answer among the choices given. All choices are restrictions on one’s capacity to act. While Marriage is the only one not mentioned in Articles 38 and 39 of the NCC as a restriction on

As such, it is covered by the Statute of

“Never Let The Odds Keep You From Pursuing What You Know In Your Heart You Were Meant To Do.”-Leroy Satchel Paige

Page 130 of 180

Civil Law Q&As (2007-2013)

[email protected]

[email protected]

capacity to act, it restricts the capacity of a

b) Yes, the will is not valid under

married person in cases of adoption.)

Philippine law. c) No, attestation clause is not an

3. This attribute or incident of a case

act of the testator.

determine whether it is a conflict-of-laws

d)

case or one covered by domestic law.

Spanish law.

No,

the

governing

law

is

a) Cause of action b) Foreign element

Note: The facts do not state the Law

c) Jurisdiction

observed by the testator in executing his

d) Forum non conveniens

will. He could have observed Spanish Law or Philippine Law (see comment of Tolentino

4. The capacity of an heir to succeed shall

to Art. 815 NCC in 3Tolentino117, 1992). If

be governed by the:

he observed Spanish Law, the opposition is

a) national law of the decedent’s

not correct because the will is valid under

heirs

Spanish Law, hence choice (d) is the correct

b) law of the country where the

answer. If he observed Philippine Law, the

decedent was a resident at the time

opposition

of his death

Philippine Law does not require the testator

c) national law of the person who

to sign the Attestation Clause of his will,

died

said clause not being his act. In such case,

d) law of the country where the

choice (c) is the correct answer).

properties

of

the

decedent

is

still

not

correct

because

are

located.

6. Ramon, a Filipino, executed a will in Manila, where he left his house and located

5. Atty. BUKO, a Filipino, executed a will

in BP Homes Parañaque in favor of his

while he was in Spain. The attestation

Filipino

clause of the said will does not contain

children RJ and Ramona, both Turkish

Buko’s signature. It is valid under Spanish

nationals, are disputing the bequest to

law. At its probate in Manila, it is being

Ramgen. They plotted to kill Ramgen.

opposed on the ground that the attestation

Ramon learned of the plot, so he tore his

clause does not contain BUKO’s signature.

will in two pieces out of anger. Which

Is the opposition correct? Choose the best

statement is most accurate?

answer.. a) Yes, because it is a fatal defect.

son,

Ramgen.

Ramon’s

other

a) The mere act of Ramon Sr. is immaterial because the will is still readable.

“Never Let The Odds Keep You From Pursuing What You Know In Your Heart You Were Meant To Do.”-Leroy Satchel Paige

Page 131 of 180

Civil Law Q&As (2007-2013)

[email protected]

[email protected]

b) The mere act of tearing the will amounts to revocation.

9. A Japanese national and a Filipino

c) The tearing of the will may

national entered into a contract for services

amount to revocation if coupled

in Thailand. The services will be rendered

with intent of revoking it.

in Singapore. In case of breach, what law

d) The act of tearing the will is

will govern?

material.

a) Thailand law b) Philippine law

7. Even if the applicable law is a foreign

c) Singapore law

law, a count in the Philippines may be

d) Japanese law

constrained to apply Philippine law under any of the following instances, except:

10. Pedro (Filipino) and his wife Jane

a) when the foreign law, judgment or

(American) executed a joint will in Canada,

contract is contrary to a sound and

where such joint will is valid. In case the

important

joint will is probated in Japan, what law

public

policy

of

the

forum;

will govern the formalities of the joint will?

b) when the property subject of

a) American law

the case is located outside of the

b) Philippine law

Philippines;

c) Canadian law

c)

when

the

foreign

law

or

d) Japanese law

judgment is penal in nature; d)

when

the

foreign

law

is

procedural in nature.

11. A French national revokes his will in Japan where he is domiciled. He then changed his domicile to the Philippines

8. If a will is executed by a testator who was

where he died. The revocation of his will in

born

became

Japan is valid under Japanese law but

naturalized Japanese citizen at the time of

invalid under Philippine law. The affected

his

heir is a Malaysian national residing in the

a

Filipino

death,

testamentary

what

citizen law

provisions

but

will if

govern the

will

its is

Philippines. What law will apply?

executed in China and the property being

a) Japanese law

disposed is located in Indonesia?

b) Philippine law

a) Chinese law

c) French law

b) Philippine law

d) Malaysian law

c) Indonesia law d) Japanese law

“Never Let The Odds Keep You From Pursuing What You Know In Your Heart You Were Meant To Do.”-Leroy Satchel Paige

Page 132 of 180

Civil Law Q&As (2007-2013)

[email protected]

[email protected]

12. In the absence of contrary stipulation in

obligation, but Bill fails or refuses to pay,

a marriage settlement, property relations of

what law will govern?

Filipino spouses shall be governed by ---

a) American law

a) Philippines laws

b) Philippine law

b) Law of the place where the

c) Australian law

spouses reside

d) Japanese law

c) Law of the place where the

(Facts for item numbers 15-18)

properties are situated

In 1989, Charice (Filipina) and Justine

d) Law of the place where they were

(American), were married in the Philippines.

married.

In 1990, they separated and Justine went to Las Vegas where he obtained a divorce in

13. The will of a Filipino executed in a

the same year. He then married another

foreign country ---

Filipina, Lea, in Canada on January 1,

a)

cannot

be

probated

in

the

Philippines; b)

may

1992. They had two (2) sons, James and John (who were both born in 1992). In

the

1993, after failing to hear from Justine,

Philippines provided that properties

Charice married Bugoy (a Filipino), by

in the estate are located in the

whom she had a daughter, Regine. In 2009,

Philippines;

Regine married James (son of Justine with

c) cannot be probated before the

Lea) in California, where such marriage is

death of the testator;

valid.

d)

may

be

be

Philippines

probated

probated provided

in

in it

the was

15. What is the current status of the

executed in accordance with the

marriage of Charice and Justine under

laws of the place where the will

Philippine laws?

was executed.

a) Valid b) Void

14. Pedro (Filipino and Bill (American)

c) Voidable

entered

d) Dissolved

into

a

contract

in

Australia,

whereby it was agreed that Pedro will build

(Note: While Art 26 of the FC does not

a commercial building for Bill in the

categorically provide that the first marriage

Philippines,

the

is dissolved by the divorce obtained by the

construction, Bill will transfer and convey

foreign spouse abroad, but provides that

his cattle ranch located in Japan in favor of

such divorce merely gives the Filipino spouse

Pedro.

the capacity to contract a second marriage, it

In

and

case

in

payment

Pedro

for

performs

his

“Never Let The Odds Keep You From Pursuing What You Know In Your Heart You Were Meant To Do.”-Leroy Satchel Paige

Page 133 of 180

Civil Law Q&As (2007-2013)

[email protected]

[email protected]

is believed that the dissolution of the first

examinations unless he marries Princess.

marriage us the necessary consequence of

As a consequence of the threat, Ricky

the foreign divorce.)

married Princess. Can the marriage be annulled on the ground of intimidation

16. What id the status of the marriage

under Article 45 of the Family Code?

between

Choose the best answer.

Charice

and

Bugoy

under

Philippine laws?

a) Yes, because without the threat,

a) Valid

Ricky

would

not

have

married

b) Void

Princess.

c) Voidable

b) Yes, because the threat to enforce

d) Unenforceable

the claim of Princess vitiates the consent of Ricky in contracting the

17. What is the status of the marriage

marriage.

between

c) No, because the threat made by

Charice

and

Bugoy

under

Philippine laws?

Marforth is just and legal.

a) Valid

d) No, because Marforth is not a

b) Void

party to the contract of marriage

c) Voidable

between Princess and Ricky.

d) Unenforceable 20. Audrey, single, bought a parcel of land 18. What is the status of the marriage

in Malolos City from Franco for P 1Million.

between

A contract was executed between them

Regine

and

James

under

Philippine laws?

which already vested upon Audrey full

a) Valid

ownership

of

the

property,

although

b) Void

payable in monthly installments for a

c) Voidable

period of four (4) years. One (1) year after

d) Unenforceable

the execution of the contract, Audrey got married to Arnel. They executed a marriage

19. Ricky and Princess were sweethearts.

settlement whereby they agreed that their

Princess became pregnant. Knowing that

properties shall be governed by the regime

Ricky is preparing for the examinations,

of conjugal partnership of gains. Thereafter,

Marforth, a lawyer and cousin of Princess,

subsequent installments were paid from the

threatened

conjugal partnership funds. Is the land

Ricky

with

the

filing

of

a

complaint for immorality in the Supreme

conjugal or paraphernal?

Court, thus preventing him from taking

“Never Let The Odds Keep You From Pursuing What You Know In Your Heart You Were Meant To Do.”-Leroy Satchel Paige

Page 134 of 180

Civil Law Q&As (2007-2013)

[email protected]

[email protected]

a) The land is conjugal because the

the laws of such consular official. Under

installments were paid from the

Philippine law, what is the status of the

conjugal partnership funds.

marriage of Agay and Topacio? Choose the

b)

best answer.

The

land

is

paraphernal

because ownership thereof was

a)

Void,

because

acquired before the marriage.

official

c) The land is both conjugal and

solemnize

paraphernal funds of installments

Filipinos.

were paid from both the personal

b) Valid, because according to the

funds of Audrey and the conjugal

laws of Australia, such consular

partnership funds.

official has authority to celebrate the

d) The land is paraphernal because

marriage.

it was Audrey who purchased the

c) Voidable, because there is an

same.

irregularity in the authority of the

only

consular 21.

Ernesto

donated

worth P 32,000

mobile

between

solemnize

marriages.

delivered the unit to Hubert who accepted.

recognized as valid in the place

Which statement is most accurate?

where it was celebrated.

donation

orally

to

to

d) Valid, because such marriage is

The

Hubert

phone

authority

marriages

official

consular

and

a)

to

a

has

the

is

void

and

Ernesto may get mobile phone

(Note: The issues in the problem is whether

back.

or not the fact that one of the parties to the

b) The donation is void but Ernesto

marriage was an alien constituted absence

cannot get the mobile phone back.

of authority or mere irregularity of authority.

c) The donation is voidable and may

The problem only give the choice, letter (a),

be anulled.

in case it is interpreted as absence of

d) The donation is valid.

authority. The problem does not give a choice in case it is interpreted as an

22. Agay, a Filipino citizen and Topacio, an

irregularity thereby making all the other

Australian

answers wrong).

consular Australia.

citizen, office

got

of

According

married

the to

in

Philippines the

laws

the in of

Australia, a marriage solemnized by a

23. Separation of property between spouses

consular official is valid, provided that such

during the marriage may take place only:

marriage is celebrated in accordance with

a) by agreement of the spouses.

“Never Let The Odds Keep You From Pursuing What You Know In Your Heart You Were Meant To Do.”-Leroy Satchel Paige

Page 135 of 180

Civil Law Q&As (2007-2013)

[email protected]

[email protected]

b) If one of the spouses has given

b) No trial shall be held without the

ground for legal separation.

6-month cooling off period being

c) Upon order of the court.

observed.

d) If one spouse has abandoned the

c) The spouses will be entitled to live

other.

separately upon the start of the trial.

24.

The

husband

may

impugn

the

d) The prosecuting attorney has

legitimacy of his child but not on the

to conduct his own investigation.

ground that: a)

the

wife

is

suspected

of

27.

A

husband

infidelity.

hidden

treasure

b) the husband had a serious illness

property

that prevented him from engaging in

discovered treasure?

of

his

The

by

chance

on

the

wife.

paraphernal

Who

owns

a)

c) they were living apart.

husband (finder)

d) he is physically incapable of

conjugal partnership.

sexual intercourse.

b) The half pertaining to the wife (as

25. A marriage is void if: a)

solemnized

pertaining

belongs

to

the

sexual intercourse.

owner)

half

discovered

the

belongs to the

to

the

conjugal

partnership. marriage

c) One half shall belong to the

license issued without complying

husband as finder and the other

with the required 10-day posting.

half shall belong to the wife as

b) solemnized by a minister whom

owner of the property.

the

d) a and b

parties

with

a

believe to

have the

authority. c) between parties both 23 years of

28. Which of the following marriages is void

age but without parental advice.

for reasons of public policy?

d) none of the above

a) Between brothers and sisters, whether of the full or half blood.

26.

In

legal

separation,

which is not

correct?

b) Between step-parents and step children.

a) The aggrieved spouse may file the

c)

Between

parents-in-law

action within five (5) years from the

children-in-law.

time of the occurrence of the cause.

d) b and c

and

“Never Let The Odds Keep You From Pursuing What You Know In Your Heart You Were Meant To Do.”-Leroy Satchel Paige

Page 136 of 180

Civil Law Q&As (2007-2013)

[email protected]

[email protected]

29. The following constitute the different

a)

Children

conceived

or

born

circumstances or case of fraud which will

outside a valid marriage.

serves as ground for the annulment of a

b) Children born under a valid

marriage, except?

marriage, which was later declared

a) Non-disclosure of the previous

void because of the psychological

conviction by final judgment of the

incapacity of either or both of the

other party of a crime involving

spouses.

moral turpitude.

c) Children conceived and born

b)

Concealment

of

a

sexually-

outside a valid marriage.

transmissible disease, regardless of

d) Children born under a valid

its nature, existing at the time of the

marriage,

marriage.

obtained a legal separation.

but

the

parents

later

c) Concealment of drug addiction, habitual alcoholism, homosexuality

32. An illegitimate child may use the

or lesbianism existing at the time of

surname of his father when his filiation is

marriage.

established

d) Concealment by the wife or the

instances, except:

in

any

of

the

following

husband of the fact of sexual

a) Filiation has been recognized by

relations prior to the marriage.

the father through the record of birth appearing in the civil register

30. Which of the following is not a requisite

b) Admission of filiation by the

for a valid donation propter nuptias?

father in a public document.

a) The donation must be made

c) Private handwritten instrument is

before

made by the father acknowledging

the

celebration

of

the

marriage. b)

The

his filiation. donation

shall

be

automatically revoked in case of

d) Affidavit by the mother stating the name of his true father.

non-celebration of the marriage. c) The donation must be made in

33. Under RA 8043, an adopter is required

consideration of the marriage.

to be at least ____ years old and ____ years

d) The donation must be made in

older than the child to be adopted at the

favor of one or both of the future

time of the application unless the adopter is

spouses.

the parent by nature of the child. a) 30 and 15

31. Who are illegitimate children?

b) 27 and 16

“Never Let The Odds Keep You From Pursuing What You Know In Your Heart You Were Meant To Do.”-Leroy Satchel Paige

Page 137 of 180

Civil Law Q&As (2007-2013)

[email protected]

[email protected]

c) 50 and 10

d) Subjects the child or allows him

d) 18 and 15

to

be

subjected

to

acts

of

lasciviousness. 34. Under RA 8043, a child qualified to be adopted is any person below _____ years

37. Which of the following statements

old.

is wrong? a) 18

a) The possessor in bad faith shall

b) 21

reimburse the fruits received and

c) 15

those which the legitimate possessor

d) 16

could have received. b) The possessor in bad faith has

35. Which of the following DOES NOT

right

of

reimbursement

result in permanent termination of parental

necessary expenses and those for

authority?

the

production,

gathering

for and

a) Death of the parents.

preservation of the fruits.

b) Death of the child.

c) The possessor in bad faith is not

c) Emancipation of the child.

entitled to a refund of ornamental

d) Conviction of the parents of a

expenses.

crime which carries with it the

d) The possessor in bad faith is

penalty of civil interdiction.

entitled to a refund of useful expenses.

36. The court, in an action filed for the purpose, may suspend parental authority if

38.

the parent or the person exercising parental

completes the statement – The expenses

authority commits any of the following

incurred in improvements for the luxury or

acts, except:

mere pleasure shall not be refunded to thew

a) Treats the child with excessive

Which

phrase

most

accurately

possessor in bad faith:

harshness or cruelty.

a) but he may remove the objects

b) Gives the child corrupting orders,

for which such expenses have

counsel or example.

been incurred, provided that the

c) Compels the child to take up a

thing suffers no injury thereby,

course in college against his/her

and that the lawful possessor does

will.

not prefer to retain them.

“Never Let The Odds Keep You From Pursuing What You Know In Your Heart You Were Meant To Do.”-Leroy Satchel Paige

Page 138 of 180

Civil Law Q&As (2007-2013)

[email protected]

[email protected]

b) and he may not remove the

of the dominion over the property as owner.

objects for which such expenses

What action is being referred to?

have been incurred.

a) Accion publiciana

c) and he may not remove the

b) Accion reinvindicatoria

objects for which such expenses

c) Accion interdictal

have been incurred, unless he pays

d) Quieting of Title

the value they may have at the time he entered into possession.

42. A summary action to recover physical

d) but he may remove the objects for

or material possession only and must be

which such expenses have been

brought within one (1) year from the time

incurred.

the cause of action arises. What action is being referred to?

39. The following are the limitations on the

a) Accion publiciana

right of ownership imposed by the owner

b) Accion reinvindicatoria

himself, except:

c) Accion interdictal

a) Will/Succession

d) Quieting of Title

b) Mortgage c) Pledge

43. The following things are property of

d) Lease

public dominion, except: a) ports and bridges constructed by

40. A plenary action for the recovery of the

the State.

possession

b) vehicles and weapons of the

of

real

allegation and proof

estate,

upon

mere

of a better right

Armed Forces of the Philippines.

thereto, and without allegation of proof of

c) rivers.

title. This action can only be brought after

d) lands reclaimed by the state from

the expiration of one (1) year. What action

the sea.

is being referred to? a) Accion publiciana

SUGGESTED ANSWER:

b) Accion reinvindicatoria

This

c) Accion interdictal

because there is no correct answer.

d) Quieting of Title

(Note: At first glance, one gets the impression

question

should

be

disregarded

that vehicles and weapons of the AFP are 41. Action to recover real property based on

not property of the public domain. But they

ownership. Here, the object is the recovery

are actually property of the public dominion under the second paragraph of Art 420 of

“Never Let The Odds Keep You From Pursuing What You Know In Your Heart You Were Meant To Do.”-Leroy Satchel Paige

Page 139 of 180

Civil Law Q&As (2007-2013)

[email protected]

[email protected]

the NCC. Property of the state which are not

a) the donee accepts the donation.

for public use but are intended for some

b) the donor executes the deed of

public service are properties of the public

donation.

dominion. While the vehicles and weapons of

c) the donor knows of the donee’s

the AFP are not for public use, they are used

acceptance even if the latter has

for the defense of the State which is a public

not received the copy of the deed

service.)

of donation. d) the donee confirms that the donor

44. Which of the following statements

has learned the former’s acceptance.

is wrong? a) patrimonial property of the

47. The following are the elements of an

state, when no longer intended

obligation, except:

for

public

use

or

for

public

a) Juridical/Legal Tie

service, shall become property of

b) Active subject

public dominion.

c) Passive subject

b) all property of the State, which is

d) Consideration

not

of

public

dominion,

is

patrimonial property.

48. It is a conduct that may consist of

c) The property of provinces, cities

giving, doing, or not doing something.

and municipalities is divided into

a) Obligation

property

b) Juridical necessity

for

public

use

and

patrimonial property. d)

Property

is

c) Prestation

either

of

public

dominion or of private ownership.

d) Contract 49.

It

is

a

juridical

relation

arising

from lawful, voluntary and unilateral acts 45.

The following

cannot

ask for

the

reduction of inofficious donation, except: a) Creditors of the deceased

based on the principle that no one should unjustly enrich himself at the expense of another.

b) Devisees or legatees

a) Quasi-contract

c) Compulsory heirs of the donor

b) Quasi-delict

d)

c) Cotract

The surviving spouse of the

donee.

d) Delict

46. Donation is perfected from the moment

50. The following are the elements of quasi-

---

delict, except:

“Never Let The Odds Keep You From Pursuing What You Know In Your Heart You Were Meant To Do.”-Leroy Satchel Paige

Page 140 of 180

Civil Law Q&As (2007-2013)

[email protected]

[email protected]

a) Act or omission

54. It is an international evasion of the

b) Fault/negligence

faithful performance of the obligation.

c) Damage/injury

a) Negligence

d) Pre-existing contract

b) Fraud c) Delay

51. A debtor is liable for damages in case of delay

if

he

is

guilty

of

any

of

d) Mistake

the

following, except:

55. The following are the requisites of

a) default (mora)

fortuitous event, except:

b) mistake

a) Cause is independent of the will

c) negligence (culpa)

of the debtor.

d) breach through contravention of

b)

the tenor thereof

is unforeseeable/unavoidable.

The

event

c) Occurrence renders it absolutely 52. This term refers to a delay on the part

impossible for the debtor to fulfill his

of both the debtor and creditor in reciprocal

obligation

obligations.

impossibility must be absolute not

in

a

normal

manner;

a) Mora accipiendi

partial, otherwise not force majeure.

b) Mora solvendi

d)

c) Compensation morae

aggravation of the injury to the

d) Solution indibiti

creditor.

Debtor

contributed

to

the

53. The following are the requisites of mora

56. A debtor may still be held liable for loss

solvendi, except:

or damages even if it was caused by a

a) Obligation pertains to the debtor

fortuitous event in any of the following

and

instances, except:

is

determinate,

due,

demandable, and liquidated.

a) The debtor is guilty of dolo, malice

b) Obligation was performed on its

or bad faith, has promised the same

maturity date.

thing to two or more persons who do

c) There is judicial or extrajudicial

not have the same interest.

demand by the creditor.

b) The debtor contributed to the

d) Failure of the debtor to comply

loss.

with such demand.

c) The thing to be delivered is generic.

“Never Let The Odds Keep You From Pursuing What You Know In Your Heart You Were Meant To Do.”-Leroy Satchel Paige

Page 141 of 180

Civil Law Q&As (2007-2013)

[email protected]

[email protected]

d) The creditor is guilty of fraud,

c) No, because a motion to dismiss

negligence

is a prohibited pleading.

or

contravened

delay

the

or

tenor

if of

he the

d) Yes, because Fermin and Toti

obligation.

should also pay their share of the obligation.

57.

Buko,

themselves

Fermin solidarily

and

59. Buko, Fermin and Toti are solidarily

amount of P 5,000.00. Suppose Buko paid

debtors of Ayee. Twelve (12) years after the

the obligation, what is his right as against

obligation became due and demandable,

his co-debtors?

Buko paid Ayee and later on asked for

Buko

pay

bound the

a)

to

Toti

cas

Ayee

ask

for

reimbursement from Fermin and

reimbursement

of

Fermin’s

and

Toti’s

shares. Is Buko correct? Why?

Toti.

a) No, because the obligation has

b) Buko can sue Fermin and Toti for

already prescribed.

damages.

b) Yes, because the obligation is

c) Buko can sue for rescission.

solidary.

d) Buko can claim a refund from

c) No, because in solidary obligation

Ayee.

any one of the solidary debtors can pay the entire debt.

58.

Buko,

Fermin

and

Toti

bound

d) Yes, because Fermin and Toti will

themselves solidarily to pay Ayee the sum

be unduly enriched at the expense

of P 10,000.00. When the obligation became

of Buko.

due and demandable, Ayee sued Buko for the

payment

of

the P 10,000.00.

Buko

60. Buko, Fermin and Toti are solidary

moved to dismiss on the ground that there

debtors

under

a

loan

obligation

was failure to implead Fermin and Toti who

of P 300,000.00 which has fallen due. The

are indispensable parties. Will the motion

creditor has, however, condoned Fermin’s

to dismiss prosper? Why?

entire share in the debt. Since Toti has

a) Yes, because Fermin and Toti

become insolvent, the creditor makes a

should have been impleaded as their

demand on Buko to pay the debt. How

obligation is solidary.

much, if any, may Buko be compelled to

b) No, because the creditor may

pay?

proceed against any one of the

a) P 200.000.00

solidary debtors or some or all of

b) P 300,000.00

them simultaneously.

c) P 100,000.00

“Never Let The Odds Keep You From Pursuing What You Know In Your Heart You Were Meant To Do.”-Leroy Satchel Paige

Page 142 of 180

Civil Law Q&As (2007-2013)

[email protected]

d) P 150,000.00

[email protected]

d) That they be liquidated and demandable.

61. Dina bought a car from Jai and delivered a check in payment of the same.

63. Which of the following statements

Has Dina paid the obligation? Why?

is correct?

a) No, not yet. The delivery of

a) All contracts are perfected by

promissory

mere consent.

notes

payable

to

order, or bills of exchange or

b) All contracts are perfected by

other mercantile documents shall

delivery of the object.

produce the effect of payment

c) All contracts are required to be in

only when they have been cashed,

writing.

or when through the fault of the

d) All contracts are required to

creditor they have been impaired.

have a valid consideration.

b) Yes, because a check is a valid legal tender of payment.

64. It is a principle which holds that parties

c) It depends. If the check is a

are bound not only by what has been

manager’s check or cashier’s check

expressly provided for in the contract but

it will produce the effect of payment.

also to the natural consequences that flow

If

out of such agreement.

it’s

an

ordinary

check,

no

payment.

a) Obligatory force of contracts

d) Yes, because a check is as good

b) Mutuality of contracts

as cash.

c) Autonomy of contracts

62. The following are the requisites of legal

d) Relativity of contracts

compensation, except: a) That each of the obligors is bound

65. It is a principle which holds that

principally and that he be the same

contracts must be binding to both parties

time a principal creditor of the

and its validity and effectivity can never be

other.

left to the will of one of the parties.

b) That both debts consist in a sum

a) Obligatory force of contracts

of money, or if the things due are

b) Mutuality of contracts

consumable, they be the same kind,

c) Autonomy of contracts

and also of the same quality if the

d) Relativity of contracts

latter has been stated. c) That the two (2) debts are not

66. It refers to the rule that a contract

yet due.

is binding

not

only between

“Never Let The Odds Keep You From Pursuing What You Know In Your Heart You Were Meant To Do.”-Leroy Satchel Paige

Page 143 of 180

Civil Law Q&As (2007-2013)

[email protected]

[email protected]

parties but extends to the heirs, successors

b) Acceptance of the offer by the

in

offeree.

interest,

and

assignees

of

the

parties, provided that the contract involved

c) Qualified/conditional acceptance

transmissible rights by their nature, or by

of the offer, which becomes counter-

stipulation or by law.

offer.

a) Obligatory force of contracts

d)

Subject

matter

becomes

b) Mutuality of contracts

illegal/impossible before acceptance

c) Autonomy of contracts

is communicated.

d) Relativity of contracts 70. Which of the following statements 67. It is rule which holds that the freedom

is correct?

of the parties to contract includes the

a) Offers in interrelated contracts

freedom

are perfected upon consent.

to

stipulate,

provided

the

stipulations are not contrary to law, morals,

b) Offers in interrelated contracts

good customs, public order or public policy.

require a single acceptance.

a) Obligatory force of contracts

c) Business

b) Mutuality of contracts

are definite

c) Autonomy of contracts

specific acceptance.

d) Relativity of contracts

d) Advertisements are only

advertisements offers

that for

invitations

require Bidders

to

make

68. The following are the ways by which

proposals and the advertiser is

innominate

not

contracts

are

regulated, except: By

the

to

accept

the

highest/lowest bidder, unless it

a) By the stipulation of the parties. b)

bound

appears otherwise.

general principles of

quasi-contracts and delicts

71.

c) By the rules governing the most

contracts (Contracts which must appear in

analogous nominate contracts.

writing), except:

d) By the customs of the place.

The

following

a) Donations of real estate or of

69. An offer becomes ineffective on any of

movables

the following grounds, except:

exceeds P 5,000.00.

a)

Death,

civil

interdiction,

are solemn

if

the

value

b) Stipulation to pay interest in

insanity/insolvency of either party

loans.

before acceptance is conveyed.

c) Sale of land through an agent (authority must be in writing).

“Never Let The Odds Keep You From Pursuing What You Know In Your Heart You Were Meant To Do.”-Leroy Satchel Paige

Page 144 of 180

Civil Law Q&As (2007-2013)

d)

[email protected]

Construction

contract

of

a

building.

[email protected]

74. The following are the characteristics of a voidable contract, except: a) Effective until set aside.

72.

The

following

are

rescissible

b) May be assailed/attacked only in

contracts, except: a)

an action for that purpose.

Entered

into

by

guardian

c) Can be confirmed or ratified.

whenever ward suffers damage more

d) Can be assailed only by either

than ¼ of value of property.

party.

b) Agreed upon in representation of absentees, if absentee suffers lesion

75.

by more than ¼ of value of property.

contracts, except:

c)

Contracts

committed

on

where

fraud

creditor

is

following

are

void

a) Pactum commissorium

(accion

b) Pactum de non alienando

pauliana). d)

The

c) Pactum leonina

Contracts

entered

into

by

d) Pacto de retro

minors. 76. The borrower in a contract of loan or 73. The following are the requisites before a

mutuum must pay interest to the lender.

contract entered into in fraud of creditors

a) If there is an agreement in

may be rescinded, except:

writing to the effect.

a) There must be credited existing

b) As a matter of course.

prior

c) If the amount borrowed is very

to

the

celebration

of

the

contract.

large.

b) There must be fraud, or at least,

d) If the lender so demands at the

the intent to commit fraud to the

maturity date.

prejudice of the creditor seeking rescission.

77.

The

liability

c) The creditor cannot in any legal

administrators

manner collect his credit (subsidiary

individual, entity or institution engaged in

character of rescission)

child care over the minor child or damage

d) The object of the contract must

caused by the acts or omissions of the

be legally in the possession of a

unemancipated minor while under their

3rd person in good faith.

supervision, instruction or custody shall be:

and

of

the

school,

teachers,

or

its the

a) Joint and subsidiary b) Principal and solidary

“Never Let The Odds Keep You From Pursuing What You Know In Your Heart You Were Meant To Do.”-Leroy Satchel Paige

Page 145 of 180

Civil Law Q&As (2007-2013)

[email protected]

[email protected]

c) Principal and joint

b) quasi-contract

d) Subsidiary and solidary.

c) civil d) natural

78. The creditor has the right to the fruits of the thing from the time:

83.

Consent

was of

given

a) the thing is delivered.

representation

b) the obligation to deliver the

authority. The contract is:

things arises.

a) voidable

c) the contract is perfected.

b) rescissible

d) the fruits are delivered.

c) void

another

by

one

in

but

without

d) unenforceable 79. If one of the parties to the contract is

84. Michael Fermin, without the authority

without juridical capacity, the contract is:

of Pascual Lacas, owner of a car, sold the

a) voidable

same car in the name of Mr. Lacas to Atty.

b) rescissible

Buko. The contract between Atty. Buko and

c) void

Mr. Lacas is ---

d) unenforceable

a) void because of the absence of consent from the owner, Mr. Lacas.

80. When both parties to the contract are

b) valid because all of the essential

minors, the contract is:

requisites of a contract are present.

a) voidable

c) unenforceable because Michael

b) rescissible

Fermin had no authority but he

c) void

sold the car in the name of Mr.

d) unenforceable

Lacas, the owner. d) rescissible because the contract

81. When the consent of one of the parties

caused lesion to Atty. Buko.

was vitiated, the contract is: a) voidable

85. Which of the following contracts is void?

b) rescissible

a) An oral sale of a parcel of land.

c) void

b) A sale of land by an agent in a

d) unenforceable

public

instrument

where

his

authority from the principal is 82. An obligation which is based on equity

oral.

and natural law is known as:

c) A donation of a wrist watch

a) pure

worth P 4,500.00.

“Never Let The Odds Keep You From Pursuing What You Know In Your Heart You Were Meant To Do.”-Leroy Satchel Paige

Page 146 of 180

Civil Law Q&As (2007-2013)

[email protected]

d) A relatively simulated contract

[email protected]

for fulfillment of the obligation even if he has not tendered payment of

86. Which of the following expresses a

the purchase price.

correct principle of law? Choose the best

c) The contract between the parties

answer.

is rescissible.

a) Failure to disclose facts when

d)

The

contract

between

the

there is a duty to reveal them, does

parties is subject to ratification

not constitute fraud.

by the parties.

b) Violence or intimidation does not render

a

contract

annullable

if

employed not by a contracting party

88. Which of the following statements is wrong?

but by a third person.

a) Creditors are protected in cases of

c) A threat to enforce one’s claim

contracts intended to defraud them.

through competent authority, if the

b)

claim is legal or just, does not vitiate

between the parties, their assign

consent.

and heirs, except in case where the

d)

Absolute

simulation

of

Contracts

take

effect

only

a

rights and obligations arising from

contract always results in a void

the contract are not transmissible

contract.

by their nature, or by stipulation or by provision of law.

87. Aligada orally offered to sell his two-

c) If a contract should contain some

hectare rice land to Balane for P 10Million.

stipulation

The

person,

offer

was

orally

accepted.

By

in

he

favor may

of

a

third

demand

agreement, the land was to be delivered

fulfillment

(through execution of a notarized Deed of

communicated his acceptance to the

Sale) and the price was to be paid exactly

obligor before its revocation.

one-month

d)

from

their

oral

agreement.

Which statement is most accurate?

In

provided

its

contracts

he

creating

real

rights, third persons who come

a) If Aligada refuses to deliver the

into possession of the object of

land on the agreed date despite

the

payment by Balane, the latter may

thereby.

not

successfully

sue

contract

are

not

bound

Aligada

because the contract is oral.

89.

Which

phrase

most

accurately

b) If Aligada refused to deliver the

completes the statement – Any third person

land, Balane may successfully sue

who induces another to violate his contract:

“Never Let The Odds Keep You From Pursuing What You Know In Your Heart You Were Meant To Do.”-Leroy Satchel Paige

Page 147 of 180

Civil Law Q&As (2007-2013)

[email protected]

[email protected]

a) shall be liable for damages only if he is a party to the same contract.

93. The attestation clause contains the

b) shall be liable for damages to

following, except:

the other contracting party.

a) the number of pages used;

c) shall not be liable for damages to

b) that the testator signed or caused

the other contracting party.

another to sign the will and every

d) shall not be liable for damages if

page thereof in the presence of the

the parties are in pari delicto.

instrumental witnesses; c) notary public;

90. The requisites of succession are as

d)

the

instrumental

witnesses

follows, except:

witnessed and signed the will and

a) Death of decedent

all the pages thereof in the presence

b) Transmissible estate

of the testator and one another.

c)

Existence

and

capacity

of

successor, designated by decedent

94.

The

following

are

the

formalities

or law

required in the execution of holographic

d) Payment of Taxes

will, except: a) Entirely written;

91. The characteristics of succession are as

b) Dated;

follows, except:

c) Signed by testator himself

a) It is a legal contract. b)

Only

property,

rights

d) Notarized by a notary public. and

obligations to the extent of the value

95. The following are the grounds for

of the inheritance are transmitted.

disallowance of wills, except:

c) The transmission takes place only

a) The formalities required by law

at the time of death.

have not been complied with.

d) The transmission takes place

b)

either by will or by operation of law.

mentally incapable of making will.

The

testator

was

insane

or

c) The will was executed through 92. The following rights are extinguished by

force or under duress, or influence

death, except:

of fear or threats.

a) Legal support

d) The will contains an attestation

b) Parental authority

clause.

c) Right to inherit d) Agency

“Never Let The Odds Keep You From Pursuing What You Know In Your Heart You Were Meant To Do.”-Leroy Satchel Paige

Page 148 of 180

Civil Law Q&As (2007-2013)

[email protected]

[email protected]

96. It is the omission in the testator’s will of

consideration, which gives him the right to

one, some or all of the compulsory heirs in

buy

direct line, whether living at the time of

property, from another person, at anytime

execution of the will or born after the death

within the agreed period, at a fixed price.

of the testator. What principle is being

What contract is being referred to?

referred to?

certain

merchandise

or

specified

a) Option Contract

a) reserva troncal

b) Contract to Sell

b) preterition

c) Contract of Sale

c) fideicommissary

d) Lease

d) disposicion captatoria 97.

Any

disposition

made

upon

the

condition that the heir shall make some

100. Which of the following contracts of sale is void?

provision in his will in favor of the testator

a) Sale of EGM’s car by KRP, EGM’s

or of any other person shall be void. Here,

agent,

both the condition and the disposition are

reduced into writing.

void. What principle is being referred to?

b) Sale of EGM’s piece of land by

whose

authority

EGM’s

not

a) reserva troncal

KRP,

b) preterition

authority

c) fideicommissary

writing.

d) disposicion captatoria

c) Sale of EGM’s car by KRP, a

is

agent,

is

not

whose

reduced

into

person stranger to EGM, without 98.

Which

phrase

most

accurately

EGM’s consent or authority.

completes the statement – If at the time the

d) Sale of EGM’s piece of land by

contract of sale is perfected, the thing

KRP, a person stranger to EGM,

which is the object of the contract has been

without EGM’s consent or authority.

entirely lost: a) the buyer bears the risk of loss. b) the contract shall be without any effect.

MCQ (November 13, 2011)

c) the seller bears the risk of loss. d) the buyer may withdraw from the contract.

for

which

(1)When does a declaration of absence of a missing person take effect?

99. A contract granting a privilege to a person,

2011 Taxation Law Exam

he

has

paid

(A) Immediately from the issuance of the declaration of absence.

a

“Never Let The Odds Keep You From Pursuing What You Know In Your Heart You Were Meant To Do.”-Leroy Satchel Paige

Page 149 of 180

Civil Law Q&As (2007-2013)

[email protected]

[email protected]

(B) 3 months after the publication of

accepted the substitution. Later, however,

the declaration of absence.

the

(C) 6 months after the publication

defaulted in his obligation. What is the

of the declaration of absence.

effect of the new debtor’s default upon the

(D) 15 days from the issuance of the

original debtor?

new

declaration of absence.

debtor

became insolvent

and

(A) The original debtor is freed of liability since novation took place

(2) The authority that school administrators

and

exercise over school children under their

obligation.

supervision,

(B) The original debtor shall pay or

instruction,

or

custody

is

called

this

relieved

him

of

his

perform the obligation with recourse (A) legal parental authority.

to the new debtor.

(B) substitute parental authority.

(C)

(C) ordinary parental authority.

liable since he gave no consent to

(D) special parental authority.

the substitution.

The

original

debtor

remains

(D) The original debtor shall pay or (3) Can future inheritance be the subject of

perform 50% of the obligation to

a contract of sale?

avoid unjust enrichment on his

(A)

No,

since

it

will

put

the

part.

predecessor at the risk of harm from a tempted buyer, contrary to public

(5) Lennie bought a business class ticket

policy.

from Alta Airlines. As she checked in, the

(B) Yes, since the death of the

manager downgraded her to economy on

decedent is certain to occur.

the ground that a Congressman had to be

(C) No, since the seller owns no

accommodated

inheritance while his predecessor

Lennie

lives.

embarrassment of the downgrade. She sued

(D) Yes, but on the condition that

the airlines for quasi-delict but Alta Airlines

the amount of the inheritance can

countered

only

governed by a contract between them, no

be

ascertained

after

the

in

the

suffered

obligations of the estate have been

quasi-delict

paid.

correct?

that,

business

the

discomfort

since

could

class.

her

arise.

Is

and

travel the

was

airline

(4) Upon the proposal of a third person, a

(A) No, the breach of contract may

new debtor substituted the original debtor

in fact be tortious as when it is

without the latter’s consent. The creditor

tainted

as

in

this

case

with

“Never Let The Odds Keep You From Pursuing What You Know In Your Heart You Were Meant To Do.”-Leroy Satchel Paige

Page 150 of 180

Civil Law Q&As (2007-2013)

[email protected]

[email protected]

arbitrariness, gross bad faith, and

(A) No, since the marriage did not

malice.

take place.

(B) No, denying Lennie the comfort

(B) Yes, since all the requisites of a

and amenities of the business class

donation

as provided in the ticket is a tortious

present.

act.

(C) No, since the donation and its

(C) Yes, since the facts show a

acceptance are not in a public

breach of contract, not a quasi-

instrument.

delict.

(D) Yes, since X freely donated the

(D)

Yes,

since

quasi-delict

of

an

immovable

are

property to Y who became its owner.

presupposes the absence of a preexisting

contractual

relation

between the parties.

(8) Rene and Lily got married after a brief courtship. After one month, Lily discovered that while Rene presented himself as a

(6)

Which

of

the

following

is

an

macho man he was actually gay. He would

indispensable requirement in an action for

not go to bed with her. He kept obscene

"quieting of title" involving real property?

magazines of nude men and always sought

The plaintiff must

the company of handsome boys. What legal

(A) be in actual possession of the

remedy does Lily have?

property.

(A) She can file an action for

(B) be the registered owner of the

annulment of marriage on ground

property.

of fraud.

(C) have legal or equitable title to

(B) She can seek a declaration of

the property.

nullity of the marriage based on

(D) be the beneficial owner of the

Rene’s psychological incapacity.

property.

(C) She can go abroad and file for divorce in a country that can grant

(7) X and Y were to marry in 3 months.

it.

Meantime,

X

(D) She has none since she had the

donated a house and lot to Y, which

opportunity to examine the goods

donation X wrote in a letter to Y. Y wrote

and freely entered into the marriage.

to

express

his

affection,

back, accepting the donation and took possession of the property. Before the

(9)

Lucio

executed

a

simple

deed

of

wedding, however, Y suddenly died of heart

donation of P50 million on time deposit

attack. Can Y’s heirs get the property?

with a bank in favor of A, B, C, D, and E,

“Never Let The Odds Keep You From Pursuing What You Know In Your Heart You Were Meant To Do.”-Leroy Satchel Paige

Page 151 of 180

Civil Law Q&As (2007-2013)

[email protected]

[email protected]

without indicating the share of each donee.

(D)

No,

since

All the donees accepted the donation in

impediment

writing. A, one of the donees, died. Will B,

inheritance to a stranger.

to

there Raul

was

no

selling

his

C, D, and E get A’s share in the money? (A) Yes, accretion will automatically

(11) When one exercises a right recognized

apply to the joint-donees in equal

by law, knowing that he thereby causes an

shares.

injustice to another, the latter is entitled to

(B) Yes, since the donor’s intention

recover damages. This is known as the

is to give the whole of P50 million to

principle of

the jointdonees in equal shares.

(A) res ipsa loquitur.

(C) No, A"s share will revert to the

(B) damnum absque injuria.

donor

(C) vicarious liability.

because

accretion

applies

only if the joint-donees are spouses.

(D) abuse of rights.

(D) No, A’s share goes to his heirs since

the

donation

did

not

provide for reversion to donor.

(12) Which of the following is NOT a basis for rendering a disinheritance defective or imperfect?

(10) Raul, Ester, and Rufus inherited a 10-

(A) Its cause comes from the guilt

hectare land from their father. Before the

of a spouse in a legal separation

land could be partitioned, however, Raul

case, the innocent-spouse having

sold his hereditary right to Raffy, a stranger

died.

to the family, for P5 million. Do Ester and

(B) The truth of its cause is denied

Rufus have a remedy for keeping the land

and

within their family?

evidence.

not

sufficiently

proved

by

(A) Yes, they may be subrogated

(C) Its cause is not authorized by the

to Raffy’s right by reimbursing to

law.

him

(D) Its cause is not specified.

within

the

required

time

what he paid Raul. (B) Yes, they may be subrogated to

(13) Manuel came to Manila and married

Raffy’s right provided they buy him

Marianne. Unknown to Marianne, Manuel

out before he registers the sale.

had been previously convicted in Palawan

(C) No, they can be subrogated to

of theft and served time for it. After

Raffy’s

Marianne

right

conformity.

only

with

his

learned

of

his

previous

conviction, she stopped living with him. Can Marianne seek the annulment of the

“Never Let The Odds Keep You From Pursuing What You Know In Your Heart You Were Meant To Do.”-Leroy Satchel Paige

Page 152 of 180

Civil Law Q&As (2007-2013)

[email protected]

[email protected]

marriage based on Manuel’s nondisclosure

since

Philippine

of his previous crime?

recognize divorce.

law

does

not

(A) No, since the assumption is that

(D) All the children are legitimate

marriage forgives all past wrongs.

since they were born of the same

(B) Yes, since the non-disclosure

father and mother.

of that crime is the equivalent of fraud,

which

is

a

ground

for

(15) Who can make a donation?

annulment.

(A) All persons who can enter into

(C) No, in case of doubt, the law

contracts and dispose of their

must be construed to preserve the

property.

institution of marriage.

(B) All persons who are of legal age

(D) No, since Manuel already served

and suffer from no civil interdiction.

the penalty for his crime.

(C) All persons who can make a last will and testament.

(14) Arthur and Helen, both Filipinos, got

(D) All persons, whether natural or

married and had 2 children. Arthur later

artificial, who own property.

worked in Rome where he acquired Italian citizenship. He got a divorce from Helen in

(16) The liability of the partners, including

Rome but, on returning to the Philippines,

industrial

he realized his mistake, asked forgiveness

contracts entered into in its name and for

of his wife, and resumed living with her.

its account, when all partnership assets

They had 2 more children. What is the

have been exhausted is

status of their 4 children?

partners

for

partnership

(A) Pro-rata.

(A) The children born before the

(B) Joint.

divorce are legitimate but those

(C) Solidary.

born after it are not since Arthur

(D) Voluntary.

got the divorce when he had ceased to be a Filipino.

(17) When can a missing person who left

(B) The divorce rendered illegitimate

someone to administer his property be

the children born before it since the

declared an absentee by the court? When

marriage that begot them had been

he has been missing for

nullified.

(A) 2 years from the receipt of the

(C) The children born before and

last news about him.

after the divorce are all legitimate

(B) 7 years from the receipt of the last news about him.

“Never Let The Odds Keep You From Pursuing What You Know In Your Heart You Were Meant To Do.”-Leroy Satchel Paige

Page 153 of 180

Civil Law Q&As (2007-2013)

[email protected]

[email protected]

(C) 10 years from the receipt of the

principal two days after the principal died,

last news about him.

an event that neither the agent nor the

(D) 5 years from the receipt of the

buyer knew at the time of the sale. What is

last news about him.

the standing of the sale? (A) Voidable.

(18) Which of the following claims against

(B) Valid.

the debtor enjoys preference over the others

(C) Void.

with respect to his specific immovable

(D) Unenforceable.

property and real rights? (A) Unpaid price of real property

(21) Spouses A and B leased a piece of land

sold, upon the immovable property.

belonging to B's parents for 25 years. The

(B) Mortgage credits recorded in the

spouses built their house on it worth

registry

P300,000.00. Subsequently, in a case that

of

property,

upon

the

mortgaged real estate.

C filed against A and B, the court found the

(C) Taxes due, upon the land or

latter liable to C for P200,000.00. When the

building.

sheriff was attaching their house for the

(D) Expenses for the preservation

satisfaction of the judgment, A and B

and improvement of property, when

claimed that it was exempt from execution,

the law authorizes reimbursement,

being a family home. Is this claim correct?

upon the preserved or improved

(A) Yes, because while B’s parents

immovable.

own the land, they agreed to have their

daughter

build

her

family

(19) When bilateral contracts are vitiated

home on it.

with vices of consent, they are rendered

(B) No, because there is no judicial

(A) rescissible.

declaration that it is a family home.

(B) void.

(C) No, since the land does not

(C) unenforceable.

belong to A and B, it cannot

(D) voidable.

qualify as a family home. (D) Yes, because the A and B’s

(20) An agent, authorized by a special

family actually lives in that house.

power of attorney to sell a land belonging to the principal succeeded in selling the same

(22) Solomon sold his coconut plantation to

to a buyer according to the instructions

Aragon, Inc. for P100 million, payable in

given the agent. The agent executed the

installments of P10 million per month with

deed of absolute sale on behalf of his

6% interest per annum. Solomon married

“Never Let The Odds Keep You From Pursuing What You Know In Your Heart You Were Meant To Do.”-Leroy Satchel Paige

Page 154 of 180

Civil Law Q&As (2007-2013)

[email protected]

[email protected]

Lorna after 5 months and they chose

(24) When A and B married, they chose

conjugal partnership of gains to govern

conjugal partnership of gains to govern

their

they

their property relations. After 3 years, B

married, Aragon had an unpaid balance of

succeeded in getting her marriage to A

P50 million plus interest in Solomon’s

annulled

favor. To whom will Aragon’s monthly

psychological incapacity. What liquidation

payments go after the marriage?

procedure will they follow in disposing of

property

relations.

When

(A) The principal shall go to the conjugal

of

the

latter’s

their assets? will

interests to Solomon.

governing

the

(B) Both principal and interests

conjugal partnership of gains where

shall go to Solomon since they are

the party who acted in bad faith

his exclusive properties.

forfeits his share in the net profits.

(C) Both principal and interests

(B) Since the marriage has been

shall go to the conjugal partnership

declared

since these become due after the

liquidation of absolute community of

marriage.

property shall be followed. principal

but

ground

They

(D) The

partnership

on

shall

go

the

(A)

to

(C) The

follow

the

liquidation

void,

the

liquidation

of

rule

of

applies

rule

a

since

a

for

co-

Solomon but the interests to the

ownership

the

conjugal partnership.

annulment brought their property relation under the chapter on

(23) X and Y, although not suffering from

property

any impediment, cohabited as husband and

marriage.

wife

(D)

without

the

benefit

of

marriage.

Following the birth of their child, the couple

regimes

The

law

on

without

liquidation

of

partnerships applies.

got married. A year after, however, the

(25) X and Y agreed verbally before their

court annulled the marriage and issued a

marriage

decree of annulment. What is the present

illegitimate child of Y and (b) on the

status of the child?

economic regime that will govern X and Y’s

(a)

on

the

paternity

of

the

(A) Legitimated.

property relations. Is the verbal agreement

(B) Illegitimate.

valid?

(C) Natural child.

(A) No,

because

a

marriage

(D) Legitimate.

settlement to be valid should be in writing.

“Never Let The Odds Keep You From Pursuing What You Know In Your Heart You Were Meant To Do.”-Leroy Satchel Paige

Page 155 of 180

Civil Law Q&As (2007-2013)

(B)

Yes,

[email protected]

since

ante-nuptial

[email protected]

(B) Illegitimate, because by the color

agreements need not be in writing.

of its skin, the child could not

(C)

possibly be that of Fidel.

No,

settlement

because cannot

a

marriage

include

an

(C) Legitimate, because the child

agreement on the paternity of an

was born within a valid marriage.

illegitimate child.

(D) Legitimate, because Fidel agreed

(D) Yes, since even if it is not a valid

to treat the child as his own after

marriage settlement, it is a valid

Gloria told him who the father was.

verbal contract. (28) The husband’s acts of forcibly ejecting (26) Spouses X and Y have a minor

his wife without

daughter, Z, who needs support for her

conjugal dwelling and refusing to take her

education.

back constitutes

Both

X

and

Y,

who

are

just

cause from the

financially distressed, could not give the

(A) desertion.

needed support to Z. As it happens, Z’s

(B) recrimination.

other relatives are financially capable of

(C) constructive abandonment.

giving that support. From whom may Z first

(D) de facto separation.

rightfully demand support? From her (A) grandfather.

(29) In his will, the testator designated X as

(B) brother.

a legatee to receive P2 million for the

(C) uncle.

purpose of buying an ambulance that the

(D) first cousin.

residents of his Barangay can use. What kind of institution is this?

(27) Fidel, a Filipino with fair complexion,

(A) a fideicomissary institution.

married Gloria. Before the marriage, Gloria

(B) a modal institution.

confessed to Fidel that she was two-month

(C) a conditional institution.

pregnant with the child of a black African

(D) a collective institution.

who had left the country for good. When the child was born, Fidel could not accept it

(30) X insured himself for P5 million,

being too black in complexion. What is the

designating

status of the child?

beneficiary.

(A)

Illegitimate,

because

Y,

his The

wife,

as

his

designation

sole was

Gloria

irrevocable. A few years later, X had their

confessed that the child is not

marriage annulled in court on the ground

Fidel’s.

that Y had an existing prior marriage. X

“Never Let The Odds Keep You From Pursuing What You Know In Your Heart You Were Meant To Do.”-Leroy Satchel Paige

Page 156 of 180

Civil Law Q&As (2007-2013)

[email protected]

[email protected]

subsequently died, Is Y entitled to the

(C) Marlon gets 1/2 and Cecilia

insurance benefits?

gets 1/2.

(A) Yes, since the insurance was not

(D) Marlon gets 3/4 and Cecilia 1/4.

dependent on the marriage. (B) Yes, since her designation as

(33) Contracts take effect only between the

beneficiary was irrevocable.

parties or their assigns and heirs, except

(C) No, X’s designation of Y is

where the rights and obligations arising

revoked by operation of law upon

from the contract are not transmissible by

the annulment of their marriage

their nature, by stipulation, or by provision

based on Y’s fault.

of law. In the latter case, the assigns or the

(D)

Yes,

since

revocation,

X’s

without

judicial

designation

of

Y

remains valid and binding.

heirs are not bound by the contracts. This is known as the principle of (A) Relativity of contracts.

(31) May a spouse freely donate communal

(B) Freedom to stipulate.

or conjugal property without the consent of

(C) Mutuality of contracts.

the other?

(D) Obligatory force of contracts.

(A) Absolutely not, since the spouses co-own such property.

(34) A buyer ordered 5,000 apples from the

(B) Yes, for properties that the

seller at P20 per apple. The seller delivered

family

6,000 apples. What are the rights and

may

spare,

regardless

of

value.

obligations of the buyer?

(C) Yes, provided the donation is

(A) He can accept all 6,000 apples

moderate

and pay the seller at P20 per

and

intended

for

charity or family rejoicing.

apple.

(D) Yes, in a donation mortis causa

(B) He can accept all 6,000 apples

that the donor may still revoke in

and pay a lesser price for the 1,000

his lifetime.

excess apples. (C) He can keep the 6,000 apples

(32) The decedent died intestate leaving an

without paying for the 1,000 excess

estate of P10 million. He left the following

since

heirs: a) Marlon, a legitimate child and b)

anyway.

Cecilia, the legal spouse. Divide the estate.

(D)

the He

seller

can

delivered

cancel

the

them whole

(A) Marlon gets 1/4 and Cecilia gets

transaction since the seller violated

3/4.

the terms of their agreement.

(B) Marlon gets 2/3 and Cecilia 1/3.

“Never Let The Odds Keep You From Pursuing What You Know In Your Heart You Were Meant To Do.”-Leroy Satchel Paige

Page 157 of 180

Civil Law Q&As (2007-2013)

[email protected]

from

[email protected]

(35) Lino entered into a contract to sell with

suffered

poisoning

caused

by

a

Ramon, undertaking to convey to the latter

noxious substance found in the sardines.

one of the five lots he owns, without

Mylene filed a case for damages against

specifying which lot it was, for the price of

Acme. Which of the following defenses will

P1 million. Later, the parties could not

hold?

agree which of five lots he owned Lino

(A) The expiry date of the "Sards"

undertook to sell to Ramon. What is the

was clearly printed on its can,

standing of the contract?

still the store sold and Mylene

(A) Unenforceable.

bought it.

(B) Voidable.

(B) Mylene must have detected the

(C) Rescissible.

noxious substance in the sardines

(D) Void.

by smell, yet she still ate it. (C) Acme had no transaction with

(36) Knowing that the car had a hidden

Mylene; she bought the "Sards" from

crack in the engine, X sold it to Y without

a store, not directly from Acme.

informing the latter about it. In any event,

(D) Acme enjoys the presumption of

the deed of sale expressly stipulated that X

safeness of its canning procedure

was not liable for hidden defects. Does Y

and Mylene has not overcome such

have the

presumption.

right

to

demand

from X

a

reimbursement of what he spent to repair the engine plus damages?

(38) Fernando executed a will, prohibiting

(A) Yes. X is liable whether or not he

his wife Marina from remarrying after his

was aware of the hidden defect.

death, at the pain of the legacy of P100

(B) Yes, since the defect was not

Million in her favor becoming a nullity. But

hidden; X knew of it but he acted

a year after Fernando’s death, Marina was

in bad faith in not disclosing the

so overwhelmed with love that she married

fact to Y.

another man. Is she entitled to the legacy,

(C) No, because Y is in estoppel,

the amount of which is well within the

having changed engine without prior

capacity of the disposable free portion of

demand.

Fernando’s estate?

(D)

No,

because

Y

waived

the

warranty against hidden defects.

(A) Yes, since the prohibition against remarrying is absolute, it is deemed

(37) Acme Cannery produced sardines in

not written.

cans known as "Sards." Mylene bought a

(B) Yes, because the prohibition is

can of Sards from a store, ate it, and

inhuman

and

oppressive

and

“Never Let The Odds Keep You From Pursuing What You Know In Your Heart You Were Meant To Do.”-Leroy Satchel Paige

Page 158 of 180

Civil Law Q&As (2007-2013)

[email protected]

[email protected]

violates Marina’s rights as a free

(A) It

is

an

ordinary

woman.

since it was not given to the bride

(C) No, because the nullity of the

or groom.

prohibition also nullifies the legacy.

(B) It is donation propter nuptias

(D) No, since such prohibition is

since it was given with the marriage

authorized by law and is not

in mind.

repressive; she could remarry but

(C) It is an indirect donation propter

must give up the money.

nuptias

since

the

donation

bride

would

eventually inherit the property from (39) X, the owner, constituted a 10-year

her parents.

usufruct on his land as well as on the

(D) It is a remunatory donation.

building standing on it in Y’s favor. After flood totally destroyed the building 5 years

(41) X and Y, both Filipinos, were married

later, X told Y that an act of God terminated

and resided in Spain although they intend

the usufruct and that he should vacate the

to return to the Philippines at some future

land. Is X, the owner of the land, correct?

time. They have not executed any marriage

(A)

No,

since

the

building

was

destroyed through no fault of Y.

settlements.

What

law

governs

their

property relations?

(B) No, since Y still has the right

(A)

to use the land and the materials

Spanish law and Philippine law.

left on it.

(B) Philippine law since they are

(C) Yes, since Y cannot use the land

both Filipinos.

without the building.

(C) No regime of property relations

(D) Yes, since the destruction of the

will apply to them.

building

(D) Spanish law since they live in

without

the

X’s

fault

terminated the usufruct.

They

may

choose

between

Spain.

(40) In gratitude, the groom’s parents made

(42) Birth determines personality. Death

a donation of a property in writing to the

extinguishes it. Under what circumstances

bride’s

may the personality of a deceased person

parents

shortly

children’s

wedding.

The

accepted.

What

the

donation?

is

before

their

donation

was

nature

of

the

continue to exist? (A) In case of re-appearance of a missing person presumed dead.

“Never Let The Odds Keep You From Pursuing What You Know In Your Heart You Were Meant To Do.”-Leroy Satchel Paige

Page 159 of 180

Civil Law Q&As (2007-2013)

[email protected]

[email protected]

(B) In protecting the works of a

(C) the brother or sister in need

deceased under intellectual property

stops

laws.

reason.

(C)

In

case

of

presumptive death

declaration of

of

a missing

spouse.

schooling

(D) the

need for

without

valid

support

of

a

brother or sister, already of age, is due to the latter's fault.

(D) In the settlement of the estate of a deceased person.

(45) Virgilio owned a bare and simple swimming pool in his garden. MB, a 7-year

(43) Six tenants sued X, the landowner, for

old child, surreptitiously entered the garden

willfully denying them water for their farms,

and merrily romped around the ledges of

which water happened to flow from land

the pool. He accidentally tripped, fell into

under X’s control, his intention being to

the pool, and drowned. MB’s parents sued

force them to leave his properties. Is X

Virgilio for damages arising from their

liable for his act and why?

child’s death, premised on the principle of

(A) No, because the tenants must be

"attractive nuisance". Is Virgilio liable for

content with waiting for rainfall for

the death of MB?

their farms.

(A) No, the child was 7 years old and

(B) No, since X owns both the land

knew the dangers that the pool

and the water.

offered.

(C) Yes, because the tenants’ farms

(B)

have the natural right of access to

nuisance, Virgilio had the duty to

water wherever it is located.

prevent children from coming near

(D) Yes, since X willfully caused

it.

injury to his tenants contrary to

(C) No, since the pool was bare

morals, good customs or public

and had no enticing or alluring

policy.

gadgets, floats, or devices in it

Yes,

being

an

attractive

that would attract a 7-year old (44)

Illegitimate

brothers

and

sisters,

child.

whether of full or half-blood, are bound to

(D) Yes, since Virgilio did not cover

support each other, EXCEPT when

the swimming pool while not in use

(A) the brother or sister who needs

to prevent children from falling into

support lives in another place.

it.

(B) such brothers and sisters are not recognized by their father.

“Never Let The Odds Keep You From Pursuing What You Know In Your Heart You Were Meant To Do.”-Leroy Satchel Paige

Page 160 of 180

Civil Law Q&As (2007-2013)

[email protected]

[email protected]

(46) The term of a 5-year lease contract

(D) Yes, the donation is not deemed

between X the lessor and Y the lessee,

made until the suspensive condition

where rents were paid from month to

has been fulfilled.

month, came to an end. Still, Y continued using the property with X’s consent. In

(48)

Illegitimate

children,

those

not

such a case, it is understood that they

recognized by their biological fathers, shall

impliedly renewed the lease

use the surname of their

(A) from month to month under

(A) biological father subject to no

the same conditions as to the

condition.

rest.

(B) mother or biological father, at

(B)

under

the same

terms and

the mother’s discretion.

conditions as before.

(C) mother.

(C) under the same terms except the

(D)

rent which they or the court must

judicially opposes it.

biological

father

unless

he

fix. (D) for only a year, with the rent

(49) Asiong borrowed P1 million from a

raised by 10% pursuant to the

bank, secured by a mortgage on his land.

rental control law.

Without his consent, his friend Boyong paid the whole loan. Since Asiong benefited from

(47)

a

the payment, can Boyong compel the bank

valuable lot to the municipality on the

to subrogate him in its right as mortgagee

condition that it will build a public school

of Asiong's land?

on

Rex,

such

acceptance

a

philanthropist,

lot

within of

the

donated

2 years from

its

donation.

The

(A) No, but the bank can foreclose and pay Boyong back.

municipality properly accepted the donation

(B) No,

but did not yet build the public school after

Asiong’s

2 years. Can Rex revoke the donation?

approval.

(A) Yes,

since

the

donation

since

Boyong

loan

paid

without

for his

is

(C) Yes, since a change of creditor

subject to a resolutory condition

took place by novation with the

which was not fulfilled.

bank’s consent.

(B) No, but Rex is entitled to recover

(D) Yes, since it is but right that

the value of the land from the

Boyong be able to get back his

municipality.

money and, if not, to foreclose the

(C) No, the transfer of ownership has

mortgage in the manner of the bank.

been completed.

“Never Let The Odds Keep You From Pursuing What You Know In Your Heart You Were Meant To Do.”-Leroy Satchel Paige

Page 161 of 180

Civil Law Q&As (2007-2013)

[email protected]

[email protected]

(50) Congress passed a law imposing taxes

(D) Yes, since Fernando was a

on income earned out of a particular

solidary creditor, payment to him

activity that was not previously taxed. The

extinguished the obligation.

law, however, taxed incomes already earned within the fiscal year when the law took

(52) What happens to the property regimes

effect. Is the law valid?

that were subsisting under the New Civil

(A) No, because laws are intended to

Code when the Family Code took effect?

be prospective, not retroactive.

(A) The original property regimes

(B) No, the law is arbitrary in that it

are

taxes income that has already been

effective.

spent.

(B) Those enjoying specific regimes

(C) Yes, since tax laws are the

under the New Civil Code may adopt

lifeblood of the nation.

the regime of absolute community of

(D) Yes, tax laws are an exception;

property under the Family Code.

they

(C) Those that married under the

can

be

given

retroactive

effect.

immutable

and

remain

New Civil Code but did not choose any of its regimes shall now be

(51)

Rudolf

Rodrigo

borrowed

and

Fernando

P1

million

who

from

acted

as

governed by the regime of absolute community of property.

solidary creditors. When the loan matured,

(D) They are superseded by the

Rodrigo wrote a letter to Rudolf, demanding

Family Code which has retroactive

payment of the loan directly to him. Before

effect.

Rudolf could comply, Fernando went to see

(53) The testator executed a will following

him personally to collect and he paid him.

the formalities required by the law on

Did Rudolf make a valid payment?

succession without designating any heir.

(A) No, since Rudolf should have

The only testamentary disposition in the

split the payment between Rodrigo

will is the recognition of the testator's

and Fernando.

illegitimate child with a popular actress. Is

(B) No, since Rodrigo, the other

the will valid?

solidary creditor, already made a

(A) Yes, since in recognizing his

prior demand for payment from

illegitimate child, the testator has

Rudolf.

made him his heir.

(C) Yes, since the payment covers

(B) No, because the non-designation

the whole obligation.

of heirs defeats the purpose of a will.

“Never Let The Odds Keep You From Pursuing What You Know In Your Heart You Were Meant To Do.”-Leroy Satchel Paige

Page 162 of 180

Civil Law Q&As (2007-2013)

[email protected]

[email protected]

(C) No, the will comes to life only

(C) Voidable, because the Judge

when

acted

the

proper

heirs

are

instituted. (D) Yes,

beyond

his

territorial

jurisdiction and is administratively

the

illegitimate

recognition heir

is

of

an

an

liable for the same.

ample

(D) Void, because the Judge did not

reason for a will.

solemnize the marriage within the premises of his court.

(54) A left B, his wife, in the Philippines to work in Egypt but died in that country after

(56) X and Y, Filipinos, got married in Los

a year’s continuous stay. Two months after

Angeles, USA, using a marriage license

A’s death, B gave birth to a child, claiming

issued by the Philippine consul in Los

it is A’s child. Who can assail the legitimacy

Angeles, acting as Civil Registrar. X and Y

of the child?

did not know that they were first cousins

(A) A’s other heirs apart from B.

because their mothers, who were sisters,

(B) The State which has interest in

were

the welfare of overseas contract

young. Since X did not want to continue

workers.

with the relation when he heard of it, he left

(C) Any one who is outraged by B’s

Y, came to the Philippines and married Z.

claim.

Can X be held liable for bigamy?

(D) No one since A died.

separated

when

they

were

quite

(A) No since X’s marriage to Y is void ab initio or did not exist.

(55) QR and TS who had a marriage license

(B) No since X acted in good faith,

requested a newly appointed Judge in

conscious that public policy did not

Manila to marry them on the beach of

approve of marriage between first

Boracay.

cousins.

Since

the

Judge

maintained

Boracay as his residence, he agreed. The

(C) Yes since he married Z without

sponsors were all public officials. What is

first

the status of the marriage.

declaration

(A) Valid,

since

the

improper

securing of

a nullity

judicial of

his

marriage to Y.

venue is merely an irregularity;

(D) Yes since his first marriage to Y

all

in Los Angeles is valid.

the

elements

of

a

valid

marriage are present. (B) Void, because the couple did not

(57) Allan bought Billy’s property through

get local permit for a beach wedding.

Carlos, an agent empowered with a special power of attorney (SPA) to sell the same.

“Never Let The Odds Keep You From Pursuing What You Know In Your Heart You Were Meant To Do.”-Leroy Satchel Paige

Page 163 of 180

Civil Law Q&As (2007-2013)

[email protected]

[email protected]

When Allan was ready to pay as scheduled,

(B) No, the buyer is entitled to a

Billy called, directing Allan to pay directly

customary 30-day extension of his

to him. On learning of this, Carlos, Billy's

obligation to take delivery of the

agent, told Allan to pay through him as his

goods.

SPA

(C) No, since there was no express

provided

and

to

protect

his

commission. Faced with two claimants,

agreement

Allan consigned the payment in court. Billy

rescission.

protested, contending that the consignation

(D)

is ineffective since no tender of payment

determine that Y was not justified in

was made to him. Is he correct?

failing to appear.

No,

regarding the

seller

automatic should

first

(A) No, since consignation without

(59) The wife filed a case of legal separation

tender of payment is allowed in

against her husband on the ground of

the face of the conflicting claims

sexual infidelity

on the plaintiff.

without previously exerting earnest efforts

(B) Yes, as owner of the property

to come to a compromise with him. The

sold, Billy can demand payment

judge dismissed the case for having been

directly to himself.

filed without complying with a condition

(C)

Yes,

since

Allan

made

no

precedent. Is the dismissal proper?

announcement of the tender.

(A) No, efforts at a compromise will

(D) Yes, a tender of payment is

only deepen the wife’s anguish.

required for a valid consignation.

(B) No, since legal separation like validity of marriage is not subject

(58) X sold Y 100 sacks of rice that Y was to

to

compromise

pick up from X’s rice mill on a particular

purposes of filing.

date. Y did not, however, appear on the

(C) Yes, to avoid a family feud that is

agreed date to take delivery of the rice. After

hurtful to everyone.

one week, X automatically rescinded the

(D) Yes, since the dispute could have

sale without notarial notice to Y. Is the

been

rescission valid?

agreeing to legal separation.

settled

agreement

with

the

for

parties

(A) Yes, automatic rescission is allowed

since,

character

of

consumables, deteriorate.

having

the

(60) An Australian living in the Philippines

movables

and

acquired shares of stock worth P10 million

easily

in food manufacturing companies. He died

rice

can

in Manila, leaving a legal wife and a child in Australia and a live-in partner with whom

“Never Let The Odds Keep You From Pursuing What You Know In Your Heart You Were Meant To Do.”-Leroy Satchel Paige

Page 164 of 180

Civil Law Q&As (2007-2013)

[email protected]

[email protected]

he had two children in Manila. He also left

(D) Yes, after full payment, the

a will, done according to Philippine laws,

action became imprescriptible.

leaving all his properties to his live-in partner and their children. What law will

(62) A court declared Ricardo, an old

govern the validity of the disposition in the

bachelor, an absentee and appointed Cicero

will?

administrator of his property. After a year, (A) Australia law since his legal wife

it was discovered that Ricardo had died

and legitimate child are Australians

abroad. What is the effect of the fact of his

and domiciled in Australia.

death

(B) Australian

administration

of

his

property?

intrinsic validity of the provisions

(A)

of

absentee but a deceased person,

will

is

since

the

the

a

law

on

governed

by

the

With

Ricardo

no

longer

an

decedent’s national law.

Cicero will cease to be administrator

(C) Philippine law since the decedent

of his properties.

died in Manila and he executed his

(B) The administration shall be

will according to such law.

given

(D)

Philippine

law

since

by

the

court

over

the

the

jurisdiction

decedent’s properties are in the

proceedings

to

Philippines.

administrator

whom

having intestate

a

new it

will

appoint. (61) X bought a land from Y, paying him

(C) Cicero automatically becomes

cash. Since they were friends, they did not

administrator

execute any document of sale. After 7 years,

until judicially relieved.

the heirs of X asked Y to execute a deed of

(D) Cicero’s alienations of Ricardo's

absolute sale to formalize the verbal sale to

property will be set aside.

of

Ricardo’s

estate

their father. Unwilling to do so, X’s heirs filed an action for specific performance

(63) Baldo, a rejected suitor, intimidated

against Y. Will their action prosper?

Judy into marrying him. While she wanted

(A) No, after more than 6 years,

to question the validity of their marriage

the action to enforce the verbal

two years after the intimidation ceased,

agreement has already elapsed.

Judy decided in the meantime to freely

(B) No, since the sale cannot under

cohabit with Baldo. After more than 5 years

the Statute of Frauds be enforced.

following their wedding, Judy wants to file a

(C) Yes, since X bought the land and

case for annulment of marriage against

paid Y for it.

“Never Let The Odds Keep You From Pursuing What You Know In Your Heart You Were Meant To Do.”-Leroy Satchel Paige

Page 165 of 180

Civil Law Q&As (2007-2013)

[email protected]

[email protected]

Baldo on ground of lack of consent. Will her

all heirs in the higher level are disqualified

action prosper?

or unable to inherit?

(A) Yes, the action for annulment is

(A) Nephews and nieces.

imprescriptible.

(B) Brothers and sisters.

(B) No, since the marriage was

(C) State.

merely voidable and Judy ratified

(D) Other collateral relatives up to

it by freely cohabiting with Baldo

the 5th degree of consanguinity.

after the force and intimidation had ceased.

(66) Roy and Carlos both undertook a

(C) No, since the action prescribed 5

contract to deliver to Sam in Manila a boat

years

docked in Subic. Before they could deliver

from

the

date

of

the

celebration of the marriage.

it, however, the boat sank in a storm. The

(D) Yes, because the marriage was

contract provides that fortuitous event shall

celebrated without Judy's consent

not exempt Roy and Carlos from their

freely given.

obligation. Owing to the loss of the motor boat, such obligation is deemed converted

(64) Is the wife who leaves her husband

into one of indemnity for damages. Is the

without just cause entitled to support?

liability of Roy and Carlos joint or solidary?

(A)

No,

because

must

(A) Neither solidary nor joint since

always be submissive and respectful

they cannot waive the defense of

to the husband.

fortuitous event to which they are

(B) Yes. The marriage not having

entitled.

been

(B)

dissolved,

the

wife

the

husband

Solidary

or

joint

upon

the

continues to have an obligation to

discretion of Sam.

support his wife.

(C) Solidary since Roy and Carlos

(C) No, because in

leaving the

failed to perform their obligation to

conjugal home without just cause,

deliver the motor boat.

she forfeits her right to support.

(D) Joint since the conversion of

(D) Yes, since the right to receive

their liability to one of indemnity

support

for damages made it joint.

is

not

subject

to

any

condition. (67) Joanne married James, a person with (65) In the order of intestate succession

no known relatives. Through James' hard

where the decedent is legitimate, who is the

work, he and his wife Joane prospered.

last intestate heirs or heir who will inherit if

When

James

died,

his

estate

alone

“Never Let The Odds Keep You From Pursuing What You Know In Your Heart You Were Meant To Do.”-Leroy Satchel Paige

Page 166 of 180

Civil Law Q&As (2007-2013)

[email protected]

[email protected]

amounted to P100 million. If, in his will,

(70) Ric and Josie, Filipinos, have been

James designates Joanne as his only heir,

sweethearts for 5 years. While working in a

what will be the free portion of his estate.

European country where the execution of

(A) Joanne gets all; estate has no

joint wills are allowed, the two of them

free portion left.

executed a joint holographic will where they

(B) Joanne gets 1/2; the other

named each other as sole heir of the other

half is free portion.

in case either of them dies. Unfortunately,

(C) Joanne gets 1/3; the remaining

Ric died a year later. Can Josie have the

2/3 is free portion.

joint will successfully

(D) Joanne gets 1/4; the remaining

Philippines?

3/4 is free portion.

probated

in the

(A) Yes, in the highest interest of comity of nations and to honor the

(68) A warranty inherent in a contract of

wishes of the deceased.

sale, whether or not mentioned in it, is

(B) No,

known as the

prohibits the execution of joint

since

Philippine

law

(A) warranty on quality.

wills and such law is binding on

(B) warranty against hidden defects.

Ric and Josie even abroad.

(C) warranty against eviction.

(C) Yes, since they executed their

(D) warranty in merchantability.

joint will out of mutual love and care,

values

that

the

generally

(69) The doctrine of stare decisis prescribes

accepted principles of international

adherence

law accepts.

to

precedents

in

order

to

promote the stability of the law. But the

(D) Yes, since it is valid in the

doctrine can be abandoned

country

where

it

was

executed,

(A) When adherence to it would

applying the principle of "lex loci

result in the Government’s loss of its

celebrationis."

case. (B) When the application of the

(71) ML inherited from his father P5 million

doctrine

in legitime but he waived it in a public

would

cause

great

prejudice to a foreign national.

instrument in favor of his sister QY who

(C) When necessary to promote the

accepted the waiver in writing. But as it

passage of a new law.

happened, ML borrowed P6 million from PF

(D) When

the

precedent

has

before the waiver. PF objected to the waiver

ceased to be beneficial and useful.

and filed an action for its rescission on the ground that he had the right to ML’s P5

“Never Let The Odds Keep You From Pursuing What You Know In Your Heart You Were Meant To Do.”-Leroy Satchel Paige

Page 167 of 180

Civil Law Q&As (2007-2013)

[email protected]

[email protected]

million legitime as partial settlement of

(B) Yes,

insofar

as

what ML owed him since ML has proved to

acknowledged

be insolvent. Does PF, as creditor, have the

illegitimate child.

right to rescind the waiver?

(C) None, since the marriage did not

Mary

Arnold as

his

(A) No, because the waiver in favor

take place.

of his sister QY amounts to a

(D) Yes, if they acquired properties

donation and she already accepted

while living together as husband

it.

and wife.

(B) Yes,

because

the

waiver

is

(73) Joseph, a 17-year old Filipino, married

prejudicial to the interest of a

Jenny, a 21-year old American in Illinois,

third person whose interest is

USA, where the marriage was valid. Their

recognized by law.

parents gave full consent to the marriage of

(C) No, PF must wait for ML to

their children. After three years, Joseph

become solvent and, thereafter, sue

filed a petition in the USA to promptly

him for the unpaid loan.

divorce Jenny and this was granted. When

(D) Yes, because a legitime cannot

Joseph turned 25 years, he returned to the

be waived in favor of a specific heir;

Philippines and married Leonora. What is

it must be divided among all the

the status of this second marriage?

other heirs.

(A) Void, because he did not cause the

judicial

issuance

of

(72) While engaged to be married, Arnold

declaration of the nullity of his

and

first marriage to Jenny before

Josephine

agreed

in

a

public

instrument to adopt out the economic

marrying Leonora.

regime of absolute community of property.

(B) Valid, because Joseph's marriage

Arnold

to Jenny is void, he being only 17

acknowledged

instrument Mary,

is

that

the

Josephine’s

daughter

Josephine died before the marriage could

Leonora has all the elements of a

take place. Does the marriage settlement

valid marriage.

have any significance?

(D) Void, because Joseph is still

since

the

child.

years of age when he married her. (C) Valid, because his marriage to

None,

illegitimate

same But

(A)

his

in

instrument

considered married to Jenny since

containing the marriage settlement

the Philippines does not recognize

is essentially void for containing an

divorce.

unrelated matter.

“Never Let The Odds Keep You From Pursuing What You Know In Your Heart You Were Meant To Do.”-Leroy Satchel Paige

Page 168 of 180

Civil Law Q&As (2007-2013)

[email protected]

(74) T died intestate, leaving an estate of

(76)

P9,000,000. He left as heirs three legitimate

brother, Y, authorizing him to sell X’s

children, namely, A, B, and C. A has two

parcel of land in Pasay. X sent the title to Y

children, D and E. Before he died, A

by courier service. Acting for his brother, Y

irrevocably repudiated his inheritance from

executed a notarized deed of absolute sale

T in a public instrument filed with the

of the land to Z after receiving payment.

court. How much, if any, will D and E, as

What is the status of the sale?

A’s children, get from T’s estate?

X,

who

[email protected]

was

abroad,

phoned his

(A) Valid, since a notarized deed of

(A) Each of D and E will get

absolute

P1,500,000

by

transaction and full payment was

representation

since

right their

of father

sale

covered

the

made.

repudiated his inheritance.

(B) Void,

since

X

should

have

(B) Each of D and E will get

authorized agent Y in writing to

P2,225,000 because they will inherit

sell the land.

from the estate equally with B and

(C) Valid, since Y was truly his

C.

brother X’s agent and entrusted

(C) D and E will get none because

with the title needed to effect the

of the repudiation; "B" and "C"

sale.

will get A’s share by right of

(D) Valid, since the buyer could file

accretion.

an action to compel X to execute a

(D) Each of D and E will get

deed of sale.

P2,000,000 because the law gives them some advantage due to the

(77) In a true pacto de retro sale, the title

demise of "A".

and ownership of the property sold are immediately vested in the vendee a retro

(75) No decree of legal separation can be

subject only to the resolutory condition of

issued

repurchase by the vendor a retro within the (A) unless the children’s welfare is attended to first. (B) without

prior

stipulated period. This is known as (A) equitable mortgage.

efforts

at

(B) conventional redemption.

reconciliation shown to be futile.

(C) legal redemption.

(C) unless the court first directs

(D) equity of redemption.

mediation of the parties.

(78) A natural obligation under the New

(D)

Civil Code of the Philippines is one which

without

prior

investigation

conducted by a public prosecutor.

“Never Let The Odds Keep You From Pursuing What You Know In Your Heart You Were Meant To Do.”-Leroy Satchel Paige

Page 169 of 180

Civil Law Q&As (2007-2013)

(A)

the

[email protected]

obligor

has

a

moral

(80)

When

[email protected]

the

donor

gives

donations

obligation to do, otherwise entitling

without reserving sufficient funds for his

the obligee to damages.

support

(B) refers to an obligation in writing

dependents, his donations are

or

for

the

support

of

his

to do or not to do.

(A) Rescissible, since it results in

(C) the obligee may enforce through

economic lesion of more than 25% of

the court if violated by the obligor.

the value of his properties.

(D) cannot be judicially enforced

(B) Voidable, since his consent to

but

the donation is vitiated by mindless

authorizes

the

obligee

to

retain the obligor’s payment or

kindness.

performance.

(C) Void, since it amounts to wanton expenditure beyond his means.

(79)

The

husband

administration

of

the

assumed family’s

sole

(D) Reducible to the extent that

mango

the

donations

impaired

the

plantation since his wife worked abroad.

support due to himself and his

Subsequently, without his wife’s knowledge,

dependents.

the husband entered into an antichretic transaction

with

a

company,

possession

and

management

giving

it

(81) Anne owed Bessy P1 million due on

the

October 1, 2011 but failed to pay her on

plantation with power to harvest and sell

due date. Bessy sent a demand letter to

the fruits and to apply the proceeds to the

Anne giving her 5 days from receipt within

payment of a loan he got. What is the

which to pay. Two days after receipt of the

standing of the contract?

letter, Anne personally offered to pay Bessy

of

(A) It is void in the absence of the

in manager's check but the latter refused to

wife’s consent.

accept the same. The 5 days lapsed. May

(B) It is void absent an authorization

Anne’s

from the court.

extinguished?

obligation

be

considered

(C) The transaction is void and can

(A) Yes, since Bessy’s refusal of the

neither be ratified by the wife nor

manager’s

authorized by the court.

presumed funded, amounts to a

(D) It is considered a continuing

satisfaction of the obligation.

offer by the parties, perfected

(B) No, since tender of payment

only upon the wife’s acceptance

even in cash, if refused, will not

or the court’s authorization.

discharge the obligation without

check,

which

is

proper consignation in court.

“Never Let The Odds Keep You From Pursuing What You Know In Your Heart You Were Meant To Do.”-Leroy Satchel Paige

Page 170 of 180

Civil Law Q&As (2007-2013)

(C)

Yes,

[email protected]

since

Anne

tendered

[email protected]

(83) The owner of a thing cannot use it in a

payment of the full amount due.

way that will injure the right of a third

(D) No, since a manager’s check is

person. Thus, every building or land is

not considered legal tender in the

subject to the easement which prohibits its

Philippines.

proprietor or possessor from committing nuisance like noise, jarring, offensive odor,

(82) The residents of a subdivision have

and smoke. This principle is known as

been using an open strip of land as passage

(A) Jus vindicandi.

to the highway for over 30 years. The owner

(B) Sic utere tuo ut alienum non

of that land decided, however, to close it in

laedas.

preparation for building his house on it.

(C) Jus dispondendi.

The residents protested, claiming that they

(D) Jus abutendi.

became

owners

of

the

land

through

acquisitive prescription, having been in

(84) Janice and Jennifer are sisters. Janice

possession of the same in the concept of

sued

owners,

and

business partner for recovery of property

continuously for more than 30 years. Is this

with damages. The complaint did not allege

claim correct?

that Janice exerted earnest efforts to come

publicly,

peacefully,

Jennifer

and

Laura,

Jennifer’s

(A) No, the residents have not

to a compromise with the defendants and

been in continuous possession of

that

the land since they merely passed

dismissed the complaint outright for failure

through

to comply with a condition precedent. Is the

it

in

going

to

the

highway.

such

efforts

failed.

The

judge

dismissal in order?

(B) No, the owner did not abandon

(A) No, since Laura is a stranger to

his right to the property; he merely

the sisters, Janice has no moral

tolerated his neighbors’ use of it for

obligation to settle with her.

passage.

(B) Yes, since court should promote

(C) Yes, residents of the subdivision

amicable

have become owners by acquisitive

relatives.

prescription.

(C) Yes, since members of the same

(D) Yes, community ownership by

family, as parties to the suit, are

prescription prevails over private

required to exert earnest efforts to

claims.

settle their disputes before coming

settlement

among

to court.

“Never Let The Odds Keep You From Pursuing What You Know In Your Heart You Were Meant To Do.”-Leroy Satchel Paige

Page 171 of 180

Civil Law Q&As (2007-2013)

[email protected]

[email protected]

(D) No, the family council, which

(87) When does the regime of conjugal

would

partnership of gains begin to exist?

ordinarily

mediate

the

dispute, has been eliminated under

(A) At the moment the parties

the Family Code.

take and declare each other as husband

and

wife

before

(85) X borrowed money from a bank,

officiating officer.

secured by a mortgage on the land of Y, his

(B) At the time the spouses acquire

close friend. When the loan matured, Y

properties through joint efforts.

offered to pay the bank but it refused since

(C) On the date the future spouses

Y was not the borrower. Is the bank’s action

executed their marriage settlements

correct?

because this is the starting point of

(A) Yes, since X, the true borrower,

their marital relationship.

did not give his consent to Y’s offer

(D) On the date agreed upon by the

to pay.

future spouses in their marriage

(B) No, since anybody can discharge

settlements since their agreement is

X’s obligation to his benefit.

the law between them.

(C) No, since Y, the owner of the collateral, has an interest in the

(88) Josie, 18, married Dante, 25, without

payment of the obligation.

her parents’ knowledge and consent, and

(D) Yes, since it was X who has an

lived with him. After a year, Josie returned

obligation to the bank.

to her parents’ home, complained of the unbearable battering she was getting from

(86) The right of a mortgagor in a judicial

Dante, and expressed a desire to have her

foreclosure

marriage with him annulled. Who may

property

to after

redeem

the

mortgaged the

bring the action?

of

the

(A) Dante.

of

the

(B) Her parents.

mortgaged property or confirmation of the

(C) Josie herself.

sale by the court, is known as

(D) The State.

performance mortgage

of

but

his

default

the

conditions

before

the

sale

in

(A) accion publiciana. (B) equity of redemption.

(89) X, a married man, cohabited with Y, an

(C) pacto de retro.

unmarried woman. Their relation bore them

(D) right of redemption.

BB, a baby boy. Subsequently, after X became a widower, he married Y. Was BB legitimated by that marriage?

“Never Let The Odds Keep You From Pursuing What You Know In Your Heart You Were Meant To Do.”-Leroy Satchel Paige

Page 172 of 180

Civil Law Q&As (2007-2013)

[email protected]

[email protected]

(A) Yes, since his parents are now

(D) Yes, as long as they leave

lawfully married.

sufficient property for themselves

(B) Yes, since he is an innocent

and for their dependents.

party and the marriage rectified the wrong done him.

(92) X owed Y P1.5 million. In his will, X

(C) No, since once illegitimate, a

gave Y legacy of P1 million but the will

child

provided that this legacy is to be set off

shall

always

remain

illegitimate.

against the P1.5 million X owed Y. After the

(D) No, since his parents were not

set off, X still owed Y P500,000. Can Y still

qualified

collect this amount?

to

marry

each

other

when he was conceived.

(A) Yes, because the designation of Y as legatee created a new and

(90) The presence of a vice of consent

separate

juridical

relationship

vitiates the consent of a party in a contract

between them, that of testator-

and this renders the contract

legatee.

(A) Rescissible.

(B) It depends upon the discretion of

(B) Unenforceable.

the probate court if a claim is filed

(C) Voidable.

in the testate proceedings.

(D) Void.

(C) No, because the intention of the testator in giving the legacy is to

(91)

Can

properties

common-law of

spouses

substantial value

donate to

one

another?

abrogate his entire obligation to Y. (D)

No,

because

X

had

no

instruction in his will to deliver

(A) No, they are only allowed to

more than the legacy of P1 million to

give moderate gifts to each other

Y.

during family rejoicing. (B) No, they cannot give anything of

(93) Josie owned a lot worth P5 million

value

prevent

prior to her marriage to Rey. Subsequently,

placing their legitimate relatives at a

their conjugal partnership spent P3 million

disadvantage.

for the construction of a house on the lot.

(C) Yes, unlike the case of legally

The construction resulted in an increase in

married spouses, such donations

the value of the house and lot to P9 million.

are not prohibited.

Who owns the house and the lot?

to

each

other

to

“Never Let The Odds Keep You From Pursuing What You Know In Your Heart You Were Meant To Do.”-Leroy Satchel Paige

Page 173 of 180

Civil Law Q&As (2007-2013)

(A)

Josie

[email protected]

and

conjugal

(C) When fortuitous circumstances

partnership of gains will own both

prevented the plaintiff from filing the

on a 50-50 basis.

case sooner.

(B) Josie will own both since the

(D) When

value

possession of the property.

of

the

the

[email protected]

house

and

the

the

plaintiff

is

in

increase in the property’s value is less than her lot’s value; but she

(95) Conrad and Linda, both 20 years old,

is

applied for a marriage license, making it

to

reimburse

conjugal

partnership expenses.

appear

(C) Josie still owns the lot, it being

married without their parents’ knowledge

her

the

before an unsuspecting judge. After the

conjugal

couple has been in cohabitation for 6 years,

exclusive

house

property,

belongs

to

the

but

that they were over 25.

They

partnership.

Linda’s parents filed an action to annul the

(D) The house and lot shall both

marriage on ground of lack of parental

belong to the conjugal partnership,

consent. Will the case prosper?

with Josie entitled to reimbursement

(A) No, since only the couple can

for the value of the lot.

question

the

validity

of

their

marriage after they became 21 of (94)

An

action

for

reconveyance

of

a

age;

their

cohabitation

also

registered piece of land may be brought

convalidated the marriage.

against the owner appearing on the title

(B) No, since Linda’s parents made

based on a claim that the latter merely

no allegations that earnest efforts

holds such title in trust for the plaintiff.

have been made to come to a

The action prescribes, however, within 10

compromise with Conrad and Linda

years from the registration of the deed or

and which efforts failed.

the date of the issuance of the certificate of

(C)

title of the property as long as the trust had

voidable, the couple being below 21

not been repudiated. What is the exception

years of age when they married.

to this 10-year prescriptive period?

(D) Yes, since Linda’s parents never

(A) When the plaintiff had no notice

Yes,

since

the

marriage

is

gave their consent to the marriage.

of the deed or the issuance of the certificate of title.

(96) Pepito executed a will that he and 3

(B) When the title holder concealed

attesting witnesses signed following the

the matter from the plaintiff.

formalities of law, except that the Notary Public failed to come. Two days later, the

“Never Let The Odds Keep You From Pursuing What You Know In Your Heart You Were Meant To Do.”-Leroy Satchel Paige

Page 174 of 180

Civil Law Q&As (2007-2013)

[email protected]

[email protected]

Notary Public notarized the will in his law

(B) Separate since their property

office where all signatories to the will

relations with their legal spouses

acknowledged that the testator signed the

are still subsisting.

will in the presence of the witnesses and

(C) Co-ownership since they agreed

that the latter themselves signed the will in

to work for their mutual benefit.

the presence of the testator and of one

(D) Communal since they earned the

another. Was the will validly notarized?

same as common-law spouses.

(A) No, since it was not notarized on the occasion when the signatories

(98) What is the prescriptive period for filing

affixed their signatures on the will.

an action for revocation of a donation based

(B) Yes, since the Notary Public

on acts of ingratitude of the donee?

has to be present only when the

(A) 5 years from the perfection of the

signatories acknowledged the acts

donation.

required of them in relation to

(B) 1 year from the perfection of

the will.

the donation.

(C) Yes, but the defect in the mere

(C) 4 years from the perfection of the

notarization of the will is not fatal to

donation.

its execution.

(D) Such action does not prescribe.

(D) No, since the notary public did not require the signatories to sign

(99)

Before

Karen

married

Karl,

she

their respective attestations again.

inherited P5 million from her deceased mother which amount she brought into the

(97) Venecio and Ester lived as common-law

marriage. She later used part of the money

spouses since both have been married to

to buy a new Mercedes Benz in her name,

other persons from whom they had been

which Karen and her husband used as a

separated

family car. Is the car a conjugal or Karen’s

in

Hardworking

fact and

for

several

years.

bright,

each

earned

exclusive property?

incomes from their respective professions

(A) It is conjugal property since the

and enterprises. What is the nature of their

spouses use it as a family car.

incomes?

(B) It is Karen’s exclusive property

(A) Conjugal since they earned the

since it is in her name.

same while living as husband and

(C) It is conjugal property having

wife.

been bought during the marriage.

“Never Let The Odds Keep You From Pursuing What You Know In Your Heart You Were Meant To Do.”-Leroy Satchel Paige

Page 175 of 180

Civil Law Q&As (2007-2013)

(D) It

is

[email protected]

Karen’s

exclusive

property since she bought it with

[email protected]

should the lessees sue for damages? (1%) (1). A, the owner

her own money. (2). B, the engineer (100) Because of X’s gross negligence, Y suffered

injuries

that

resulted

in

the

abortion of the foetus she carried. Y sued X for, among other damages, P1 million for the death of a family member. Is Y entitled to indemnity for the death of the foetus she carried? (A) Yes, since the foetus is already regarded as a child from conception, though unborn. (B) No, since X’s would not have known

that

the

accident

would

result in Y’s abortion. (C) No,

since

birth

determines

personality, the accident did not result in the death of a person. (D) Yes, since the mother believed in her heart that she lost a child.

(3). both A & B SUGGESTED ANSWER: 3. Both A & B. The lessee may proceed against A for breach of contract, and against B for tort or statutory liability. Under Article 1654 (2) of the New Civil Code, the lessor is obliged to make all the necessary repairs in order to keep the leased property suitable for the use to which it has been devoted.

Consequently,

under

Article

1659 NCC, the proprietor of a building or structure is responsible for the damages resulting

from

its

total

or

partial

collapse, if it is due to lack of necessary repairs. Under Article 1723, NCC, the engineer

2010 Civil Law Exam MCQ

or architect who drew up the plans and

(September 12, 2010)

specifications for a building is liable for

No.II. Multiple choice.

damage if 15 years from the completion of

the

structure

the

same

should

collapse by a reason of a defect by those

(A). A had a 4-storey building which was

plans and specifications, or due to the

constructed by Engineer B. After five years,

defects in the ground. This liability

the

developed cracks and its

maybe enforced against the architect or

stairway eventually gave way and collapsed,

engineer even by a third party who has

resulting to injuries to some lessees. Who

no privity of contract with the architect

building

or engineer under Article 2192, NCC. “Never Let The Odds Keep You From Pursuing What You Know In Your Heart You Were Meant To Do.”-Leroy Satchel Paige

Page 176 of 180

Civil Law Q&As (2007-2013)

[email protected]

ALTERNATIVE ANSWER:

[email protected]

(B) O, owner of Lot A, learning that Japanese soldiers may have buried gold

No.1. A , the owner .

and other treasures at the adjoining vacant

The lessee can sue only the lessor for

Lot

breach of contract under Article 1659 in

excavated in Lot B where she succeeded in

relation to Article 1654, NCC. The lessee

unearthing gold and precious stones. How

cannot sue the architect or the engineer

will the treasures found by O be divided?

because there was no privity of contracts

(1%)

between them. When sued, however, the lessor may file a third party claim against the architect or the engineer.

B

belonging

to

spouses

No. 2. B, the Engineer .

(3). 50% to O and 50% to the state

1723

the

engineer

Y,

(2). 50% to O and 50% to the spouses X and Y

Article

&

(1). 100% to O as finder

ANOTHER ALTERNATIVE ANSWER:

Under

X

or

architect who drew up the plans and specifications for a building is liable for

(4). None of the above. SUGGESTED ANSWER:

damages if within 15 years from the completion of the structure, the same should collapse by reason of a defect in those plans and specifications, or due to the defects in the ground. Under Article 2192 (NCC), however, if the damages should be the result of any of the defects in the construction mentioned in Art 1723, NCC, the third person suffering damages may proceed only against the engineer

or

architect

or

contractor

within the period fixed therein. The damages suffered by the lessee in the problem are clearly those resulting from defects in the construction plans or specifications.

No. 4. None of the above. The general rule is that the treasure shall belong to the spouses X and Y, the owner of Lot B. Under Article 438 (NCC), the exception is that when the discovery of a hidden treasure is made on the property of another and by chance, onehalf thereof shall belong to the owner of the

land

and

the

other

one-half

is

allowed to the finder. In the problem, the finding of the treasure was not by chance because O knew that the treasure was in Lot B. While a trespasser is also not entitled to any share, and there is no indication in the problem whether or not O was a trespasser, O is not entitled to a

“Never Let The Odds Keep You From Pursuing What You Know In Your Heart You Were Meant To Do.”-Leroy Satchel Paige

Page 177 of 180

Civil Law Q&As (2007-2013)

[email protected]

mother

[email protected]

share because the finding was not “by

or

alone,

even

in

a

public

chance.”

document, is not sufficient because the father and mother did not have a special power of attorney for the purpose. Under

(C) A executed a Deed of Donation in favor of B, a bachelor, covering a parcel of land valued at P1 million. B was, however, out of the country at the time. For the donation to be valid, (1%)

Article 745 (NCC), the donee must accept the donation personally, or through an authorized person with a special power of attorney for the purpose; otherwise, the donation shall be void. No.3 is also false. B cannot accept the

(1). B may e-mail A accepting the donation. (2). The donation may be accepted by B’s father with whom he lives.

convenient to him.

attorney may accept the donation for him. (5). None of the above is sufficient to make B’s acceptance valid

No. 5 None of the above is sufficient to make B's acceptance valid . donation must

them signed each and every page of the will. One of the witnesses was B, the father of one of the legatees to the will. What is the effect of B being a witness to the will? (1%)

(2). The will is valid and effective (3). The legacy given to B’s child is not valid

covered

an

immovable property, the donation and acceptance

(D) A executed a 5-page notarial will before

(1). The will is invalidated

SUGGESTED ANSWER:

the

accept the donation only during the

a notary public and three witnesses. All of

(4). B’s mother who has a general power of

the

Under Article 749 NCC, the donee may lifetime of the donor.

(3). B can accept the donation anytime

Since

donation anytime at his convenience.

be

in

public

document and e-mail is not a public document. Hence, No.1 is false. No. 2 and No.4 are both false. The acceptance by the donee’s father alone

SUGGESTED ANSWER: No. 3.

The legacy given to B's child is

not valid. The validity of the will is not affected by the legacy in favor of the son of an

“Never Let The Odds Keep You From Pursuing What You Know In Your Heart You Were Meant To Do.”-Leroy Satchel Paige

Page 178 of 180

Civil Law Q&As (2007-2013)

[email protected]

attesting witness to the will. However,

[email protected]

ALTERNATIVE ANSWER:

the said legacy is void under Article 823 NCC.

(D). all the above

ALTERNATIVE ANSWER:

(2). A deposit made in compliance with a legal obligation is:

No. 2 .The will is valid and effective. Under Article 823 ( NCC ),the legacy given

in

favor

of

the

son

of

an

instrumental witness to a will has no effect on the validity of the will. Hence,

(A). an extrajudicial deposit; (B). a voluntary deposit; (C). a necessary deposit;

the will is valid and effective. (D). a deposit with a warehouseman; (E). letters a and b

2007 Civil Law Exam MCQ (September 09, 2007)

SUGGESTED ANSWER: (C). a necessary deposit

No.IX. Multiple choice: Choose the right answer. (2% each)

(3). A contract of antichresis is always:

(1). The parties to a bailment are the:

(A). a written contract;

(A). bailor;

(B). a contract, with a stipulation that the debt will be paid through receipt of the

(B). bailee; (C) comodatario; (D). all the above; (E). letters a and b SUGGESTED ANSWER: (E). letters a and b

fruits of an immovable; (C). Involves the payment of interests, if owing; (D). All of the above; (E). Letters a and b SUGGESTED ANSWER:

“Never Let The Odds Keep You From Pursuing What You Know In Your Heart You Were Meant To Do.”-Leroy Satchel Paige

Page 179 of 180

Civil Law Q&As (2007-2013)

[email protected]

(D). All of the above;

(D).

1/3

of

[email protected]

the

total

debts

must

be

represented by the approving creditors; (4). An, assignee in a proceeding under the Insolvency Law does not have the duty of:

(E). Letters a and b

(A). suing to recover the properties of the

SUGGESTED ANSWER:

state of the insolvent debtor; (C). 3/5 of the number of creditors (B). selling property of the insolvent debtor;

should agree to the settlement;

(C). ensuring that a debtor corporation

[Note: Items 4&5 on Insolvency Law are

operate

and

not included within the coverage of Civil

effectively while the proceedings are

Law but Commercial Law. It is therefore

pending;

suggested that the examinees be given

the

business

efficiently

full credit for the two items regardless of (D). collecting and discharging debts owed

their answers.]

to the insolvent debtor. SUGGESTED ANSWER: References: (C). ensuring that a debtor corporation operate

the

business

efficiently

and

Answers

to

Bar

Examination

effectively while the proceedings are

Questions by the UP LAW COMPLEX

pending;

(2007, 2009, 2010)

(5). In order to obtain approval of the proposed settlement of the debtor in an insolvency proceeding. (A). the court must initiate the proposal (B). 2/3 of the number of creditors should

UP LAW REVIEW PHILIPPINE ASSOCIATION OF LAW SCHOOLS (2008) lawphil.net

agree to the settlement; (C). 3/5 of the number of creditors should agree to the settlement;

“Never Let The Odds Keep You From Pursuing What You Know In Your Heart You Were Meant To Do.”-Leroy Satchel Paige

Page 180 of 180

UNIVERSITY OF SANTO TOMAS SUGGESTED ANSWERS IN CIVIL LAW BAR EXAMINATIONS 2013 By: Assoc. Dean Viviana M. Paguirigan

QUESTION NO. I. You are a Family Court judge and before you is a Petition for the Declaration of Nullity of Marriage (under Article 36 of the Family Code) filed by Maria against Neil. Maria claims that Neil is psychologically incapacitated to comply with the essential obligations of marriage because Neil is a drunkard, a womanizer, a gambler, and a mama's boy- traits that she never knew or saw when Neil was courting her. Although summoned, Neil did not answer Maria's petition and never appeared in court. To support her petition, Maria presented three witnesses- herself, Dr. Elsie Chan, and Ambrosia. Dr. Chan testified on the psychological report on Neil that she prepared. Since Neil never acknowledged n9r responded to her invitation for interviews, her report is solely based on her interviews with Maria and the spouses' minor children. Dr. Chan concluded that Neil is suffering from Narcissistic Personality Disorder, an ailment that she found to be already present since Neil's early adulthood and one that is grave and incurable. Maria testified on the specific instances when she found Neil drunk, with another woman, or squandering the family's resources in a casino. Ambrosia, the spouses' current household help, corroborated Maria's testimony. On the basis of the evidence presented, will you grant the petition? (8%)

SUGGESTED ANSWER: If I were the judge, I will not grant the petition. Although psychological incapacity has not been defined by the Family Code, the Supreme Court in several cases (Republic vs. San Jose February 28, 2007; Zamora v. CA an Zamora G.R. No. 141917 February 7, 2007; Benjamin Ting v. Carmen Ting G.R. No. 166562; March 31, 2009) has ruled that the intendment of the law is to confine psychological incapacity to the most serious cases of personality disorders clearly demonstrative of an utter insensitivity or inability to give meaning and significance to the marriage. What the law requires is downright incapacity and not refusal or neglect or difficulty but a failure to perform essential marital obligations due to causes psychological in nature. Further, the presentation of expert proof presupposes a thorough and in-depth assessment of the parties by the psychologist or expert for a conclusive diagnosis of grave, severe, and incurable presence of psychological incapacity. (Paz vs. Paz – February 18, 2010) In this case, the report of Dr. Chan is solely based on her interviews with Maria and the children. She did not actually hear, see and evaluate Neil. Hence, the report cannot constitute a reasonable basis to reach a conclusion as to Neil’s psychological incapacity. QUESTION II. A collision occurred at an intersection involving a bicycle and a taxicab. Both the bicycle rider (a businessman then doing his morning exercise) and the taxi driver claimed that the other was at fault. Based on the police report, the bicycle crossed the intersection first but the taxicab, crossing at a fast clip from the bicycle's left, could not brake in time and hit the bicycle's rear wheel, toppling it and throwing the bicycle rider into the sidewalk 5 meters away. 1

The bicycle rider suffered a fractured right knee, sustained when he fell on his right side on the concrete side walk. He was hospitalized and was subsequently operated on, rendering him immobile for 3 weeks and requiring physical rehabilitation for another 3 months. In his complaint for damages, the rider prayed for the award ofP1,000,000 actual damages,P200,000 moral damages, P200,000 exemplary damages, P1 00,000 nominal damages and P50,000 attorney's fees. Assuming the police report to be correct and as the lawyer for the bicycle rider, what evidence (documentary and testimonial) and legal arguments will you present in court to justify the damages that your client claims? (8%)

SUGGESTED ANSWER: As lawyer for the bicycle rider, I will present in addition to the police report, the medical abstract as to the injuries sustained by my client as well as copies of receipts of expenses incurred in connection with the treatment of his injuries. I will also present the testimony of my client and perhaps a bystander who witnessed the incident as to the circumstances surrounding the accident. As for the legal argument, I will rebut the claim of negligence on my client’s part by presenting evidence that my client has actually crossed the intersection ahead of the taxicab and it was the taxicab driver who rapidly cut the path of the bicycle which caused the collision. Also, even assuming that there was contributory negligence on the part of my client, I will argue that it will not preclude the recovery of damages but may only mitigate the damages to which he is entitled. QUESTION III. Sergio is the registered owner of a 500-square meter land. His friend, Marcelo, who has long been interested in the property, succeeded in persuading Sergio to sell it to him. On June 2, 2012, they agreed on the purchase price of P600,000 and that Sergio would give Marcelo up to June30, 2012 within which to raise the amount. Marcelo, in a light tone usual between them, said that they should seal their agreement through a case of Jack Daniels Black and P5,000 "pulutan" money which he immediately handed to Sergio and which the latter accepted. The friends then sat down and drank the first bottle from the case of bourbon. On June 15, 2013, Sergio learned of another buyer, Roberto, who was offering P800,000 in ready cash for the land. When Roberto confirmed that he could pay in cash as soon as Sergio could get the documentation ready, Sergio decided to withdraw his offer to Marcelo, hoping to just explain matters to his friend. Marcelo, however, objected when the withdrawal was communicated to him, taking the position that they have a firm and binding agreement that Sergio cannot simply walk away from because he has an option to buy that is duly supported by a duly accepted valuable consideration. (A) Does Marcelo have a cause of action against Sergio? (5%) (B) Can Sergio claim that whatever they might have agreed upon cannot be enforced because any agreement relating to the sale of real property must be supported by evidence in writing and they never reduced their agreement to writing? (3%)

2

SUGGESTED ANSWER: A) Yes, Marcelo has a cause of action against Sergio. As a rule, an offer can be withdrawn at any time before acceptance by communicating such withdrawal (Art. 1324) except when the option is founded upon a consideration as something paid or promised. In this case, although there was no separate consideration for the option, the offer had already been accepted and thus, it resulted into a perfected contract of sale between Marcelo and Sergio. Sale being a consensual contract is perfected by mere consent. B) No, Sergio cannot claim that the agreement cannot be enforced because it was not reduced into writing. Contracts shall be obligatory, in whatever form they may have been entered into, provided all the essential requisites for their validity are present. (Art. 1356) In fact when the law requires a document or other special form, as in the acts and enumerated by law, the contracting parties may compel each other to observe that form, once the contract has been perfected, and this right may be exercised simultaneously with the action upon the contract. (Art. 1357) Even an oral sale of a parcel of land is valid between the parties (Campillo vs. CA 129 SCRA 513; Zaide v. CA 163 SCRA 71) QUESTION IV. Anselmo is the registered owner of a land and a house that his friend Boboy occupied for a nominal rental and on the condition that Boboy would vacate the property on demand. With Anselmo's knowledge, Boboy introduced renovations consisting of an additional bedroom, a covered veranda, and a concrete block fence, at his own expense. Subsequently, Anselmo needed the property as his residence and thus asked Boboy to vacate and turn it over to him. Boboy, despite an extension, failed to vacate the property, forcing Anselmo to send him a written demand to vacate. In his own written reply, Boboy signified that he was ready to leave but Anselmo must first reimburse him the value of the improvements he introduced on the property as he is a builder in good faith. Anselmo refused, insisting that Boboy cannot ask for reimbursement as he is a mere lessee. Boboy responded by removing the improvements and leaving the building in its original state. (A) Resolve Boboy's claim that as a builder in good faith, he should be reimbursed the value of the improvements he introduced. (4%) (B) Can Boboy be held liable for damages for removing the improvements over Anselmo's objection? (4%)

SUGGESTED ANSWER: A) Boboy’s claim that he is a builder in good faith is without merit. The contract between the parties remains to be a lease despite the nominal rentals paid by Boboy. As such, Boboy’s right with regard to the improvements he introduced on the property should not be resolved on the basis of the provisions of the Civil Code on builder in good faith under Article 448 but by the provision on lease, particularly Article 1678. A lessee who makes improvements on the property cannot be considered a builder in good faith for he knows that he does not own the property and his possession is merely temporary. Boboy may only claim one-half of the value of the improvements from Anselmo but if the latter refuses to reimburse him, Boboy may remove the improvements even if it may cause damage to the property.

3

B) No, Boboy cannot be held liable for damages except if he caused unnecessary impairment to the property leased. Since Anselmo refused to appropriate the improvements and to reimburse Boboy, the latter may exercise his right to remove the improvements provided he shall not cause any more impairment to the property leased than is necessary. QUESTION V. Josefa executed a deed of donation covering a one-hectare rice land in favor of her daughter, Jennifer. The deed specifically provides that:

"For and in consideration of he love and service Jennifer has shown and given to me, I hereby freely, voluntarily and irrevocably donate to her my one-hectare rice land covered by TCT No. 11550, located in San Fernando, Pampanga. This donation shall take effect upon my death." The deed also contained Jennifer's signed acceptance, and an attached notarized declaration by Josefa and Jennifer that the land will remain in Josefa's possession and cannot be alienated, encumbered, sold or disposed of while Josefa is still alive. Advise Jennifer on whether the deed is a donation inter vivos or mortis causa and explain the reasons supporting your advice. (8%)

SUGGESTED ANSWER: I will advise Jennifer that the deed of donation executed in her favor by Josefa is a donation inter vivos. An inter vivos donation is generally irrevocable once accepted, and the law requires that if it involves immovable property, it must be in a public document and there must be a deed of acceptance which must be in the same deed of donation. If the acceptance is in a separate instrument, it has to be noted in both instruments. (Art. 749) In this case, the deed of acceptance clearly signifies that it is a donation inter vivos because a donation mortis causa need not be accepted by the donee during the lifetime of the donor although the donee in the case of mortis causa donation is free to accept or repudiate it after the death of the donor. Also, the prohibition on alienation during Josefa’s lifetime all the more indicates that the donation is inter vivos because the fact that Josefa reserved the lifetime usufruct of the land shows that her intent is to transfer the ownership of the donated property to Jennifer or else there would have been no need for her to reserve the lifetime usufruct thereof if it were a donation mortis causa. (Gestopa v. CA 342 SCRA 105 citing Reyes vs. Mosqueda, 187 SCRA 661, 671 (1990); Concepcion vs. Concepcion, 91 Phil. 823, 827 (1952).) QUESTION VI. Lito obtained a loan of P1,000,000 from Ferdie, payable within one year. To secure payment, Lito executed a chattel mortgage on a Toyota Avanza and a real estate mortgage on a 200square meter piece of property. (A) Would it be legally significant - from the point of view of validity and enforceability - if the loan and the mortgages were in public or private instruments? (6%) (B) Lito's failure to pay led to the extra-judicial foreclosure of the mortgaged real property. Within a year from foreclosure, Lito tendered a manager's check to Ferdie to redeem the property. Ferdie refused to accept payment on the ground that he wanted payment in cash: the 4

check does not qualify as legal tender and does not include the interest payment. Is Ferdie's refusal justified? (4%)

SUGGESTED ANSWER: A) With respect to the loan, the same is both valid and enforceable regardless of whether it is in a private or public document because as a rule, contracts shall be obligatory in whatever form they may have been entered into provided all the essential requisites for their validity are present. A loan is a contract which the law does not require to be in a particular form in order that it may be valid or enforceable. However, with regard to the chattel mortgage, since the law (Act 1508) requires an affidavit of good faith stating that the chattel mortgage is supposed to stand as security for the loan, it is submitted that for validity of the chattel mortgage, it must be in a public document. A real estate mortgage under the provisions of Article 2125 requires that in order that a mortgage may be validly constituted that the document in which it appears must be recorded. If it is not recorded, the mortgage is nevertheless valid and binding between the parties. Hence, for validity both chattel and real estate mortgages must be in a public document. But for purposes of enforceability, it is submitted that the form of the contract whether in a public or private document would be immaterial. (Mobil Oil vs. Diocares 29 SCRA 656). B) Ferdie’s refusal to accept the check on the ground that it does not qualify as legal tender is correct because a check, whether a manager's check or ordinary check, is not legal tender, and an offer of a check in payment of a debt is not a valid tender of payment and may be refused receipt by the obligee or creditor. (Philippine Airlines vs. CA and Amelia Tan – January 30, 1990) Mere delivery of checks does not discharge the obligation under a judgment. The obligation is not extinguished and remains suspended until the payment by commercial document is actually realized (Art. 1249, Civil Code, par. 3). Also, redemption within the period allowed by law is not a matter of intent but a question of payment or valid tender of full redemption price within the said period. Whether the redemption is being made under Act 3135 or under the General Banking Law, the mortgagor or his assignee is required to tender payment to make said redemption valid. (Heirs of Quisumbing vs. PNB aand SLDC –G.R. No. 178242 January 20, 2009) QUESTION VII. In 2005, Andres built a residential house on a lot whose only access to the national highway was a pathway crossing Brando's property. Andres and others have been using this pathway (pathway A) since 1980. In 2006, Brand0 fenced off his property, thereby blocking Andres' access to the national highway. Andres demanded that part of the fence be removed to maintain his old access route to the highway (pathway A), but Brando refused, claiming that there was another available pathway (pathway B) for ingress and egress to the highway. Andres countered that pathway B has defects, is circuitous, and is extremely inconvenient to use. To settle their dispute, Andres and Brando hired Damian, a geodetic and civil engineer, to survey and examine the two pathways and the surrounding areas, and to determine the shortest and the least prejudicial way through the servient estates. After the survey, the engineer concluded that pathway B is the longer route and will need improvements and repairs, but will not significantly affect the use of Brando's property. On the other hand, pathway A that had long been in place, is the shorter route but would significantly affect the use of Brando's property.

5

In light of the engineer's findings and the circumstances of the case, resolve the parties' right of way dispute. (6%)

SUGGESTED ANSWER: I will rule in favor of Brando. The easement of right of way should be established at a point least prejudicial to the servient estate where the distance from the dominant estate to the public highway may be the shortest. (Art. 650) If these two conditions do not concur in one estate, the criterion of least prejudice prevails over shortest distance. (Anastacia Quimen vs. CA and Yolanda Oliveros May 29, 1996) In this case, to establish the easement on the property of Brando would significantly affect his use of his property whereas while Pathway B may prove to be the longer route, it will cause least prejudice to Brando. Andres’ argument that Pathway B is circuitous and inconvenient to use should not be given weight because the true test of the establishment of an easement is adequacy. Convenience of the dominant estate has never been the gauge for the establishment of the easement. (Costabella Corporation v. CA 193 SCRA 333; Cristobal vs. Ledesma 291 SCRA 122)

QUESTION VIII. Ciriaco Realty Corporation (CRC) sold to the spouses Del a Cruz a500-square meter land (Lot A) in Paranaque. The land now has a fair market value of Pl,200,000. CRC likewise sold to the spouses Rodriguez, a 700-square meter land (Lot B) which is adjacent to Lot A. Lot B has a present fair market value of P1,500,000. The spouses Dela Cruz constructed a house on Lot B, relying on there presentation of the CRC sales agent that it is the property they purchased. Only upon the completion of their house did the spouses Dela Cruz discover that they had built on Lot B owned by the spouses Rodriguez, not on Lot A that they purchased. They spent P 1 000,000 for the house. As their lawyer, advise the spouses Dela Cruz on their rights and obligations under the given circumstances, and the recourses and options open to them to protect their interests. (8%)

SUGGESTED ANSWER: I will advise Spouses Dela Cruz that they have the right to retain possession of the premises until Rodriguez exercises any of the options under Article 448 of the Civil Code. (Tecnogas Manufacturing vs. CA February 10, 1997) Spouses Dela Cruz are builders in good faith because before constructing the house, they exercised due diligence by asking the agent of CRC the location of Lot A and they relied on the information given by the agent who is presumed to know the identity of the lot purchased by the Dela Cruz. (Pleasantville vs. CA 253 SCRA 10) The owner of the land on which anything has been built in good faith by another has the right to appropriate as his own the works, sowing or planting after payment of the indemnity or to oblige the builder to pay the price of the land if its value is not considerably higher than the building or trees, or to ask the sower to pay proper rent. I will also advise my clients that Rodriguez may not compel them to remove the improvements because it is not one of the options granted to the landowner if the builder is in good faith.

6

QUESTION IX. Rica petitioned for the annulment of her ten-year old marriage to Richard. Richard hired Atty. Cruz to represent him in the proceedings. In payment for Atty. Cruz's acceptance and legal fees, Richard conveyed to Atty. Cruz a parcel of land in Taguig that he recently purchased with his lotto winnings. The transfer documents were duly signed and Atty. Cruz immediately took possession by fencing off the property's entire perimeter. Desperately needing money to pay for his mounting legal fees and his other needs and despite the transfer to Atty. Cruz, Richard offered the same parcel of land for sale to the spouses Garcia. After inspection of the land, the spouses considered it a good investment and purchased it from Richard. Immediately after the sale, the spouses Garcia commenced the construction of a threestory building over the land, but they were prevented from doing this by Atty. Cruz who claimed he has a better right in light of the prior conveyance in his favor. Is Atty. Cruz's claim correct? (8%)

SUGGESTED ANSWER: No, Atty. Cruz is not correct. At first glance, it may appear that Atty. Cruz is the one who has a better right because he first took possession of the property. However, as a lawyer of Richard he is prohibited under Article 1491 from acquiring the property and rights which may the object of any litigation in which they may take part by virtue of their profession. While the suit is for annulment of marriage and it may be argued that the land itself is not the object of the litigation, the annulment of marriage if granted, will carry with it the liquidation of the absolute community or conjugal partnership of the spouses as the case may be (Article 50 in relation to Article 43 of the Family Code). Richard purchased the land with his lotto winnings during the pendency of the suit for annulment and on the assumption that the parties are governed by the regime of absolute community or conjugal partnership, winnings from gambling or betting will form part thereof. Also, since the land is part of the absolute community or conjugal partnership of the Richard and Rica it may not be sold or alienated without the consent of the latter and any disposition or encumbrance of the property of the community or conjugal property without the consent of the other spouse is void. (Article 96 and Article 124, Family Code). QUESTION X. Manuel was born on 12 March 1940 in a 1 000-square meter property where he grew up helping his father, Michael, cultivate the land. Michael has lived on the property since the land was opened for settlement at about the time of the Commonwealth government in 193 5, but for some reason never secured any title to the property other than a tax declaration in his name. He has held the property through the years in the concept of an owner and his stay was uncontested by others. He has also conscientiously and continuously paid the realty taxes on the land. Michael died in 2000 and Manuel - as Michael’s only son and heir -now wants to secure and register title to the land in his own name. He consults you for legal advice as he wants to perfect his title to the land and secure its registration in his name. (A) What are the laws that you need to consider in advising Manuel on how he can perfect his title and register the land in his name? Explain the relevance of these laws to your projected course of action. (4%)

7

(B) What do you have to prove to secure Manuel's objectives and what documentation are necessary? (4%)

SUGGESTED ANSWER: A) For purposes of confirmation of imperfect title, I will have to consider the provisions of Commonwealth Act No. 141 as well as the Property Registration Decree or P.D. 1529 in giving my advice to Manuel. C.A. No. 141 which amended the second Public Land Act (Act 2874) provides that there are two requisites for judicial confirmation of imperfect title namely:1) open and continuous, exclusive and notorious possession and occupation of the land by himself or through his predecessor in interest under bona fide claim of ownership since June 12, 1945; and 2) the classification of the land as alienable and disposable land of the public domain. (Secretary of DENR v. Yap -G.R. NO. 167707, October 8, 2008) The Property Registration Decree or P.D. 1529 provides who may file an application for registration of title to the land under Section 14 1 thereof which provides that those who by themselves or their predecessors-in-interest have been in open, continuous, exclusive and notorious possession and occupation of alienable and disposable lands for the public domain under a bona fide claim of ownership since June 12, 1945 or earlier. Since Manuel’s father Michael had been in open, continuous, exclusive and notorious possession of the land since 1935, and that the land was declared alienable in the same year, his possession has ripened into ownership which entitles him or his successor Manuel to file an application for judicial confirmation of imperfect title. B) I have to prove that the land was already declared alienable at the time that Manuel or his father Michael took possession of the land and that their possession was open, continuous, exclusive and notorious which started prior to or on June 12, 1945 as required by C.A. No. 141. To prove the first requisite, the original classification of the land as approved by the DENR Secretary (Republic v. T.A. N. Properties 555 SCRA 4777 (2008) or in lieu thereof, a Certification by the DENR Regional office attesting to the alienable and disposable character of the land (Republic v. Serrano G.R. No. 183063 – February 24, 2010) must have to be submitted. I also have to file together with the application for registration all original muniments of title or copies thereof and a survey plan of the land approved by the Bureau of Lands in accordance with Section 17 of P.D. 1529.2 Manuel may also submit the tax declarations and tax payment receipts which have been ruled to be good indications of possession in the concept of owner (Republic vs. Candy Maker, Inc. G.R. No. 163766, June 22, 2006). 1

Section 14. Who may apply. The following persons may file in the proper Court of First Instance an application for registration of title to land, whether personally or through their duly authorized representatives: (1) Those who by themselves or through their predecessors-in-interest have been in open, continuous, exclusive and notorious possession and occupation of alienable and disposable lands of the public domain under a bona fide claim of ownership since June 12, 1945, or earlier. (2) Those who have acquired ownership of private lands by prescription under the provision of existing laws. (3) Those who have acquired ownership of private lands or abandoned river beds by right of accession or accretion under the existing laws. (4) Those who have acquired ownership of land in any other manner provided for by law. Section 17. What and where to file. The application for land registration shall be filed with the Court of First Instance of the province or city where the land is situated. The applicant shall file together with the application all original muniments of titles or copies thereof and a survey plan of the land approved by the Bureau of Lands. 2

The clerk of court shall not accept any application unless it is shown that the applicant has furnished the Director of Lands with a copy of the application and all annexes.

8

2014 BAR EXAMINATIONS UNIVERSITY of the PHILIPPINES LAW CENTER SUGGESTED ANSWERS IN CIVIL LAW Assoc. Dean Viviana M. Paguirigan I. Ariz and Paz were officemates at Perlas ng Silangan Bank (PSB). They fell in love with each other and had a civil and church wedding. Meanwhile, Paz rapidly climbed the corporate ladder of PSB and eventually became its Vice President, while Ariz remained one of its bank supervisors, although he was short of 12 units to finish his Masters of Business Administration (MBA) degree. Ariz became envious of the success of his wife. He started to drink alcohol until he became a drunkard. He preferred to join his "barkadas"; became a wifebeater; would hurt his children without any reason; and failed to contribute to the needs of the family. Despite rehabilitation and consultation with a psychiatrist, his ways did not change. After 19 years of marriage, Paz, a devout Catholic, decided to have their marriage annulled by the church. Through the testimony of Paz and a psychiatrist, it was found that Ariz was a spoiled brat in his youth and was sometimes involved in brawls. In his teens, he was once referred to a psychiatrist for t reatment due to his violent tendencies. In due time, the National Appellate Matrimonial Tribunal (NAMT) annulled the union of Ariz and Paz due to the failure of Ariz to perform and fulfill his duties as a husband and as a father to their children. The NAMT concluded that it is for the best interest of Paz, Ariz and their children to have the marriage annulled. In view of the NAMT decision, Paz decided to file a Petition for Declaration of Nullity of Marriage of their civil wedding before the Regional Trial Court (RTC) of Makati City using the NAMT decision and the same evidence adduced in the church annulment proceedings as basis. If you are the judge, will you grant the petition? Explain. (5%)

SUGGESTED ANSWER:

If I were the judge, I will not grant the petition. While the decision of the church tribunal annulling the marriage of the parties may be persuasive, it is not however, binding upon the civil courts. For psychological incapacity to be a ground for nullity, it must be shown that it was rooted in the history of the party alleged to be suffering from it, must be grave and serious, and incurable such that it renders the person incapacitated to perform the essential marital obligations due to causes psychological in nature. In the case presented, it appears that Ariz fulfilled his marital obligations at the beginning and it was only after feeling envious about the success of Paz that he started exhibiting violent tendencies and refused to comply with marital obligations. Psychological incapacity is not mere refusal but outright incapacity to perform marital obligations which does not appear to be present in the case of Ariz. (Marcos v. Marcos G.R. No. 136490- October 19, 2000) II. Crispin died testate and was survived by Alex and Josine, his children from his first wife; Rene and Ruby, his children from his second wife; and Allan, Bea, and Cheska, his children from his third wife. One important provision in his will reads as follows: "Ang lupa at bahay sa Lungsod ng Maynila ay ililipat at ilalagay sa pangalan nila Alex at Rene hindi bilang pamana ko sa kanila kundi upang pamahalaan at pangalagaan lamang nila at nang ang sinuman sa aking mga anak, sampu ng aking mga apo at kaapuapuhan ko sa habang panahon, ay may tutuluyan kung magnanais na mag-aral sa Maynila o sa kalapit na mga lungsod." Is the provision valid? (4%)

9

SUGGESTED ANSWER:

No, the provision is not valid. At first glance, the provision may appear valid as it provides for the transfer of title in favor of Alex and Rene over the parcel of land. A legacy or devise is to be construed as a donation effective mortis causa, and it is intended to transfer ownership to the legatee or devisee. Since the ownership is legally transferred to the Alex and Rene, they cannot be prohibited by the testator from alienating or partitioning the same perpetually. The dispositions of the testator declaring all or part of the estate inalienable for more than twenty years are void. (Article 870) III. The Roman Catholic Church accepted a donation of a real property located in Lipa City. A deed of donation was executed, signed by the donor, Don Mariano, and the donee, the Church, as represented by Fr. Damian. Before the deed could be notarized, Don Mariano died. Is the donation valid? (4%)

SUGGESTED ANSWER:

The donation is void. The donation of an immovable property must be in a public instrument in order for it to be valid. In this case, the donor died even before the notarization of the deed of donation. Hence, it does not satisfy the requirement of being in a public instrument for the donation to be valid.

IV. Nante, a registered owner of a parcel of land in Quezon City, sold the property to Monica under a deed of sale which reads as follows: "That for and in consideration of the sum of P500,000.00, value to be paid and delivered to me, and receipt of which shall be acknowledged by me to the full satisfaction of Monica, referred to as Vendee, I hereby sell, transfer, cede, convey, and assign, as by these presents, I do have sold, transferred, ceded, conveyed and assigned a parcel of land covered by TCT No. 2468 in favor of the Vendee." After delivery of the initial payment of P100,000.00, Monica immediately took possession of the property. Five (5) months after, Monica failed to pay the remaining balance of the purchase price. Nante filed an action for the recovery of possession of the property. Nante alleged that the agreement was one to sell,which was not consummated as the full contract price was not paid. Is the contention of Nante tenable?

SUGGESTED ANSWER:

The contention of Nante is not tenable. The deed itself states that for consideration received, he sells, transfers, and conveys the land to Monica and there was delivery of the property to the latter. The contract is clearly one of sale as there was no reservation of ownership on the part of the seller Nante. The non-payment of the price in a contract of sale would only entitle the seller to rescind the contract but it does not thereby prevent the transfer of ownership particularly so as in this case, where there was already delivery to the buyer. V. What is the effect of preterition ? (1%) (A) It annuls the devise and legacy (B) It annuls the institution of heir (C) It reduces the devise and legacy (D) It partially annuls the institution of heir

10

Answer is letter B (preterition annuls the institution of heirs)

VI. Miko and Dinah started to live together as husband and wife without the benefit of marriage in 1984. Ten (10) years after, they separated. In 1996, they decided to live together again, and in 1998, they got married. On February 17, 2001, Dinah filed a complaint for declaration of nullity of her marriage with Miko on the ground of psychological incapacity under Article 36 of the Family Code. The court rendered the following decision: "1. Declaring the marriage null and void; 2. Dissolving the regime of absolute community of property; and 3. Declaring that a decree of absolute nullity of marriage shall only be issued after liquidation, partition and distribution of the parties’ properties under Article 147 of the Family Code." Dinah filed a motion for partial reconsideration questioning the portion of the decision on the issuance of a decree of nullity of marriage only after the liquidation, partition and distribution of properties under Article 147 of the Code. If you are the judge, how will you decide petitioner’s motion for partial reconsideration? Why? (4%)

SUGGESTED ANSWER:

I will grant partial reconsideration. If the marriage is declared void under Article 36, the provisions of the Family Code on liquidation, partition, and distribution of the properties on absolute community or conjugal partnership will not apply but rather Article 147 or Article 148 depending on the presence or absence of a legal impediment between them. In Dino v. Dino,3 the SC ruled that Art. 50 of the Family Code and Section 19 of the Rules on Declaration of Nullity applies only to marriages which are declared void ab initio or annulled by final judgment under Articles 40 and 45 of the Family. In short, Art. 50 of the Family Code does not apply to marriages which are declared void ab initio under Art. 36 of the FC which should be declared void without waiting for the liquidation of the properties of the parties. VII. Due to the continuous heavy rainfall, the major streets in Manila became flooded. This compelled Cris to check-in at Square One Hotel. As soon as Crisgot off from his Toyota Altis, the Hotel’s parking attendant got the key of his car and gave him a valet parking customer’s claim stub. The attendant parked his car at the basement of the hotel. Early in the morning, Cris was informed by the hotel manager that his car was carnapped. (4%) (A) What contract, if any, was perfected between Cris and the Hotel when Cris surrendered the key of his car to the Hotel’s parking attendant? (B) What is the liability, if any, of the Hotel for the loss of Cris’ car?

3

Alain Dino vs. Ma. Caridad Dino G.R. No. 178044, January 19, 2011

11

SUGGESTED ANSWER: a) The contract between Cris and Square One Hotel is one of necessary deposit. Deposit of effects made by travelers or guests in hotels or inns is considered a necessary deposit. 4 This includes not only the personal effects brought inside the hotel premises but also vehicles or animals and articles which have been introduced or placed in the annexes of the hotel. b) In the case of Durban Apartments vs. Pioneer Insurance,5 the Supreme Court held the hotel liable for the loss of the vehicle of the guest after its valet parking attendant parked the vehicle in front of a bank near the hotel premises. The court ruled that the bank’s parking area became an annex of the hotel when the management of the bank allowed the hotel to park vehicles there on the night in question. The contract of deposit was perfected when the guest surrendered the keys to his vehicle to the parking attendant and the hotel is under obligation of safely keeping and returning it. Ultimately, Square One Hotel is liable for the loss of the vehicle. VIII. Tess leased her 1,500 sq. m. lot in Antipolo City to Ruth for a period of three (3) years, from January 2010 to February 2013. On March 19, 2011, Tess sent a letter to Ruth, part of which reads as follows: "I am offering you to buy the property you are presently leasing at P5,000.00 per sq. m. or for a total of P7,500,000.00. You can pay the contract price by installment for two (2) years without interest. I will give you a period of one (1) year from receipt of this letter to decide whether you will buy the property." After the expiration of the lease contract, Tess sold the property to her niece for a total consideration of P4 million. Ruth filed a complaint for the annulment of the sale, reconveyance and damages against Tess and her niece. Ruth alleged that the sale of the leased property violated her right to buy under the principle of right of first refusal. Is the allegation of Ruth tenable? (4%)

SUGGESTED ANSWER:

No, the allegation of Ruth is not tenable. The letter written by Tess did not grant a right of first refusal to Ruth. At most, it is to be construed as an option contract whereby Ruth was given the right to buy or not to buy the leased property. An option is itself not a purchase but it merely secures the privilege to buy. However, the option is not valid because it was not supported by a cause or consideration distinct from the price of the property. (Article 1479) Also, Ruth does not appear to have exercised her option before the offer was withdrawn by the subsequent sale of the property to the niece of Tess. IX.

4

5

Article 1998, Civil Code G.R. No. 179419 January 12, 2011

12

Spouses Macario and Bonifacia Dakila entered into a contract to sell with Honorio Cruz over a parcel of industrial land in Valenzuela, Bulacan for a price of Three Million Five Hundred Thousand Pesos (P3,500,000.00). The spouses would give a downpayment of Five Hundred Thousand Pesos (P500,000.00) upon the signing of the contract, while the balance would be paid for the next three (3) consecutive months in the amount of One Million Pesos (P1,000,000.00) per month. The spouses paid the first two (2) installments but not the last installment. After one (1) year, the spouses offered to pay the unpaid balance which Honorio refused to accept. The spouses filed a complaint for specific performance against Honorio invoking the application of the Maceda Law. If you are the judge, how will you decide the case? (4%)

SUGGESTED ANSWER:

I will rule in favor of Honorio. The invocation of the Maceda Law is misplaced. The law applies only to sale or financing of realty on installment payments including residential units or residential condominium apartments and does not apply to sales of industrial units or industrial lands like in the case presented. Another reason why the Maceda law will not apply is that, the sale in the case at bar is not the sale on installment as contemplated by the law. The sale on installment covered by the Maceda Law is one where the price is paid or amortized over a certain period in equal installments. The sale to the Spouses Dakila is not a sale on installment but more of a straight sale where a down payment is to be made and the balance to be paid in a relatively short period of three months. X. Dorotea leased portions of her 2,000 sq. m. lot to Monet, Kathy, Celia, and Ruth for five (5) years. Two (2) years before the expiration of the lease contract, Dorotea sold the property to PM Realty and Development Corporation. The following month, Dorotea and PM Realty stopped accepting rental payments from all the lessees because they wanted to terminate the lease contracts. Due to the refusal of Dorotea to accept rental payments, the lessees , Ruth, et al., filed a complaint for consignation of the rentals before the Regional Trial Court (RTC) of Manila without notifying Dorotea. Is the consignation valid? (4%) SUGGESTED ANSWER: No, the consignation is not valid. For consignation of the thing or sum due to be proper, there must be prior notice to the creditor that the debtor is going to consign the payment in court. This notice is intended to give the creditor the opportunity to accept payment and thus avoid liability for costs in case it is found that the act of consignation was properly made. Even on the assumption that Dorotea was no longer the creditor as she had already sold the property to DM Realty, the facts do not state that the realty corporation was also given notice before filing the case for consignation. XI. An easement that can be acquired by prescription: (1%) (A) Right of way (B) Watering of an animal (C) Lateral and subjacent support (D) Light and view Correct answer – letter D – only continuous and apparent easements may be acquired by prescription

13

XII. J.C. Construction (J.C.) bought steel bars from Matibay Steel Industries (MSI) which is owned by Buddy Batungbacal. J.C. failed to pay the purchased materials worth P500,000.00 on due date. J.C. persuaded its client Amoroso with whom it had receivables to pay its obligation to MSI. Amoroso agreed and paid MSI the amount of P50,000.00. After two (2) other payments, Amoroso stopped making further payments. Buddy filed a complaint for collection of the balance of the obligation and damages against J.C. J.C. denied any liability claiming that its obligation was extinguished by reason of novation which took place when MSI accepted partial payments from Amoroso on its behalf. Was the obligation of J.C. Construction to MSI extinguished by novation? Why? (4%)

SUGEGSTED ANSWER:

No, the obligation of JC was not extinguished by novation. Novation may either be objective or subjective. Subjective novation takes place by the substitution of debtor or subrogation of a third person to the rights of the creditor. Novation by substituting a new debtor may take place even without the knowledge or against the will of the original debtor but not without the consent of the creditor. Moreover, novation must be expressed and it cannot be implied and there must be an agreement that the old obligation is extinguished. In the case of JC, it does not appear that MSI had agreed to release JC from the obligation. Hence, the obligation of JC was not extinguished. XIII. Esteban and Martha had four (4) children: Rolando, Jun, Mark, and Hector. Rolando had a daughter, Edith, while Mark had a son, Philip. After the death of Esteban and Martha, their three (3) parcels of land were adjudicated to Jun. After the death of Jun, the properties passed to his surviving spouse Anita, and son Cesar. When Anita died, her share went to her son Cesar. Ten (10) years after, Cesar died intestate without any issue. Peachy, Anita’s sister, adjudicated to herself the properties as the only surviving heir of Anita and Cesar. Edith and Philip would like to recover the properties claiming that they should have been reserved by Peachy in their behalf and must now revert back to them. Is the contention of Edith and Philip valid? (4%)

SUGGESTED ANSWER:

No, the contention is not valid. The property adjudicated to Jun from the estate of his parents which he in turn left to Anita and Cesar is not subject to reservation in favor of Edith and Philip. In Mendoza et. al. vs.Policarpio, et. al. 6 the court ruled that lineal character of the reservable property is reckoned from the ascendant from whom the propositus received the property by gratuitous title. The ownership should be reckoned only from Jun, as he is the ascendant from where the first transmission occurred or from whom Cesar inherited the properties. Moreover, Article 891 provides that the person obliged to reserve the property should be an ascendant. Peachy is not Cesar’s ascendant but a mere collateral relative. On the assumption that the property is reservable, Edith and Philip being first cousins of Cesar who is the propositus are disqualified to be reservatarios as they are not third degree relatives of Cesar. XIV.

6

G.R. NO. 176422 -March 20, 2013

14

A pedestrian, who was four (4) months pregnant, was hit by a bus driver while crossing the street. Although the pedestrian survived, the fetus inside her womb was aborted. Can the pedestrian recover damages on account of the death of the fetus? (1%) (A) Yes, because of Article 2206 of the Civil Code which allows the surviving heirs to demand damages for mental anguish by reason of the death of the deceased. (B) Yes, for as long as the pedestrian can prove that she was not at fault and the bus driver was the one negligent. (C) No, because a fetus is not a natural person. (D) No, if the fetus did not comply with the requirements under Article 41 of the Civil Code. Correct Answer is letter D – Article 41 of the Civil Code requires that to be considered a person, a fetus with an intrauterine life of less than seven months must survive for the full twenty-four hours from complete separation from the mother’s womb. XV. Mr. Bong owns several properties in Pasig City. He decided to build a condominium named Flores de Manila in one of his lots. To fund the project, he obtained a loan from the National Bank (NB) secured by a real estate mortgage over the adjoining property which he also owned. During construction, he built three (3) pumps on the mortgaged property to supply water to the condominium. After one (1) year, the project was completed and the condominium was turned over to the buyers. However, Mr. Bong failed to pay his loan obligation to NB. Thus, NB foreclosed the mortgaged property where the pumps were installed. During the sale on public auction of the mortgaged property, Mr. Simon won in the bidding. When Mr. Simon attempted to take possession of the property, the condominium owners, who in the meantime constituted themselves into Flores de Manila Inc. (FMI), claimed that they have earlier filed a case for the declaration of the existence of an easement before the Regional Trial Court (RTC) of Pasig City and prayed that the easement be annotated in the title of the property foreclosed by NB. FMI further claimed that when Mr. Bong installed the pumps in his adjoining property, a voluntary easement was constituted in favor of FMI. Will the action prosper? (4%)

SUGGESTED ANSWER:

No, the action will not prosper. The essence of a mortgage is that it immediately subjects the property upon which it is imposed, and whoever the possessor may be, to the fulfillment of the obligation for whose security it was constituted.7 There was no voluntary easement in this case because at the time the water pumps were constructed, the subject lot where the water pumps were constructed and the condominium belong to the same person. No one can have an easement over his own property. (Bogo- Medellin vs. CA G.R. 124699, July 31, 2003.) Even of the assumption that an easement was created in favor of FMI that alone will not defeat the right of the mortgagee to enforce the security if the debtor defaults. XVI. A congregation for religious women, by way of commodatum, is using the real property owned and registered in the name of Spouses Manuel as a retreat house. Maria, a helper of the congregation discovered a chest in the backyard. When she opened the chest, it contained several pieces of jewelry and money. (4%)

7

Article 2126

15

(A) Can the chest containing the pieces of jewelry and money be considered as hidden treasure? (B) Who has the right to claim ownership of it?

SUGGESTED ANSWER: a) No, for property to be considered hidden treasure it must consist of money, jewelry or other precious objects, the lawful ownership of which does not appear. In the case at bar, the chest was just lay in the backyard and the real property where it was found belongs to the Spouses Manuel. They are thus presumed the owner of the chest where the jewelry was found. b) Since it does not come within the purview of hidden treasure, the spouses Manuel have the right to claim ownership over the chest as well as its contents. XVII. On March 30, 2000, Mariano died intestate and was survived by his wife, Leonora, and children, Danilo and Carlito. One of the properties he left was a piece of land in Alabang where he built his residential house. After his burial, Leonora and Mariano’s children extrajudicially settled his estate. Thereafter, Leonora and Danilo advised Carlito of their intention to partition the property. Carlito opposed invoking Article 159 of the Family Code. Carlito alleged that since his minor child Lucas still resides in the premises, the family home continues until that minor beneficiary becomes of age. Is the contention of Carlito tenable? (4%)

SUGGESTED ANSWER:

No, the contention of Carlito is not tenable. In the case of Patricio v. Dario,8 with similar facts to the case at bar, the court ruled that to qualify as beneficiary of the family home the person must be among those mentioned in Article 154, he/she must be actually living in the family home and must be dependent for legal support upon the head of the family. While Lucas, the son of Carlito satisfies the first and second requisites, he cannot however, directly claim legal support from his grandmother, Leonora because the person primarily obliged to give support to Lucas is his father, Carlito. Thus, partition may be successfully claimed by Leonora and Danilo. XVIII. Spouses Magtanggol managed and operated a gasoline station on a 1,000 sq.m. lot which they leased from Francisco Bigla-awa. The contract was for a period of three (3) years. When the contract expired, Francisco asked the spouses to peacefully vacate the premises. The spouses ignored the demand and continued with the operation of the gasoline station. One month after, Francisco, with the aid of a group of armed men, caused the closure of the gasoline station by constructing fences around it. Was the act of Francisco and his men lawful? Why? (4%)

SUGGESTED ANSWER:

No, the act was not lawful. Even if the lessee’s right to occupy the premises has expired, the lessor cannot physically oust the lessee from the leased premises if the latter refuses to vacate. The lessor must go through the proper channels by filing an appropriate case for unlawful detainer or recovery of possession. Every possessor has a right to be respected in his possession (Article 539) and in no case my possession be acquired through force or intimidation as long as there is a possessor who objects thereto. (Article 8

G.R. No. 170829 November 20, 2006

16

536) The act of Francisco is an abuse of rights because even if he has the right to recover possession of his property, he must act with justice and give the lessees their day in court and observe honesty and good faith. XIX. Who enjoys the Right of Retention? (1%) (A) Depositary until full payment of what may be due him in deposit. (B) Lessee if he advances the expenses for the repair of the leased premises. (C) Bailee if bailor owes him something. (D) Builder in bad faith for the recovery of necessary and useful expenses.

Correct answer is letter A – depositary (Article 1994) XX.

Mabuhay Elementary School organized a field trip for its Grade VI students in Fort Santiago, Manila Zoo, and Star City. To be able to join, the parents of the students had to sign a piece of paper that reads as follows: "I allow my child (name of student), Grade – Section, to join the school’s field trip on February 14, 2014. I will not file any claim against the school, administrator or teacher in case something happens to my child during the trip." Joey, a 7-year-old student of Mabuhay Elementary School was bitten by a snake while the group was touring Manila Zoo. The parents of Joey sued the school for damages. The school, as a defense, presented the waiver signed by Joey’s parents. Was there a valid waiver of right to sue the school? Why? (4%) SUGGESTED ANSWER:

No, there was no valid waiver of the right to sue the school. A waiver to be valid must have three requisites 1) existence of the right; 2) legal capacity of the person waiving the right and 3) the waiver must not be contrary to law, morals, good customs, public order or public policy or prejudicial to a third person with a right recognized by law. In the case presented, the waiver may be considered contrary to public policy as it exonerates the school from liability for future negligence. The waiver in effect allows the school to not exercise even ordinary diligence. XXI.

A delayed accession is: (1%) (A) formation of an island (B) avulsion (C) alluvium 17

(D) change in the course of the riverbed Correct answer is letter B (Article 459 Civil Code) XXII.

On March 27, 1980, Cornelio filed an application for land registration involving a parcel of agricultural land that he had bought from Isaac identified as Lot No. 2716 with an area of one (1) hectare. During the trial, Cornelio claimed that he and his predecessors-in-interest had been in open, continuous, uninterrupted, public and adverse possession and occupation of the land for more than thirty (30) years. He likewise introduced in evidence a certification dated February 12, 1981 citing a presidential declaration to the effect that on June 14, 1980, agricultural lands of the public domain, including the subject matter of the application, were declared alienable and disposable agricultural land. (4%) (A) If you are the judge, will you grant the application for land registration of Cornelio? (B) Can Cornelio acquire said agricultural land through acquisitive prescription, whether ordinary or extraordinary? SUGGESTED ANSWER: a) No, I will not grant the application. To be entitled to registration of the parcel of land, the applicant must show that the land being applied for is alienable land. At the time of the filing of the application, the land has not yet been declared alienable by the state. (Republic v. CA, G.R. No. 144057, January 17, 2005) b) Cornelio can acquire the land by acquisitive prescription only after it was declared part of alienable land by the state by possession for the required number of years for ordinary prescription, ten years possession in good faith with just title or extraordinary prescription by possession for thirty years without need of any other condition. (Article 1134, Civil Code) XXIII.

After undergoing sex reassignment in a foreign country, Jose, who is now using the name of "Josie," married his partner Ador. Is the marriage valid? (1%) (A) Yes, the marriage is valid for as long as it is valid in the place where it is celebrated following Article 17 of the Civil Code. (B) Yes, the marriage is valid if all the essential and formal elements of marriage under the Family Code are present. (C) No, the marriage is not valid because one essential element of marriage is absent. (D) No, the marriage is not valid but is voidable because "Josie" concealed her real identity. Correct answer is letter C – not valid for lack of one essential requirement (Silverio v. Republic G.R. No. 174689, October 22, 2007) XXIV.

18

Ted, married to Annie, went to Canada to work. Five (5) years later, Ted became a naturalized Canadian citizen. He returned to the Philippines to convince Annie to settle in Canada. Unfortunately, Ted discovered that Annie and his friend Louie were having an affair. Deeply hurt, Ted returned to Canada and filed a petition for divorce which was granted. In December 2013, Ted decided to marry his childhood friend Corazon in the Philippines. In preparation for the wedding, Ted went to the Local Civil Registry of Quezon City where his marriage contract with Annie was registered. He asked the Civil Register to annotate the decree of divorce on his marriage contract with Annie. However, he was advised by the National Statistics Office (NSO) to file a petition for judicial recognition of the decree of divorce in the Philippines. Is it necessary for Ted to file a petition for judicial recognition of the decree of divorce he obtained in Canada before he can contract a second marriage in the Philippines? (4%) SUGGESTED ANSWER:

Yes, a divorce decree even if validly obtained abroad cannot have effect in the Philippines unless it is judicially recognized through an appropriate petition filed before Philippine courts. In Corpuz v. Sto. Tomas,9 the SC ruled that the foreigner must file a petition under Rule 108 and prove therein the fact of divorce by presenting an official copy attested by the officer having custody of the original. He must also prove that the court which issued the divorce has jurisdiction to issue it and the law of the foreign country on divorce. XXV.

Mario executed his last will and testament where he acknowledges the child being conceived by his live-in partner Josie as his own child; and that his house and lot in Baguio City be given to his unborn conceived child. Are the acknowledgment and the donation mortis causa valid? Why? (4%) SUGGESTED ANSWER:

Yes, the acknowledgment is considered valid because a will (although not required to be filed by the notary public) may still constitute a document which contains an admission of illegitimate filiation. Article 834 also provides that the recognition of an illegitimate child does not lose its legal effect even though the will wherein it was made should be revoked. This provision by itself warrants a conclusion that a will may be considered as proof of filiation. The donation mortis causa may be considered valid because although unborn, a fetus has a presumptive personality for all purposes favorable to it provided it be born under the conditions specified in Article 41.

XXVI.

Isaac leased the apartment of Dorotea for two (2) years. Six (6) months after, Isaac subleased a portion of the apartment due to financial difficulty. Is the sublease contract valid? (1%) (A) Yes, it is valid for as long as all the elements of a valid sublease contract are present. (B) Yes, it is valid if there is no express prohibition for subleasing in the lease contract. (C) No, it is void if there is no written consent on the part of the lessor.

9

Gerbert Corpuz vs. Daisylyn Sto. Tomas G.R. No. 186571; August 11, 2010

19

(D) No, it is void because of breach of the lease contract. Correct answer is letter B – Article 1650 XXVII.

Fe, Esperanza, and Caridad inherited from their parents a 500 sq. m. lot which they leased to Maria for three (3) years. One year after, Fe, claiming to have the authority to represent her siblings Esperanza and Caridad, offered to sell the leased property to Maria which the latter accepted. The sale was not reduced into writing, but Maria started to make partial payments to Fe, which the latter received and acknowledged. After giving the full payment, Maria demanded for the execution of a deed of absolute sale which Esperanza and Caridad refused to do. Worst, Maria learned that the siblings sold the same property to Manuel. This compelled Maria to file a complaint for the annulment of the sale with specific performance and damages. If you are the judge, how will you decide the case? (4%) SUGGESTED ANSWER:

I will dismiss the case for annulment of the sale and specific performance filed by Maria with respect to the shares pertaining to Esperanza and Caridad. Since the object of the sale is a co-owned property, a co-owner may sell his undivided share or interest in the property owned in common but the sale will be subject to the result of the partition among the co-owners. In a co-ownership there is no mutual agency except as provided under Article 487. Thus, Fe cannot sell the shares of Esperanza and Caridad without a special power of attorney from them and the sale with respect to the shares of the latter without their written authority is void under Article 1874. Hence, the sale of the property to Manuel is not valid with respect to the shares of Esperanza and Caridad. Maria can only assail the portion pertaining to Fe as the same has been validly sold to her by Fe. XXVIII.

Spouses Esteban and Maria decided to raise their two (2) nieces, Faith and Hope, both minors, as their own children after the parents of the minors died in a vehicular accident. Ten (10) years after, Esteban died. Maria later on married her boss Daniel, a British national who had been living in the Philippines for two (2) years. With the permission of Daniel, Maria filed a petition for the adoption of Faith and Hope. She did not include Daniel as her co-petitioner because for Maria, it was her former husband Esteban who raised the kids. If you are the judge, how will you resolve the petition? (4%) SUGGESTED ANSWER:

I will dismiss the petition for adoption. The rule is that the husband and wife must jointly adopt and there are only three recognized exceptions to joint adoption by the husband and wife: 1) if one spouse seeks to adopt the legitimate child of the other; 2) if one spouse seeks to adopt his or her own illegitimate child; 3) if the spouses are legally separated. The case of Maria and Daniel does not appear to fall under any of the recognized exceptions, accordingly the petition filed by the wife alone should be dismissed. XXIX

20

Timothy executed a Memorandum of Agreement (MOA) with Kristopher setting up a business venture covering three (3) fastfood stores known as "Hungry Toppings" that will be established at Mall Uno, Mall Dos, and Mall Tres. The pertinent provisions of the MOA provides: 1. Timothy shall be considered a partner with thirty percent (30%) share in all of the stores to be set up by Kristopher; 2. The proceeds of the business, after deducting expenses, shall be used to pay the principal amount of P500,000.00 and the interest therein which is to be computed based on the bank rate, representing the bank loan secured by Timothy; 3. The net profits, if any, after deducting the expenses and payments of the principal and interest shall be divided as follows: seventy percent (70%) for Kristopher and thirty percent (30%) for Timothy; 4. Kristopher shall have a free hand in running the business without any interference from Timothy, his agents, representatives, or assigns , and should such interference happen, Kristopher has the right to buy back the share of Timothy less the amounts already paid on the principal and to dissolve the MOA; and 5. Kristopher shall submit his monthly sales report in connection with the business to Timothy. What is the contractual relationship between Timothy and Kristopher? (4%) SUGGESTED ANSWER:

The contractual relationship between Timothy and Kristopher is a contract of partnership as defined under Article 1767 of the Civil Code, since they have bound themselves to contribute money, property or industry to a common fund, with the intention of dividing the profits of the partnership between them. With a seed money of P500, 000.00 obtained by Timothy through a bank loan, they agreed to divide the profits, 70% for Kristopher and 30% for Timothy. However, to be more specific, theirs is a limited partnership as defined under Article 1843 of the Civil Code because Timothy does not take part in the control of the business pursuant to Article 1848, Civil Code. Nevertheless, Timothy is entitled to monthly sales reports in connection with the business, a right enshrined in Article 1851 of the Civil Code. XXX.

Joe Miguel, a well-known treasure hunter in Mindanao, executed a Special Power of Attorney (SPA) appointing his nephew, John Paul, as his attorney-infact. John Paul was given the power to deal with treasure-hunting activities on Joe Miguel’s land and to file charges against those who may enter it without the latter’s authority. Joe Miguel agreed to give John Paul forty percent (40%) of the treasure that may be found on the land. Thereafter, John Paul filed a case for damages and injunction against Lilo for illegally entering Joe Miguel’s land. Subsequently, he hired the legal services of Atty. Audrey agreeing to give the latter thirty percent (30%) of Joe Miguel’s share in whatever treasure that may be found in the land. Dissatified however with the strategies implemented by John Paul, Joe Miguel unilaterally revoked the SPA granted to John Paul. 21

Is the revocation proper? (4%) SUGGESTED ANSWER:

No, the revocation was not proper. As a rule, a contract of agency may be revoked by the principal at will.10 However, an agency ceases to be revocable at will if it is coupled with an interest or if it is a means of fulfilling an obligation already contracted. (Article 1922). In the case at bar, the agency may be deemed an agency coupled with an interest not only because of the fact that John Paul expects to receive 40% of whatever treasure may be found but also because he also contracted the services of a lawyer pursuant to his mandate under the contract of agency and he therefore stands to be liable to the lawyer whose services he has contracted. (Sevilla v. Tourist World Service, G.R. No. L-41182-3 April 16, 1988)

UNIVERSITY OF SANTO TOMAS SUGGESTED ANSWERS 2015 CIVIL LAW BAR EXAMINATIONS By: Assoc. Dean Viviana M. Paguirigan I. Alden and Stela were both former Filipino citizens. They were married in the Philippines but they later migrated to the United States where they were naturalized as American citizens. In their union they were able to accumulate several real properties both in the US and in the Philippines. Unfortunately, they were not blessed with children. In the US, they executed a joint will instituting as their common heirs to divide their combined estate in equal shares, the five siblingsand of Alden the seven siblings of Stela. Alden passed away in 2013 and a year later, Stela also died. The siblings of Alden who were all citizens of the US instituted probate proceedings in a US court impleading the siblings of Stela who were all in the Philippines. a)

Was the joint will executed by Alden and Stela who were both former Filipinos valid? Explain with legal basis. (3%)

b)

Can the joint will produce legal effect in the Philippines with respect to the propertiesand of Alden Stela found here? If so, how? (3%)

c)

Is the situation presented in Item I an example of depe9age? (2%)

SUGGESTED ANSWER: a) Yes, the joint will of Alden and Stela is considered valid. Being no longer Filipino citizens at the time they executed their joint will, the prohibition under our Civil Code on joint wills will no longer apply to Alden and Stela. For as long as their will was executed in accordance with the law of the place where they reside, or the law of the country of which they are citizens or even in accordance with the Civil Code, a will executed by an alien is considered valid in the Philippines. (Article 816) b) Yes, the joint will of Alden and Stela can take effect even with respect to the properties located in the Philippines because what governs the distribution of their estate is no longer Philippine law but their national law at the time of their demise. Hence, the joint will produces legal effect even with respect to the properties situated in the Philippines.

10

Article 1920

22

c) No, because depecage is a process of applying rules of different states on the basis of the precise issue involved. It is a conflict of laws where different issues within a case may be governed by the laws of different states. In the situation in letter (a) no conflict of laws will arise because Alden and Stela are no longer Filipino citizens at the time of the execution of their joint will and the place of execution is not the Philippines.

II. Marco and Gina were married in 1989. Ten years later, or in 1999, Gina left Marco and lived with another man, leaving their two children of school age with Marco. When Marco needed money for their children's education he sold a parcel of land registered in his name, without Gina's consent, which he purchased before his marriage. Is the sale by Marco valid, void or voidable? Explain with legal basis. (4%)

SUGGESTED ANSWER: The sale made by Marco is considered void. The parties were married in 1989 and no mention was made whether they executed a marriage settlement. In the absence of a marriage settlement, the parties shall be governed by absolute community of property whereby all the properties owned by the spouses at the time of the celebration of the marriage as well as whatever they may acquire during the marriage shall form part of the absolute community. In ACP, neither spouse can sell or encumber property belonging to the ACP without the consent of the other. Any sale or encumbrance made by one spouse without the consent of the other shall be void although it is considered as a continuing offer on the part of the consenting spouse upon authority of the court or written consent of the other spouse. (Article 96 FC)

III. Julie had a relationship with a married man who had legitimate children. A son was born out of that illicit relationship in 1981. Although the putative father did not recognize the child in his certificate of birth, he nevertheless provided the with child all the support he needed and spent time regularly with the child and his mother. When the man died in 2000, the child was already 18 years old so he filed a petition to be recognized as an illegitimate child of the putative father and sought to be given a share in his putative father's estate. The legitimate family opposed, saying that under the Family Code his action cannot prosper because he did not bring the action for recognition during the lifetime of his putative father. a)

If you were the judge in this case, would how you rule? (4%)

b) Wishing to keep the peace, the child during the pendency of the case decides to compromise with his putative father's family by abandoning his petition in exchange for Yi of what he would have received as inheritance if he were recognized as an illegitimate child. As the judge, would you approve such a compromise? (2%)

SUGGESTED ANSWER:

23

a) If I were the judge, I will not allow the action for recognition filed after the death of the putative father. Under the Family Code, an illegitimate child who has not been recognized by the father in the record of birth, or in a private handwritten instrument, or in a public document and may prove his filiation based on open and continuous possession of the status of an illegitimate child but pursuant to Article 175, he or she must file the action for recognition during the lifetime of the putative father. The provision of Article 285 of the Civil Code allowing the child to file the action for recognition even after the death of the father will not apply because in the case presented, the child was no longer a minor at the time of death of the putative father. b) No, I will not approve the compromise agreement because filiation is a matter to be decided by law. It is not for the parties to stipulate whether a person is a legitimate or illegitimate child of another. (De Jesus v. Estate of Dizon 366 SCRA 499) In all cases of illegitimate children, their filiation must be duly proved. (Article 887, Civil Code) ALTERNATIVE ANSWER: Yes, I would approve the compromise because it is no longer considered future inheritance. What the law prohibits is a compromise with respect to future legitime. In this case, the father is already dead so the compromise is considered valid. IV. Bert and Joe, both male and single, lived together as common law spouses and agreed to raise a son of Bert's living brother as their child without legally adopting him. Bert worked while Joe took care of their home and the boy. In their 20 years of cohabitation they were able to acquire real estate assets registered in their names as co-owners. Unfortunately, Bert died of cardiac arrest, leaving no will. Bert was survived by his biological siblings, Joe, and the boy. a) Can Article 147 on co-ownership apply to Bert and Joe, whereby all properties they acquired will be presumed to have been acquired by their joint industry and shall be owned by them in equal shares? (2%) b) What are the successional rights of the boy Bert Joe and raised as their son? (2%) c) If Bert and Joe had decided in the early years of their cohabitation to jointly adopt the boy, would they have been legally allowed to do so? Explain with legal basis. (3%)

SUGGESTED ANSWER: a) No, Article 147 cannot apply to Bert and Joe because the law only applies to a man and a woman who are capacitated to marry each other who live together as husband and wife without the benefit of marriage or under a void marriage. In the case of Bert and Joe, they are both men so the law does not apply. b) Neither of the two will inherit from Bert. Joe cannot inherit because the law does not recognize the right of a stranger to inherit from the decedent in the absence of a will. Their cohabitation will not vest Joe with the right to inherit from Bert. The child will likewise not inherit from Bert because of the lack of formal adoption of the child. A mere ward or “ampon” has no right to inherit from the adopting parents. (Manuel v. Ferrer, 247 SCRA 476) 24

c) No, because joint adoption is allowed between husband and wife. Even if Bert and Joe are cohabiting with each other, they are not vested with the right to jointly adopt under the Family Code or even under the Domestic Adoption Act. (Section 7, R.A. 8552) V. Mrs. L was married to a ship captain who worked for an international maritime vessel. For her and her family's support, she would claim monthly allotments from her husband's company. One day, while en route from Hong Kong to Manila, the vessel manned by Captain L encountered a severe typhoon at sea. The captain was able to send radio messages of distress to the head office until all communications were lost. In the weeks that followed, the search operations yielded debris of the lost ship but the bodies of the crew and the passengers were not recovered. The insurance company thereafter paid out the death benefits to all the heirs of the passengers and crew. Mrs. L filed a complaint demanding that her monthly allotments continue for the next four years until her husband may be legally presumed dead because of his absence. If you were the magistrate would how you rule? (3%)

SUGGESTED ANSWER: I would rule against Mrs. L. There is no merit in her contention that the monthly allotments to her should continue despite the presumptive death of the husband. In case of disappearance where there is danger of death, the person shall be presumed to have died at the beginning of the four (4) year period although his succession will be opened only at the end of the four year period. (Article 391, Civil Code) Since the husband of Mrs. L is presumed to have died at about the time of disappearance, he is no longer entitled to receive his salary from the day the presumption of death arises. VI. Kardo met Glenda as a young lieutenant and after a whirlwind courtship, they were married. In the early part of his military career, Kardo was assigned to different places all over the country but Glenda refused to accompany him as she preferred to live in her hometown. They did not live together until the 12th year of their marriage when Kardo had risen up the ranks and was given his own command. They moved to living quarters in Fort Gregorio. One day, while Kardo was away on official business, one of his military aides caught Glenda having sex with the corporal assigned as Kardo's driver. The aide immediately reported the matter to Kardo who rushed home to confront his wife. Glenda readily admitted the affair and Kardo sentawayher in anger. Kardo would later come to know the true extent of Glenda's unfaithfulness from his aides, his household staff, and former neighbors who informed him that Glenda has had intimate relations with various men throughout their marriage whenever Kardo was away on assignment. Kardo filed a petition for declaration of nullity of marriage under Article 36. Based on interviews from Kardo, his aide, and the housekeeper, a psychologist testified that Glenda's habitual infidelity was due to her affliction with Histrionic Personality Disorder, an illness characterized by excessive emotionalism and uncontrollable attention-seeking behavior rooted in Glenda's abandonment as a child by her father. Kardo himself, his aide, and his housekeeper also testified in court. The RTC granted the petition, relying on the liberality espoused by Te v. Te and Azcueta v. Republic. However, the OSG filed an appeal, arguing that sexual infidelity was only a ground for legal separation and that the RTC failed to abide by the guidelines laid down in the Molina case. How would you decide the appeal? (5%) 25

SUGGESTED ANSWER: I will resolve the appeal in favor of the Republic. In the case of Dedel v. Dedel, (G.R. No. 151867 January 29, 2004) the Supreme Court refused to declare the marriage of the parties void on the ground of sexual infidelity of the wife Sharon. In case mentioned, the wife committed infidelity with several men up to the extent of siring two illegitimate children with a foreigner. The court, however, said that it was not shown that the sexual infidelity was a product of a disordered personality and that it was rooted in the history of the party alleged to be psychologically incapacitated. Also, the finding of psychological incapacity cannot be based on the interviews conducted by the clinical psychologist on the husband or his witnesses and the person alleged to be psychologically incapacitated must be personally examined to arrive at such declaration. (Marcos v. Marcos, 343 SCRA 755; Agraviador v. Agraviador, G.R. No. 170729- December 8, 2010) VII. Mr. and Mrs. X migrated to the US with all their children. As they had no intention of coming back, they offered their house and lot for sale to their neighbors, Mr. and Mrs. A (the buyers) who agreed to buy the property for 128 Million. Because Mr. and Mrs. A needed to obtain a loan from a bank first, and since the sellers were in a hurry to migrate, the latter told the buyers that they could already occupy the house, renovate it as it was already in a state of disrepair, and pay only when their loan is approved and released. While waiting for the loan approval, the buyers spent .Pl Million in repairing the house. A month later, a person carrying an authenticated special power of attorney from the sellers demanded that the buyers either immediately pay for the property in full now or vacate it and pay damages for having made improvements on the property without a sale having been perfected. a) What are the buyers' options or legal rights with respect to the they expenses incurred in improving the property under circumstances? (3%) b) Can the buyers be made to immediately vacate on the ground that the sale was not perfected? Explain briefly. (3%)

SUGGESTED ANSWER: a) The buyers here may be deemed possessors or builders in good faith because they were made to believe that they were allowed to make repairs or renovation by the sellers themselves. As builders in good faith, they have the right to seek reimbursement for the value of the improvements in case the owner decides to appropriate them. They cannot be asked to remove the improvements because that is not one of the options given by law to the landowner in case the builder is in good faith. b) No, the buyers cannot be made to vacate on the ground that the sale was not perfected for the fact of the matter is that a contract of sale is consensual and is perfected by mere consent. (Article 1315, Civil Code) In this case, there was an agreement to deliver a determinate thing for a price certain in money. When the owners made an offer to sell their property to Mr. and Mrs. A and the latter accepted the offer, there was already a meeting of the minds between the parties resulting in the perfection of the contract of sale.

26

VIII. X, Y, Z are siblings who inherited a IO-storey building from their parents. They agreed in writing to maintain it as a co-owned property for leasing out and to divide the net profits among themselves equally for a period of 20 years. On the gth year, X wanted to get out of the coownership so he could get his 1/3 share in the property. Y and Z refused, saying X is bound by their agreement to keep the co-ownership for 20 years. Are Y and Z correct? Explain. (3%)

SUGGESTED ANSWER:

Y and Z are partly correct. The law provides that none of the co-owners shall be obliged to remain in the co-ownership and it is the right of a co-owner to ask for partition of the coownership anytime. One exception to the rule is if the co-owners agree to keep the thing undivided which period shall not exceed ten years. In this case, the agreement to keep the thing undivided shall be valid at the most for ten years. (Article 494, Civil Code) IX. Jose, single, donated a house and lot to his only niece, Maria, who was of legal age and who accepted the donation. The donation and Maria's acceptance thereof were evidenced by a Deed of Donation. Maria then lived in the house and lot donated to her, religiously paying real estate taxes thereon. Twelve years later, when Jose had already passed away, a woman claiming to be an illegitimate daughter of Jose filed a complaint against Maria. Claiming rights as an heir, the woman prayed that Maria be ordered to reconvey the house and lot to Jose's estate. In her complaint she alleged that the notary public who notarized the Deed of Donation had an expired notarial commission when the Deed of Donation was executed by Jose. Can Maria be made to reconvey the property? What can she put up as a defense? (4%) SUGGESTED ANSWER: No. Maria cannot be compelled to reconvey the property. The Deed of Donation was void because it was not considered a public document. However, a void donation can trigger acquisitive prescription. (Solis v. CA 176 SCRA 678; Doliendo v. Biarnesa 7 Phil. 232) The void donation has a quality of titulo colorado enough for acquisitive prescription especially since 12 years had lapsed from the deed of donation. ALTERNATIVE ANSWER: Yes, Maria can be made to reconvey the property. The law provides that no person may give or receive by way of donation more than what he may give or receive by will. On the assumption that the property donated to Maria is the only property of Jose, the legitime of his illegitimate child would be impaired if Maria would be allowed to keep the entire property. After taking into account the value of the property, Maria can be made to reconvey the property to the extent necessary to satisfy the legitime of Jose’s illegitimate daughter provided that the woman claiming to be Jose’s child can prove her filiation to the deceased. Maria can set up the defense that the action has prescribed. An action for revocation of the donation on the ground that it impaired the legitime of a compulsory heir may only be filed within ten (10) years from the time the cause of action accrues which is at the time of the death of Jose. The facts are not clear as to when Jose died but on the assumption that he died ten years prior to the filing of the action, the same has clearly prescribed. X. X, a dressmaker, accepted clothing materials from Karla to make two dresses for her. dayOn the X was supposed to deliver Karla's dresses, X called up Karla to say that she had an 27

urgent matter to attend to and will deliver them the next day. That night, however, a robber broke into her shop and took everything including Karla's two dresses. X claims she is not liable to deliver Karla's dresses or to pay for the clothing materials considering she herself was a victim of the robbery which was a fortuitous event and over which she had no control. Do you agree? Why? (3%) SUGGESTED ANSWER: No, I do not agree with the contention of X. The law provides that except when it is otherwise declared by stipulation or when the law provides or the nature of the obligation requires the assumption of risk, no person shall be liable for those events which could not be foreseen or which though foreseen were inevitable. (Article 1174, Civil Code) In the case presented, X cannot invoke fortuitous event as a defense because she had already incurred in delay at the time of the occurrence of the loss. (Article 1165, Civil Code) XI. Jackie, 16, inherited a townhouse. Because she wanted to study in an exclusive school, she sold her townhouse by signing a Deed of Sale and turning over possession of the same to the buyer. Whenthatthe buyer discovered she was still a minor, she promised to execute another Deed of Sale when she turns 18. When Jackie turned 25 and was already working, she wanted to annul the sale and return the buyer's money to recover her townhouse. Was the sale contract void, voidable or valid? Can Jackie still recover the property? Explain. (4%) SUGGESTED ANSWER: The contract of sale was voidable on the ground that Jackie is incapable of giving consent at the time of the execution of the sale. (Article 1390 and Article 1327) Jackie can no longer recover the townhouse unit because if a contract is voidable on the ground of minority, the action to annul it must be filed within four (4) years from attainment of the age of majority. Since Jackie was already 25 years old, the action has clearly prescribed because she should have filed it before she reached the age of 22. (Article 1391, Civil Code) XII. A. Iya and Betty owed Jun P500,000.00 for advancing their equity in a corporation they joined as incorporators. Iya and Betty bound themselves solidarily liable for the debt. Later, Iya and Jun became sweethearts so Jun condoned the debt of P500,000.00. May lya demand from Betty ~250,000.00 as her share in the debt? Explainlegal with basis. (2%) B. Juancho, Don and Pedro borrowed ~150,000.00 from their friend Cita to put up an internet cafe orally promising to pay her the full amount after one year. Because of their lack of business know-how, their business collapsed. Juancho and Don ended up penniless but Pedro was able to borrow money and put up a restaurant which did well. Can Cita demand that Pedro pay the entire obligation since he, together with the two others, promised to pay the amount in full after one year? Defend your answer. (2%) SUGGESTED ANSWER: a) No, Iya may not demand the 250,000 from Betty because the entire obligation has been condoned by the creditor Jun. In a solidary obligation the remission of the whole obligation obtained by one of the solidary debtors does not entitle him to reimbursement from his codebtors. (Article 1220, Civil Code) 28

b) No, Cita cannot demand that Pedro pay the entire obligation because the obligation in this case is presumed to be joint. The concurrence of two or more creditors or of two or more debtors in one and the same obligation does not imply that each one of the former has a right to demand, or that each one of the latter is bound to render, entire compliance with the prestation. (Article 1207) In a joint obligation, there is no mutual agency among the joint debtors such that if one of them is insolvent the others shall not be liable for his share. XIII. A. X and Y are partners in a shop offering portrait painting. Y provided the capital and the marketing while X was the portrait artist. They accepted the PS0,000.00 payment of Kyla to do her portrait but X passed away without being able to do it. Can Kyla demand that Y deliver the portrait she had paid for because she was dealing the with business establishment and not with the artist personally? Why or why not? (3%) B. In this jurisdiction, is a joint venture (i.e., a group of corporations contributing resources for a specific project and sharing the profits therefrom) considered a partnership? (3%)

SUGGESTED ANSWER: a) No Kyla cannot demand that Y deliver the portrait. The death of X has the effect of dissolving the partnership. (Article 1830, Civil Code) Also, while the obligation was contracted by the partnership, it was X who was supposed to create the portrait for Kyla. Since X died before creating the portrait, the obligation can no longer be complied because of impossibility of performance. (Article 1266) In obligations to do, the debtor shall be released when the prestation becomes legally or physically impossible without the debtor’s fault. b) Yes, under Philippine law, a joint venture is understood to mean an organization formed for some temporary purpose and is hardly distinguishable form a partnership since its elements are similar which are: community of interest in business, sharing of profits, and losses, and a mutual right of control. (Primelink Properties v. Lazatin June 27, 2006 citing Blackner v. Mcdermott, 176 F. 2d 498[1949]) XIV. A driver of a bus owned by company Z ran over a boy who died instantly. A criminal case for reckless imprudence resulting in homicide was filed against the driver. He was convicted and was ordered to pay P2 Million in actual and moral damages to the parents of the boy who was an honor student and had a bright future. Without even trying to find out if the driver had assets or means to pay the award of damages, the parents of the boy filed a civil action against the bus company to make it directly liable for the damages. a) Will their action prosper? (4%) b) If the parents of the boy do not wish to file a separate civil action against. the bus company, can they still make the bus company liable if the driver cannot' pay the award for damages? If so, what is the nature of the employer's liability and how may civil damages be satisfied? (3%)

SUGGESTED ANSWER: 29

a) Yes, the action will prosper. The liability of the employer in this case may be based on quasidelict and is included within the coverage of independent civil action. It is not necessary to enforce the civil liability based on culpa aquiliana that the driver or employee be proven to be insolvent since the liability of the employer for the quasi-delicts committed by their employees is direct and primary subject to the defense of due diligence on their part. (Article 2176; Article 2180) b) Yes, the parents of the boy can enforce the subsidiary liability of the employer in the criminal case against the driver. The conviction of the driver is a condition sine qua non for the subsidiary liability of the employer to attach. Proof must be shown that the driver is insolvent. (Article 103, Revised Penal Code) XV. A. Sara borrowed PS0,000.00 from Julia and orally promised to pay it within six months. When Sara tried to pay her debt on the gth month, Julia demanded the payment of interest of 12o/o per annum because of Sara's delay in payment. Sara paid her debt and the interest claimed by Julia. After rethinking, Sara demanded back from Julia the amount she had paid as interest. Julia claims she has no obligation to return the interest paid by Sara because it was a natural obligation which Sara voluntarily performed and can no longer recover. Do you agree? Explain. (4%) B.

Distinguish civil and natural obligations. (2%)

SUGGESTED ANSWER: a) No, the case is not one of a natural obligation because even if the contract of loan is verbal, the delay of Julia made her liable for interest upon demand by Sara. This is not a case of a natural obligation but a civil obligation to pay interest by way of damages by reason of delay. (Article 1956; Article 1169; Article 2209 Civil Code) b) A civil obligation is based on positive law which gives a right of action to compel their performance in case of breach. A natural obligation is based on equity and natural law and cannot be enforced by court action but after voluntary fulfilment by the obligor, they authorize the retention of what may have been delivered or rendered by reason thereof. (Article 1423, Civil Code) XVI. Donna pledged a set of diamond ring and earrings to Jane for P200,000.00 She was made to sign an agreement that if she cannot pay her debt within six months, Jane could immediately appropriate the jewelry for herself. After six months, Donna failed to pay. Jane then displayed the earrings and ring set in her jewelry shop located in a mall. A buyer, Juana, bought the jewelry set for P300,000.00. a) b) c)

Was the agreement which Donna signed with Jane valid? Explain with legal basis. (2%) Can Donna redeem the jewelry set from Juana by paying the amount she owed Jane to Juana? Explain with legal basis. (2%) Give an example of a pledge created by operation of law. (2%)

30

SUGGESTED ANSWER: a) appropriate the jewelry upon default of Donna is considered pactum commissorium and it is considered void by law. ( Article 2088) b) No, Donna cannot redeem it from Juana because the pledge contract is between her and Jane. Juana is not a party to the pledge contract. (Article 1311, Civil Code) c) One example of a pledge created by operation of law is the right of the depositary to retain the thing deposited until the depositor shall have paid him whatever may be due to the depositary by reason of the deposit. (1994) Another is the right of the agent to retain the thing which is the object of the agency until the principal reimburses him the expenses incurred in the execution of the agency. (Article 1914, Civil Code) XVII. Z, a gambler, wagered and lost P2 Million in baccarat, a card game. He was pressured into signing a Deed of Absolute Sale in favor of the winner covering a parcel ·of land with improvements worth P20 Million. One month later, the supposed vendee of the property demanded that he and his family vacate the property subject of the deed of sale. Was the deed of sale valid? What can Z do? (4%)

SUGGESTED ANSWER: The sale is valid. Being pressured to sign the deed of sale is not equivalent to vitiation of consent. Z however, can recover his losses from the winner because the law provides that no action can be maintained by the winner for the collection of what he has won in any game of chance. But any loser in a game of chance may recover his loss from the winner, with legal interests from the time he paid the amount lost. (Article 2014)

XVIII. A lawyer was given an authority by means of a Special Power of Attorney by his client to sell a parcel of land for the amount of P3 Million. Since the client owed the lawyer Pl Million in attorney's fees in a prior case he handled, the client agreed that if the property is sold, the lawyer was entitled to get 5% agent's fee plus Pl Million as payment for his unpaid attorney's fees. The client, however, subsequently found a buyer of his own who was willing to buy the property for a higher amount. Can the client unilaterallythe rescind authority he gave in favor of his lawyer? Why or why not? (4%)

SUGGESTED ANSWER: No, the agency in the case presented is one which is coupled with an interest. As a rule, agency is revocable at will except if it was established for the common benefit of the agent and the principal. In this case, the interest of the lawyer is not merely limited to his commission for the sale of the property but extends to his right to collect his unpaid professional fees. Hence, it is not revocable at will. (Article 1927)

XIX. 31

Mr. A, a businessman, put several real estate properties under the name of his eldest son X because at that time, X was the only one of legal age among his four children. He told his son he was to hold those assets for his siblings until they become adults themselves. X then got married. After 5 years, Mr. A asked X to transfer the titles over three properties to his three siblings, leaving two properties for himself. To A’s surprise, X said that he can no longer be made to transfer the properties to his siblings because more than 5 years have passed since the titles were registered in his name. Do you agree? Explain. ( 4%) SUGGESTED ANSWER: No, the transfer of the properties in the name of X was without cause or consideration and it was made for the purpose of holding these properties in trust for the siblings of X. If the transfer was by virtue of a sale, the same is void for lack of cause or consideration. Hence, the action to declare the sale void is imprescriptible. (Article Heirs of Ureta vs. Ureta September 14, 2011- G.R. No. 165748 September 14, 2011 ALTERNATIVE ANSWER: No, I do not agree. A trust was created in favor of the siblings of X when their father A transferred the titles in his name. The facts are clear that X was to hold these assets for his siblings until they reach the age of majority. An action to recover property based on an implied trust prescribes in ten years from the time the title was issued in favor of the trustee. In the case presented, only five years had lapsed from the issuance of the title hence, the action has not yet prescribed.

XX. A. Mr. and Mrs. Roman and Mr. and Mrs. Cruz filed an application for registration of a parcel of land which after due proceedings was granted by the RTC acting registration as land court. However, before the decree of registration could be issued, the spouses Roman and the spouses Cruz sold the lot to Juan. In the notarized deed of sale, the sellers expressly undertook to submit the deed of sale to the land registration court so that the title to the property would be directly issued in Juan's name. Is such a stipulation valid? (2%) B.

Distinguish a direct attack from a collateral attack on a title. (2%)

C. If the title in Item XX.A is issued in the names of the original sellers, would a motion filed by Juan in the same case to correct or amend the title in order to reflect his name as owner considered be collateral attack? (2%) SUGGESTED ANSWER: a) Yes, because when one who is not the owner of the property sells or alienates it and later the seller or grantor acquires title, such title passes by operation of law to the buyer or grantee. (Article 1434, Civil Code) b) A direct attack on a title is one where the action filed is precisely for the purpose of pointing out the defects in the title with a prayer that it be declared void. A collateral attack is one where the action is not instituted for the purpose of attacking the title but the nullity of the title is raised as a defense in a different action.

32

c) No, because Juan is not attacking the title but merely invoking his right as transferee. Hence, it does not involve a collateral attack on the title.

33

I. State whether the following marital unions are valid, void, or voidable, and give the corresponding justifications for your answer: a. Ador and Becky’s marriage wherein Ador was afflicted with AIDS prior to the marriage. (2%) SUGGESTED ANSWER: Voidable. Under the Family Code, a marriage is voidable if either of the party was afflicted with a sexually transmissible disease which is serious and incurable, such as AIDS. Here, Ador was afflicted with AIDS at the time of the celebration of the marriage, a sexually transmissible disease considered to be serious and incurable. [Basis: Article 45(6), Family Code; discussed in p. 122, Vol. 1, Rabuya’s Civ Reviewer Book] b. Carlos’ marriage to Dina which took place after Dina had poisoned her previous husband Edu in order to free herself from any impediment in order to live with Carlos. (2%) SUGGESTED ANSWER: Void. Under the Family Code, a marriage is declared void by reason of public policy when one, with the intention to marry the other, killed that other spouse or his or her own spouse. Here, the wife killed her previous husband for the purpose of marrying the second husband. [Basis: Article 38 (9), Family Code; discussed in p. 94, Vol. 1, Rabuya’s Civ Reviewer Book] c. Eli and Fely’s marriage solemnized seven years after the disappearance of Chona, Eli’s previous spouse, after the plane she had boarded crashed in the West Philippine Sea. (2%) SUGGESTED ANSWER: If the marriage took place during the effectivity of the Family Code and Chona is in fact alive, the subsequent marriage is void for being bigamous because Eli failed to obtain a judicial declaration of presumptive death of the absentee spouse prior to contracting the subsequent marriage. Under the Family Code, a judicial declaration of presumptive death of the absentee is required to be obtained by the spouse present to make the subsequent marriage valid. However, had Chona really died when the plane crashed, the subsequent marriage of Eli is valid because the prior marriage was already terminated. [Basis: Article 41, Family Code; Armas v. Calisterio, 330 SCRA 201 (2000); discussed in pp. 99-100, Vol. 1, Rabuya’s Civ Reviewer Book] But if the subsequent marriage took place during the effectivity of the Civil Code, the marriage is valid until annulled (voidable) because no judicial declaration of presumptive death was required under the Civil Code.

d. David who married Lina immediately the day after obtaining a judicial decree annulling his prior marriage to Elisa. (2%) SUGGESTED ANSWER: Void. Under the Family Code, David is required to record the judgment of annullment and the partition and distribution of the properties of the spouses, as well as the delivery of the presumptive legitimes of their children, in the appropriate civil registry and registries of property prior to contracting the second marriage; otherwise, the subsequent marriage is void. [Basis: Article 35(6), in relation to Artcicles 53 and 52, Family Code; discussed in p. 83, Vol. 1, Rabuya’s Civ Reviewer Book] e. Marriage of Zoren and Carmina who did not secure a marriage license prior to their wedding, but lived together as husband and wife for 10 years without any legal impediment to marry. (2%) SUGGESTED ANSWER: Valid because their marriage is exceptional and exempt from the requirement of a marriage license. Under the Family Code, the marriage of a man and woman who lived exclusively as husband and wife for at least five years and without impediment is exempt from the requirement of a marriage license. [Basis: Article 34, Family Code; discussed in pp. 57-58, Vol. 1, Rabuya’s Civ Reviewer Book] II. In 1960, Rigor and Mike occupied two separate but adjacent tracts of land in Mindoro. Rigor’s tract was classified as timber land while Mike’s was classified as agricultural land. Each of them fenced and cultivated his own tract continuously for 30 years. In 1991, the Government declared the land occupied by Mike as alienable and disposable, and the one cultivated by Rigor as no longer intended for public use or public service. Rigor and Mike now come to you today for legal advice in asserting their right of ownership of their respective lands based on their long possession and occupation since 1960. a. What are the legal consequences of the 1991 declarations of the Government respecting the lands? Explain your answer. (2%) SUGGESTED ANSWER: As to the land occupied Mike, the same remains property of the public dominion. According to jurisprudence, the classification of the property as alienable and disposable land of the public domain does not change its status as property of the public dominion. There must be an express declaration by the State that the public dominion property is no longer intended

for public service or the development of the national wealth or that the property has been converted into patrimonial. Without such express declaration, the property, even if classified as alienable or disposable, remains property of the public dominion. [Basis: Heirs of Mario Malabanan v. Republic, 587 SCRA 172 (2009); Heirs of Mario Malabanan v. Republic, 704 SCRA 561 (2013); discussed in my FB wall on October 15 and 16, 2017 and pp. 338-344, Vol. 1, Rabuya’s Civil Law Reviewer] As to the land occupied by Rigor, the declaration that it is no longer intended for public use or public service converted the same into patrimonial property provided that such express declaration was in the form of a law duly enacted by Congress or in a Presidential Proclamation in cases where the President was duly authorized by law. According to jurisprudence, when public land is no longer intended for public use, public service or for the development of the national wealth it is thereby effectively removed from the ambit of public dominion and converted into patrimonnial provided that the declaration of such conversion must be made in the form of a law duly enacted by Congress or by a Presidential proclamation in cases where the President is duly authorized by law to that effect. [Basis: Heirs of Mario Malabanan v. Republic, 587 SCRA 172 (2009); Heirs of Mario Malabanan v. Republic, 704 SCRA 561 (2013); discussed in my FB wall on October 15 and 16, 2017 and pp. 338-344, Vol. 1, Rabuya’s Civil Law Reviewer] b. Given that, according to Section 48(b) of Commonwealth Act No. 141, in relation to Section 14(1) of Presidential Decree No. 1529, the open, continuous, exclusive, and notorious possession and occupation of alienable and disposable lands of the public domain as basis for judicial confirmation of imperfect title must be from June 12, 1945, or earlier, may Mike nevertheless validly base his assertion of the right of ownership on prescription under the Civil Code? Explain your answer. (4%) SUGGESTED ANSWER: No, because the land remains property of public dominion and, therefore, not susceptible to acquisition by prescription. According to jurisprudence, the classification of the subject property as alienable and disposable land of the public domain does not change its status as property of the public dominion. In order to convert the property into patrimonial, there must be an express declaration by the State that the public dominion property is no longer intended for public service or the development of the national wealth or that the property has been converted into patrimonial. Without such express declaration, the property, even if classified as alienable or disposable, remains property of the public dominion, and thus incapable of acquisition by prescription. [Basis: Heirs of Mario Malabanan v. Republic, 587 SCRA 172 (2009); Heirs of Mario Malabanan v. Republic, 704 SCRA 561 (2013); discussed in my FB wall on October 15 and 16, 2017 and pp. 338-344, Vol. 1, Rabuya’s Civil Law Reviewer]

Here, the declaration of the property into alienable and disposable land of the public domain in 1991 did not convert the property into patrimonial in the absence of an express declaration of such conversion into patrimonial in the form of a law duly enacted by Congress or by a Presidential proclamation in cases where the President is duly authorized by law to that effect. c. Does Rigor have legal basis for his application for judicial confirmation of imperfect title based on prescription as defined by the Civil Code given that, like Mike, his open, continuous, exclusive, and notorious possession and occupation was not since June 12, 1945, or earlier, and his tract of land was timber land until the declaration in 1991? Explain your answer. (4%) SUGGESTED ANSWER: None, because Rigor’s possession was short of the period required by the Civil Code for purposes of acquisitive prescription which requires ten (10) years of continuous possession, if possession was in good faith and with a just title, or thirty years, in any event. While the property may be considered converted into patrimomial because of the 1991 declaration that it is no longer intended for public use or public service (provided that the declaration be in the form of a law of a law duly enacted by Congress or by a Presidential proclamation in cases where the President is duly authorized by law to that effect), Rigor failed to complete the 30-year period required by law in case of extra-ordinary prescription. Since the property was converted into patrimonial only in 1991, the period of presciption commenced to run beginning that year only. Rigor’s possession prior to the conversion of the property into patrimonial cannot be counted for the purpose of completing the prescriptive period because prescription did not operate against the State at that time, the property then being public dominion property. Rigor may not likewise acquire ownership by virtue of the shorter 10-year ordinary prescription because his possession was not in good faith and without a just title. [Basis: Heirs of Mario Malabanan v. Republic, 587 SCRA 172 (2009); Heirs of Mario Malabanan v. Republic, 704 SCRA 561 (2013); discussed in my FB wall on October 15 and 16, 2017 and pp. 338-344, Vol. 1, Rabuya’s Civil Law Reviewer] III. Josef owns a piece of land in Pampanga. The National Housing Authority (NHA) sought to expropriate the property for its socialized housing project. The trial court fixed the just compensation for the property at P50 million. The NHA immediately

deposited the same at the authorized depository bank and filed a motion for the issuance of a writ of possession with the trial court. Unfortunately, there was delay in the resolution of the motion. Meanwhile, the amount deposited earned interest. When Josef sought the release of the amount deposited, NHA argued that Josef should only be entitled to P50 million. Who owns the interest earned? (3%) SUGGESTED ANSWER: The interest earned belongs to Josef because bank interest partakes of the nature of civil fruits under Article 442 of the Civil Code and shall belong to the owner of the principal thing. When the National Housing Authority deposited the P50 Million as payment for the just compensation with an authorized depositary bank for the purpose of obtaining a writ of possession, it is deemed to be a constructive delivery of the said amount to Josef. Since Josef is entitled to the P50 Million and undisputably the owner of the said principal amount, the interest yield, as accession, in a bank deposit should likewise pertain to the owner of the money deposited. Being an attribute of ownership (jus fruendi), Josef’s right over the fruits, that is the bank interests, must be respected. [Basis: Republic v. Holy Trinity Realty Development Corp., G.R. No. 172410, April 14, 2008] IV. a. Distinguish antichresis from usufruct. (3%) SUGGESTED ANSWER: They are distinguished as follows: (1) Antichresis is always a contract while usufruct need not arise from a contract because it may also be constituted by law or by other acts inter vivos, such as donation, or in a last will and testament, or by prescription. (2) The subject matter of antichresis is always a real property while the subject matter of usufruct may either be real property or personal property. (3) Antichresis is an accessory contract or contract of security while usufruct is a real right.

(4) While in both, the fruits do not pertain to the owner, the usufructuary is entitled to enjoy the fruits while the antichretic creditor has the obligation to apply the fruits to the payment of the interest, if owing, and therefatre to the principal of the credit. b. Distinguish commodatum from mutuum. (3%) They are distinguished, as follows: (1) As to subject matter: The subject matter of commodatum is ordinarily non-consumable while the subject matter of mutuum is either money or consumable; (2) As to compensation: Commodatum is essentially gratuitous while mutuum may be gratuitous or with a stipulation to pay interest; (3) As to right in subject matter: In commodatum, there is no transmission fo ownership of the thing loaned while in mutuum, the borrower acquires ownership of the thing borrowed. (4) As to duty of borrower: In commodatum, the same thing borrowed is required to be returned while in mutuum, the borrower discharges himself, not by returning the identical thing loaned, but by paying its equivalent in kind, quality and quantity. [Discussed in pp. 725-726, Vol. 1, Rabuya’s Civil Law Reviewer] V. Jacob has owned a farm land in Ramos, Tarlac. In 2012, Liz surreptitiously entered and cultivated the property. In 2014, Jacob discovered Liz’s presence in and cultivation of the property. Due to his being busy attending to his business in Cebu, he tolerated Liz’s cultivation of the property. Subsequently, in December 2016, Jacob wanted to regain possession of the property; hence, he sent a letter to Liz demanding that she vacate the property. Liz did not vacate despite the demand. Jacob comes to enlist your legal assistance to bring an action against Liz to recover the possession of the property. What remedies are available to Jacob to recover possession of his property under the circumstances? Explain your answer. (4%) SUGGESTED ANSWER: The remedy available to Jacob is accion publiciana, or an action for the recovery of the better right of possession. It also refers to an ejectment suit filed after the expiration of one

year from accrual of the cause of action or from the unalwful withholding of possession of the realty. Since the entry made by Liz is through stealth, Jacob could have filed an action for forcible entry. Ordinarily, the one-year period within which to bring an action for forcible entry is generally counted from the date of actual entry on the land, except that when the entry is through stealth, the one-year period is counted from the time the plaintiff learned thereof. Here, since more than one year had elapsed since Jacob learned of the entry made by Liz through stealth, the action that may be filed by Jacob is no longer forcible entry, but an accion publiciana. [Basis: Canlas v. Tubil, 601 SCRA 147 (2009); Valdez v. CA, 489 SCRA 369 (2006); discussed in pp. 353-354, Vol. 1, Rabuya’s Civil Law Reviewer] VI. Tyler owns a lot that is enclosed by the lots of Riley to the North and East, of Dylan to the South, and of Reece to the West. The current route to the public highway is a kilometer’s walk through the northern lot of Riley, but the route is a rough road that gets muddy during the rainy season, and is inconvenient because it is only 2.5 meters wide. Tyler’s nearest access to the public highway would be through the southern lot of Dylan. May Dylan be legally required to afford to Tyler a right of way through his property? Explain your answer. (4%) SUGGESTED ANSWER: No, Dylan is not entitled to a grant of compulsory right of way because he has an adequate outlet going to the public highway. One of the requisites for a compulsory grant of right of way is that the estate of the claimant of a right of way must be isolated and without adequate outlet to a public highway. The true standard for the grant of compulsory right of way is “adequacy” of outlet going to a public highway and not the convenience of the dominant estate. In the case at bar, there is already an existing adquate outlet from the dominant estate to a public highway. Even if said outlet be incovenient, the need to open up another servitude is entirely unjustified. [Basis: Article 649, Civil Code; Dichoso, Jr. v. Marcos, 647 SCRA 495 (2011); Costabella Corp. v. CA, 193 SCRA 333 (1991); discussed in pp. 559-561, Vol. 1, Rabuya’s Civil Law Reviewer]

VII. Alice agreed to sell a parcel of land with an area of 500 square meters registered in her name and covered by TCT No. 12345 in favor of Bernadette for the amount of P900,000. Their agreement dated October 15, 2015, reads as follows: I, Bernadette, agree to buy the lot owned by Alice covered by TCT No. 12345 for the amount of P900,000 subject to the following schedule of payment: Upon signing of agreement – P100,000 November 15, 2015 – P200,000 December 15, 2015 – P200,000 January 15, 2016 – P200,000 February 15, 2016 – P200,000 Title to the property shall be transferred upon full payment of P900,000 on or before February 15, 2016. After making the initial payment of P100,000 on October 15, 2015, and the second installment of P200,000 on November 15, 2015, Bernadette defaulted despite repeated demands from Alice. In December 2016, Bernadette offered to pay her balance but Alice refused and told her that the land was no longer for sale. Due to the refusal, Bernadette caused the annotation of her adverse claim upon TCT No. 12345 on December 19, 2016. Later on, Bernadette discovered that Alice had sold the property to Chona on February 5, 2016, and that TCT No. 12345 had been cancelled and another one issued (TCT No. 67891) in favor of Chona as the new owner. Bernadette sued Alice and Chona for specific performance, annulment of sale and cancellation of TCT No. 67891. Bernadette insisted that she had entered into a contract of sale with Alice; and that because Alice had engaged in double sale, TCT No. 67891 should be cancelled and another title be issued in Bernadette’s favor. a. Did Alice and Bernadette enter into a contract of sale of the lot covered by TCT No. 12345? Explain your answer. (4%) SUGGESTED ANSWER:

No, because in the agreement between Alice and Bernadette the ownership is reserved in the vendor and is not to pass to the vendee until full payment of the purchase price, which makes the contract one of contract to sell and not a contract of sale. Distinctions between a contract to sell and a contract of sale are well-established in jurisprudence. In a contract of sale, the title to the property passes to the vendee upon the delivery of the thing sold; in a contract to sell, ownership is, by agreement, reserved in the vendor and is not to pass to the vendee until full payment of the purchase price. Otherwise stated, in a contract of sale, the vendor loses ownership over the property and cannot recover it until and unless the contract is resolved or rescinded; whereas, in a contract to sell, title is retained by the vendor until full payment of the price. In the latter contract, payment of the price is a positive suspensive condition, failure of which is not a breach but an event that prevents the obligation of the vendor to convey title from becoming effective. [Saberon v. Ventanilla, Jr., 722 SCRA 287 (2014); Spouses Torrecampo v. Alindogan, 545 Phil. 686 (2007); discussed in pp. 363-366, Vol. 2, Rabuya’s Civil Law Reviewer] In the case at bar, the contract entered between the parties is a contract to sell because ownership is retained by the vendor and is not to pass to the vendee until full payment of the purchase price. b. Did Alice engage in double sale of the property? Explain your answer. (4%) SUGGESTED ANSWER: NO, because there was no previous sale of the same property prior to its sale to Chona. Despite the earlier transaction of Alice with Bernadette, the former is not guilty of double sale because the previous transaction with Bernadette is charactrerized as a contract to sell. In a contract to sell, there being no previous sale of the property, a third person buying such property despite the fulfillment of the suspensive condition such as the full payment of the purchase price, for instance, cannot be deemed a buyer in bad faith and the prospective buyer cannot seek the relief of reconveyance of the property. There is no double sale in such case. Title to the property will transfer to the buyer after registration because there is no defect in the owner-sellers title per se, but the latter, of course, may be sued for damages by the intending buyer. [Basis: Coronel v. CA, 263 SCRA 15 (1996); discussed in pp. 363-366, Vol. 2, Rabuya’s Civil Law Reviewer]

VIII. Pedro had worked for 15 years in Saudi Arabia when he finally decided to engage in farming in his home province where his 10-hectare farmland valued at P2,000,000 was located. He had already P3,000,000 savings from his long stint in Saudi Arabia. Eagerly awaiting Pedro’s arrival at the NAIA were his aging parents Modesto and Jacinta, his common-law spouse Veneranda, their three children, and Alex, his child by Carol, his departed legal wife. Sadly for all of them, Pedro suffered a stroke because of his over-excitement just as the plane was about to land, and died without seeing any of them. The farmland and the savings were all the properties he left. (a) State who are Pedro’s legal heirs, and the shares of each legal heir to the estate? Explain your answer. (4%) SUGGESTED ANSWER: Pedro’s legal heirs are his legitimate child, Alex, and his three illegitimate chidlren with Veneranda. Pedro’s chidlren with Veneranda are illegitimate because they were conceived and born outside of a valid marriage. Alex, on the other hand, is a legitimate child because she was conceived or born inside a valid marriage. Pedro’s surviving parents are not legal heirs because they are excluded by Alex. In intestate succession, the legitimate ascendants do not become legal heirs if there is a surviving legitimate descendant, such as Alex in the problem. Veneranda is not a legal heir of Pedro because she and Pedro were not married. Ordinarily, the share of an illegitimate child in intestate succession is one-half of the share of the legitimate child. Considering, however, that the three illegitimate chidlren will impair the legitime of Alex if the foregoing formula is followed, Alex is entitled instead to get his legitime, which is ½ of the estate, or P2.5 Million, while the remaining P2.5 Million is to be divided equally among the three illegitimate children of Pedro. Their legitimes in this case will likewise be their shares in intestate succession. [Discussed in pp. 944, Vol. 1, Rabuya’s Civil Law Reviewer] (b) Assuming that Pedro’s will is discovered soon after his funeral. In the will, he disposed of half of his estate in favor of Veneranda, and the other half in favor of his children and his parents in equal shares. Assuming also that the will is admitted to probate by the proper court. Are the testamentary dispositions valid and effective under the law on succession? Explain your answer. (4%) SUGGESTED ANSWER:

No, because the testamentary dispositions impair the legitimes of Pedro’s compulsory heirs. Following the provisions of the Civil Code, only Alex and Pedro’s three illegitimate children are Pedro’s compulsory heirs. Since Alex is Pedro’s legitimate descendant and a primary compulsory heir, she excludes Pedro’s parents as compulsory heirs, the latter being merely secondary compulsory heirs. However, the three illegitimate chidlren are considered concurring compulsory heirs who are also entitled to a share of the legitime. Under the law, the legitime of Alex, being a legitimate descendant, is ½ of Pedro’s estate, or P2.5 Million. The legitime of each of the illegitimate children is supposed to be ½ of the share of Alex, or P1.25 Million each. Considering, however, that the remaining portion of the estate is no longer sufficient to cover the supposed legitimes of the three illegitimate children, they will simply share equally in the remaining P2.5 Million. Consequently, there is no disposable free portion that Pedro may validly give to Veneranda or to his parents. Hence, the will is intrinsically invalid. [Discussed in pp. 859, Vol. 1, Rabuya’s Civil Law Reviewer] IX. Danny and Elsa were married in 2002. In 2012, Elsa left the conjugal home and her two minor children with Danny to live with her paramour. In 2015. Danny sold without EIsa’s consent a parcel of land registered in his name that he had purchased prior to the marriage. Danny used the proceeds of the sale to pay for his children’s tuition fees. Is the sale valid, void or voidable? Explain your answer. (3%) SUGGESTED ANSWER: The sale is void because the subject property is a community property which was sold without the consent of one of the spouses. Since the marriage of Danny and Elsa was celebrated during the effectivity of the Family Code without a marriage settlement, their property regime is absolute community of property, which is the property regime that applies by default under the Family Code in the absence of a marriage settlement. Under the regime of absolute community, properties acquired by the future spouses prior to the celebration of the marriage shall become community property after the marriage. Hence the subject property is a community property.

Under the regime of absolute community, the disposition or encumbrance of community property must have the written consent of the other spouse or the authority of the court without which the disposition or encumbrance is void Here, the sale of the absolute community property by the husband without the consent of the wife or the authority of the court renders the sale void, whatever may be the reason for such sale. The husband should have obtained court authorization in selling the community property for the purpose of using the proceeds thereof to pay his children’s tuition fees. [[Basis: Articles 75, 91 and 96, Family Code; discussed in pp. 145, 147 and 153, Vol. 1, Rabuya’s Civil Law Reviewer]. X. Briefly explain whether the following contracts are valid, rescissible, unenforceable, or void: (a) A contract of sale between Lana and Andy wherein 16-year old Lana agreed to sell her grand piano for 25,000.00. (2%) SUGGESTED ANSWER: Voidable. Under the Civil Code, a contract where one of the parties is incapable of giving consent to a contract is voidable. A minor, like Andy in this case, is incapable of giving consent to a contract. Hence, the contract is voidable. [Basis: Articles 1390(1) and 1327, Civil Code; discussed in p. 278, Vol. 2, Rabuya’s Civil Law Reviewer] (b) A contract of lease of the Philippine Sea entered by and between Mitoy and Elsa. (2%) SUGGESTED ANSWER: Void. Under the Civil Code, a contract whose cause, object or purpose is contrary to law, morals, good customs, public order or public policy is void. The Philippine Sea is either a property of public dominion (if within Philippine territory) or a common thing (if outside of Philippine territory) and, therefore, outside the commerce of men. Hence, it cannot be made the object of a contract. [Basis: Articles 1409(1) and 1347, Civil Code; discussed in pp. 217-218, Vol. 2, Rabuya’s Civil Law Reviewer] (c) A barter of toys executed by 12-year old Clarence and 10-year old Czar (2%) SUGGESTED ANSWER: Unenforceable. Under the Civil Code, a contract where both parties are incapable of giving consent to a contract is unenforceable. Here, both parties to the contract are minors and, therefore, incapable of giving consent to a contract. [Basis: Articles 1403(3) and 1327, Civil Code; discussed in p. 278, Vol. 2, Rabuya’s Civil Law Reviewer] (d)A sale entered by Barri and Garri, both minors, which their parents later ratified. (2%)

SUGGESTED ANSWER: Valid. Under the Civil Code, while both parties to the contract are minors and, therefore, incapable of giving consent, the ratification made by the parents of both the contracting parties shall nonetheless validate the contract from the inception. [Basis: Article 1407, Civil Code; discussed in p. 297, Vol. 2, Rabuya’s Civil Law Reviewer] (e) Jenny’s sale of her car to Celestine in order to evade attachment by Jenny’s creditors. (2%) SUGGESTED ANSWER: Rescissible. Under the Civil Code, a contract undertaken in fraud of creditors is rescissible when the latter cannot in any other manner collect the claims due them. [Basis: Article 1381 (3), Civil Code; discussed in p. 256, Vol. 2, Rabuya’s Civil Law Reviewer] XI. Zeny and Nolan were best friends for a long time already. Zeny borrowed 310,000.00 from Nolan, evidenced by a promissory note whereby Zeny promised to pay the loan “once his means permit.” Two months later, they had a quarrel that broke their longstanding friendship. Nolan seeks your advice on how to collect from Zeny despite the tenor of the promissory note. what will your advice be? Explain your answer. (3%) SUGGESTED ANSWER: I will advice Nolan to file first an action to fix the term or period because the fulfillment of the obligation itself cannot be demanded unti after the court has fixed the period for compliance therewith, and such period has arrived. Any action to compel performance brought before that would be premature. Under the Civil Code, when the debtor binds himself when his means permit to do so, the obligation shall be deemed to be one with a period, but which period shall be fixed by the court. In such a situation, the court is authorized to fix the period because the duration of the period depends exclusively upon the will of the debtor. Any action filed prior to the expiration of the period to be fixed by the court would be premature. [Basis: Articles 1180 and 1197, Civil Code; Concepcion v. People, 74 Phil. 63; Gonzales v. Jose, 66 Phil. 369; dicussed in pp. 70-72, Vol. 2, Rabuya’s Civil Law Reviewer]

XII. Krystal owns a parcel of land covered by TCT No. 12345 in Angeles City, Due to severe financial constraints, Krystal was lorc based in the property to RBP Corporation, a foreign corporation based in South Korea. Subsequently, RBP Corporation sold the property to Gloria, one of its most valued clients. Wanting her property back, Krystal, learning of the transfer of the property from RBP Corporation to Gloria, sued both of them in the Regional Trial Court (RTC) for annulment of sale and for reconveyance. She alleged that the sale by RBP Corporation to Gloria was void because RBP Corporation was a foreign corporation prohibited by the Constitution from acquiring and owning lands in the Philippines. Will KrystaI’s suit for annulment of sale and reconveyance prosper? Explain your answer. (4%) SUGGESTED ANSWER: No, because the flaw in the original transaction is considered cured by the subsequent transfer of the property to a Filipino citizen who is constitutionally qualified to own land in the Philippines. While the Constitutuion prohibits an alien from acquiring or holding title to private lands or to lands of the public domain in the Philippines, except only by way of hereditary succession, jurisprudence is consistent that if land is invalidly transferred to an alien who subsequently becomes a citizen or transfers it to a citizen, the flaw in the original transaction is considered cured and the title of the transferee is rendered valid. In the case at bar, the subsequent transfer of the property to Gloria, a Filipino citizen, has the effect of curing the defect of the original transaction in favor of RBP Corporation because the land has since become the property of a Filipino citizen who is constitutionally qualified to own land. As such, the prior invalid transfer can no longer be assailed because the objective of the constitutional provision -- to keep our land in Filipino hands -- has been served. [Basis: United Church Board of World Ministries vs. Sebastian, 159 SCRA 446, 451452, March 30, 1988; per Cruz, J. See also Tejido vs. Zamacoma, 138 SCRA 78, August 7, 1985; Sarsosa vda. de Barsobiavs. Cuenco, 113 SCRA 547, April 16, 1982; Godinez vs. Fong Pak Luen, 120 SCRA 223, January 27, 1983; Yap vs. Maravillas, 121 SCRA 244, March 28, 1983; De Castro vs. Tan, 129 SCRA 85, April 30, 1984]

XIII. TRUE or FALSE – Explain your answers. (a) All rights are considered as property. (2%) SUGGESTED ANSWER: False, because rights which are not patrimonial in nature, such as the right to liberty, the right to honor, family rights, etc., cannot be considered as property. [Basis: II Tolentino, Civil Code of the Philippines, 1992 ed., pp. 4-5] (b) A lessee cannot bring a case for quieting of title respecting the property that he leases. (2%) SUGGESTED ANSWER: False, because the action may be filed by anyone who has legal or equitable title to, or interest in, the property which is the subject matter of the action. Hence, any holder of interest to the property or right to possession of the land, including the interest of a lessee, may bring an action for quieting of title. [Basis: Article 477, Civil Code] (c) Only the city or municipal mayor can file a civil action to abate a public nuisance. (2%) SUGGESTED ANSWER: False, because under the law it is the district health officer and not the chief executive of the local government who has been authorized to file a civil action to abate a public nuisance. [Basis: Article 700, in relation to Article 699, Civil Code; Cruz v. Pandacan Hiker’s Club, Inc., 778 SCRA 385 (2016), discussed in p. 601, Vol. 1, Rabuya’s Civil Law Reviewer]. (d) Possession of a movable property is lost when the location of the said movable is unknown to the owner. (2%) SUGGESTED ANSWER: False, because possession of movables is not deemed lost so long as they remain under the control of the possessor, even though for the time being he may not know their whereabouts. [Basis: Article 556, Civil Code; discussed in pp. 485-486, Vol. 1, Rabuya’s Civil Law Reviewer] (e) Continuous non-apparent easements can be acquired either through title or by prescription. (2%) SUGGESTED ANSWER: False, because only continuous and apparent easements can be acquired either by virtue of a title or by prescription. [Basis: Article 620, Civil Code; discussed in p. 533, Vol. 1, Rabuya’s Civil Law Reviewer]

XIV. Plutarco owned land that borders on a river. After several years the action of the water of the river caused the deposit of soil, and increased the area of Plutarco’s property by 200 square meters. a. If Plutarco wants to own the increase in area, what will be his legal basis for doing so? Explain your answer. (2%) SUGGESTED ANSWER: Plutarco acquires ownership over the increased area by virtue of accession. According to the Civil Code, the accretion gradually receive from the effects of the current of the waters shall belong to the owner of the lands adjoining the banks of rivers. In order for the above rule to apply, however, the following requisites must be present: (1) that the deposit of soil be gradual and imperceptible; (2) that it be made through the effects of the current of the waters; and (3) that the land where accretion takes place is adjacent to the banks of the rivers. All foregoing requirements are present in this case. Hence, Plutarco aquires ownership over the increased area by operation of law. [Basis: Article 457, Civil Code; Republic v. CA, 132 SCRA 514 (1984); discussed in pp. 402-405, Vol. 1, Rabuya’s Civil Law Reviewer] b. On the other hand, if the river dries up, may Plutarco validly claim a right of ownership of the dried-up river bed? Explain your answer. (2%) SUGGESTED ANSWER: No, because the dried-up river bed shall continue to belong to the State as its property of public dominion. As such, it is not susceptible to private appropriation and acquisitive prescription. Therefore, Plutarco may not validly claim a right of ownership of the dried-up river bed. [Republic v. Santos III, 685 SCRA 51 (2012); Celestial v. Cachopero, 431 SCRA 469 (2003); 657 SCRA 499 (2011); discussed in p. 409, Vol. 1, Rabuya’s Civil Law Reviewer] XV. Kevin signed a loan agreement with ABC Bank. To secure payment, Kevin requested his girlfriend Rosella to execute a document entitled “Continuing Guaranty Agreement” whereby she expressly agreed to be solidarily liable for the obligation of Kevin.

Can ABC Bank proceed directly against Rosella upon Kevin’s default even without proceeding against Kevin first? Explain your answer. (3%) SUGGESTED ANSWER: Yes, ABC Bank may proceed directly against Rosella upon Kevin’s default even without proceeding against Kevin first because Rosella is a surety after she bound herself solidarily with the principal debtor. Notwithstanding the use of the word “guaranty” circumstances may be shown which convert the contract into one of suretyship. Under the Civil Code, when the guarantor binds himself solidarily with the principal debtor, the contract becomes one of suretyship and not of guaranty proper. In a contract of suretyship, the liability of the surety is direct, primary and absolute. He is directly and equally bound with the principal debtor. Such being the case, a creditor can go directly against the surety although the principal debtor is solvent and is able to pay or no prior demand is made on the principal debtor. [Basis: Article 2047, Civil Code; Ong v. PCIB, 448 SCRA 705; discussed in pp. 810-812, Vol. 2, Rabuya’s Civil Law Reviewer] In this case, since Rosella is a surety, ABC Bank can go directly against her even without proceeding against the principal debtor because the surety insures the debt, regardless of whether or not the principal debtor is financially capable to fulfil his obligation. XVI. Jovencio operated a school bus to ferry his two sons and five of their schoolmates from their houses to their school, and back. The parents of the five schoolmates paid for the service. One morning, Porfirio, the driver, took a short cut on the way to school because he was running late, and drove across an unmanned railway crossing. At the time, Porfirio was wearing earphones because he loved to hear loud music while driving. As he crossed the railway tracks, a speeding PNR train loudly blared its horn to warn Porfirio, but the latter did not hear the horn because of the loud music. The train inevitably rammed into the school bus. The strong impact of the collision between the school bus and the train resulted in the instant death of one of the classmates of Jovencio’s younger son. The parents of the fatality sued Jovencio for damages based on culpa contractual alleging that Jovencio was a common carrier; Porfirio for being negligent; and the PNR for damages based on culpa aquiliana.

Jovencio denied being a common carrier. He insisted that he had exercised the diligence of a good father of a family in supervising Porfirio, claiming that the latter had had no history of negligence or recklessness before the fatal accident. (a) Did his operation of the school bus service for a limited clientele render Jovencio a common carrier? Explain your answer. (3%) SUGGESTED ANSWER: Yes, because a common carrier is one who is engaged in the business of carrying or transporting passengers or goods or both, or one who holds himself or itself out to the public as being engaged in said business. In Perena v. Zarate [679 SCRA 208 (2012)], the Court definitively ruled that the operators of a school bus service are common carriers even if they are catering to a limited clientele because of the following reasons: (1) they are engaged in transporting passengers generally as a business, not just as a casual occupation; (2) they are undertaking to carry passengers over established roads by the method by which the business was conducted; and (3) they are transporting students for a fee. The Court additionally explained that despite catering to a limited clientè le, they operate as common carriers because they held themselves out as a ready transportation indiscriminately to the students of a particular school living within or near where they operated the service and for a fee. [Discussed and posted on my FB wall as early as October 23, 2017] (b) In accordance with your answer to the preceding question, state the degree of diligence to be observed by Jovencio, and the consequences thereof. Explain your answer. (3%) SUGGESTED ANSWER: Being a common carrier, Jovencio is required to observe extraordinary diligence, and is presumed to be at fault or to have acted negligently in case of the loss of the effects of passengers, or the death or injuries to passengers. In this case, Jovencio is liable for the death of the student because, acting as a common carrier, he is already presumed to be negligent at the time of the accident because death had occurred to the passenger. Here, Jovencio failed to fend off liability because he failed to prove that he observed extraordinary diligence in ensuring the safety of the passengers. [Basis: Perena v. Zarate, 679 SCRA 208 (2012); discussed and posted on my FB wall as early as October 23, 2017]

(c) Assuming that the fatality was a minor of only 15 years of age who had no earning capacity at the time of his death because he was still a student in high school, and the trial court is minded to award indemnity, what may possibly be the legal and factual justifications for the award of loss of earning capacity? Explain your answer. (4%) SUGGESTED ANSWER: The basis for the computation of the deceased’s earning capacity should be the minimum wage in effect at the time of his death, pursuant to the ruling of the Court in Perena v. Zarate [679 SCRA 208 (2012)]. In the same case, the Court also ruled that the computation of the victim’s life expectancy rate should not be reckoned from his age of 15 years at the time of his death, but on 21 years, his age when he would have graduated from college. In the same case, the Court justified the indemnification of the victim’s loss of earning capacity despite him having been unemployed because compensation of this nature is awarded not for loss of time or earnings but for loss of the deceased’s power or ability to earn money.

Related Documents


More Documents from "Maria"